Sunteți pe pagina 1din 271

Probleme alese

de matematicã
pentru
,
pregãtirea Olimpiadei Nationale
lista scurt@ 2002-2010

30
Probleme alese de matematică pentru
pregătirea Olimpiadei Naţionale
lista scurtă 2002-2010

Editura GIL
©2011 Editura Gil

Titlu: Probleme de matematică pentru pregătirea Olimpiadei Naţionale


(lista scurtă 2002-2010)

Au colaborat:

Ismail Andrei Vlad Matei


Beniamin Bogoşel Tudor Micu
Iurie Boreico Andrei Neguţ
Radu Bumbăcea Tudor Pădurariu
Omer Cerrahoglu Cosmin Pohoaţă
Filip Chindea Călin Popa
Alexandru Ciolan Corneliu Prodescu
Andrei Ciupan Claudiu Raicu
Gabriel Dospinescu Cătălin Ţurcaş
Michnea Dragoş Andrei Ungureanu
Andrei Frimu Adrian Zahariuc
Livia Ilie Alexandru Zamorzaev
Cezar Lupu

ISBN 978-606-500-052-0

Toate drepturile rezervate Editurii Gil. Nici o parte din acest volum nu poate fi
copiată fãră permisiunea scrisă a Editurii Gil.
Copyright © 2011 by Gil. All rights reserved.

Departament difuzare:

Editura GIL,
CP 44 O.P.3,
Zalău, Sălaj,
cod 450200

Tel. 0260/616314;
Mobil 0744/612106
Tel/Fax.: 0260/616414
E-mail: comenzi@gil.ro

Editor: Mircea LASCU


www.gil.ro
Prefaţă

Olimpiada de matematică reuneşte an de an tot mai mulţi elevi şi profesori, dornici
de a-şi demonstra măiestria ı̂n rezolvarea celor mai dificile, dar frumoase probleme de
matematică.
Prezenta culegere reuneşte ı̂n premieră problemele aflate ı̂n atenţia Comisiei Cen-
trale a Olimpiadei Naţionale de Matematică, ı̂n ultimii nouă ani. Deşi nu au fost
folosite ı̂n concurs, problemele din listele scurte sunt valoroase, originale şi dificile,
propuse de cei mai buni profesori de la noi din ţară.
Cartea se adresează elevilor talentaţi şi pasionaţi de matematica concursurilor şcolare,
profesorilor care ı̂i antrenează pentru aceste frumoase şi dificile confruntări, şi nu ı̂n
ultimul rând oricărui iubitor de matematică. Soluţiile problemelor sunt originale şi
reunesc idei noi şi tehnici inedite, bazate pe bogata noastră experienţă competiţională.
Mulţumim profesorilor noştri, care au avut generozitatea de a ne ajuta ı̂n pregătirea
pentru concursuri, precum şi tuturor celor care au contribuit la realizarea acestei cărţi
şi aşteptăm sugestii sau soluţii, prin care puteţi construi la reeditarea acestei culegeri.
De asemenea, mulţumim editurii GIL, care a făcut posibilă apariţia acestei cărţi.
Tuturor vă dorim mult succes la concursurile la care veţi participa!

Autorii

3
Cuprins

I Enunţuri 9
1 Probleme avute ı̂n atenţia comisiei, 2002 11
1.1 Clasa a VII-a . . . . . . . . . . . . . . . . . . . . . . . . . . . . . . . . . . . 11
1.2 Clasa a VIII-a . . . . . . . . . . . . . . . . . . . . . . . . . . . . . . . . . . 13
1.3 Clasa a IX-a . . . . . . . . . . . . . . . . . . . . . . . . . . . . . . . . . . . 15
1.4 Clasa a X-a . . . . . . . . . . . . . . . . . . . . . . . . . . . . . . . . . . . . 16

2 Probleme avute ı̂n atenţia comisiei, 2003 19


2.1 Clasa a VII-a . . . . . . . . . . . . . . . . . . . . . . . . . . . . . . . . . . . 19
2.2 Clasa a VIII-a . . . . . . . . . . . . . . . . . . . . . . . . . . . . . . . . . . 21
2.3 Clasa a IX-a . . . . . . . . . . . . . . . . . . . . . . . . . . . . . . . . . . . 22
2.4 Clasa a X-a . . . . . . . . . . . . . . . . . . . . . . . . . . . . . . . . . . . . 24

3 Probleme avute ı̂n atenţia comisiei, 2004 29


3.1 Clasa a VII-a . . . . . . . . . . . . . . . . . . . . . . . . . . . . . . . . . . . 29
3.2 Clasa a VIII-a . . . . . . . . . . . . . . . . . . . . . . . . . . . . . . . . . . 31
3.3 Clasa a IX-a . . . . . . . . . . . . . . . . . . . . . . . . . . . . . . . . . . . 34
3.4 Clasa a X-a . . . . . . . . . . . . . . . . . . . . . . . . . . . . . . . . . . . . 40

4 Probleme avute ı̂n atenţia comisiei, 2005 43


4.1 Clasa a VII-a . . . . . . . . . . . . . . . . . . . . . . . . . . . . . . . . . . . 43
4.2 Clasa a VIII-a . . . . . . . . . . . . . . . . . . . . . . . . . . . . . . . . . . 47
4.3 Clasa a IX-a . . . . . . . . . . . . . . . . . . . . . . . . . . . . . . . . . . . 51
4.4 Clasa a X-a . . . . . . . . . . . . . . . . . . . . . . . . . . . . . . . . . . . . 56

5 Probleme avute ı̂n atenţia comisiei, 2006 59


5.1 Clasa a VII-a . . . . . . . . . . . . . . . . . . . . . . . . . . . . . . . . . . . 59
5.2 Clasa a VIII-a . . . . . . . . . . . . . . . . . . . . . . . . . . . . . . . . . . 60
5.3 Clasa a IX-a . . . . . . . . . . . . . . . . . . . . . . . . . . . . . . . . . . . 62
5.4 Clasa a X-a . . . . . . . . . . . . . . . . . . . . . . . . . . . . . . . . . . . . 65

5
6 CUPRINS

6 Probleme avute ı̂n atenţia comisiei, 2007 69


6.1 Clasa a VII-a . . . . . . . . . . . . . . . . . . . . . . . . . . . . . . . . . . . 69
6.2 Clasa a VIII-a . . . . . . . . . . . . . . . . . . . . . . . . . . . . . . . . . . 73
6.3 Clasa a IX-a . . . . . . . . . . . . . . . . . . . . . . . . . . . . . . . . . . . 78
6.4 Clasa a X-a . . . . . . . . . . . . . . . . . . . . . . . . . . . . . . . . . . . . 80

7 Probleme avute ı̂n atenţia comisiei, 2008 83


7.1 Clasa a VII-a . . . . . . . . . . . . . . . . . . . . . . . . . . . . . . . . . . . 83
7.2 Clasa a VIII-a . . . . . . . . . . . . . . . . . . . . . . . . . . . . . . . . . . 87
7.3 Clasa a IX-a . . . . . . . . . . . . . . . . . . . . . . . . . . . . . . . . . . . 89
7.4 Clasa a X-a . . . . . . . . . . . . . . . . . . . . . . . . . . . . . . . . . . . 92

8 Probleme avute ı̂n atenţia comisiei, 2009 97


8.1 Clasa a VII-a . . . . . . . . . . . . . . . . . . . . . . . . . . . . . . . . . . . 97
8.2 Clasa a VIII-a . . . . . . . . . . . . . . . . . . . . . . . . . . . . . . . . . . 97
8.3 Clasa a IX-a . . . . . . . . . . . . . . . . . . . . . . . . . . . . . . . . . . . 98
8.4 Clasa a X-a . . . . . . . . . . . . . . . . . . . . . . . . . . . . . . . . . . . . 99

9 Probleme avute ı̂n atenţia comisiei, 2010 101


9.1 Clasa a VII-a . . . . . . . . . . . . . . . . . . . . . . . . . . . . . . . . . . . 101
9.2 Clasa a VIII-a . . . . . . . . . . . . . . . . . . . . . . . . . . . . . . . . . . 101
9.3 Clasa a IX-a . . . . . . . . . . . . . . . . . . . . . . . . . . . . . . . . . . . 102
9.4 Clasa a X-a . . . . . . . . . . . . . . . . . . . . . . . . . . . . . . . . . . . . 103

II Soluţii 105

10 Probleme avute ı̂n atenţia comisiei, 2002 107


10.1 Clasa a VII-a . . . . . . . . . . . . . . . . . . . . . . . . . . . . . . . . . . . 107
10.2 Clasa a VIII-a . . . . . . . . . . . . . . . . . . . . . . . . . . . . . . . . . . 110
10.3 Clasa a IX-a . . . . . . . . . . . . . . . . . . . . . . . . . . . . . . . . . . . 112
10.4 Clasa a X-a . . . . . . . . . . . . . . . . . . . . . . . . . . . . . . . . . . . . 116

11 Probleme avute ı̂n atenţia comisiei, 2003 121


11.1 Clasa a VII-a . . . . . . . . . . . . . . . . . . . . . . . . . . . . . . . . . . . 121
11.2 Clasa a VIII-a . . . . . . . . . . . . . . . . . . . . . . . . . . . . . . . . . . 124
11.3 Clasa a IX-a . . . . . . . . . . . . . . . . . . . . . . . . . . . . . . . . . . . 125
11.4 Clasa a X-a . . . . . . . . . . . . . . . . . . . . . . . . . . . . . . . . . . . . 132
CUPRINS 7

12 Probleme avute ı̂n atenţia comisiei, 2004 141


12.1 Clasa a VII-a . . . . . . . . . . . . . . . . . . . . . . . . . . . . . . . . . . . 141
12.2 Clasa a VIII-a . . . . . . . . . . . . . . . . . . . . . . . . . . . . . . . . . . 150
12.3 Clasa a IX-a . . . . . . . . . . . . . . . . . . . . . . . . . . . . . . . . . . . 150
12.4 Clasa a X-a . . . . . . . . . . . . . . . . . . . . . . . . . . . . . . . . . . . . 162

13 Probleme avute ı̂n atenţia comisiei, 2005 169


13.1 Clasa a VII-a . . . . . . . . . . . . . . . . . . . . . . . . . . . . . . . . . . . 169
13.2 Clasa a VIII-a . . . . . . . . . . . . . . . . . . . . . . . . . . . . . . . . . . 183
13.3 Clasa a IX-a . . . . . . . . . . . . . . . . . . . . . . . . . . . . . . . . . . . 193
13.4 Clasa a X-a . . . . . . . . . . . . . . . . . . . . . . . . . . . . . . . . . . . . 205

14 Probleme avute ı̂n atenţia comisiei, 2006 211


14.1 Clasa a VII-a . . . . . . . . . . . . . . . . . . . . . . . . . . . . . . . . . . . 211
14.2 Clasa a VIII-a . . . . . . . . . . . . . . . . . . . . . . . . . . . . . . . . . . 214
14.3 Clasa a IX-a . . . . . . . . . . . . . . . . . . . . . . . . . . . . . . . . . . . 219
14.4 Clasa a X-a . . . . . . . . . . . . . . . . . . . . . . . . . . . . . . . . . . . . 226

15 Probleme avute ı̂n atenţia comisiei, 2007 233


15.1 Clasa a VII-a . . . . . . . . . . . . . . . . . . . . . . . . . . . . . . . . . . . 233
15.2 Clasa a VIII-a . . . . . . . . . . . . . . . . . . . . . . . . . . . . . . . . . . 241
15.3 Clasa a IX-a . . . . . . . . . . . . . . . . . . . . . . . . . . . . . . . . . . . 250
15.4 Clasa a X-a . . . . . . . . . . . . . . . . . . . . . . . . . . . . . . . . . . . . 250

16 Probleme avute ı̂n atenţia comisiei, 2008 255


16.1 Clasa a VII-a . . . . . . . . . . . . . . . . . . . . . . . . . . . . . . . . . . . 255
16.2 Clasa a VIII-a . . . . . . . . . . . . . . . . . . . . . . . . . . . . . . . . . . 255
16.3 Clasa a IX-a . . . . . . . . . . . . . . . . . . . . . . . . . . . . . . . . . . . 255
16.4 Clasa a X-a . . . . . . . . . . . . . . . . . . . . . . . . . . . . . . . . . . . . 259

17 Probleme avute ı̂n atenţia comisiei, 2009 261


17.1 Clasa a VII-a . . . . . . . . . . . . . . . . . . . . . . . . . . . . . . . . . . . 261
17.2 Clasa a VIII-a . . . . . . . . . . . . . . . . . . . . . . . . . . . . . . . . . . 261
17.3 Clasa a IX-a . . . . . . . . . . . . . . . . . . . . . . . . . . . . . . . . . . . 261
17.4 Clasa a X-a . . . . . . . . . . . . . . . . . . . . . . . . . . . . . . . . . . . . 263

18 Probleme avute ı̂n atenţia comisiei, 2010 267


18.1 Clasa a VII-a . . . . . . . . . . . . . . . . . . . . . . . . . . . . . . . . . . . 267
18.2 Clasa a VIII-a . . . . . . . . . . . . . . . . . . . . . . . . . . . . . . . . . . 267
18.3 Clasa a IX-a . . . . . . . . . . . . . . . . . . . . . . . . . . . . . . . . . . . 267
8 CUPRINS

18.4 Clasa a X-a . . . . . . . . . . . . . . . . . . . . . . . . . . . . . . . . . . . . 270


Partea I

Enunţuri

9
Capitolul 1

Probleme avute ı̂n atenţia comisiei, 2002

1.1 Clasa a VII-a


1. Dacă 1 + 2 + . . . + n = abcabc (ı̂n baza 10), aflaţi numerele n şi abc.
Gheorghe Moraru, Bucureşti

2. Să se determine cifrele x şi y, x < y astfel ca


q
0, xx(y) + 0, yy(x) ∈ Q.

Romeo Zamfir, Galaţi



3. Să se arate că numărul a = 52002 · n2002 + 32001 este iraţional oricare ar fi n ∈ N.
Fogarassy Mihály, Gheorgheni
2 4 6 2000
4. Să se arate că numărul a = · · ··· satisface inegalităţile:
3 5 7 2001
1 1
√ <a< √ .
2001 1001

Virginica Tică şi Vasile Tică, Câmpulung

5. În triunghiul ABC punctul D este mijlocul segmentului (AC). Fie n ∈ N∗ şi
Cn ∈ (BC cu C ∈ (BCn ) astfel ı̂ncât CCn = n · BC şi BD ∩ AC = {Dn }. Să se
arate că:
ACn
a) = n + 2 pentru orice n ∈ N∗ ;
ADn
b) BDn < 2 · BD pentru orice n ∈ N∗ .
Nicoale Viziroiu, Piteşti

11
12

6. Rezolvaţi ı̂n N inecuaţia ax + 3 < 4x + 2a, unde parametrul a ∈ Z \ {4}. Pentru ce


valori ale lui a inecuaţia are cele mai multe soluţii numere naturale ?
Emilian Ionescu, Slobozia

7. Să se rezolve ı̂n mulţimea numerelor reale ecuaţia:


√ √
4x2 + 6y 2 − 10 2 · x − 10 3y + 25 = 0.

Mariana Coadă, Galaţi

8. Considerăm triunghiul ABC şi punctele M ∈ (AB, N ∈ (BC, P ∈ (CA, astfel


ı̂ncât au loc relaţiile: AM = kAB, BN = kBC, CP = kCA unde k este o con-
stantă > 1. Să se arate că centrele de greutate ale triunghiurilor ABC şi M N P
coincid.
Simon Jozsef, Miercurea Ciuc

9. Se consideră trapezul dreptunghic ABCD cu AB paralelă cu CD şi unghiul din


A drept. Arătaţi că AB 2 + CD2 = BC 2 + AD2 , dacă şi numai dacă AC ⊥ BD.
Dorin Popa, Slatina

10. Să se arate că dintr-o foaie de hârtie ı̂n formă de pătrat cu latura de 10 cm se
poate decupa un triunghi echilateral cu latura de 10,3 cm dar nu se poate decupa
un triunghi echilateral de latură 10,4 cm.
Laurenţiu Panaitopol, Bucureşti

11. Dacă x ∈ R şi x2002 = x2003 + 1, atunci să se arate că x nu este raţional şi x < 1.
P. Simion, S. Smarandache, Bucureşti

12. Într-un triunghi dreptunghic, a este lungimea ipotenuzei, iar b, c sunt lungimile
catetelor. Arătaţi că: √
a bc 1+ 2
1< + ≤ .
b + c a2 2
Petre Stângescu, Bucureşti

13. Fie r un număr natural, r > 1 şi punctele raportate la reperul cartezian XOY :
A(−r, 0), B(r, 0), P (1 − r, 0) şi M astfel ı̂ncât AM 2 = 2r şi M B 2 = 2r(2r − 1). Fie
P1 şi P2 proiecţiile lui P pe dreptele AM respectiv BM . Determinacţi lungimea
segentului [P1 P2 ] ı̂n funcţie de r.
Ion Cheşcă, Lehliu - Gară
13

14. Un elev are 10 bile numerotate cu numerele 1, 2, . . . , 9, 10. El trebuie să le pună
ı̂n trei urne identice astfel ı̂ncât ı̂n nici o urnă să nu fie două bile numerotate cu
numere consecutive. In câte moduri poate face acest lucru?
***

15. Să se determine numerele naturale a, b, c care verifică relaţile:

a2 + 3 = bc, b2 + 14 = ca, c2 = ab + 30.

***

1.2 Clasa a VIII-a


2000 2002
1. Să se arate că numerele 55 + 1 şi 55 + 26 sunt prime ı̂ntre ele.
Daniel Streţcu, Turnu Severin

2. a) Fie a, b ∈ R , a < b. Să se arate că oricare ar fi c ∈ [a, b] există t ∈ [0, 1] astfel
ı̂ncât c = ta + (1 − t)b;
b) Dacă c, d ∈ (a, b) şi c + d = a + b atunci ab < cd.
Dan Marinescu şi Viorel Cornea, Hunedoara

3. Se dă a ∈ R. Calculaţi ı̂n funcţie de a numerele reale x, y, z, ştiind că ı̂ndeplinesc


condiţiile:
x + y + z = 3a − 1, xy + xz + yz + x + y = 3a2 .

Gheorghe Molea, Curtea de Argeş

4. Fie a, b, c ∈ R astfel ı̂ncât a2 + b2 + c2 = x > 0. Să se arate că



a3 + b3 + c3 − 3abc ≤ x x.

Marcel Chiriţă, Bucureşti

5. Să se arate că dacă a, b, c ∈ (0, ∞) şi abc = 1, atunci are loc inegalitatea
 6
a+b+c
(a + b)(b + c)(c + a) ≤ .
2

Valer Pop, Şanţ, Bistriţa Năsăud


14

6. Demonstraţi că dacă a, b, c ∈ (1, ∞) astfel ı̂ncât abc = 2 2, atunci avem inegali-
tatea:
(a + 1)(b + 1)(c + 1) > 8(a − 1)(b − 1)(c − 1).

Gheorghe Molea, Curtea de Argeş

7. Pentru orice număr natural notăm cu p(n) cel mai mare pătrat perfect cel mult
egal cu n.
a + 1 a + 2
1) Determinaţi numerele naturale a pentru care p = .
2 3
2) Arătaţi că nu existănumere naturle b care să verifice egalitatea:

p(b2 ) + p(b2 + 1) + p(b2 + 2) + . . . + p((b + 1)2 ) = 2002.

***

8. Se dă un poligon convex cu n laturi şi fie a ∈ N∗ . Determinaţi poligonul ştiind că
numărul diagonalelor sale este egal cu raportul dintre n şi a.
Valer Pop, Şanţ, Bistriţa Năsăud

9. Se consideră un paralelipiped dreptunghic cu diagonala egală cu 1 şi M un punct


abitrar interior paralelipipedului. Se notează cu S(M ) suma pătratelor distanţelor
de la M la cele opt vârfuri ale sale. Să se afle cea mai mică şi cea mai mare valoare
a sumei S(M ).
Valentin Matrosenco, Bucureşti

10. Pe planul triunghiului ABC ı̂n punctele A şi B se ridică de o parte şi de alta a lui
\ = 90◦ . Demonstraţi că:
perpendicularele AD şi BE, astfel ca m(DCE)

1√
Aria(ABC) = AC 2 · BC 2 − AD2 · BE 2 .
2

Gheorghe Molea, Curtea de Arge

11. În tetraedrul ABCD punctele E şi F sunt mijloacele medianelor AM şi AN ale tri-
unghiurilor ABC respectiv ACD. Dacă CE ∩ AB = {P }, CF ∩ AD = {Q}, DF ∩
AC = {R}, demonstraţi că:
a) 9Aria(P QR) = Aria(BCD);
b) 12(P Q + EF + M N ) = 13BD.
Virginia şi Vasile Tică, Câmpulung
15

12. Fie m, n numere naturale. Să se arate că există numerele naturale a şi b astfel ı̂ncât:

(m4 − m2 + 1)(n4 − n2 + 1) = a2 + b2 .

Bogdan Enescu, Buzău

1.3 Clasa a IX-a


1. Determinaţi perechile (x, y) de numere ı̂ntregi care satisfac:

2(x3 − xy + y 3 ) = 3(x2 + y 2 ).

***

2. Să se arate că 5(a2 + b2 + c2 ) ≤ 6(a3 + b3 + c3 ) + 1, pentru orice numere reale


nenegative a, b, c care satisfac relaţia a + b + c = 1.
Mihai Piticari, Câmpulung Moldovenesc; Dan Popescu, Suceava

3. Să se demonstreze că, pentru orice numere naturale nenule şi distincte două câte
două a1 , a2 , . . . , an are loc inegalitatea:

(a1 + a2 + . . . + an )2 n3 − n
a21 + a22 + . . . + a2n ≥ + .
n 12

Mihai Piticari, Câmpulung Moldovenesc; Dan Popescu, Suceava

4. Fie a, b, c numere reale pozitive astfel ı̂ncât a2 + b2 + c2 + 2abc = 1. Să se arate că:

a+b+c
ab + bc + ca ≤ .
2

Octavian Purcaru, Ploieşti

5. Fie A, B, C, D puncte distincte pe un cerc de centru O. Dacă există x, y ∈ R∗ astfel


ca:
−→ −−→ −−→ −→ −→ −−→ −−→ −→
||xOA + y OB|| = ||xOB + y OC|| = ||xOC + y OD|| = ||xOD + y OA||

să se arate că ABCD este pătrat.


Manuela Prajea, Drobeta Turnu-Severin
16

6. Se consideră mulţimea parabolelor de forma y = ax2 + bx + c care intersectează


axele de coordonate ı̂n trei puncte distincte. Dacă cele trei puncte determină un
triunghi dreptunghic, numim parabola pitagorică, iar dacă ele determină un tri-
unghi echilateral, respectiva parabola va fi numită echilateră.
a) Determinaţi ecuaţiile generale ale acestor clase.
b) Fiind dată o parabolă echilateră determinaţi toate parabolele pitagorice ce au
ı̂n comun cu aceasta două dintre cele trei puncte de intersecţie cu axele.
Adrian Ghioca, Sinaia

7. Arătaţi că dacă numerele strict pozitive a, b, c satisfac a + b + c = 1, atunci:


5 5 5 1
(ab) 4 + (bc) 4 + (ca) 4 < .
4
Dinu Teodorescu

1.4 Clasa a X-a


1. Fie n ∈ N, n ≥ 3. Determinaţi z ∈ C\R astfel ı̂ncât z n şi (z + 1)n să fie numere
reale.
Gheorghe Iurea, Iaşi

2. Determinaţi n ∈ N∗ astfel ı̂ncât să existe x1 , x2 , . . . , xn ∈ [0, ∞) cu proprietatea

(1 + x1 )(1 + x2 ) · · · (1 + xn )(x1 + x2 + . . . + xn ) = 2n2 x1 x2 · · · xn .

Dan Ştefan Marinescu; V. Cornea, Hunedoara

3. Dacă A, B sunt două submulţimi nevide arbitrare ale lui C, notăm

AB = {xy|x ∈ A, y ∈ B}.

Fiind date numerele a ∈ C∗ şi n ∈ N∗ să se determine mulţimile X ⊆ C cu n


elemente care satisfac proprietatea:

XX ⊆ {a}X.

Marcel Ţena, Bucureşti

4. Dacă a, b, c ∈ C distincte şi |a| = |b| = |c| = R iar k > 0, arătaţi că:

| − a + kb + kc| + | − b + kc + ka| + | − c + ka + kb| ≤ 3(k + 1)R.

Călin Burduşel, Târgovişte


17

5. Fie a, b, c ∈ C cu |a| = |b| = |c| = 1. Se notează Sm = |a + b|m + |b + c|m + |c + a|m .


Dacă există p ∈ N∗ astfel ı̂ncât S2p ≤ 3, atunci pentru orice n ∈ N avem Sn = 3.
Marcel Chiriţă, Bucureşti

6. Fie tetraedrul ABCD, m, n, p, k > 0, iar M ∈ (AD), N ∈ (BD), P ∈ (CD), K ∈


AM BN CP
(DG), cu = m, = n, = p, unde G este centrul de greutate al
MD ND PD
triunghiului ABC. Să se arate că:

K ∈ (M N P ) ⇔ m + n + p = 3k.

Vasile Popa, Galaţi


p
7. Să se arate că şirul de numere pozitive (an )n≥1 ce satisface relaţia an+1 = 6 − 2a2n ,
pentru orice n ≥ 1, este constant.
Laurenţiu Panaitopol, Bucureşti

8. Se consideră numerele complexe a, b, c distincte două câte două, astfel ı̂ncât |a| =
|b| = |c| = 1 şi |a − b|2 + |b − c|2 + |c − a|2 > 8. Să se arate că |(a + b)(b + c)(c + a)| ≤ 1.
Dan Nedeianu, Drobeta Turnu-Severin

9. Fie ABC un triunghi şi A1 ∈ (BC), B1 ∈ (CA), C1 ∈ (AB) astfel ı̂ncât:

AB + BA1 = AC + CA1

AB + AB1 = BC + CB1
AC + AC1 = BC + BC1 .
Să se arate că SABC ≥ 4SA1 B1 C1 .
Marcel Chiriţă, Bucureşti
18
Capitolul 2

Probleme avute ı̂n atenţia comisiei, 2003

2.1 Clasa a VII-a


1. Demonstraţi că dacă x şi y sunt numere ı̂ntregi, atunci numerele x3 + y 3 şi x + y +
2x+y nu pot fi egale.
Valer Pop, Şanţ

2. Determinaţi numerele naturale a, b, c diferite oricare două, dacă


a+1 b+2 c+3 2 3 4
= = şi + + ∈ N.
2 3 4 a+1 b+2 c+3

Valer Pop, Şanţ

3. Pe o dreaptă d se consideră punctele A0 , A1 , A2 , . . . , A25 ı̂n această ordine şi astfel


ı̂ncât A0 A1 = 1 cm, A2 A3 = 3 cm, . . . , A24 A25 = 25 cm. Să se determine segmentele
de lungime 1 m, cu capetele ı̂n puncte dintre A0 , A1 , A2 , . . . , A25 .
Ioan Bogdan

4. Determinaţi a ∈ Z astfel ı̂ncât inecuaţia


√ √
x− 2 + x− 3 <a

să aibă exact două soluţii ı̂ntregi.


Marius Ghergu, Slatina

5. Determinaţi numerele naturale x, y, z, t ştiind că

xyzt(x + y + z) = xy + yz + zx.

Gheorghe Molea

19
20

6. Determinaţi a, b astfel ı̂ncât numărul n = abab, scris ı̂n baza 7 să fie pătrat perfect.
Viorel Cornea, Dan Marinescu, Hunedoara
b
7. Să se arate că dacă a, b, c sunt numere reale şi ax + + c este număr ı̂ntreg oricare
x
ar fi x real nenul, atunci a = b = 0, iar c este ı̂ntreg.
Mariana Coadă, Galaţi

8. Determinaţi numerele ı̂ntregi nenule x, y, z pentru care


1 1 1
+ +
x y z
este ı̂ntreg nenul.
Gheorghe Iurea, Iaşi

9. Arătaţi că pentru orice număr natural nenul n, numărul real


p
A = 49n2 + 0, 35n
are primele trei zecimale aceleaşi.
Cecilia Deaconu, Piteşti

10. Determinaţi toate numerele abc scrise ı̂n baza 10 astfel ı̂ncât a este media ar-
monică a numerelor b şi c.
Daniel Cojocaru, Slatina

11. Fie N punctul de intersecţie al bisectoarelor exterioare ale unghiurilor B şi C ale
triunghiului ABC şi D proiecţia lui N pe dreapta BC. Arătaţi că:
a) AB + BD = AC + CD;
b) AB 2 + BD2 + AM 2 ≥ 2N A · AM − 2AB · BD.
***

12. Fie Γ1 şi Γ2 două cercuri de centre O1 şi respectiv O2 , care se intersectează ı̂n A şi
B. Fie CD o tangentă comună (CD este mai apropiată de A decât de B). Fie O
centrul cercului circumscris triunghiului ACD. Demonstraţi că CBD \ = O\ 1 OO2 .

Valentin Vornicu

13. Fie ABCD un pătrat şi M un punct pe segmentul (AB). Bisectoarea unghiului
M
\ DC intersectează latura BC ı̂n N . Arătaţi că pentru orice punct M de pe seg-
mentul (AB) avem
M B + BN < AM + CN.

Cecilia Deaconu, Piteşti


21

14. În exteriorul triunghiului ABC considerăm triunghiurile dreptunghice isoscele


ABD şi ACE cu ipotenuzele AD, respectiv AE. Să se demonstreze că ı̂nălţimea
din A a triunghiului ABC, dreptele DC şi BE sunt concurente.
Alexandru Zăgrăian, Cluj

15. Fie ABC un triunghi şi M un punct variabil pe latura BC. Paralelele din B şi
C la AM taie dreptele AC şi AB ı̂n punctele N şi P . Să se determine minimul
expresiei
BN + CP
.
AM
Mircea Lascu, Zalău

2.2 Clasa a VIII-a


1. Demonstraţi inegalitatea
(a + b)3 (b + c)3 (c + a)3
+ + ≥ 8(a2 + b2 + c2 ).
c a b
Nicolae Papacu, Slobozia

2. Să se arate că numărul


17 + 27 + 37 + . . . + 10007
este divizibil prin 500500.
Simona Stoicoiu şi Costin Zălog, Tg. Jiu

3. Dacă p este un număr prim fixat, să se rezolve ecuaţia

p{x} = x + [x].

Daniel Cojocaru, Slatina

4. a) Să se demonstreze că pentru numere strict pozitive a, b, c avem


X X 2
2003 100
3 a ≥ abc a .

b) Determinaţi cel mai mic număr nenul k astfel ca pentru orice numere strict
pozitive a, b, c să avem
X X 16
2003 125
k a ≥ abc a .

Doina şi Aurelian Ionescu, Topliţa


22

5. Pe planul rombului (ABCD) se√ridică, de aceeaşi parte, perpendicularele AA0 şi


2
CC 0 astfel ı̂ncât AA0 = CC 0 ≥ AB. Arătaţi că dacă măsura unghiului format
2
de planele (A00 BD) şi (C 0 BD) este egală cu măsura unghiului format de planele
(A0 BC 0 ) şi (A0 DC 00 ), atunci ABCD este pătrat.
Marius Ghergu, Slatina

6. În sistemul de axe ortogonale xOy se consideră punctele A(a, b) şi B(c, d) unde
a 6= b 6= c 6= d 6= a. Care este condiţia necesară şi suficientă ca
min (M A + M B) = min (N A + N B)?
M ∈Oy N ∈Ox

Cecilia Deaconu, Piteşti

7. În triunghiul ABC se consideră punctele M ∈ (AC), D ∈ (BM ) astfel ı̂ncât


BM = 2M D şi CD k AB. Punctul N este mijlocul lui (AB) şi 6M N = AB. În ex-
teriorul planului ABC se consideră punctul V cu proprietatea V A = V B = V E,
unde {E} = AD ∩ BC. Dacă distanţele de la V la AD respectiv BC sunt egale,
demonstraţi că:
a) V E ⊥ AB;
b) σ(4V N D)2 + σ(4V N C)2 + σ(4CDN )2 = σ(4V CD)2 .
Nicolae Vizuroiu, Piteşti

2.3 Clasa a IX-a


1 1 1 a b c
1. Fie a, b, c trei numere reale strict pozitive satisfăcând + + ≥ + + .
a b c b c a
Demonstraţi că
a2 b 2 c 2
+ + ≥ ab + bc + ca.
c a b
Tudorel Lupu, Constanţa

2. Determinaţi numerele x, y, z ∈ [1/3, ∞) ştiind că


√ p √ √
x + y + z ≤ 4, 3x − 1 + 3y − 1 + 3z − 1 ≥ 3 3.

Gheorghe F. Molea, Curtea de Argeş

3. Să se rezolve ecuaţia    


49 16
= x+ .
x2 + 9 x
Romeo Zamfir, Galaţi
23

4. Determinaţi numerele reale x1 , x2 , . . . , xn , n ≥ 2, dacă

x1 (2004 − x2 ) = x2 (2004 − x3 ) = . . . = xn (2004 − x1 ) = 2003.

Gheorghe Iurea, Iaşi

5. Fie a1 , a2 , . . . , an numere reale astfel ı̂ncât diferenţa dintre oricare două dintre ele
este un număr iraţional. Fie Ai = {ai + r|r ∈ Q}, i = 1, 2, . . . , n. Să se arate că
[n
Ai 6= R\Q.
i=1

Mihai Piticari

6. Fie a, b, c numere reale strict pozitive şi n ∈ N∗ . Comparaţi numerele

a b c 3
+ + şi .
na + b nb + c nc + a n + 1

Marin Chirciu, Piteşti

7. Dacă x, y, z, m, n > 0 şi x + y + z = 1, demonstraţi că


X x4 3
≥ .
(mx + ny)(my + nx) (m + n)2

Marin Chirciu
n
X n
X
8. Fie xi , yi ∈ (0, ∞), i = 1, 2, . . . , n, astfel ı̂ncât xi = yi . Demonstraţi că
i=1 i=1

n
X
n
xi
X x2i i=1
≥ .
i=1
xi + y i 2

Gigel Buth, Satu Mare

9. Să se arate că pentru orice număr real pozitiv x avem:


s r

q
x + x + x + x + x2 + x + . . . + x2 + x < x + 1,
2 2

unde numărul radicalilor este n.


D.M. Bătineţu-Giurgiu
24

10. Aflaţi soluţiile reale ale sistemului de ecuaţii


 √ √
 3 x + 2√ 6 yz = 13
√3 y + 2 6 zx = 13 .
 √ √
3
z + 2 6 xy = 13

Marin Chirciu

11. Fie n ≥ 2 un număr natural. Calculaţi minimul expresiei ab + bc + cd după toate


scrierile n = a + b + c + d, cu a, b, c, d numere naturale nenule.
V. Zidaru

12. Să se determine funcţiile f : Z → R care satisfac relaţiile:


a) f (x + 2003) ≤ f (x) + 2003;
b) f (x + 1987) ≥ f (x) + 1987, ∀x ∈ Z.
Dan Popescu, Suceava

13. Dacă a, b, c sunt numere reale strict pozitive satisfăcând abc = 1, demonstraţi
inegalitatea a2 + b2 + c2 + 3 ≥ ab + bc + ca + a + b + c.
Mircea Lascu, Marian Tetiva

14. Pe laturile triunghiului ABC se consideră punctele A1 ∈ (BC), B1 ∈ (CA), C1 ∈


(AB) care le ı̂mpart ı̂n acelaşi raport pozitiv neunitar. Fie {A2 } = BB1 ∩CC1 , {B2 } =
CC1 ∩ AA1 , {C2 } = AA1 ∩ BB1 . Arătaţi că:
a) Triunghiurile ABC şi A2 B2 C2 au acelaşi centru de greutate;
−−→ −−→ −−→
b) Vectorii AA2 , BB2 , CC2 pot fi laturile unui triunghi.
Nicolae Papacu, Slobozia

15. Fie ABCDE un pentagon convex şi P ∈ [DE]. Notăm cu G1 , G2 , G3 , G4 , re-


spectiv, centrele de greutate ale triunghiurilor ADE, AP B, ABC şi AP C. Să
se arate că G1 G2 G3 G4 este un paralelogram dacă şi numai dacă P este mijlocul
segmentului [DE].
Marian Ionescu, Piteşti

2.4 Clasa a X-a


1. Fie f ∈ C[X] polinomul f (X) = X 2003 + a2002 X 2002 + . . . + a1 X + a0 cu toate
rădăcinile de modul 1. Dacă a0 6= 1, atunci numărul
a2002 − a1
1 − a0
25

este real.
Valentin Vornicu

2. Se consideră mulţimile

An = {x ∈ R+ |1x + 2x + . . . + nx = (n + 1)x + (n + 2)x }

şi B = A1 ∪ A2 ∪ . . . ∪ A2002 . Câte elemente are B?


Mariana Coadă, Galaţi

3. Rezolvaţi ecuaţia
2
5x + 3 x = 28.

Romeo Zamfir, Galaţi

4. Rezolvaţi ecuaţia
[2x ] + [3x ] = [6x ].

Nicolae Papacu, Slobozia

5. Rezolvaţi ecuaţia

(2x + 3x + 4x )(5x + 6x + 7x + 8x ) = (4x + 5x )(8x + 9x + 10x ).

Nicolae Papacu

6. Fie şirul de numere reale (an )n≥1 cu a1 = 1 astfel ca pentru orice număr natural
p(p + 1)
nenul p, notând m = , numerele am , am+1 , . . . , am+p+1 sunt ı̂n progresie
2
aritmetică cu raţia p.
a) Calculaţi a2003 .
b) Există n astfel ı̂ncât an = 2003?
Nicolae Papacu

7. Arătaţi că pentru orice n ≥ 2 natural, numerele

xn = C0n − C3n + C6n − . . . , yn = C2n − C5n + C8n − . . . ,

verifică x2n + yn2 ≥ 2 · 3n−2 .


Marian Ursărescu, Roman
26

8. Fie a, b, c numere strict pozitive, subunitare cu suma 1. Demonstraţi că:


X log2 c 9
ab
≥ .
a+b 8
Tudorel Lupu, Constanţa

9. Rezolvaţi ecuaţia
3 −1 1
3x + 3 x = 4.

Tudorel Lupu

10. Rezolvaţi ı̂n R sistemul de ecuaţii:


 x y z
 7 + 7 + 7 = 21
3y − z 3 =2 .
 x 5
5 −z =4

Daniel Jinga, Piteşti

11. Fie a, b, c numere strict pozitive satisfăcând a3 + b3 + c3 = 1. Arătaţi că


X √3
loga ( 3bc) > 0.

I.V. Maftei
X X
12. Determinaţi numerele x, y, z din (1, ∞) pentru care xlogy z = x.
Călin Burduşel, Târgovişte

13. Demonstraţi că pentru orice z1 , z2 , z3 ∈ C avem


|z1 |2 + |z2 |2 + |z3 |2 ≥ Re(z1 z2 + z2 z3 + z3 z1 ).

Traian Tămâian, Carei

14. Rezolvaţi ecuaţia


2 2
5x + 5x = 4x + 6x .

Nicolae Dragomir

15. a) Se dă şirul cu termenul general an , n ≥ 3, dat de


an + an−1 + 2an−2 = 0, a1 = 1, a2 = −1.
Determinaţi formula termenului general.
b) Arătaţi că pentru orice n ∈ N, ecuaţia x2 + 7y 2 = 2n are soluţii ı̂n mulţimea
numerelor ı̂ntregi.
Nicolae Dragomir
27

16. Pentru n număr natural nenul, calculaţi

[ n2 ]
X 2
Ckn − Ck−1
n .
k=0

AMM

17. Fie n, k ∈ N∗ , k ≤ n şi M = {1, 2, . . . , n}. Pentru o submulţime cu k elemente a


lui M , ordonată a1 < a2 < . . . < an , fie λA = max(ai+1 − ai ) şi lA = min(ai+1 − ai ).
Calculaţi X
(λA + lA ).
A⊆M, |A|=k

V. Zidaru

18. Pentru n natural considerăm suma S a numerelor de 10n+1 cifre ce ı̂n reprezentarea
lor ı̂n baza 10 nu conţin cifra 0. Calculaţi restul ı̂mpărţirii numărului S la ı̂mpărţirea
prin 11.
V. Zidaru

19. Fie z1 , z2 , z3 numere complexe


 de acelaşi
 modul
 nenul
 cu z1 + z2 + z3 6= 0. Se
z2 + z3 z3 + z1 z1 + z2
consideră punctele A , B , C .
z1 z2 z3
a) Determinaţi centrul cercului circumscris triunghiului ABC;
b) În ce condiţii este triunghiul echilateral?
Florin Cârjan, Braşov

20. În triunghiul ABC fie M un punct interior şi fie da , db , dc şi respectiv ta , tb , tc
distanţele de la M la laturile respectiv vârfurile triunghiului. Arătaţi că
X X A
da ≤ ta sin .
2

Nicolae Stăniloiu

21. Fie z1 , z2 , . . . , zn , n ≥ 4, numere complexe (nu neapărat distincte), pentru care

|z1 | + |z2 | + . . . + |zn | = 1.

Demonstraţi că se pot alege unele dintre acestea pentru care modulul sumei este
strict mai mare decât 1/4.
Valentin Vornicu
28

22. Arătaţi că patru numere pozitive a, b, c, d sunt ariile feţelor unui tetraedru dacă şi
numai dacă au loc toate inegalităţile de tipul a < b + c + d.
Dumitru Mihalache şi Marian Tetiva

23. Demonstraţi că pentru orice numere reale x, y, avem

| cos x| + | cos y| + | cos(x + y)| ≥ 1.

Stabiliţi condiţiile ı̂n care are loc egalitatea.


Dorin Arventiev, Constantin Caragea
Capitolul 3

Probleme avute ı̂n atenţia comisiei, 2004

3.1 Clasa a VII-a


1. Se colorează cu roşu n dintre vârfurile unui octogon regular, n ≤ 8. Care este cea
mai mică valoare a lui n astfel ı̂ncât oricum s-ar face această colorare, să existe un
triunghi isoscel cu toate vârfurile roşii? Justificaţi.
Radu Gologan

2. Dată fiind o mulţime X cu 10 elemente, câte perechi (A, B) cu A ⊆ B ⊆ X se pot


forma?
Valentin Vornicu

3. Considerăm numărul a = 999 . . . 9 cu 2003 cifre şi numărul b natural cu b ≤ a.


Arătaţi că suma cifrelor numărului ab nu poate fi pătrat perfect.
Florin Nicoară

4. Fie a un număr natural. Arătaţi că dacă există b şi c numere ı̂ntregi nenule astfel
ı̂ncât bc > a şi ac + b > 3a, atunci ab + c > 2a.
Lucian Dragomir

5. Fie a un număr natural cu 2005 cifre pentru care 2003 cifre aparţin mulţimii

{0, 3, 6, 9} iar o cifră aparţine mlţimii {2, 5, 8}. Arătaţi că a este iraţional.
Romeo Zamfir

6. Se se determine cifrele a şi b, unde a ∈


/ {0, 9}, pentru care numărul raţional
a, (ba) + b, (ab) + a, b(a)
se pot scrie ca fracţie zecimală finită.
Romeo Zamfir

29
30

7. Pentru un număr natural ce nu se termină cu cifra 0 definim răsturnatul său ca


fiind numărul format scriind ı̂ordine inversă cifrele numărului iniţial (răsturnatul
lui 1234 este 4321). Să se determine toate numerele naturale nenule n pentru care
a) n2 se poate scrie ca diferenţa dinte un număr de trei cifre şi răsturnatul său;
b) n3 se poate scrie ca diferenţa dintre un număr de patru cifre şi răstrunatul său.
Valentin Vornicu
√ 1
8. Fie a, b numere naturale astfel ı̂ncât a + √ să fie număr natural. Să se arate că
b
a şi b sunt pătrate perfecte.
Mircea Becheanu

9. Numărul N = 124578 . . . 20022003 este format din toate numerele nedivizibile cu


3, cel
√ mult egale cu 2003, scrise unul după altul ı̂ordine crescătoare. Demonstraţi
că N ∈ R \ Q.
Petre Simion

10. Se consideră triunghiul ABC cu m(∠B) > 90◦ , D ∈ [CB, astfel ı̂ncât m(∠ADC) =
m(∠B) − m(∠A), [DP bisectoarea unghiului ∠ADB, P ∈ AB, iar {Q} = CP ∩
AD, {T } = BQ ∩ AC. Să se arate că T D ⊥ DP .
Gabriel Popa, Ghorghe Iurea

11. Se consideră triunghiul ascuţitunghic ABC având ortocentrul H şi ı̂nălţimile


AM, BN, CP . Fie Q şi R mijloacele segmentelor BH şi CH. Notăm {U } =
M Q ∩ AB, {V } = M R ∩ AC, {T } = AH ∩ P N . Să se arate că
MH TH
a) = ;
MA TA
b) T este ortocentrul triunghiului U AV .
Manuela Prajea

12. Se consideră un punct P variabil pe conturul unui dreptunghi ABCD ı̂care AB =


CD = a, BC = AD = b şi a > b. Să se determine poziţia lui P pentru care suma
P A + P B + P C + P D este minimă.
Mircea Becheanu

13. Fie ABCD un paralelogram ı̂care m(∠BCD) ≥ 90◦ şi BC ≥ CD. Să se arate
că dacă pentru orice puncte M ∈ (DA) şi N ∈ (CB) avem CN ⊥ DN implică
CM ≡ DN , atunci patrulaterul ABCD este pătrat.
Alexandru Blaga
31

14. În triunghiul oarecare ABC avem AD ⊥ BC, D ∈ (BC), BE ⊥ AC, E ∈


(AC), CF ⊥ AB, F ∈ (AB). Ducem AM ⊥ EF, M ∈ (EF ), BN ⊥ DF, N ∈
(DF ), CP ⊥ DE, P ∈ (DE).
a) Arătaţi că [EP ] ≡ [F N ].
b) Arătaţi că DM, EN, F P sunt concurente.
Petre Bătrâneţu

15. În paralelogramul BCDE, avem CE = BE = CD = b şi BC = DE = c. Con-


struim triunghiul DEA astfel ca ∠ADE = ∠DCE şi E ∈ (AB). Arătaţi că:
a) ABCD este trapez isoscel.
b) dacă AB = a, rezultă ab = b2 + c2 .
Ion Morteanu

16. Se consideră triunghiul ABC şi cevienele (AA0 ), (BB 0 ), (CC 0 ), concurente ı̂punctul
M.
a) Să se arate că dacă BA · BC 0 = BC · BA0 şi CA · CB 0 = CB · CA0 atunci AB 0 M C 0
este patrulaterul inscriptibil.
b) Dacă, ı̂plus, AB · AC 0 = AC · AB 0 , atunci AA0 este perpendiculară pe BC.
Constantin Apostol

17. În patrulaterul convex ABCD diagonalele se intersectează ı̂punctul O. Se ştie că

BC = CD, AO = OC = 2 2, BO = 3, OD = 4.
a) Să se demonstreze că (AC este bisectoarea unghiului ∠BAD.
b) Să se calculeze lungimile laturilor patruletrului.
Constantin Apostol

18. Fie BCD un dreptunghi şi fie M un punct pe diagonale BD astfel ı̂ncât ∠AM C =
∠CM D. Fie N intersecţia dintre AM şi paralela la CM prin B. Să se arate că
triunghiul BM N este echilateral dacă şi numai dacă ABCD este pătrat.
Valentin Vornicu

3.2 Clasa a VIII-a


1. Dată fiind o mulţime X cu n elemente (n ≥ 2), câte perechi (A, B) cu A ⊂ B ⊂ X
se pot forma?
Valentin Vornicu
32

2. Vârfurile unui cub se colorează ı̂roşu, galben sau albastru. Putem proceda ı̂aşa fel
ı̂ncât fiecare mulţime formată din patru vârfuri colplanar e să conţină toate cele
trei culori?
Gabriel Popa

3. Un cub de latură 3 se ı̂mparte ı̂27 de cubuleţe congruente eliminându-se astfel de


cubuleţ care nu este colţ al cubului mare.
Se poate umple exact corpul obţinut astfel cu paralelipipedul dreptunghic 2 × 1 ×
1?
 2
n
4. Se consideră n, p ∈ N∗ şi f (n, p) = .
p
a) Să se arate că f (n, 3) + f (n + 1, 3) + f (n + 2, 3) este pătrat perfect pentru oricare
n numnăr natural nenul.
b) Să se arate că pentru oricare n număr natural nenul numărul f (n) = f (n, 2) +
f (n + 1, 2) nu este pătrat perfect.
c) Să se arate că dacă f (n) = f (n, p) + f (n + 1, p) + f (n + 2, p) este pătrat perfect
pentru oricare n număr natural nenul, atunci p = 3.
Marius Burtea

5. Se ştie că numerele reale pozitive a, b, c au proprietatea că a + b + c = abc + 2.


Demonstraţi că max{a, b, c} ≥ 1.
Valentin Vornicu

6. Determinaţi funcţiile f : R → R, pentru care

f (2x − 5) ≤ 2x − 3 ≤ f (2x) − 5,

pentru orice x real.


Liliana Antonescu

7. Să se determine numerele naturale n care au ı̂Z cel puţin n divizori distincţi.
Romanţa şi Ioan Ghiţă

8. În tetraedrul ABCD punctele E, M, N sunt mijloacele segmentelor [CD], [AE]


respectiv [BM ].
Dacă AC ∩ DM = {F }, EN ∩ AB = {P }, AN ∩ BE = {Q} şi CQ ∩ BD = {L},
demonstraţi că P, L, E, F sunt coplanare.
Virginia şi Vasile Tică
33

9. Se consideră prisma patrulateră regulată ABCDA0 B 0 C 0 D0 ı̂care AC ∩ BD = {O1 },


A0 C ∩ C 0 O1 = {O2 } şi O3 este centrul cercului ı̂nscris ı̂triunghiul ACC 0 Ştiind că

AB = 4 cm şi AA0 = 3 2 cm, aflaţi măsura unghiului format de planele (O1 B 0 B)
şi (O2 O3 B 0 ).
Marius Nicoară

10. Fie V ABCD o piramidă patrulateră regulată şi fie O centrul bazei. Considerăm
punctele P ∈ (V C), M ∈ (AB) şi N ∈ (V M ) astfel ı̂ncât OP ⊥ V C şi M respectiv
N sunt mijloacele segmentelor (AB) repsectiv (V M ) iar R ∈ N O esye astfel ı̂ncât
AB
M R ⊥ N O. Pe paralela prin V la AB se ia un punct U astfel ı̂ncât U V = şi U
2
se află ı̂acelaşi semispaţiu delimitat de planul (V M O) ca şi B.
Demonstraţi că dreptele DP, AR, U C sunt concurente pe sfera de centru O şi
razş AO.
Claudiu-Ştefan Popa

11. Fie ABCD un tetraedru, şi fie Ga , Gb , Gc Gd centrele de greutate ale feţelor
BCD, CDA, DAB şi respectiv ABC.
a) Să se arate că dreptele AGa , BGb , CGc , DGd sunt concurente ı̂ntr-un punct G.
b) Dacă M este un punct interior tetraedrului ABCD, şi Ma , Mb , Mc , Md sunt
intersecţiile dreptelor AM, BM, CM, DM cu feţele opuse vârfurilor A, B, C, D
respectiv atunci
AM BM CM DM
= = =
M Ma M Mb M Mc Md
dacă şi numai dacă G ≡ M .
Valentin Vornicu

12. Se consideră patrulaterul inscriptibil ABCD. În punctele M, N, P, Q 0 mi-


jloacele laturilor AB, BC, CD şi, respectiv DA - se ridică perpendiculare pe
planul patrulaterului. Pe aceste perpendiculare, de aceeaşi parte a planului, se
1 1 1
iau punctele M1 , N1 , P1 , Q1 , astfel ca M M1 = AB, N N1 = BC, P P1 = CD
2 2 2
1
şi QQ1 = DA. Să se arate că dacă punctele M1 , N1 , P1 , Q1 sunt coplanare,
2
atunci patrulaterul M1 N1 P1 Q1 este inscriptibil.
Constantin Apostol

13. Să se arate că ı̂ntr-un tetraedru echifacial, centrul sferei ı̂nscrise ı̂tetraedru coin-
cide cu centrul de greutate al tetraedrului.
Valentin Vornicu
34

3.3 Clasa a IX-a


1. Fie k, n1 , n2 , . . . , nk numere naturale impare, fie A = {(ni , nj ) | 1 ≤ i < j ≤ k} şi
f : A → N definită prin
 n +n
1, dacă i 2 j este impar,
f (ni , nj ) =
0, altfel.
P
Să se arate că dacă S = f (x) este număr par, atunci 4 divide k − 1.
x∈A

Romeo Zamfir

2. Fie numerele strict pozitive a, b astfel ca a + b = 1. Aflaţi minimul expresiei


1 1
√ + √ .
1− a 1− b

Ioan V. Maftei
3. [a)]
Determinaţi numerele naturale p pentru care
     
n n+1 n+2
+ + =n
p p p
oricare ar fi numărul natural n.
(b)
(a) Determinaţi numerele naturale nenule p, q, r pentru care
     
n n+1 n+2
+ + =n
p q r
pentru orice n număr natural.
Marius Burtea

4. Numerele pozitive a, b, c verifică abc = 1. Demonstraţi inegalitatea


Xa+b X X1 
≥2 a+ −3 .
c a
(a se vedea problema asemănătoare de la baraj 2003-juniori)
Gabriel Dospinescu

5. Fie a, b ∈ R şi f R → R strict crescătoare. Determinaţi funcţiile g : [a, b] →


[f (a), f (b)] ştiind că g(x) − g(y) ≥ |f (x) − f (y)|, pentru orice x, y ∈ [a, b].
Călin Burduşel
35

6. Determinaţi funcţiile de gradul al doilea f (x) = ax2 + bx + c, cu f (0) ∈ Z şi care


au proprietatea că
   
1 2 n−1
f n+ >n −n+1 şi f n + < n2 + n − 1,
n n

pentru o infinitate de numere naturale n.


Cristinel Mortici

7. Pentru orice număr natural n ≥ 2 notăm an = 2 + √1 + √1 + ··· + √1 .


2 3 n
a2 a3 an
Demonstraţi că 1 + + + ··· + < an−1 .
2 3 n
Cristinel Mortici

8. [a)] Fie f : N∗ → N∗ o funcţie monotonă astfel ı̂ncât pentru orice n ∈ N∗


avem f (f (n)) = n + f (2).
Arătaţi că f este crescătoare.
(b)
(a) Determinaţi funcţia f .
Nicolae Papacu

9. Determinaţi x, y ∈ N pentru care


hxi 
  
2x 3x h xy i
+ + + ··· + = 4.
π π π π

Marian Teler

10. [a)]

Determinaţi toate funcţiile injective f : N∗ → N∗ care ı̂ndeplinesc condiţia:


pentru orice n ∈ N∗ există i1 , i2 , . . . , i3n ∈ N∗ distincte astfel ı̂ncât

[f (i1 ) + f ◦ f (i1 )] + · · · + [f (i3n + f ◦ f (i3n )] = 3n(3n + 1).

(a) Fie g : N∗ → N∗ . Arătaţi că există o funcţie F : N∗ → N∗ ce satisface condiţia:


(b)
pentru orice n ∈ N∗ , există i1 , i2 , . . . , ig(n) ∈ N∗ distincte astfel ı̂ncât

[f (i1 ) + f ◦ f (i1 )] + · · · + [f (ig(n) + f ◦ f (ig(n) )] = g(n)(g(n) + 1)

dacă şi numai dacă g este surjectivă.


Alexandru Take
36

11. Dacă a, b, c > 0 demonstraţi că


X a 27
≥ .
bc(c + a) 2(a + b + c)2

Petre Bătrâneţu

12. Determinaţi toate funcţiile de gradul al doilea f (x) = ax2 +bx+c, cu a, b, c numere
reale, a 6= 0, care transformă intervalele [0, 1] şi [4, 5] ı̂n două intervale care au un
singur punct comun şi a căror reuniune este intervalul [1, 9].
Cristinel Mortici

13. Dacă x, y, z, a, b, c, d sunt numere reale pozitive nenule, arătaţi că


xyz 1
≤ √
4
,
(1 + ax)(x + by)(y + cz)(z + d) (1 + abcd)4

unde se consideră că expresia are sens.


Marin Chirciu

14. Dacă a, b, c sunt numere pozitive, arătaţi că


X√ √
a(b3 + c3 ) ≥ 2 abc(a2 + b2 + c2 ).

Marin Chirciu

15. Găsiţi toate perechile (x, y) de numere reale, soluţii ale sistemului
√ 1 √ 1
x4 + 2x3 − y = 3− , y 4 + 2y 3 − x = − 3 − .
4 4

Titu Andreescu

16. Fie a, b, c trei numere reale pozitive. Să se arate că


√ √ √ √ p
abc( a + b + c) + (a + b + c)2 ≥ 4 3abc(a + b + c).

Valentin Vornicu

17. Să se demonstreze că dacă a, b, c ∈ R, a2 + b2 + c2 = 9 atunci

3 min{a, b, c} ≤ 1 + abc.

Virgil Nicula
37

18. Fie a, b, c numere reale strict pozitive pentru care

1 1 1 a b c
+ + ≥ + + .
a b c b c a

Să se demonstreze inegalitatea

a b c 3
+ + ≥ .
bc(b + c) ca(c + a) ab(a + b) 2

Tudorel şi Cezar Lupu



19. Fie p ∈ N∗ , q ∈ {p2 + 1, p2 + 2, . . . , p2 + 2p} şi xn = [(p + q)n ], pentru orice n ∈ N.
Să se arate că există o bijecţie ı̂ntre

{n ∈ N | xn este par} şi {n ∈ N | xn este impar}.

Radu Miculescu

20. Determinaţi mulţimea S a numerelor naturale n nenule care ı̂ndeplinesc condiţia:


pentru orice a, b ∈ R \ {0} există numerele reale x1 , x2 , . . . , xn astfel ı̂ncât
(
x1 + x2 + · · · + xn = a
1
x1
+ x12 + x1n = b.

Mihai Bălună

21. Determinaţi coordonatele vârfurilor triunghiului ABC ı̂nscris ı̂ntr-un cerc de cen-
tru O(1/2, −1/2) ştiind că piciorul ı̂nălţimii din A este H(1, 4), iar piciorul bisec-
toarei din A este D(−1, 2).
Gabriel Popa

22. Fie A0 , B 0 , C 0 respectiv, puncte pe laturile (BC), (AC), (AB) ale unui triunghi ABC
şi numărul k, k ≥ 1 astfel ca

1 AC 0 1 BA0 1 CB 0
≤ ≤ k, ≤ ≤ k, ≤ ≤ k.
k BC 0 k CA0 k AB 0
Demonstraţi că
max(A0 B 0 , A0 C 0 , B 0 C 0 ) k
≤ .
max(AB, AC, BC) k+1

Dan Ismailescu
38

23. Fie O centrul cercului circumscris triunghiului ABC pentru care există pe laturile
−−→ −−→ −−→ −−→ −→ −→
sale punctele M, N, P astfel ı̂ncât 3AM = 2M B, 3BN = 2N C, 3CP = 2P A iar
OM ≡ ON ≡ OP .
Arătaţi că triunghiul ABC este echilateral.
Marian Teler

24. Fie P un poligon convex. Pentru un punct M din interiorul poligonului, notăm
cu s(M ) suma distanţelor de la M la laturile poligonului. Se ştie că există trei
puncte necoliniare A, B, C astfel ca

s(A) = s(B) = s(C) = k.

Să se arate că s(M ) = k, pentru orice punct.


Bogdan Enescu

25. Care sunt patrulaterele convexe pentru care există un punct ı̂n planul lor cu pro-
prietatea că orice dreapta ce trece prin punctul respectiv ı̂mparte patrulaterul ı̂n
două poligoane de arii egale?
Gheorghe Iurea

26. Fie n un număr natural şi X1 , X2 , . . . , Xn mulţimi disjuncte ı̂n plan de cardinal
n
S
cel puţin n. Pentru X = Xi notăm cu G centru de greutate al punctelor din
i=1
X şi pentru fiecare i = 1, 2, . . . , n cu Gi centrul de greutate al punctelor din Xi .
Fie S suma pătratelor distanţelor de la G la punctele din X şi si suma pătratelor
Pn n
P
distanţelor de la Gi la punctele din Xi . Dacă s = si şi d = GGi să se arate că
i=1 i=1
S ≥ s + d2 .
Claudiu Raicu

27. Un triunghi ABC este echilateral dacă şi numai dacă există puncte M, N, P pe
laturile (AB), (BC), (CA) respectiv, astfel ı̂ncât triunghiurile ABC şi M N P au
acelaşi centru de greutate şi acelaşi ortocentru.
Marian Ionescu

28. Fie T = {∆ABC | 2ha = a}. Arătaţi că

a) T 6= ∅.
b) T conţine triunghiuri neisoscele.
39

c) Dacă ı̂ntr-un triunghi ABC avem 2a2 = b2 + c2 , b2 − c2 = 2bc, atunci ∆ABC ∈


T.
Alexandru Blaga

29. Demonstraţi că

1 3 9
+ + +
cot 9◦ − 3 tan 9◦ cot 27◦ − 3 tan 27◦ cot 81◦ − 3 tan 81◦

27
+ = 10 tan 9◦ .
cot 243◦ − 3 tan 243◦

Titu Andreescu

30. Fie patrulaterul convex ABCD ı̂n care AD = DB, AC = CD,


m(∠BAC) = 2m(∠BDC).
Să se arate că m(∠CBD) = 30◦ şi că există x ∈ (0, 30◦ ) astfel ı̂ncât m(∠A) = 60+x,
m(∠B) = 90 + x, m(∠C) = 150 − x.
Virgil Nicula

31. Fie ABCD un patrulater ı̂nscris ı̂ntr-un cerc de centru O şi X un punct ı̂n planul
său. Se notează cu AH , BH , CH , DH , respectiv AG , BG , CG , DG simetricele punctu-
lui X faţă de ortocentrele respectiv centrele de greutate ale triunghiurilor BCD,
CDA, DAB, ABC.
Să se demonstreze că:

1) Dreptele AHA , BHB , CHC , DHD sunt concurente ı̂ntr-un punct notat cu H.
2) Dreptele AAG , BBG , CCG , DDG sunt concurente ı̂ntr-un punct notat cu G.
3) Punctele Y , H, G sunt colineare, unde Y este simetricul lui X faţă de O.
Daniel Văcăreţu

32. Demonstraţi că un patrulater ABCD este romb dacă şi numai dacă pentru orice
punct M din plan avem

4(M A · M C + M B · M D) ≥ AC 2 + BD2 .

Laurenţiu Panaitopol
40

3.4 Clasa a X-a


1. Fie A = {1, 2, . . . , n}. Câte soluţii (X, Y, Z) cu |Z| > |Y | > |X| ≥ 2 are ecuaţia
X ∪ Y ∪ Z = A.
***

2. Fie A, B, C mulţimi de cardinal m, n, p respectiv astfel ı̂ncât p ≥ m. Fie f : A →


C o funcţie injectivă. În câte moduri poate fi scrisă funcţia f ca f = g ◦ h unde
h : A → B şi g : B → C?
Cornel Berceanu

3. Determinaţi progresiile geometrice de numere naturale (an )n≥0 pentru care suma
n  
X n
ak
k=0
k

este pătrat perfect pentru orice n natural.


Lucian Dragomir
1 1
4. Să se arate că ecuaţia 5x + 6 x + 7x+ x = 58 nu admite soluţii ı̂mulţimea numerelor
reale.
Mariana Coadă

5. Fie k ≥ 1 natuarl şi a1 , a2 , . . . , ak , b1 , b2 , . . . , bk numere raţionale având următoarea


proprietate pentru orice numere iraţionale supraunitare x1 , x2 , . . . , xk există nu-
mere naturale nenule n1 , . . . , nk , m1 , . . . , mk astfel ı̂ncât
a1 [xn1 1 ] + . . . + ak [xnnk ] = b1 [xm mk
1 ] + . . . + bk [xk ].
k

Să se demonstreze că ai = bi pentru orice i = 1, 2, . . . , k.


Gabriel Dospinescu
p−1
n   [X2 ]
X p+k
6. Dacă p este un număr prim p ≥ 5 şi an = , atunci numărul a2k se
k=1
k k=1
divide prin p.
Nicolae Papacu

7. Fie a, b, c ∈ (0, 1). Să se determine numerele reale x, y, z pentru care


1 − ax 1 − by 1 − cy x+y+z
= = = .
1−a 1−b 1−c 3
Manuela Prajea
41

8. Fie a ∈ (0, ∞) \ {1}. Să se determine soluţiile reale ale sistemului

ax + loga y = aay = loga z = aaz + loga x = a.

***

9. Se consideră numerele complexe z0 , z1 , . . . , zn−1 cu proprietatea că pentru orice


j, k ∈ {0, 1, . . . , n − 1} avem Im((zj − zk )(zk+1 − zk )) = 0. Să se arate că
n−1
X n−1
X
|zk+1 − zk−1 | ≥ |zk − zk−1 |,
k=0 k=0

unde se consideră z−1 = zn−1 .


Călin Popescu

10. Găsiţi toate perechile de numere complexe (z1 , z2 ) care satisfac simultan condiţiile:
a) |1 + z1 + z2 | = |1 + z1 | = 1;
b) |z1 z2 (z1 + z2 )| = 2(|z1 | + |z2 |).
Valentin Vornicu

11. Să se rezolve ecuaţia

2x (ax + bx ) + x(ax + bx ) = (a + b)x + x(a + b)

unde a, b ∈ (1, ∞), a 6= b.


Dan Popescu

12. Să se determine z ∈ C ştiind că |1 + z 2n+1 | ≤ 1, pentru orice n ∈ N.


Costel Chiteş
π
13. Fie x, y, z numere strict astfel ca x + y + z = . Arătaţi că
2
(1 − sin x)(1 − cos y)(! − cos z) ≥ sin x sin y sin z.

În ce caz avem egalitate?


Ghorghe Szölösy

14. Să se arate că ı̂ntr-un tetraedru echifacial, centrul sferei ı̂nscrise ı̂tetraedru coin-
cide cu centrul de greutate al tetraedrului.
Valentin Vornicu
42

15. Să se arate că un tetraedru ce are muchiile ı̂progresie aritmetică şi perechile de
muchii opuse sunt perpendiculare, este regulat.
Mircea Becheanu

16. Spunem că un tetraedru este median dacă are proprietatea următoare:
(P) Pentru orice vârf al său planul ce trece prin mijloacele muchiilor ce pornesc
din acel vârf este tangent la sfera ı̂nscrisă ı̂tetraedru.
Să se aret că un tetraedru este median dacă şi numai dacă este echifacial.
Valentin Vornicu

17. Fie M un punct ı̂interiorul tetraedrului ABCD şi fie A0 , B 0 , C 0 , D0 intersecţiile


dreptelor AM, BM, CM, DM cu planele opuse. Să se arate că
X X 1
AM 2 ≥ 9.
AA02

Andrei Chiteş

18. Demonstraţi că ı̂orice triunghi are loc inegalitatea


X√
r
X A B C
sin A ≥ 2 cos sin sin .
2 2 2

Traian Tămâian

19. Fie tetraedrul V ABC ı̂care muchiile laterale V A, V B, V C sunt perpendiculare


AM 3 AN 1
două câte două. Fie M ∈ (AB), N ∈ (BC) astfel ı̂ncât = şi = . Dacă
√ AB 4 AC√ 5
π
V A = 5 arătaţi că m(V AB) = dacă şi numai dacă V M · V N = 2.
4
Călin Burduşel

20. Fie ABCD un patrulater ı̂nscris ı̂ntr-un cerc de centru O şi fie P un punct BCD,
CDA, DAB, ABC respectiv.
a) Să se demonstreze că dreptele AHA , BHB , CHC , DHD sunt concurente ı̂ntr-un
punct notat cu H şi
1
V OH = (V OA + V OB + V OC + V OD).
2

b) Să se demonstreze că expresia P A2 + P B 2 + P C 2 + P D2 − 2P H2 este constantă.


Daniel Văcăreţu
Capitolul 4

Probleme avute ı̂n atenţia comisiei, 2005

4.1 Clasa a VII-a


1. Să se afle suma tuturor numerelor de trei cifre scrise ı̂n baza zece ce au atât cifre
pare cât şi cifre impare.
Petre Bătrăneţu, Galaţi

2. Fie date ı̂n plan 2005 puncte astfel ı̂ncât oricare trei sunt necoliniare. Arătaţi că
există un cerc ce conţine ı̂n interiorul lui 1002 puncte şi ı̂n exterior tot 1002 puncte.
Petre Bătrăneţu, Galaţi

3. Determinaţi numerele naturale cu scrierea ı̂n baza 10 de forma ab şi numerele


naturale n, care satisfac egalitatea:

(n − 1)ab2 + n = 2006

Ion Safta, Piteşti

4. Câte numere de patru cifre abcd, a 6= 5, au proprietatea că numărul N = a + 4b +


3c − d se divide cu 13 ?
Nicolae Stăniloiu, Bocşa

5. Fie ABCD un paralelogram. Bisectoarea unghiului B intersectează diagonala


0 0
AC ı̂n B , iar paralela dusă prin B la AB intersectează dreptele AD şi BD ı̂n P
respectiv Q. Demonstraţi că triunghiul P AQ este isoscel.
Ion Safta, Piteşti

6. În pătratul ABCD, M şi N sunt mijloacele laturilor AB şi respectiv AD, {Q} =
CM ∩ BN, {O} = AC ∩ BD, {P } = DE ∩ BC, E ∈ QC. Arătaţi că:

43
44

a) CM ⊥ BN
b) DQ = AB
c) P este mijlocul lui BC dacă şi numai dacă E este mijlocul lui QC.
d) punctele N, O, P sunt coliniare dacă şi numai dacă E este mijlocul segmentului
QC.
Ion Trandafir, Hunedoara

7. O dreaptă ce trece prin punctul O de intersecţie a diagonalelor patrulaterului


AE CF
convex ABCD intersectează [AB] şi [CD] ı̂n E, respectiv F . Ştiind că AB = CD =
1
r ∈ (0, 1), demonstraţi că AB şi CD sunt paralele. Dacă ı̂n plus r = 2 şi EF =
AD+BC
2
, demonstraţi că ABCD este paralelogram.
Mihai Boloş, Baia Mare

8. Fiind date numerele a = 1 + 3 + 32 + 33 + . . . + 32004 şi b = 2501 + 2502 + . . . + 22004 ,


aflaţi restul ı̂mpărţirii numărului 2a − b + 22004 · 32004 la 211.
Dorina Zaharia, Braşov

9. Pentru fiecare număr natural nenul n şi x număr ı̂ntreg definim expresia
√ √ √
En (x) = |x − 1| + |x − 2| + . . . + |x − n|

a) Determinaţi x şi n astfel ı̂ncâtEn (x) < n3 .


b) Determinaţi x şi n astfel ı̂ncâtEn (x) < n4 .
Marius Ghergu, Slatina

10. Să se determine cea mai mare şi cea mai mică valoare a expresiei

abc
E= ,
ab + bc + ca
unde a, b, c sunt cifre ı̂n sistemul zecimal.
Tuzson Zoltan, Odorheiu Secuiesc

11. Într-o urnă se află 50 de bile numerotate de la 1 la 50. Dacă se extrag două bile,
care este probabilitatea ca produsul numerelor bilelor să fie un număr divizibil
cu 12.
Stelian Ionescu, Piteşti

12. Pe baza AB a trapezului ABCD (de baze AB şi CD), AB > CD se consideră
punctele M şi N . Fie {P } = AC ∩ DM, {Q} = BD ∩ CN, {S} = AC ∩ DN, {R} =
BD ∩ CM .
45

a) Arătaţi că patrulaterul cu vârfurile P, Q, R, S este trapez dacă şi numai dacă
AM = BN .
b) În ce condiţii P QRS este trapez ı̂n care linia mijlocie este conţinută ı̂n linia
mijlocie a trapezului ABCD?
Petru Braica şi Constantin Voicu, Satu Mare

13. În trapezul ABCD cu AB k CD, AB < CD unghiul din C este drept, unghiul
din D are măsura de 45◦ , iar 2DC = 5BC. Dacă N aparţine laturii BC astfel
ı̂ncât BN = N C şi F este intersecţia dreptelor DN şi AC, să se calculeze masura
unghiului AF N .
Gheorghe Molea, Curtea de Argeş

14. Fie M un punct interior triunghiului echilateral ABC. Demonstraţi că următoarele
două afirmaţii sunt echivalente:
a) M A2 + M B 2 = M C 2
b) măsura unghiului ^AM B este 150◦ .
Gheorghe Niţoiu, Râmnicu Vâlcea

15. Să se arate că numărul


r
44 . . . 4} − 88
| {z . . . 8}
| {z
2k cifre k cifre

este natural şi divizibil cu 3 pentru orice număr natural k > 1.


Emilia Velcea, Lupeni, Hunedoara

16. Fie T un punct exterior rombului M N P Q. Dacă T N ∩ M Q = {A}, T N ∩ P Q =


{B}, T Q ∩ M N = {C}, T Q ∩ P N = {E}, arătaţi că ABEC este trapez.
Emilia Velcea, Lupeni, Hunedoara

17. Fie M N P Q un paralelogram şi T un punct oarecare pe latura N P . Dreptele M T şi


QT intersectează dreptele QP şi M N ı̂n R respectiv S. Demonstraţi că lungimea
segmentului P Q este medie proporţională ı̂ntre lungimile segmentelor P R şi N S.
Emilia Velcea, Lupeni, Hunedoara

18. Fie M respectiv N mijloacele laturilor AB şi CD ale patrulaterului convex ABCD.
Dacă bisectoarele interioare ale unghiurilor din A şi B se taie ı̂n E ∈ M N iar
EM = AB 2
, să se arate că
a) patrulaterul ABCD este trapez.
b) dacă bisectoarele interioare ale unghiurilor C şi D se taie ı̂n F , calculaţi lungimea
segmentului EF ı̂n funcţie de lungimile laturilor trapezului ABCD.
46

Szöcs Péter, Miercurea Ciuc

19. Fie ABC un triunghi oarecare şi D un punct pe latura BC. Notăm cu M şi N
picioarele bisectoarelor duse din A ı̂n triunghiurile ABD şi respectiv ADC. Dacă
există P un punct situat pe AD astfel ca M P k AB şi N P k AC, arătaţi că tri-
unghiul ABC este isoscel.
Cezar Lupu, student, Bucureşti

20. Fie ABCD un trapez isoscel ı̂nscris ı̂ntr-un cerc. Notăm cu {E} = AC ∩ BD,
{F } = AD ∩ BC, şi fie T1 şi T2 intersecţiile cu cercul ale paralelei prin E la AB.
Să se arate că F T1 şi F T2 sunt tangente la cerc.

21. Pe bisectoarea unghiului ascuţit aOb se consideră ı̂n această ordine punctele O1 , O2 ,
. . . , On astfel ca cercurile C1 , C2 , . . . , Cn cu centrele ı̂n aceste puncte, tangente la-
turilor unghiului, să fie distincte. Notăm cu T1 şi U1 punctele de tangenţă ale
cercului C1 cu laturile Oa şi Ob şi cu Tn şi Un punctele de tangenţă cu Cn . Tan-
gentele comune interioare cercurilor Ck şi Ck+1 intersectează laturile unghiului ı̂n
Bk ∈ Oa şi Ck ∈ Ob, k = 1, 2, . . . , n − 1. Să se arate că
T1 Tn + U1 Un = 2 · (B1 C1 + B2 C2 + . . . + Bn−1 Cn−1 ).
Vasile Pop, Cluj

22. Fiind date numerele naturale a1 , a2 , . . . , a100 , astfel că ak ≤ k pentru k ≤ 100, să se
demonstreze că:

a1 a2 . . . a100 + (1 − a1 ) · (2 − a2 ) · . . . · (100 − a100 ) < 1 · 2 · 3 · . . . · 99 · 100.

Dorin Mărghidanu, Corabia

23. Fie M mijlocul laturii (AB) a triunghiului ABC şi N punctul din plan pentru care
N M ⊥ AC şi N B ⊥ BC. Fie P punctul de pe latura (BC) astfel ı̂ncât lungimea
segmentului N P este egală cu raza cercului circumscris triunghiului ABC. Să se
arate că P este mijlocul laturii (BC).
Constantin Apostol, Rm. Sărat

24. În triunghiul ABC se duce bisectoarea (AD), D ∈ BC şi fie DE k AC, E ∈ AB
şiDF k AB, F ∈ AC.
Să se arate că
1
SAEDF ≤ AB · AC.
4
Constantin Apostol, Rm. Sărat
47

25. Spunem că o pereche de numere naturale (a, b) este magică dacă unul dintre cele
două numere se scrie ca produsul cifrelor celui de-al doilea.
Spunem că un număr este prietenos daca el face parte din cel puţin o pereche
magică. Demonstraţi că:
a) numărul 2005 nu este prietenos.
b) numărul 200 . . . 0}5 nu este prietenos pentru orice n ≥ 3.
| {z
de n ori
Valentin Vornicu, student, Bucureşti

26. Fie triunghiul ABC cu AB < AC.


a) Să se arate că pentru orice punct M ∈ (AB) există un singur punct N ∈ (AC),
astfel ca perimetrul triunghiului AM N să fie AB.
b) Să se arate că există cel puţin un punct N ∈ (AC) astfel ca, pentru orice punct
M ∈ (AB), perimetrul triunghiului AM N depăşeşte 2AB.
Dan Popescu, Suceava

4.2 Clasa a VIII-a


1. Fie triunghiul ABC echilateral. Pe perpendicularele ı̂n A şi C de aceeaşi parte
a planului ABC luăm respectiv punctele M şi N astfel ı̂ncât AM = AB = a şi
M N = BN . Să se determine:
(a) distanţa de la punctul A la planul M N B;
(b) sinusul unghiului dintre M N şi BC.
Gianina Busuioc, Niculai Solomon

2. Fie ABCD un tetraedru regulat de muchie a. Ştiind că O şi M sunt proiecţiile lui
D pe (ABC), respectiv pe BC, iar N este simetricul lui M faţă de CO, aflaţi:
(a) cosinusul unghiului dintre DM şi AB;
(b) tangenta unghiului dintre planele (DM N ) şi (ABC);
(c) distanţa de la C la planul (DM N ).
Ion Trandafir

3. Fie funcţia f : R → R astfel ca


f (2x − 1) − 5 ≤ 4x − 8 ≤ 2f (x + 2) − 14,
oricare ar fi x ∈ R. Determinaţi valoarea minimă a expresiei (f (x − 1))2 +(f (3 + x))2 +
16x; arătaţi că (f (2005))2 + 4f (1005) − 17 este pătrat perfect.
Gianina Busuioc, Niculai Solomon
48

4. În cubul ABCDA0 B 0 C 0 D0 , punctele M, N, P sunt situate respectiv pe muchiile


AA0 , C 0 D0 şi CC 0 . Fie S punctul ı̂n care dreapta CD0 intersectează planul (M N P ).
Arătaţi că dacă D0 este mijlocul segmentului [CS], atunci unghiul pe care ı̂l face
planul (M N P ) cu planul (BCC 0 ) este mai mare decât unghiul dintre (M N P ) şi
(ABC).
Dorina Zaharia

5. Fie ABCDA0 B 0 C 0 D0 un paralelipiped cu baza ABCD romb, ı̂n care AB = AA0 =


10cm, AC = 16cm şi fie O intersecţia diagonalelor AC şi BD. Se ştie că A0 O⊥ (ABCD).
Să se afle:
(a) distanţa de la punctul D la planul (ABB 0 );
(b) tangenta unghiului diedru determinat de planele (ABC) şi (ABB 0 );
(c) distanţa dintre dreptele AC şi BC 0 .
Mariana Coadă

6. Fie a, b, c ∈ (0, 1] şi x, y, z ≥ 1 astfel ı̂ncât


√ p √ 1 x y z 
2
x−a + y−b + z−c = 2 2 + + .
2 a b c
(a) Demonstraţi că x + y + z ≤ 6.
(b) În ce caz avem egalitate la a)?
Cecilia Diaconescu

7. Să se arate că pentru fiecare număr natural n, n ≥ 2 ecuaţia


1 1 1 1
+ + ... + =
x1 x 2 xn y
are o infinitate de soluţii x1 , x2 , ..., xn , y ı̂n mulţimea numerelor naturale.
Traian Tămâian

8. Pe muchiile AB, BC, CD, DA ale unui tetraedru regulat ABCD se iau punctele
M, N, P, respectiv Q astfel ı̂ncât AM = BN = CP = DQ. Fie O mijlocul segmen-
tului [N Q]. Să se arate că (M OP ) ⊥N Q.
Emilia Lungu

9. În prisma triunghiulară ABCA0 B 0 C 0 oarecare, punctul M este mijlocul muchiei


A0 B 0 . Fie P ∈ (M C). Arătaţi că dacă A0 P ∩ B 0 C = {D} şi B 0 P ∩ A0 C = {E}, atunci
dreptele AE, BD şi CC 0 sunt concurente.
Virginia şi Vasile Tică
49

10. Pe muchia AD a piramidei ABCD se ia un punct oarecare M . Fie M N kAC, N ∈


DC şi N P kBC, P ∈ DB, iar X, Y , respectiv Z mijloacele segmentelor [AN ] , [CP ],
respectiv [BM ].
(a) Demonstraţi că planele (M N P ) , (XY Z) şi (ABC) sunt paralele şi echidis-
tante.
MD
(b) Determinaţi raportul pentru ca ı̂n ipoteza că ABC este triunghi echila-
MA
teral, măsura unghiului dintre dreptele BC şi XY să fie de 450 .
Petre Simion

11. Fie A = {1, 2, 3, ..., 2005}. Se consideră funcţiile f, g : A → N date de

f (x) = cel mai mic număr de forma 2k cel puţin egal cu x,


g (x) = cel mai mare număr de forma 2k cel mult egal cu x.

(a) Determinaţi cardinalul mulţimii {x ∈ A | f (x) = g (x)}.


(b) Demonstraţi că

1 1 1
+ + ... + ∈ (6, 25; 6, 5) .
f (1) f (2) f (2005)

Petre Simion

12. Se consideră un tetraedru OABC ı̂n care OA⊥OB⊥OC⊥OA. Arătaţi că:


 
1 1 1 1 1 1
+ + ≥ 2 + + .
OA OB OC AB BC CA

Cezar Lupu

13. Arătaţi că există un număr a natural nenul, par, astfel ı̂ncât pentru o infinitate de
numere ı̂ntregi pare b, expresia a + b + ab să fie pătrat perfect.
Gheorghe Gherasim

14. În cubul ABCDA0 B 0 C 0 D0 cu latura de lungime 3 cm se consideră mijloacele A00 , B 00 , C 00 , D00 ,
ale segmentelor [AA0 ] , [BB 0 ] , [CC 0 ] , respectiv [DD0 ]. Să se calculeze volumul
poliedrului convex determinat de vârfurile A, A”, B0, B”, C, C”, D0, D”. Câte
feţe are acest poliedru?
A. M. Ionescu
50

15. Fie a, b ∈ R∗ . Să se determine funcţia f : R → R pentru care


   
b a 2 2b b
f x− + 2x ≤ x + ≤f x+ − 2x,
a b a a

pentru orice x ∈ R.
Dorin Mărghidanu

16. Fie numerele reale a, b, c astfel ı̂ncât a + b + c = abc.


(a) Dacă a, b, c ∈ R∗+ , determinaţi valoarea minimă a lui a + b + c.
(b) Care sunt tripletele de numere ı̂ntregi (a, b, c) cu a, b, c nenule ce verifică egal-
itatea dată?
Adrian P. Ghioca
2 −1
17. Fie a ∈ N, a ≥ 2. Arătaţi că numărul xa = a2a + 1 este produsul a două numere
naturale prime ı̂ntre ele, strict mai mari ca 1.
Dumitru Acu

18. Fie p un număr prim astfel ı̂ncât 2p − 1 este de asemenea prim. Determinaţi toate
perechile de numere naturale (x, y) pentru care

(xy − p)2 = x2 + y 2 .

Dumitru Acu

19. Fie A, B, C, D puncte necoplanare. Se notează cu HA , HB ortocentrele triunghi-


urilor BDC, respectiv ADC. Să se arate că punctele A, B, HA , HB sunt coplanare
dacă şi numai dacă sunt conciclice.
A. M. Ionescu

20. Pe planul pătratului ABCD cu latura de 48 cm, de aceeaşi parte a planului, ı̂n
punctele B, C, D se ridică perpendicularele BB 0 = 20 cm, CC 0 = 100 cm, DD0 =
40 cm.
(a) Să se determine, pe perpendiculara ı̂n A planul pătratului, punctul A0 astfel
ca punctele A0, B0, C0, D0 să fie coplanare.
(b) Să se determine distanţa de la punctul C la intersecţia planelor (A0 B 0 C 0 ) şi
(ABC).
Constantin Apostol
51

21. Determinaţi toate funcţiile strict crescătoare f : Z → Z cu proprietatea că

f x2 + y 2 = f 2 (x) + f 2 (y) , ∀x, y ∈ Z.




Valentin Vornicu

22. (a) Să se arate că dacă a, b sunt numere naturale şi 5 | a2 + 3b2 , atunci 5 | a şi 5 | b.
(b) Să se rezolve ecuaţia următoare ı̂n mulţimea numerelor naturale

x3 − y 3 = 2005 x2 − y 2 .


Valentin Vornicu

4.3 Clasa a IX-a


1. Arătaţi că ı̂ntr-un triunghi, cu notaţiile consacrate, avem
P
2 a
X a cyc
≥ rP .
cyc
ma a2
cyc

Petre Bătrâneţu, Galaţi

2. Determinaţi numerele reale pozitive a, b, c dacă a3 + b3 + c3 = 24 şi (a + b)(b +


c)(c + a) = 64.
Nicolae Muşuroia, Baia Mare

3. În triunghiul ABC se consideră cevienele concurente AM, BN, CP cu N ∈ AC, M ∈


BC, P ∈ AB. Demonstraţi că

BC · CA · AB ≥ 8 · AN · CM · BP.

Romanţa şi Ioan Ghiţă, Blaj

4. În triunghiurile dreptunghice A1 B1 C1 , A2 B2 C3 (cu unghiurile A1 , A2 drepte) se


notează cu a1 , b1 , c1 , a2 , b2 , c2 laturile corespunzătoare.
(a) Arătaţi că b1 b2 + c1 c2 ≤ a1 a2 .
(b) În ce condiţii are loc egalitatea?
Tuzson Zoltán, Odorheiu Secuiesc
52

5. Dacă a, b, c, d sunt numere reale cu a 6= 0, să se arate că

|ax + b| ≤ |cx + d|, oricare ar fi x ∈ R,

dacă şi numai dacă ad = bc şi |c| ≥ |a|.


Ovidiu Pop, Satu Mare

6. Fie a1 > a2 > · · · > an (n ≥ 3) numere naturale care dau acelaşi rest la ı̂mărţirea
cu 2005, pentru care există n numere reale x1 , x2 , ..., xn astfel ı̂ncât sunt satisfăcute
următoarele proprietăţi:
(a) x1 ≥ 0, x1 + x2 ≥ 0, ..., x1 + x2 + ... + xn ≥ 0.
(b) (n − 1)x1 + (n − 2)x2 + ... + 2xn−2 + xn−1 = 1.
(c) a1 x1 + a2 x2 + ... + an xn = 2005.
Demonstraţi că numerele a1 , a2 , ..., an sunt ı̂n progresie aritmetică.
Cristinel Mortici, Târgovişte
a b c
7. Fie a, b, c numere reale strict pozitive astfel ı̂ncât a+b+c ≥ + + . Demonstraţi
b c a
inegalitatea
X a3 c 3
≥ .
cyc
b(c + a) 2

Cezar Lupu, student, Bucureşti

8. Fie a, b, c trei numere strict pozitive astfel ı̂ncât abc = 1. Să se arate ă
X a 3
≥ .
cyc
b2 (c + 1) 2

Cezar Lupu, student, Bucureşti

9. Fie a, b, c trei numere strict pozitive astfel ı̂ncât a + b + c = 1. Să se arate că
r
X a 3
√ ≥ .
cyc
b+c 2

Cezar Lupu, student, Bucureşti

10. Pe un cerc se consideră punctele A, B, C, M şi fie A1 , B1 , C1 ortocentrele triunghi-


urilor M AB, M BC, M CA.
53

(a) Să se arate că triunghiurile ABC şi A1 B1 C1 sunt congruente, iar M este orto-
centrul triunghiului A1 B1 C1 .
(b) Să se demonstreze că dreptele AA1 , BB1 , CC1 , GG1 , HM şi OO1 sunc con-
curente, unde G, H, O, respectiv G1 , H1 , O1 sunt respectiv centrele de greutate,
ortocentrele şi centrele cercurilor circumscrise triunghiulrilor ABC şi A1 B1 C1 .
Nicolae Papacu, Slobozia

11. Fie numerele reale x1 , x2 , ..., xn , y1 , y2 , ..., yn astfel ı̂ncât


n
X n
X
xk = 1, |xk | = 2.
k=1 k=1

Să se arate că


n
X 3 1
xk yk ≤ max yk − min yk .
k=1
2 k 2 k

Marcel Chiriţă, Bucureşti

12. Determinaţi k ∈ N, k ≥ 2 pentru ca să avem


r
√ √
q
a1 + 3 a2 + · · · + k+1 ak ≥ 32 a1 a2 ...ak ,

pentru orice numere a1 , a2 , ..., ak ∈ [0, ∞).


Gabriel Popa, Paul Georgescu, Iaşi

13. Fie M ∈ (BC), N ∈ (AC), P ∈ (AB) picioarele a trei ceviene concurente ı̂n tri-
unghiul ABC. Paralela prin N la AB intersectează P M ı̂n E, iar paralela prin
M la AB intersecteaza P N ı̂n F . Să se arate că dreptele M N, EF şi P C sunt
concurente.
Gabriel Popa, Iaşi
5
14. Se consideră mulţimea A = {(1 − x, 2x − 6 + )|x ∈ R∗ }.
x
(a) Să se arate că există o funcţie f : R \ {1} → R astfel ı̂ncât mulţimea A să fie
graficul funcţiei f .
(b) Să se arate că mulţimea A nu are centru de simetrie.
(c) Să se determine minimul lungimii segmentelor SM unde S(1, −6), iar M par-
curge mulţimea A.
Virgil Nicula, Bucureşti
54

15. Fie patrulaterul convex ABCD care nu este trapez, şi fie punctele I = AC ∩
BD, E = AB ∩ CD, F = AD ∩ BC. Notăm cu M, N, P, Q, respectiv mijloacele
segmentelor AB, BC, CD, DA. Să se arate că paralela dusă prin punctul E la
dreapta M P , paralela dusă prin punctul F la dreapta N R şi dreapta BD sunt
concurente dacă şi numai dacă IA = IC.
Virgil Nicula, Bucureşti

16. Considerăm un triunghi dreptunghic ABC cu unghiul drept ı̂n A. Construim


D ∈ (BC) cu AD ⊥ BC. Prin mijlocul E al ı̂nălţimii AD se duce o dreaptă
oarecare care intersectează catetele (AB) şi (AC) ı̂n M , respectiv N .
−−→ −→ −→
(a) Să se exprime vectorul AD ı̂n funcţie de vectorii AB şi AC.
AC AB
(b) Să se arate că + = k, unde k ≥ 4 este o constantă.
AM AN
Nicolae Muşuroia, Baia Mare

17. Numerele reale pozitive a, b, c verifică relaţia 2abc + ab + bc + ca = 1. Să se arate


că
√ √ √ 3
ab + bc + ca ≥ .
2
Vasile Pop, Cluj

18. În triunghiul dreptunghic ABC cu unghiul drept ı̂n A, fie D mijlocul laturii AB
şi fie M un punct mobil pe CD. Notăm cu L intersecţia dreptei BM cu simetrica
dreptei AM faţă de AC. Să se determine locul geometric al punctului L.
Virgil Nicula, Bucureşti

19. În exteriorul triunghiului ABC se construiesc triunghiurile BM C, CN A, AP B


astfel ı̂ncât ∠BCM = ∠ACN = 30◦ , ∠M BC = ∠N AC = 40◦ , ∠P AB = ∠P BA =
20◦ . Să se arate că P M = P N şi ∠M P N = 80◦ .
Virgil Nicula, Bucureşti

20. Fie triunghiul ABC ı̂nscris ı̂n cercul de centru O şi rază R şi circumscris cercului
de centru I şi rază r. Fie M = AB ∩ IO, N = AC ∩ IO. Să se arate că patrulaterul
BCN M este inscriptibil dacă şi numai dacă ha = R + r şi ı̂n acest caz
1 1 1
= + .
MN a b+c
Virgil Nicula, Bucureşti

21. Să se găsească ultimele trei cifre ale numărului 20052005 .


A.M. Ionescu, Bucureşti
55

22. Şirurile (an )n şi (bn )n sunt definite prin relaţiile


1 1
an+1 = an + , bn+1 = bn +
2bn 2an

cu a0 , b0 > 0 date. Să se demonstreze că max{a2004 , b2004 } > 2005.
Dorin Mărghidanu, Corabia

23. Fie (an )n o progresie aritmetică cu a1 < 0. Să se arate că dacă progresia conţine
cuburile a patru numere naturale consecutive atunci ea conţine toate cuburile
numerelor naturale.
Mihai Piticari, Câmpulung

24. Fie a, b, c, d numere reale pozitive astfel ı̂ncât a ≥ b + c + d. Să se arate că
(a + b)(b + c)(c + d)(d + a) ≥ 20abcd.

Dan Marinescu, Viorel Cornea, Hunedoara

25. Fie ABCD un triunghi, M un punct ı̂n interiorul său astfel ı̂ncât ∠ABM = ∠ACM ,
şi notăm cu E proiecţia pe bisectoarea interioară a unghiului A, F simetricul lui
M faţă de E şi Q mijlocul segmentului [BC].
(a) Să se arate că dacă AB 6= AC atunci EQ||AF .
(b) Demonstraţi că proiecţia lui M pe bisectoarea exterioară a unghiului A, şi
punctele E, Q sunt coliniare.
Dan Marinescu, Ciprian Demeter

26. Fie triunghiul ABC cu AB = AC. Dacă bisectoarea unghiului B intersectează


AC ı̂n D astfel ı̂ncât BC = AB + 2AD, determinaţi măsura unghiului A.
Marin Chirciu, Piteşti

27. Fie M, N, P puncte pe laturile BC, CA respectiv AB ale triunghiului ABC, astfel
BM CN AP
ı̂ncât = = = k, k ∈ (0, 1).
BC CA AB
Arătaţi că dacă cercurile circumscrise triunghiurilor AP N, BM P, CN M sunt
congruente, atunci şi cercurile ı̂nscrise ı̂n aceste triunghiuri sunt congruente.
Marin Chirciu, Piteşti

28. Dacă a, b, c > 0, să se arate că


X a6 + b 6
≥ 3a2 b2 c2 .
cyc
a+b

Marin Chirciu, Piteşti


56

29. Fie p un număr prim. Aflaţi numărul de elemente al mulţimii


p
Mp = {x ∈ Z∗ | x2 + 2pn x ∈ Q, n ∈ N∗ }.

Dumitru Acu, Sibiu

30. Fie un triunghi neisoscel ABC cu ∠N < 45◦ . Fie AD ı̂nălţimea din A, D ∈ (BC, şi
E un punct pe latura (AC). Fie P ∈ AD şi Q ∈ BE astfel ı̂ncât BD = P D = QD,
şi fie R ∈ BE, S ∈ AD astfel ı̂ncât AE = RE = SE.
Demonstraţi că punctele P, Q, R, S sunt conciclice dacă şi numai dacă BE ⊥ AC.
Valentin Vornicu, student
p
31. Fie M = {(x, y)|x, y, x2 + y 2 ∈ N}. Să se determine mulţimea punctelor din

planul cartezian situate ı̂n M şi aflate la distanţă 2 unul faţă de celălalt.
Adrian Stoica, Bucureşti

32. Se consideră o funcţie f : [a, b] ∈ R astfel ı̂ncât f (a) = f (b) şi |f (x)−f (y)| ≤ |x−y|
pentru orice x, y ∈ [a, b]. Să se demonstreze că
b−a
|f (x) − f (y)| ≤ , ∀x, y ∈ [a, b].
2

Dan Popescu, Suceava

4.4 Clasa a X-a


1. Fie n ≥ 2 un număr natural. Să se determine p ∈ N∗ astfel ı̂ncât
jp k
Apn = n − p + 1.
p

Ionel Tudor, Giurgiu

2. Se consideră şirurile (an )n şi (bn )n astfel ı̂ncât an+1 an + 3 = an − an+1 , ∀n ≥ 1,


bn bn+1 + 1 = bn − bn+1 şi a1 = b1 . Demonstraţi că a3p+1 = a4q+1 pentru orice
p, q ∈ N∗ .
Tamas Sándor, Ciucsı̂ngeorgiu

3. Fie n un număr natural. Rezolvaţi ecuaţia

logn2 (2x2 + x) + lognx2 +1 (2x2 + 2) = 8.

Meda şi Florin Bojor, Baia Mare


57

4. Fie a, b > 1. Rezolvaţi ecuaţia


a
abx + b x = ab + ba .

Marin Chirciu, Piteşti

5. Determinaţi numerele complexe a, b astfel ı̂ncât şirul (an +bn )n , n ∈ N∗ este format
numai din două valori reale.
Virgil Nicula, Bucureşti

6. Rezolvaţi ecuaţia
(−3)blog4 xc + xblog4 xc = 1.

Daniel Jinga, Piteşti

7. Fie a, b > 0. Considerăm dezvoltarea binoamelor P1 (X) = (a + bX)n şi P2 (X) =


(b + aX)n , n ≥ 2.
(a) Dacă ki este rangul celui mai mare coeficient din dezvoltarea lui Pi (X), i = 1, 2,
arătaţi că k1 + k2 nu poate lua decât trei valori.
(b) Dacă k1 ≤ k2 atunci a ≤ b.
Marcel Chiriţă, Bucureşti

8. Se consideră ı̂n plan punctele A, B, C, M ce au ca afixe numerele complexe zA , zB , zC , zM


de acelaşi modul. Fie A1 , B1 , C1 ortocentrele triunghiurilor M BC, M AC, M AB.
Să se arate că triunghiurile ABC şi A1 B1 C1 sunt congruente şi se obţin unul din
celălalt printr-o simetrie centrală.
Nicolae Papacu, Slobozia

9. Se consideră mulţimea

Fn = {(i1 , i2 , ..., ip )|n = i1 + i2 + ... + ip , i1 ≤ i2 ≤ ... ≤ ip , ik = 1, 2, 3}.

Notăm cu xn numărul elementelor mulţimii Fn . Demonstraţi că:


(a) pentru orice n ∈ N, numerele x6n+1 , x6n+2 , ..., x6n+5 sunt ı̂n progresie arit-
metică.
(b) x6n = 3n2 − 3n + 1.
Octavian Purcaru, Ploieşti
58
Capitolul 5

Probleme avute ı̂n atenţia comisiei, 2006

5.1 Clasa a VII-a


1. Bisectoarele unghiurilor triunghiului ABC taie laturile BC, CA, AB ı̂punctele
D, E respectiv F . Demonstraţi că:

1 1 1 1
+ + = .
AB · CE BC · AF CA · BD r·R
Traian Tămı̂ian, Carei

2. Într-un triunghi ABC avem m(∠BAC) = 110◦ , m(∠ABC) = 50◦ . Dacă D este un
punct ı̂interiorul triunghiului, astfel ı̂ncât m(∠DBC) = 20◦ şi m(∠DCB) = 10◦ ,
determinaţi măsura unghiului ∠ADC.

3. Fie ABCDEF un hexagon convex. Triunghiurile 41 şi 42 vor fi numite opuse


dacă sunt formate cu vârfuri consecutive care formează mulţimi disjuncte. Să se
demonstreze că dreptele care unesc centrele de greutate ale tuturor perechilor de
triunghiuri opuse sunt concurente.
Nicolae Stăniloiu

4. Fie punctele M şi N care aparţin laturilor AC şi AB respectiv, ale triunghiului
ABC, astfel ı̂ncât M A = m · M C, N A = n · N B, unde m + n = 2. Dacă P este
mn
punctul de intersecţie a dreptelor BM şi CN , să se arate că AAM P N ≥ AABC .
3
Virgil Nicula, Bucureşti

5. Fie ABCDE un pentagon convex. Numim dreaptă centrată o dreaptă care trece
prin centrul de greutate al unui triunghi format de trei vârfuri consecutive şi prin
mijlocul laturii determinate de celelalte vărfuri. Să se arate că cele cinci drepte
centrate sunt concurente.

59
60

Nicolae Stăniloiu

6. Se consideră unghiul ascuţit XOY , bisectoarea OZ şi punctele A, B fixate, A sit-


uat ı̂interiorul unghiului XOZ iar B ı̂interiorul unghiului Y OZ. Să se determine
punctele M ∈ [OX, N ∈ [OY astfel ca suma AM + M N + N B să fie minimă.
Dan Nedeianu, Turnu Severin

7. Dacă a, b, c, d sunt numere naturale nenule distincte, astfel ı̂ncât produsul lor
este pătrat perfect, să se arate că numărul a4 + b4 + c4 + d4 se poate scrie ca suma
a cinci pătrate nenule.
Dan Nedeianu, Turnu Severin

8. Găsiţi trei numere naturale distincte pentru care media aritmetică, media geo-
metrică şi media armonică sunt de asemenea numere naturale. Rezolvaţi aceeaşi
problemă pentru n ≥ 4 numere distincte.
Laurenţiu Panaitopol, Bucureşti

9. Fie numerele reale x, y, z de sumă nenulă. Să se arate că

x(y − z) y(z − x) z(x − y)


+ + = 0,
y+z x+z x+y

dacă şi numai dacă cel puţin două dintre numerele date sunt egale.
Virgil Nicula, Bucureşti

10. La o competiţie de tenis de masă desfăşurată pe parcursul a 3 zile au partici-


pat 20 de elevi. În fiecare zi fiecare a jucat câte un meci (fiind posibil ca aceeaşi
pereche să se ı̂ntâlnească ı̂mai multe zile). La final s-a observat că există un sin-
gur câştigător şi că nici un jucător nu a pierdut toate cele 3 partide. Căţi elevi au
câştigat un singur meci şi câţi exact două?
G. Rene, Bucureşti

5.2 Clasa a VIII-a


1. Fie m, n ∈ N cu m > n > 3. Arătaţi că rădăcinile x1 , x2 ale ecuaţiei

x2 − mx + n = 0,

sunt numere naturale dacă şi numai dacă expresia [mx1 ] + [mx2 ] este pătrat per-
fect.
61

2. Se consideră a, b, c trei numere reale strict pozitive. Să se arate că


Xb+a (a2 + b2 + c2 )(ab + bc + ca)
≥3+ .
a abc(a + b + c)
Cezar Lupu. Bucureşti

3. Fie a, b ∈ N∗ , a < b a nu este divizor comun al lui b. Să se rezolve ecuaţia a[x] =
b{x}.
Ioan Ucu Crişan, Arad
(r )
1 1
4. Fie A = + a, b ∈ N∗ , a 6= b şi
a b
r 
1 1 1 ∗
B= + + : x, y, z ∈ N , x > y > z .
x y z
Demonstraţi că A ∩ B conţine o infinitate de numere raţionale şi o infinitate de
numere iraţionale.
Cristina şi Claudiu Andone, Cugir

5. Fie a, b, c numere strict pozitive. Demonstraţi inegalitatea


X a2 1
2 2
≤ .
3a + b + 2ac 2
Mircea Lascu, Zalău

6. Determinaţi numerele naturale n şi x1 , x2 , . . . , xn nenule pentru care


1 1 1 1
x1 + x2 + . . . + xn = 3n şi + + ... + =1+ .
x1 x2 xn 4n
Lucian Dragomir, Oţelu-Roşu

7. Se consideră muţimea A = {x ∈ N | x = 2(p2 = q)(q 2 − p), p, q ∈ N}. Să se arate


că:
a) 2006 ∈ A;
b) A conţine o infinitate de pătrate perfecte;
c) nu există nici o mulţime nevidă B astfel ı̂ncât |B| = p2 − q; |P (B)| = q 2 − p.
Cristian Alexandrescu şi Adrian Stoica, Bucureşi

8. Determinaţi numerele ı̂ntregi x, y, z, t care satisfac relaţiile x + y + z = t2 ,


x2 + y 2 + z 2 = t2 .
Lucian Dragomir, Oţelu-Roşu
62

9. O prismă dreaptă ABCDEF A1 B1 C1 D1 E1 F1 arw baza hexagon regulat. Arătaţi


că:

a) AE1 ⊥ B1 E dacă şi numai dacă AA1 = AB 3;

42
b) ı̂condiţiile de mai sus avem dist(AE1 , B1 E) = AB.
14
Petru Braica, Satu Mare

10. Se consideră tetraedrul ABCD de volum 1 si punctele M, N, P, Q, R, S pe


muchiile AB, BC, CD, DA, AC respectiv BD. Dacă M P, N Q şi RS sunt con-
1
curente, arătaţi că volumul prismei M N P QRS nu poate depăşi .
2
Flavian Georgescu, elev, Bucureşti

11. Fie cubul ABCDA0B0C0D0 de muchie 2. Se secţionează cubul cu două plane ce


conţin fiecare mijloacele ce pleacă din B0 şi C0. Fie E şi F centrele de greutate
ale triunghiurilor de secţiune şi {P } = A0E ∩ D0F . Calculaţi cosinusul unghiului
∠A0P D0 şi distanţele de la A0 la cele două plane de secţiune.
Romanţa şi Ioan Ghiţă, Blaj

5.3 Clasa a IX-a


1. Fie n ∈ N, n ≥ 2 şi a1 , a2 , . . . , a2n , numere reale strict pozitive astfel ı̂ncât a1 +
a2 + . . . + a2n = s. Demonstraţi inegalitatea
a1 an an+1 a2n
+ ... + + + ... + ≥ 1.
s + an+1 − a1 s + a2n − an s − a1 − an+1 s + an − a2n
Stabiliţi cazurile de egalitate.
Traian Tămâian

2. Fie a, b, c numere reale strict pozitive astfel ı̂ncât a2 + b2 + c2 = 3. Să se arate că
pentru orice numere reale strict pozitive x, y, z, avem
x y z √ √ √
+ + ≥ xy + yz + zx.
a b c
Marius Damian

3. Fie a, b, c numere pozitive astfel ı̂ncât abc = 1. Demonstraţi că


X 1 1
≤ .
a2 2
+ 2b + 3 2
Mircea Lascu
63

4. O funcţie bijectivă f : {1, 2, . . . , n} → {1, 2, . . . , n} are proprietatea P dacă relaţia


 
f (1) + . . . + f (n)
g(k) = f (1) . . . + f (k) − n ,
n

pentru orice k = 1, 2, . . . , n defineşte o funcţie dacă şi numai dacă n este par.
Emil Vasile

5. Spunem că o funcţie f : [0, ∞) → [0, ∞) are proprietatea P dacă

f (xf (y 2 )) = f (y)(f (f (x2 )),

pentru orice x, y ∈ [0, ∞).


a) Să se arate că există o infinitate de funcţii cu propeietatea P.
b) Să se arate că există o unică funcţie cu proprietatea P a cărei imagine conţine
un interval deschis centrat ı̂1.
Coste Chiteş şi Adrian Stoica

6. Rezolvaţi ı̂numere ı̂ntregi x, y ecuaţia


q p
x= y2 − y 2 + x.

Vasile Berinde

7. Să se determine

 
1 1
min n ∈ N > { n} < .
100 10
Dorin Mărghidan
1 1 1
8. Fie a, b, c trei numere reale strict pozitive astfel ı̂ncât a + b + c ≥ + + . Să
a b c
se arate că
3 2
a+b+c≥ + .
a + b + c abc
Cezar Lupu şi Valentin Vornicu
X1
9. Să se arate că dacă a, b, c sunt numere reale strict pozitive astfel ı̂ncât ≤ 3,
a
atunci
X a2 + 1
√ ≥ 6.
a2 − a + 1
I. V. Maftei
64

10. Fie a, b, c ∈ R astfel ı̂ncât |ax2 + bx + x| ≤ 1, oricare ar fi x ∈ [−1, 1]. Să se arate
că dacă α ∈ [0, 1], atunci

α(1 + α)|b| + (10 − α2 )|c| ≤ 1 + α2 .

Când are loc egalitatea?


Dumitru Barac

11. Să se determine  coordonatele


  vârfurilor
  triunghiului
 ABC cunoscând picioarele
36 26 1 1 6 8
ı̂nălţimilor D , ; E , ; F , .
17 17 2 2 5 5
Gabriel Popa şi Paul Georgescu

12. Se consideră triunghiul ascuţitunghic ABC având ortocentrul H şi ı̂nălţimile


AM , BN , CP . Fie Q şi R mijloacele segementelor BH respectiv CH şi notăm
U = M Q ∩ AB, V = M R ∩ AC, T = AH ∩ P N . Să se arate că:
MH TH
a) = ;
MA TA
b) T este ortocentrul triunghiului U AV .
Manule Prajea

MB
13. Considerăm un triunghi ABC, punctul M al laturii BC perntru care =
MC
c(b + c)
şi punctul N al laturii AM pentru care ∠BN M = ∠A. Să se arate că
b2
A
∠CN M = .
2
Virgil Nicula

14. Fie ABC un triunghi şi I centrul cercului ı̂nscris. Fie A1 , B1 , C1 respectiv centrele
cercurilor ı̂nscrise ı̂triunghiurile IBC, ICA, IAB. Arătaţi că AA1 , BB1 , CC1
sunt concurente.

15. Într-un campionat de fotbal se ı̂nscriu 18 echipe. Fiecare echipă poaate să joace cel
mult un meci cu o altă echipă. Spunem că campionatul este bun dacă la sfârşitul
său oricum am alege 12 echipe din cele 18, există cel puţin 6 meciuri disputate
ı̂ntre ele.
Determinaţi numărul minim posibil de meciuri care se desfăşoareă ı̂ntr-un campio-
nat bun.
Valentin Vornicu
65

5.4 Clasa a X-a


1 1 1
1. Fie a, b, c trei numere strict pozitive astfel ı̂ncât a + b + c ≥ + + . Să sa arate
a b c
că
3 2
a+b+c≥ + .
a + b + c abc
Cezar Lupu şi Valentin Vornicu

2. Se dau numerele a1 , a2 , . . . , an ∈ {−1, 1} astfel ı̂ncât a1 + a2 . . . + an = 0. Arătaţi


că există k ∈ {1, 2, . . . , n} astfel ı̂ncât
 
k
|a1 + 2a2 + . . . + kak | ≤ .
2
G. René

3. Fie k şi n numere naturale nenule, n ≥ 2. Câte numere de kn cifre 0 şi 1 se pot
forma pentru care dacă apare cifra 0, ea apare ı̂număr multiplu de k şi ı̂secvenţe
consecutive de lungime multiplu de k.
Romanţa şi Ioan Ghiţă
n +2m−1
4. Demonstraţi că numărul (22n + 2n+m + 22m )! este divizibil cu (2n !)2 ·
m n−1
(2m !)2 +2 , pentru orice n, m ∈ N∗ .
Dana Heuberger

5. Fie a ∈ N, a ≥ 2. Definim şirul (xn )n≥0 prin relaţiile


a2 a
x0 = , x1 = (2a3 − 4a2 − a + 4) şi xn+1 − (4a2 − 2)xn + xn−1 = 0,
4 2
a2 − 2
pentru n ≥ 1. Să se arate că nunărul 2xn − este pătrat perfect pentru orice
2
n ∈ N.
Ştefan Alexe
x +1) 
6. Să se rezolve ecuaţia log3 2log3 (2 + 1 = log2 (3x − 1).
Cezar Lupu

7. Pentru p, n ∈ N∗ se consideră mumerele naturale x1 , x2 , . . . , xn . Să se arate că:


n
!p n n n
X X 2p−1
X 2p−3
X
2p−1
xi 1
≤ Cp xi 3
+ Cp xi + . . . + Cp 2m+1
x2p−2m−1
i ,
i=1 i=1 i=1 i=1
 
p−1
unde m = .
2
Dumitru Barac
66

8. Aflaţi numerele prime p şi numerele q naturale cu p ≤ q ≤ q 2 pentru care

Cpq2 − Cqp = 1.

Costel Chiteş, Adrian Stoica

9. Fie A1 A2 A3 A4 un patrulater convex circumcritibil, cercul ı̂nscris având raza r.


a) Arătaţi că există cercurile Ci = (Ai , ri ), i = 1, 2, 3, 4 centrate ı̂Ai şi tangente la
C(Ai+1 , ri+1 ) (unde se consideră notaţia modulo 4, adocă C5 = C1 ).
4
X 1 4
b) Dacă = , arătaţi că patrulaterul este pătrat.
r
i=1 i
r
***

10. Fie d o dreaptă dată ı̂spaţiu. Pentru orice n puncte A1 , A2 , . . . , an nesituate pe d,


figura formată din reuniunea semiplanelor Sk = (dAk , 1 ≤ k ≤ n o vom numi
n-evantai dacş măsura unghiului diedru dintre oricare două semiplane Si şi Sj ,
1 ≤ i < j ≤ n, este exprimată printr-un număr ı̂ntreg nenul de grade.
a) Demonstraţi că orice 91-evantai conţine două semiplane care sau sunt ı̂prelungire
sau sunt perpendiculare.
b) Pentru fiecare 1 ≤ n ≤ 360, aflaţi câte n-evantaie există cu proprietatea că
prientre semiplanele sale există două care sau sunt ı̂prelungire sau sunt pepren-
diculare (două n-evantaie se consideră identice dacă şi numai dacă unul se obţine
din altul printr-o rotaţie ı̂jurul dreptei d).
Cristinel Mortici

11. Dacă a, b, c sunt lungimile laturilor unui triunghi, iar R este raza cercului cir-
cumscris şi S aria, să se arate că ı̂ipoteza a2 + b2 + c2 = 4 avem 6R2 + S 2 ≥ 3.
Vasile Cı̂rtoaje

12. Fie ABC un triunghi şi P un punct interior. Notăm ci r1 , r2 respectiv r3 razele
cercurilor ı̂nscrise ı̂triunghiurile P BC, P CA şi P AB. Demonstraţi că
a b c √
+ + ≥ 6(2 + 3).
r1 r2 r3
G. René

13. Determinaţi numerele complexe a, b, c care verifică relaţiile

a3 + b3 + c3 = 24, (a + b)(b + c)(c + a) = 64, |a + b| = |b + c| = |c + a|.

Nicolae Muşuroaia
67

14. Să se arate că ı̂ntr-un triunghi oarecare ABC, cu notţiile consacrate, avem
r
3R(ma + mb + mc )
p≤ .
2
Nicolae Stăniloiu
68
Capitolul 6

Probleme avute ı̂n atenţia comisiei, 2007

6.1 Clasa a VII-a


1. Pentru care numere naturale N există un număr natural n astfel ı̂ncât suma oricărui
grup de n numere consecutive să fie divizibilă cu N .

Mihai Bălună, Bucureşti

2. Să se arate că 2007 nu poate fi scris ca suma pătratelor a patru numere prime.
[Enunţ modificat: 2007 ı̂nlocuieşte 20007]

∗∗∗

3. Fie ABCD un patrulater ı̂care bisectoarele unghiurilor A şi C ı̂mpreună cu diag-


onala BD sunt concurente ı̂punctul I. Să se arate că:
a) Este verificată egalitatea AB · CD = AD · BC.
b) Bisectoarele unghiurilor B şi D se intersectează pe diagonala AC.
c) Dacă distanţele de la I la laturile AB şi BC sunt egale atunci diagonalele AC
şi BD sunt perpendiculare.

Vasile Pop, Cluj

4. Fie ABC un triunghi cu ortocentrul H şi O centrul cercului circumscris. Fie M


mijlocul laturii BC. Ştiind că AC = 2AB = 2AH, demonstraţi că:
a) AC = 4OM ;
b) segmentele AH, OM şi BM pot fi laturile unui triunghi.

Petre Simion, Bucureşti

69
70

5. Un pătrat ABCD se ı̂ndoaie astfel ı̂ncât punctul D ajunge pe latura AB ı̂ntr-un


punct M . Arătaţi că suprafaţa ı̂ndoită este mai mare decât un sfert din suprafaţa
pătratului, oriunde ar fi M pe AB.

Dorina Zaharia, Braşov

6. Arătaţi că numărul

1 2 3 22007
1+ + + + ... + 2007
2 3 4 2 +1

nu poate fi natural.

Petre Bătrâneţu, Galaţi

7. Să se rezolve ı̂n numere naturale ecuaţia 4x2 + 9y 2 = 31213.

Petre Bătrâneţu, Galaţi

8. Dacă n este un număr natural cel puţin egal cu 250874, aflaţi primele 3 zecimale

ale numărului A = 16n2 + n + 2007.
[Enunţ modificat: cel puţin egal cu 250874 ı̂nlocuieşte strict mai mic decât 250875]

Petre Bătrâneţu, Galaţi



9. Este numărul A = 44...44355...556, unde cifrele 4 şi 5 apar de câte 2007 ori,
natural?

Petre Bătrâneţu, Galaţi

10. Se consideră un triunghi ABC ı̂care AB = AC şi D ∈ (BC) astfel ı̂ncât m(∠BAD)
= 90◦ , m(∠DAC) = 30◦ şi BD
DC
= 2. Să se arate că există un număr real k astfel ı̂ncât
BC = k · AB şi BC = k · AD (sau să se arate că BC · AD = AB 2 ).
2

Adriana şi Lucian Dragomir

11. a) Arătaţi că nu există perechi de numere naturale nenule a, b care să verifice
ecuaţia 3a2 + 223b2 = 20072007.
b) Determinaţi tripletele de numere naturale nenule a, b, k cu proprietăţile
i) 3a2 + 223b2 = 2007k 2 ;
ii) a are exact 4 divizori naturali.
71

Adrian Turcanu, Piteşti

12. Fie ABC un triunghi ascuţitunghic, ω cercul său circumscris şi fie H 0 , O0 , I 0 , G0
simetricele H, O, I, G respectiv faţă de latura BC. Să se arate că:
a) H 0 ∈ ω;
b) O0 ∈ ω dacă şi numai dacă m(∠A) = 60◦ ;
c) I 0 ∈ ω dacă şi numai dacă m(∠A) = 60◦ ;
d) G0 ∈ ω dacă şi numai dacă 3bc = 4mb · mc .
[Enunţ modificat: m(∠A) = 60◦ ı̂nlocuieşte m(∠A) = 30◦ ]

Vasile Pop, Cluj

13. Fie x, y, z ∈ Z şi P = (x + y + z + 1)2 − 2(xy + xz + yz + 1) + 1.


a)Dacă x, y, z sunt numere consecutive, demonstraţi că numărul P nu este diviz-
ibil cu 3.
b)Dacă numărul P nu este divizibil cu 3, demonstraţi că cel puţin unul dintre
numerele x, y, z nu este divizibil cu 3.

Petre Simion, Bucureşti

14. Fie trapezul ABCD cu AB k CD ı̂n care AB = 2CD, M ∈ (BC) astfel ı̂ncât
BM = CM şi DM ∩ AC = {P }. Să se demonstreze că:
a) DP = P M ;
b) AC, paralela prin D la BC şi paralela prin M la AB sunt concurente.

∗∗∗

15. Fie ABCD un paralelogram, iar N ∈ (AD) astfel ı̂ncât AN = k1 · AD şi M ∈ (AB)
astfel ı̂ncât AM = k2 · AB cu k1 , k2 ∈ (0, 1). Dacă DM ∩ CN = {P }, AP ∩ CD =
CQ
{Q}, calculaţi valoarea raportului DQ ı̂n funcţie de k1 şi k2 .

Gheorghe Molea, Curtea de Argeş

16. În triunghiul ABC se consideră D, M, N, P mijloacele laturilor BD, AB, AC şi
respectiv AD. Prin centrul de greutate G se duce paralela la BC ce taie AB ı̂n E
şi AC ı̂n F .
a) Demonstraţi că punctele A, GM ∩ P F şi GN ∩ P E sunt coliniare.
b) Demonstraţi că EP, GN şi DF sunt concurente.

Romanţa şi Ioan Ghiţă


72

17. În triunghiul ABC centrul cercului circumscris coincide cu cel al cercului ı̂nscris.
Fie D pe AC astfel că patrulaterul ODCB este trapez. Determinaţi măsurile
unghiurilor triunghiului ABC.

Petru Braica, Satu Mare

18. Se consideră tetraedrul ABCD ı̂care punctul A este egal depărtat de laturile tri-
unghiului BCD şi ∠BAC = ∠BDC = 90◦ . Dacă I este centrul cercului ı̂nscris
ı̂triunghiul BCD şi P este proiecţia lui I pe dreapta BC, să se arate că:
a) BD · DC = 2 · BP · P C;
[ABC] BC
b) [BCD] = 2·AP .

∗∗∗

19. Să se arate că dacă ı̂ntr-un tetraedru cercurile ı̂nscrise ı̂trei feţe sunt tangente două
câte două, atunci cele patru cercuri ı̂nscrise ı̂feţe sunt tangente două câte două.

∗∗∗

20. Ştiind că x2 + y 2 ≤ 2x + 6y, aflaţi valoarea maximă a expresiei E = x + 3y, unde
x, y ∈ R.

∗∗∗

21. Fie n > 2 un număr natural şi fie x1 , x2 , ..., xn ∈ [1, ∞) astfel ca
n
!
X √ 1 X√
xi − 1 ≥ xi xj .
i=1
n − 1 i6=j

Să se calculeze (x1 − 1)(x2 − 1)...(xn − 1).

Gheorghe Molea, Curtea de Argeş

22. Fie a, b numere naturale nenule cu (a, b) = d > 1.


a) Să se arate că există o partiţie a lui N∗ ı̂două mulţimi nevide A, B cu propri-
etatea

(P ): x ∈ A ⇒ x + a ∈ A şi y ∈ B ⇒ y + b ∈ B.

b) Să se arate că dacă (A, B) este o partiţie a lui N∗ cu proprietatea (P ), atunci
pentru orice z ∈ N∗ numerele z şi z + d sunt ambele ı̂A sau ambele ı̂B.
[Enunţ modificat: (a, b) = d > 1 ı̂nlocuieşte (a, b) > 1]
73

Vasile Pop, Cluj

23. Fie a, b, c > 0 numere reale cu a + b + c = 1. Demonstraţi că


√ √ √  
ab bc ca 1 1 1 1
+ + ≤ 3+ + + .
1−c 1−a 1−b 8 a b c

Laura Molea şi Gheorghe Molea

24. Arătaţi că pentru orice numere reale a, b, c > 0 avem

a2 + 1 b 2 + 1 c 2 + 1
+ + ≥ 3.
b+c c+a a+b

∗∗∗

25. Fie V ABC o piramidă triunghiulară regulată şi M un punct pe arcul mic AB
al cercului C(O, R) circumscris triunghiului ABC. Dacă V D, V E, V F reprezintă
distanţele de la V la dreptele M C, M A, M B respectiv, arătaţi că

R2
 
2 2 2 2
VD +VE +VF =3 VO + .
2

Vasile Pop, Cluj

26. Se dau cubul ABCDA0 B 0 C 0 D0 şi punctele M , N , P , Q pe segmentele A0 B, B 0 C,


0 0 AQ
C 0 D, D0 A astfel ı̂ncât AMM B
= BN CN = PDP
C0
= QD 0 = k.

a) Demonstraţi că punctele M , N , P , Q sunt coplanare dacă şi numai dacă k = 1.


b) Aflaţi valoarea lui k astfel ca unghiul dintre dreapta C 0 M şi planul ADB 0 să fie
de 15◦ .

Petre Simion, Bucureşti

6.2 Clasa a VIII-a


1. Pentru care numere naturale N există un număr natural n astfel ı̂ncât suma oricărui
grup de n numere consecutive să fie divizibilă cu N .

Mihai Bălună, Bucureşti


74

2. Să se arate că 2007 nu poate fi scris ca suma pătratelor a patru numere prime.
[Enunţ modificat: 2007 ı̂nlocuieşte 20007]

∗∗∗

3. Fie ABCD un patrulater ı̂care bisectoarele unghiurilor A şi C ı̂mpreună cu diag-


onala BD sunt concurente ı̂punctul I. Să se arate că:
a) Este verificată egalitatea AB · CD = AD · BC.
b) Bisectoarele unghiurilor B şi D se intersectează pe diagonala AC.
c) Dacă distanţele de la I la laturile AB şi BC sunt egale atunci diagonalele AC
şi BD sunt perpendiculare.

Vasile Pop, Cluj

4. Fie ABC un triunghi cu ortocentrul H şi O centrul cercului circumscris. Fie M


mijlocul laturii BC. Ştiind că AC = 2AB = 2AH, demonstraţi că:
a) AC = 4OM ;
b) segmentele AH, OM şi BM pot fi laturile unui triunghi.

Petre Simion, Bucureşti

5. Un pătrat ABCD se ı̂ndoaie astfel ı̂ncât punctul D ajunge pe latura AB ı̂ntr-un


punct M . Arătaţi că suprafaţa ı̂ndoită este mai mare decât un sfert din suprafaţa
pătratului, oriunde ar fi M pe AB.

Dorina Zaharia, Braşov

6. Arătaţi că numărul

1 2 3 22007
1+ + + + ... + 2007
2 3 4 2 +1

nu poate fi natural.

Petre Bătrâneţu, Galaţi

7. Să se rezolve ı̂n numere naturale ecuaţia 4x2 + 9y 2 = 31213.

Petre Bătrâneţu, Galaţi


75

8. Dacă n este un număr natural cel puţin egal cu 250874, aflaţi primele 3 zecimale

ale numărului A = 16n2 + n + 2007.
[Enunţ modificat: cel puţin egal cu 250874 ı̂nlocuieşte strict mai mic decât 250875]

Petre Bătrâneţu, Galaţi



9. Este numărul A = 44...44355...556, unde cifrele 4 şi 5 apar de câte 2007 ori,
natural?

Petre Bătrâneţu, Galaţi

10. Se consideră un triunghi ABC ı̂care AB = AC şi D ∈ (BC) astfel ı̂ncât m(∠BAD)
= 90◦ , m(∠DAC) = 30◦ şi BD
DC
= 2. Să se arate că există un număr real k astfel ı̂ncât
BC = k · AB şi BC = k · AD (sau să se arate că BC · AD = AB 2 ).
2

Adriana şi Lucian Dragomir

11. a) Arătaţi că nu există perechi de numere naturale nenule a, b care să verifice
ecuaţia 3a2 + 223b2 = 20072007.
b) Determinaţi tripletele de numere naturale nenule a, b, k cu proprietăţile
i) 3a2 + 223b2 = 2007k 2 ;
ii) a are exact 4 divizori naturali.

Adrian Turcanu, Piteşti

12. Fie ABC un triunghi ascuţitunghic, ω cercul său circumscris şi fie H 0 , O0 , I 0 , G0
simetricele H, O, I, G respectiv faţă de latura BC. Să se arate că:
a) H 0 ∈ ω;
b) O0 ∈ ω dacă şi numai dacă m(∠A) = 60◦ ;
c) I 0 ∈ ω dacă şi numai dacă m(∠A) = 60◦ ;
d) G0 ∈ ω dacă şi numai dacă 3bc = 4mb · mc .
[Enunţ modificat: m(∠A) = 60◦ ı̂nlocuieşte m(∠A) = 30◦ ]

Vasile Pop, Cluj

13. Fie x, y, z ∈ Z şi P = (x + y + z + 1)2 − 2(xy + xz + yz + 1) + 1.


a)Dacă x, y, z sunt numere consecutive, demonstraţi că numărul P nu este diviz-
ibil cu 3.
b)Dacă numărul P nu este divizibil cu 3, demonstraţi că cel puţin unul dintre
numerele x, y, z nu este divizibil cu 3.
76

Petre Simion, Bucureşti

14. Fie trapezul ABCD cu AB k CD ı̂n care AB = 2CD, M ∈ (BC) astfel ı̂ncât
BM = CM şi DM ∩ AC = {P }. Să se demonstreze că:
a) DP = P M ;
b) AC, paralela prin D la BC şi paralela prin M la AB sunt concurente.

∗∗∗

15. Fie ABCD un paralelogram, iar N ∈ (AD) astfel ı̂ncât AN = k1 · AD şi M ∈ (AB)
astfel ı̂ncât AM = k2 · AB cu k1 , k2 ∈ (0, 1). Dacă DM ∩ CN = {P }, AP ∩ CD =
CQ
{Q}, calculaţi valoarea raportului DQ ı̂n funcţie de k1 şi k2 .

Gheorghe Molea, Curtea de Argeş

16. În triunghiul ABC se consideră D, M, N, P mijloacele laturilor BD, AB, AC şi
respectiv AD. Prin centrul de greutate G se duce paralela la BC ce taie AB ı̂n E
şi AC ı̂n F .
a) Demonstraţi că punctele A, GM ∩ P F şi GN ∩ P E sunt coliniare.
b) Demonstraţi că EP, GN şi DF sunt concurente.

Romanţa şi Ioan Ghiţă

17. În triunghiul ABC centrul cercului circumscris coincide cu cel al cercului ı̂nscris.
Fie D pe AC astfel că patrulaterul ODCB este trapez. Determinaţi măsurile
unghiurilor triunghiului ABC.

Petru Braica, Satu Mare

18. Se consideră tetraedrul ABCD ı̂care punctul A este egal depărtat de laturile tri-
unghiului BCD şi ∠BAC = ∠BDC = 90◦ . Dacă I este centrul cercului ı̂nscris
ı̂triunghiul BCD şi P este proiecţia lui I pe dreapta BC, să se arate că:
a) BD · DC = 2 · BP · P C;
[ABC] BC
b) [BCD] = 2·AP .

∗∗∗

19. Să se arate că dacă ı̂ntr-un tetraedru cercurile ı̂nscrise ı̂trei feţe sunt tangente două
câte două, atunci cele patru cercuri ı̂nscrise ı̂feţe sunt tangente două câte două.

∗∗∗
77

20. Ştiind că x2 + y 2 ≤ 2x + 6y, aflaţi valoarea maximă a expresiei E = x + 3y, unde
x, y ∈ R.

∗∗∗

21. Fie n > 2 un număr natural şi fie x1 , x2 , ..., xn ∈ [1, ∞) astfel ca

n
!
X √ 1 X√
xi − 1 ≥ xi xj .
i=1
n−1 i6=j

Să se calculeze (x1 − 1)(x2 − 1)...(xn − 1).

Gheorghe Molea, Curtea de Argeş

22. Fie a, b numere naturale nenule cu (a, b) = d > 1.


a) Să se arate că există o partiţie a lui N∗ ı̂două mulţimi nevide A, B cu propri-
etatea

(P ): x ∈ A ⇒ x + a ∈ A şi y ∈ B ⇒ y + b ∈ B.

b) Să se arate că dacă (A, B) este o partiţie a lui N∗ cu proprietatea (P ), atunci
pentru orice z ∈ N∗ numerele z şi z + d sunt ambele ı̂A sau ambele ı̂B.
[Enunţ modificat: (a, b) = d > 1 ı̂nlocuieşte (a, b) > 1]

Vasile Pop, Cluj

23. Fie a, b, c > 0 numere reale cu a + b + c = 1. Demonstraţi că


√ √ √  
ab bc ca 1 1 1 1
+ + ≤ 3+ + + .
1−c 1−a 1−b 8 a b c

Laura Molea şi Gheorghe Molea

24. Arătaţi că pentru orice numere reale a, b, c > 0 avem

a2 + 1 b 2 + 1 c 2 + 1
+ + ≥ 3.
b+c c+a a+b

∗∗∗
78

25. Fie V ABC o piramidă triunghiulară regulată şi M un punct pe arcul mic AB
al cercului C(O, R) circumscris triunghiului ABC. Dacă V D, V E, V F reprezintă
distanţele de la V la dreptele M C, M A, M B respectiv, arătaţi că
R2
 
2 2 2 2
VD +VE +VF =3 VO + .
2

Vasile Pop, Cluj

26. Se dau cubul ABCDA0 B 0 C 0 D0 şi punctele M , N , P , Q pe segmentele A0 B, B 0 C,


0 0 AQ
C 0 D, D0 A astfel ı̂ncât AMM B
= BN CN = PDP
C0
= QD 0 = k.

a) Demonstraţi că punctele M , N , P , Q sunt coplanare dacă şi numai dacă k = 1.


b) Aflaţi valoarea lui k astfel ca unghiul dintre dreapta C 0 M şi planul ADB 0 să fie
de 15◦ .

Petre Simion, Bucureşti

6.3 Clasa a IX-a


1. Fie ABCD un dreptunghi şi M, N două puncte situate ı̂n semiplane opuse lui D
faţă de AB, respectiv BC, astfel ı̂ncât M A = M B şi N B = N C. Demonstraţi că
triunghiul DM N este echilateral dacă şi numai dacă triunghiurile M AB şi N BC
sunt echilaterale.
***

2. Fie hexagonul convex M N P QRS, şi A, B, C ı̂n interiorul său, astfel ı̂ncât tri-
unghiurile ABC, N AM, P QB şi CRS să fie asemenea. Fie X, Y, Z mijloacele seg-
mentelor [N P ], [QR], respectiv [SM ] şi G, K, I centrele de greutate ale triunghi-
urilor ABC, M P R, respectiv N QS. Arătaţi că:
(a) dacă triunghiul ABC este echilateral, atunci şi triunghiul GKI este echila-teral
(b) triunghiurile ABC şi XY Z sunt asemenea dacă şi numai dacă triunghiul ABC
este echilateral
Dana Heuberger, Baia Mare

3. Fie p ∈ N, p ≥ 2. Determinaţi toate funcţiile crescătoare f : Z → Z cu proprietatea


că
1
f1 (n) + f2 (n) + ... + fp (n) = pn + p(p + 1), ∀n ∈ Z,
2
unde fk = f f ◦ f ◦ ... ◦ f .
| {z }
k ori
Marin Ionescu, Piteşti
79

4. Demonstraţi că dacă o funcţie f : R → R are proprietatea


n
X
2k (f (x + ky) − f (x − ky)) ≤ 1, ∀n ∈ N∗ , ∀x, y ∈ R
k=1

atunci f este constantă.


Farcas Csaba, Cluj Napoca

5. Demonstraţi că pentru orice numere reale a, b, c > 0 are loc inegalitatea

a2 + 1 b 2 + 1 c 2 + 1
+ + ≥ 3.
b+c a+c a+b

Petre Bătrâneţu, Galaţi

6. Demonstraţi că dacă a, b, c ∈ [0, 1] atunci


a b c 5
+ + + abc ≤ .
1 + bc 1 + ac 1 + ab 2

Vasile Pop, Cluj Napoca

7. Demonstraţi că dacă a1 , a2 , ..., an > 0 şi a1 + a2 + ... + an = 1 atunci

(a2 + a3 + ... + an )2 (a1 + a3 + ... + an )2


+ + ...+
1 + a1 1 + a2
(a1 + a2 + ... + an−1 )2 (n − 1)2
+ ≥ .
1 + an n+1
Traian Tămâian, Carei

8. Demonstraţi că ı̂n orice triunghi are loc inegalitatea

a b c 2(a + b + c)
+ + ≥√ .
ma mb mc a2 + b 2 + c 2

Petre Bătrâneţu, Galaţi


1
9. Demonstraţi că pentru orice numere reale a, b, c > − are loc inegalitatea
2
a2 + 2 b2 + 2 c2 + 2
+ + ≥ 3.
b+c+1 a+c+1 a+b+1

Marin Ionescu, Galaţi


80

6.4 Clasa a X-a


1. Fie n ∈ N ∗ . Rezolvati ı̂n N∗ ecuaţia:
h 2i h 2i
x n
x n

n
+ x
= n
+ x
+ nx .

Marcel Chirita, Bucuresti

2. Rezolvati sistemul

x (3y 2 + 1) = y (y 2 + 3)
y (3z 2 + 1) = z (z 2 + 3)
z (3x2 + 1) = x (x2 + 3).

Marcel Chirita, Bucuresti

3. Determinati toate numerele prime p ≥ 3 care au proprietatea ca, pentru orice


k ∈ 1, p−1
2
, numarul 1 + k (p − 1) este prim.
Adrian Stoica, Bucuresti

4. Fie z1 , z2 , z3 numere complexe distincte cu |z1 | = |z2 | = |z3 | > 0. Sa se arate ca


punctele de afixe z1 , z2 , z3 sunt varfurile unui triunghi echilateral daca si numai
daca exista k ∈ R\ {1} astfel incat

|kz1 + z2 + z3 | = |z1 + kz2 + z3 | = |z1 + z2 + kz3 |.

Marin Ionescu, Pitesti

5. O multime M de numere reale verifica:


a) 0 ∈ M ;
b) daca x, y > 0 si log2 (x + y) ∈ M atunci si 3x ∈ M si log4 y ∈ M .
2008
Demonstrati ca 2007
∈ M.
Lucian Dragomir, Otelul Rosu

6. Rezolvati ecuatia 2tan x + 2cot x = 2 cot 2x.


Traian Tamaian, Carei

7. Demonstrati ca in orice triunghi


81
 2
4R+r 9r
p
+ 4R+r
≥ 4.

Cosmin Pohoata, elev, Bucuresti

8. Fie z un numar complex pentru care |z| , |z 2 − 2z + 2| , |z 2 − 3z + 3| sunt cel mult


1. Demonstrati ca z = 1.
Virgil Nicula, Bucuresti

9. Fie z1 , z2 , z3 numere complexe distincte. Demonstrati ca urmatoarele afirmatii


sunt echivalente:
i)z1 , z2 , z3 sunt varfurile unui triunghi echilateral;
ii)exista θ ∈ C astfel incat polinomul (X − z1 ) (X − z2 ) (X − z3 )−θ sa nu aiba nici
o radacina simpla.
Sever Moldoveanu, Constantin Buse
82
Capitolul 7

Probleme avute ı̂n atenţia comisiei, 2008

7.1 Clasa a VII-a


1. În triunghiul ABC se consideră bisectoarea (AE a unghiului ∠BAC, E ∈ (BC)
şi punctele D ∈ (AB), F = AE ∩ CD astfel ı̂ncât 2DB = AB, 3EC = BC şi
4F C = AB. Determinaţi măsurile unghiurilor triunghiului ABC.
Lucian Dragomir, Oţelu-Roşu

2. ı̂n triunghiul ABC, M ∈ (BC) şi P ∈ (M B). Prin punctul P construim paralele
P F la AB, respectiv P E la AC cu F, E ∈ AM . Demonstraţi că punctul M este
mijlocul segmentului (BC) dacă şi numai dacă CE k BF .
Ion Diaconu, Piatra Neamţ

3. Fie triunghiul ABC şi (D, (CE bisectoarele unghiurilor ∠ABC, respectiv ∠ACB,
cu D ∈ (AC), E ∈ (AB). Demonstraţi că

1 1 1 1
− = − .
AD AE AC AB

***

4. Fie Ia punctul de intersecţie al bisectoarelor exterioare unghiurilor B şi C ale


triunghiului ABC şi D proiecţia lui Ia pe dreapta BC. Notăm AIa ∩ BC = {M }.
Arătaţi că:
AB · BC 1 + cos C
a) = ;
AC · CD 1 + cos B
b) (AB + CD)2 > AM (2AIa − AM ).
Viorel Cornea şi Dan Şt. Marinescu, Hunedoara

83
84

5. Trapezul ABCD are diagonalele perpendiculare ı̂n punctul O, baza mare [AB]
şi măsura unghiului A de 90◦ . Paralele prin punctul O la AB intersectează pe
AD şi BC ı̂n punctele P , respectiv Q, iar perpendiculara din punctul O pe BC
intersectează AD ı̂n punctul R. Demonstraţi că:
a) (P O) ≡ (QO);
b) [RQ] ≡ [AD].
Claudiu Şt. Popa, Iaşi

6. În triunghiul ABC, AD este bisectoarea ∠BAC, iar I1 şi I2 sunt centrele cercurilor
ı̂nscrise respectiv ale triunghiurilor ABD şi ADC. Demonstraţi că:
a) AI1 · DI1 = AI2 · DI2 ;
b) AD, BI2 şi CI1
D. Zaharia

7. În trapezul oarecare ABCD, cu baza mare (CD), se consideră punctele M, N, P, Q


respectiv mijloacele laturilor [AB], [BC], [CD], [AD]. Ştiind că QN ∩ BD = {E} şi
QN ∩ AC = {F }, iar [EF ] ≡ [M P ], calculaţi m(∠ADC) + m(∠BCD).
D. Nedeianu, Drobeta Turnu-Severin

8. Punctele A, B, C, D sunt coliniare şi BC = 2AB, B ∈ (AC), CD = 2BC, C ∈


(BD). De aceeaşi parte a dreptei AD se consideră, pe drepte paralele, segmentele
AE = 2n , BF = 2n+1 , CG = 2n+2 , dH = 2n+3 , n ∈ N∗ .
(i) Să se arate că punctele E, F, G, H sunt coliniare.
(ii) Ştiind că (AB) ≡ (AE) să se demonstreze că DG ⊥ EH.
***

9. Fie ABC un triunghi isoscel cu m(∠BAC) = 120◦ . Semidreapta (CD este bi-
sectoarea ∠ACB, D ∈ AB, iar punctele A0 şi D0 sunt proiecţiile ortogonale ale
punctelor A, respectiv D pe BC. Demonstraţi că (DD0 ) ≡ (D0 A0 ).
V. Popa

10. În trapezul ABCD (AB k CD, AB > CD) ducem DM ⊥ AB, M ∈ (AB). Fie N
mijlocul diagonalei BD. Demonstraţi că M N k AC dacă şi numai dacă trapezul
este isoscel.
Aurel Bârsan, Braşov
85

11. Se dă un paralelogram de arie S. Folosind numai o riglă negradată, să se con-
S
struiască un paralelogram de arie .
6
Aurel Bârsan, Braşov

12. Fie triunghiul ABC şi cevienele AM, BN şi CP concurente ı̂n O. Următoarele
afirmaţii sunt echivalente:
AP AN
(i) + = 1;
P B NC
(ii) Aria(BOC) = 12 Aria(ABC);
(iii) Aria(P ON ) = 21 Aria(AP N ).
Viorel Cornea şi Dan Şt. Marinescu, Hunedoara

13. Să se arate că un paralelogram ABCD este romb dacă şi numai dacă există un
punct M ∈ (AC) astfel ı̂ncât m(∠AM B) + m(∠CM D) = 180◦ .
Lucian Dragomir, Oţelu-Roşu

14. Fie triunghiul ABC şi D ∈ (BC), E ∈ (AC), F ∈ (AB), astfel ı̂ncât AD, BE şi CF
sunt concurente ı̂n H. Demonstraţi că H este ortocentrul triunghiului ABC dacă
şi numai dacă HA · HD = HB · HE = HC · HF .
Romanţa Ghiţă şi Ioan Ghiţă, Blaj

n2008
15. Fie p > 2 un număr natural prim. Determinaţi n ∈ N astfel ı̂ncât numărul
n+p
să fie număr natural impar.
C. Săvescu

16. Fie ABCD un patrulater ı̂nscris ı̂ntr-un cerc de diametru (AD). fie E un punct
pe cerc, situat pe semicercul opus lui B şi C faţă de AD. Fie M, N, P picioarele
perpendicularelor din E pe laturile AB, BC şi CD respectiv. Demonstraţi că tri-
unghiul M N P este dreptunghic.
Valentin Vornicu, Bucureşti
 
1 2
17. Fie mulţimea S = | abc = x + 3x + 2, x ∈ N . Calculaţi suma elementelor
abc
mulţimii S.
Nicolae Stănică, Brăila

18. Demonstraţi că 2008 nu se poate scrie sub forma an + bn , cu a, b, n ı̂ntregi, n > 2.
Valentin Vornicu, Bucureşti
86

19. În patrulaterul convex ABCD diagonalele sunt perpendiculare şi se intersectează
ı̂n punctul O. Fie P, Q, R, S proiecţiile punctului O pe laturile AB, BC, CD, DA.
Arătaţi că
1 1 1 1
+ = + .
P A · P B RC · RD QB · QC SD · SA
Mihaly Bencze, Braşov
p √ p √
20. Există numere naturale a, b nenule, astfel ı̂ncât a + 2 b şi b + 2 a să fie raţionale?
G. Rene

21. Fie a, b ∈ R, astfel ı̂ncât a(a − b) şi b(b − a) sunt raţionale. Demonstraţi că ab este
număr raţional.
Gh. F. Molea, Curtea de Argeş

22. Se consideră ecuaţia x2 + y 2 − 2xy + 3x − 3y + 2 = 0, x, y ∈ BbbN .


a) Arătaţi că ecuaţia are o infinitate de soluţii (x, y) ∈ N × N.
b) Arătaţi că, pentru orice partiţie a mulţimii numerelor naturale, ı̂n două submulţimi
N = A ∪ B, A ∩ B = ∅, există o soluţie a ecuaţiei (x, y) cu x şi y ı̂n aceeaşi
submulţime.
c) Arătaţi că se poate face o partiţie a lui N ı̂n trei submulţimi N = A ∪ B ∪ C, A ∩
B = B ∩ C = C ∩ A = ∅ astfel ca, pentru orice soluţie (x, y) a ecuaţiei, numerele
x şi y să fie ı̂n submulţimi diferite din partiţie.
Vasile Pop

23. Arătaţi că există o infinitate de numere naturale scrise ı̂n baza zece care au suma
şi produsul cifrelor egale.
Aurel Bârsan, Braşov

24. Pot fi colorate cu alb şi roşu vârfurile unui poligon convex cu 2009 laturi, astfel
ı̂ncât ı̂n orice secvenţă de şase vârfuri consecutive să fie exact unul sau exact trei
vârfuri albe?
G. Rene

25. Aflaţi câte numere naturale de zece cifre au suma şi produsul cifrelor egale.
Aurel Bârsan, Braşov

26. Arătaţi că ı̂nălţimea dusă pe latura cea mai mare a unui triunghi este mai mică
decât suma lungimilor perpendicularelor duse dintr-un punct arbitrar al acestei
laturi pe celelalte două.
Florica Banu
87

27. Răsturnatul unui număr N este R(N ) = numărul obţinut prin scrierea cifrelor ı̂n
ordine inversă. Aflaţi toate numerele N de patru cifre astfel ı̂ncât R(N ) = 4N + 3.
Florica Banu

28. Arătaţi că există un număr natural A divizibil cu 22001 şi care nu conţine nici o
cifră 0 ı̂n reprezentarea sa ı̂n baza 10.
Florica Banu

29. Secvenţa a1 , a2 , . . . , a2001 este o permutare a numerelor 1, 2, . . . , 2001. Fiecare număr


ak se ı̂nmulţeşte cu locul pe care-l ocupă ı̂n această secvenţă. Arătaţi că cel mai
mare dintre numerele astfel obţinute este mai mare sau egal cu 10012 .
Florica Banu

30. Se dau n > 5 numere reale strict pozitive cu suma 1.


a) Arătaţi că, oricum am aşeza numerele pe un cerc, suma produselor numerelor
din perechile de numere vecine este cel mult 41 .
b) Arătaţi că există o aşezare a numerelor pe cerc astfel ı̂ncât suma produselor
numerelor din perechi de numere vecine să fie cel mult 51 .
Florica Banu

31. Un trapez are diagonalele perpendiculare, ı̂nălţimea de lungime 4 şi una dintre
diagonale are lungimea 5. Aflaţi aria trapezului.
Florica Banu

7.2 Clasa a VIII-a


1. Fie prisma triunghiulară regulată ABCA0 B 0 C 0 şi P ∈ (AC) oarecare. Fie Q ∈
(AP ) astfel ı̂ncât m(∠[(BP B 0 ), (BQB 0 )]). Notăm {T } = (BQB 0 ) ∩ [(AB 0 C 0 ) ∩
(ABC)]. Demonstraţi că S4BP B 0 = S4CP C 0 + S4AT A0 .
***

2. Determinaţi numerele naturale nenule x, y, k, α şi numărul prim p astfel ı̂ncât


xy k − yxk = pα .
Cosmin Manea şi Dragoş Petrică, Piteşti

3. Se consideră cubul ABCDA0 B 0 C 0 D0 , AB = 1 cm şi O1 respectiv O2 centrele feţelor


A0 B 0 C 0 D0 respectiv CDD0 C 0 . Aflaţi aria secţiunii determinate de planul (AO1 O2 )
ı̂n cub.
Manuela Prajea, Drobeta Turnu-Severin
88

4. Se consideră mulţimea cu patru elemente A = {4, 15, 24, m}, m ∈ N. Să se arate
că există n, p ∈ A, n 6= p, astfel ı̂ncât numărul n + p − 3 nu este pătrat perfect.
Lucian Dragomir, Oţelu-Roşu

5. În paralelipipedul dreptunghic ABCDA0 B 0 C 0 D0 , M şi N sunt centrele feţelor


A0 B 0 C 0 D0 respectiv ADD0 A0 . Arătaţi că, dacă AM ⊥ A0 C şi C 0 N ⊥ BD0 , atunci
paralelipipedul este cub.
Petre Bătrâneţu, Galaţi

6. Fie pirmida patrulateră regulată V ABCD şi punctele E, G, F, H pe muchiile (V A),


(V B), (V C) respectiv (V D) astfel ı̂ncât EF ∩ AC = {P } şi GH ∩ BD = {R}.
Paralela prin E la AC intersectează V C ı̂n punctul E1 şi paralela prin H la BD
intersectează V B ı̂n H1 . Paralela prin G la CB intersectează V A ı̂n G1 şi paralela
prin F la CD intersectează V D ı̂n F1 . Notăm cu O punctul de intersecţie ı̂ntre AC
F E1 GH1
şi BD. Demonstraţi că dacă HF1 = EG1 şi = , atunci punctele E, G, F
PO RO
şi H sunt coplanare.
Claudiu Şt. Popa, Iaşi

abcd
7. Determinaţi abcd, a, c 6= 0, pentru care √ √ ∈ Q.
ab + cd
Gheorghe Iurea, Iaşi

8. Fie p, q, x, y numere naturale nenule, cu p, q prime. Să se determine cele patru


numere ştiind că p% din x este egal cu q% din y, iar x + y este (p + q)% din xy.
Gabriel Popa, Iaşi

9. A, B, C, M sunt patru puncte necoplanare. Fie D un punct oarecare pe latura BC


a triunghiului ABC, iar picioarele bisectoarelor unghiurilor ∠ADC şi ∠ADB sunt
E şi respectiv F , unde E ∈ (AC) şi F ∈ (AB). Arătaţi că planele (ADM ), (BEM )
şi (CF M ) au o dreaptă comună.
Dan Zaharia

10. Se consideră un tablou ı̂n formă de pătrat astfel ı̂ncât pe fiecare linie şi fiecare
coloană să avem n căsuţe (n > 2) care se completează cu elemente din mulţimea
1, 2, 3, . . . , 2008. Determinaţi ı̂n câte moduri poate fi completat tabelul astfel ı̂ncât
suma elementelor de pe fiecare linie şi coloană să fie un număr impar.
Marin Ionescu, Piteşti
89

11. Demonstraţi că pentru orice numere reale strict pozitive a, b, c cu abc = 1 avem
inegalitatea:
a2 + b 2 b 2 + c 2 c 2 + a2
+ + 6 a + b + c.
a4 + b 4 b 4 + c 4 c 4 + a4
Gh. F. Molea, Curtea de Argeş

12. Se ı̂mpart laturile pătratului ABCD ı̂n câte 3 segmente congruente prin punctele
E, F, G, H, I, K, L, aşezate ı̂n această ordine. Fie O centrul pătratului şi V O per-
pendiculara pe planul pătratului. Se cunosc ariile A4V GH = 5cm2 şi A4V IH =

4 3cm2 . Determinaţi aria laterală, aria totală şi volumul piramidei [V EF GHIJKL].
Marian Teler, Argeş

13. Fie A, B, C, D, O puncte ı̂n spaţiu astfel ca AB ⊥ CD, AD ⊥ BC, OA ⊥ BD.


Dacă M este proiecţia punctului C pe BD, să se arate că A, O, C, M sunt puncte
coplanare.
Dan Nedeianu, Drobeta Turnu-Severin

7.3 Clasa a IX-a


1. Determinaţi funcţiile f : N → N pentru care

f (x2 + f (y)) = xf (x) + y

pentru orice x, y ∈ N.

Lucian Dragomir

2. Fie a, b ∈ R, a < b numere reale date. Spunem că funcţia f : [a, b) → [a, b) are
proprietatea P, dacă f (x) ∈ {x − 1, x + 1} pentru orice x ∈ [a, b).
(a) Demonstraţi că există o funcţie injectivă cu proprietatea P dacă şi numai dacă
b − a este număr natural par.
(b) Dacă f, g au proprietatea P şi g ◦ f este bijectivă, arătaţi că f = g.

Dana Heuberger

3. Arătaţi că, pentru orice triunghi ascuţitunghic ABC, este adevărată inegalitatea:

X a
≥ 2(a + b + c).
cos A
90

Mircea Becheanu

4. Se consideră n ∈ N, n ≥ 3 şi mulţimile An = {1, 2, . . . , n} şi Fn = {f : An →


{0, 1}}.
(a) Să se arate că pentru niciun k ∈ N∗ , k < n − 1 sau k > 2n − n + 1 nu există
funcţiile distincte f1 , . . . , fk ∈ Fn astfel ı̂ncât funcţia gk : An → R, gk = f1 + f2 + · · · + fk
să fie injectivă.
(b) Să se arate că pentru orice k ∈ {n−1, n, . . . , 2n −n+1} există funcţiile distincte
f1 , . . . , fk ∈ Fn , astfel ı̂ncât funcţia gk : An → R, gk = f1 + f2 + · · · + fk să fie injectivă.
Dana Heuberger
5. Fie ABC un triunghi isoscel (AB = AC), M ∈ (AC) şi N ∈ (AB) astfel ı̂ncât
B ∈ (AN ).
(i) Să se arate că P este mijlocul lui [M N ] dacă şi numai dacă M C = N B.
(ii) Dacă M C = N B, să se determine locul geometric al centrului de greutate al
triunghiului AM N când M descrie (AC).
Dan Ştefan Marinescu şi Viorel Cornea, Hunedoara
6. Se notează cu D, E şi F intersecţiile ı̂nălţimilor din A, B respectiv C ale unui
triunghi ABC cu cercul circumscris acestuia.
Să se arate că dacă triunghiurile BDC, CEA, AF B au ariile sau perimetrele
egale, atunci triunghiul ABC este echilateral.
Lucian Dragomir, Oţelu Roşu
7. Se consideră trapezul ABCD (AB k CD) şi M, N ∈ (CD), picioarele perpendic-
ularelor din A şi respectiv B pe CD. Cercurile C1 (r1 ), C2 (r2 ) ı̂nscrise ı̂n triunghiurile
AM D şi respectiv BN C sunt tangente dreptelor AM şi BN ı̂n punctele P şi respec-
tiv Q. Dacă t este cealaltă tangentă comună exterioară a cercurilor date, să se arate că

P Q k t dacă şi numai dacă sau AB = BC sau M N = 2 r1 r2 .
Nicolae Stănică şi Marius Damian, Brăila
8. Fie ABCD un patrulater, punctele M ∈ [BC] şi N ∈ [AD] astfel ı̂ncât CM BC
+ DN
AD
=1
MP
şi k ∈ (0, 1). Să se afle locul geometric al punctelor P ∈ (M N ) astfel ı̂ncât M N = k când
M şi N sunt variabile.
Dan Ştefan Marinescu şi Viorel Cornea, Hunedoara
9. Pentru o funcţie f notăm cu f (k) (x) = f (f (. . . (f (x)) . . .)) unde compunerea se
efectuează de k ori. Să se afle f : N∗ → N∗ astfel ı̂ncât, pentru orice x ∈ N∗ avem
f (f (x)) (x) = x şi, pentru orice x ≥ 2, dacă f (x) 6= x, există un t ∈ N cu cmmdc(f (t) (x), x) =
1.
91

Ganea Octavian, Bucureşti

10. Fie n ∈ N, n ≥ 2 şi An = {1, 2, . . . , n}. Să se determine numărul funcţiilor


P
f : An → An care au proprietatea n| k f (k).

∗∗∗

11. Determinaţi numerele reale x care pot fi scrise sub forma:

1 a1 an−2 an−1
x= + + ··· + + ,
a1 a2 . . . an a2 a3 . . . an an−1 an an

unde n, a1 , a2 , . . . , an sunt numere naturale nenule, a1 < a2 < . . . an .

Iurea Gheorghe, Iaşi

12. Dacă a2 + b2 + c2 = 3, arătaţi că

a2 b2 c2
b+c
+ c+a
+ a+b
≥ 32 .

Cezar Lupu, student, Bucureşti

13. Fie ABCD un patrulater convex şi fie A0 ∈ (AB), B 0 ∈ (BC), C 0 ∈ (CD) şi
D0 ∈ (DA) astfel ı̂ncât AA0 = CC 0 şi BB 0 = DD0 . Arătaţi că dreapta ce uneşte mi-
jloacele segmentelor (AC 0 ) şi (A0 C) este perpendiculară pe dreapta ce uneşte mijloacele
segmentelor (BD0 ) şi (B 0 D) dacă şi numai dacă ABCD este inscriptibil.

Ganea Octavian

14. (i) Arătaţi că

1 1
xn+1 + ≥ xn + , ∀x > 0, n ∈ N.
xn+1 xn

(ii) Folosind eventual (i) arătaţi că dacă a, b, c > 0 atunci pentru orice n ∈ N,
   
n+1 n+1 n+1 1 1 1 n n n 1 1 1
(a +b +c ) + + ≥ (a + b + c ) + + .
an+1 bn+1 cn+1 an b n c n

Dan Ştefan Marinescu şi Viorel Cornea, Hunedoara

15. Se dau numerele reale x, y, z > 0. Să se demonstreze inegalitatea


92
s
x y z √ 7xyz
+ + ≥ 2· 2− .
y+z z+x x+y (x + y)(y + z)(x + z)

Când are loc egalitatea?

Andrei Ciupan

16. Fie a, b, c ≥ 0 cu proprietatea că a + b + c = 1. Să se arate că:

1 a b c 9
≤ 2 + 2 + 2 ≤ .
3 a +a+1 b +b+1 c +c+1 13

Ion Nedelcu, Piteşti

7.4 Clasa a X-a


1. Fie p1 , p2 , p3 numere prime diferite două câte două şi n ∈ N∗ . Să se determine
numărul funcţiilor f : {1, 2, . . . , n} → {p1 , p2 , p3 } pentru care numărul f (1)f (2) · · · f (n)
este pătrat perfect.

Dorin Andrica şi Mihai Piticari

2. Determinaţi toate perechile (z, n) cu z ∈ C şi n ∈ N astfel ı̂ncât

z + z 2 + · · · + z n = n|z|n .

Dorin Andrica şi Mihai Piticari

3. Fie ABC un triunghi dreptunghic isoscel ı̂n care AB = AC. Să se arate că, oricum
am lua punctele M pe cateta AB şi N pe cateta AC, se poate construi un triunghi cu
segmentele CM , BN , M N .

Mircea Becheanu

4. Determinaţi z ∈ C pentru care funcţia fz : N → N, fz (n) = [|z + n|] este injectivă.

Emil Vasile

5. (a) Să se arate că dacă şirul (xn )n∈N verifică relaţia de recurenţă

xn+1 = −xn + 6xn−1 , ∀n ≥ 1,


93

atunci există A, B ∈ R astfel ca

xn = A · 2n + B · (−3)n , ∀n ∈ N.

(b) Să se determine funcţiile f : (0, ∞) → (0, ∞) care verifică relaţia

f (f (x) − x) = 6x, ∀x ∈ (0, ∞).

Vasile Pop

6. Dacă z ∈ C şi |z| = 1 atunci

n
X hni  h n i
(n − k + 1)|1 + z k | ≥ n− |1 − z|.
k=1
2 2

Mihaly Bencze, Braşov

7. Să se determine toate perechile (z, n) astfel ı̂ncât z + z 2 + · · · + z n = n|z|, unde z


este un număr complex de modul natural şi n este un număr natural nenul.

Dorin Andrica şi Mihai Piticari

8. Fie a1 , a2 , . . . , an numere reale strict pozitive şi distincte cu suma lor egală cu n.
Să se arate că funcţia f : [1, ∞) → R definită prin f (x) = ax1 + ax2 + · · · + axn este strict
crescătoare.

Cezar Lupu, student, Bucureşti

9. Fie x1 , x2 , . . . , xn > 0. Să se arate că:


p

n x x ...x ≤
n
nn−1 x1 x2 . . . xn (x1 + · · · + xn ) x1 + x2 + · · · + xn
1 2 n √n x + ··· + n x
√ ≤ ,
1 n n

cu egalitate dacă şi numai dacă x1 = x2 = · · · = xn . Interpretaţi geometric


rezultatul pentru n = 2.

Lucian Petrescu, Tulcea

10. Să se arate că dacă x, y, z ∈ (0, ∞), atunci:

xy yz zx
p +p +p > 1.
(x2 + z 2 )(y 2 + z 2 ) (y 2 + x2 )(z 2 + x2 ) (z 2 + y 2 )(x2 + y 2 )
94

Robert Sasz

11. Un alfabet conţine a litere. Aflaţi numărul cuvintelor de lungime m, care folos-
esc exact p litere distincte.

Vasile Popa

12. Să se demonstreze că nu există numere naturale n şi k astfel ı̂ncât
√ √
[(2 + 3)2n+1 ] = [(4 + 15)k ].

Manea Cosmin şi Petrică Dragoş, Piteşti

13. Aflaţi toate numerele naturale a pentru care mulţimea

Ma = {f : N → N|f (k) (n) = n + a, ∀n ∈ N}

este nevidă — f (k) denotă compunerea lui f cu ea ı̂nsăşi de k ori.

Dorel Miheţ, Timişoara

14. Să se arate că ı̂n orice triunghi ABC are loc inegalitatea (cu notaţiile cunoscute):

9R2 ≥ a2 + b2 + c2 + (a − b)2 + (b − c)2 + (c − a)2 .

Nica Nicolae şi Nica Cristina, Craiova

15. Fie a > 1. Să se rezolve ı̂n R∗ ecuaţia:


2
ax + (2a + 1) x = (a + 1)2 .

Nedelcu Ion, Ploieşti

16. Fie triunghiul ABC şi punctele distincte M, N ∈ (BC), P, Q ∈ (CA) astfel ı̂ncât
BM
MC
= CP
PA
= k şi BN
NC
= CQ
QA
= t. Să se arate că 4AM N ∼ 4BP Q dacă şi numai dacă
4ABC este echilateral.

Dana Heuberger

17. Fie [AB] un segment, A 6= B, M un punct pe (AB) şi punctele C, D, E ı̂n acelaşi
plan cu A şi B astfel ı̂ncât punctele C şi E, respectiv D şi E sunt de o parte şi de alta a
dreptei AB, iar triunghiurile CAM , DM B, EBA sunt asemenea.
(a) Arătaţi că centrul de greutate G al triunghiului CDE se află pe (AB).
(b) Să se afle locul geometric al lui G când M variază pe [AB].
95

Dan Ştefan Marinescu şi Viorel Cornea, Hunedoara

18. Fie n ∈ N, n ≥ 3 şi poligoanele regulate A1 A2 . . . An , B1 B2 . . . Bn situate ı̂n acelaşi


plan şi având acelaşi centru.
(a) Arătaţi că nk=1 Bi Ak = nk=1 Aj Bk , ∀i, j ∈ {1, 2, . . . , n}.
Q Q

(b) Aflaţi min{M A1 · M A2 · · · M An + M B1 · M B2 · · · M Bn }, când M parcurge


mulţimea punctelor aflate ı̂n planul celor două poligoane.

Dan Ştefan Marinescu şi Viorel Cornea, Hunedoara

19. Fie z1 , z2 , . . . zn ∈ C, n ∈ N, n ≥ 2, numere complexe de modul 1 astfel ı̂ncât


originea O a planului complex se află ı̂n ı̂nfăşurătoarea convexă a punctelor de afixe
z1 , z2 , . . . , zn . Să se arate că singurul punct comun al discurilor D(zk , 1) = {z ∈ C :
|z − zk | ≤ 1}, k ∈ {1, 2, . . . , n} este O.

Dan Ştefan Marinescu şi Viorel Cornea, Hunedoara

20. Să se rezolve ecuaţia exponenţială 2x + 21−x = 3 · ax(x−1) , unde a ≥ 2.

Virgil Nicula

21. Se consideră şirul definit recurent astfel: a0 = 2, a1 = −1, an+2 = −an+1 − an ,


n ∈ N. Să se arate că:
n   n   n  
X n n
X n 2 n n+1
X n −1 (−1)n
ak = (−1) a2n , ak = (−1) an + 2 , ak = an − 2n−1 .
k=0
k k=0
k k=0
k 2

Virgil Nicula

22. Fie OA1 A2 . . . An , unde n ≥ 5 o piramidă având ca bază un poligon inscriptibil.


Un plan α intersectează muchiile laterale OAi ı̂n punctele Bi , i = 1, . . . , n. Demonstraţi
că dacă poligonul B1 B2 . . . Bn este regulat, atunci A1 A2 . . . An este poligon regulat.

Beniamin Bogoşel

23. Fie ABCD un tetraedru cu proprietatea că AB = CD = a, AC = BD = b şi


AD = BC = c. Demonstraţi că următoarele afirmaţii sunt echivalente:
(a) ABCD este tetraedru regulat.
(b) Două perechi de muchii opuse sunt perpendiculare.
(c) V [ABCD] = 6abc √ .
2
(d) Există o sferă cu centrul ı̂n centrul de greutate al tetraedrului care trece prin
mijloacele tuturor muchiilor tetraedrului.

∗∗∗
96

24. Pentru orice numere pozitive n şi k, fie S(n, k) mulţimea tuturor numerelor
pozitive din intervalul [nk , (n + 1)k ]. Demonstrţi că există elementele a1 , a2 , . . . , ak+1 ∈
S(n, k) ı̂n progresie geometrică dacă şi numai dacă ai = nk−i+1 (n+i)i−1 , i ∈ {1, 2, . . . , k+
1} cu a1 < a2 < . . . ak+1 .

Mihaly Bencze, Braşov


Capitolul 8

Probleme avute ı̂n atenţia comisiei, 2009

8.1 Clasa a VII-a


1. Care este numărul tripletelor (a, b, c) de numere naturale pentru care a + b + c =
2009? Dintre acestea găsiţi-le pe acelea pentru care produsul nu este maxim.
Cosmin Manea şi Dragoş Petru

2. Se consideră triunghiul ABC şi fie A1 , B1 , C1 punctele de contact ale cercului


ı̂nscris triunghiului ABC cu laturile BC, CA respectiv AB. Bisectoarea unghi-
ului ∠BAC intersectează cercul circumscris triunghiului ABC ı̂n A2 iar dreapta
A1 A2 intersectează acelaşi cerc ı̂n A3 . Demonstraţi că punctele A, A3 , B1 , C1 sunt
conciclice.
Manuela Prajea

3. Pentru orice punct P din interiorul unui triunghi ABC notăm cu d(P ) suma
distanţelor de la P la laturi. Presupunem că există două puncte M şi N ı̂n in-
teriorul triunghiului ABC astfel ı̂ncât d(M ) şi d(N ) reprezintă lungimile a două
dintre ı̂nălţimile triunghiului. Să se arate că triunghiul ABC este echilateral.
Cristian Manolescu

8.2 Clasa a VIII-a


1. a) Să se arate că pentru orice numere naturale m, n cu m + n 6= 0 numărul
√ √
m 2+n 5
a=
m+n
√ √
este iraţional şi 2 ≤ a ≤ 5.

97
98
" √ √ # "√ √ #
k 2+ 5 2+k 5
b) Să se calculeze şi pentru k ∈ N, unde k reprezintă
1+k 1+k
partea ı̂ntreagă a numărului real a.
Dan Negulescu

2. Se consideră tetraedrul echifacial ABCD cu AB = a, BC = b, CD = c, DA = d,


a+b+c+d a+c+e+f b+d+e+f
AC = e, BD = f . Arătaţi că dacă , şi
e+f b+d a+c
sunt numere naturale, atunci tetraedrul este regulat.
Nicolae Muşuroaia

3. Dacă a, b, c sunt numere reale pozitive cu abc = 1, arătaţi că


X a
≥ 1.
2 + bc
Manuela Prajea

4. Pătratul ABCD şi triunghiul ABE sunt situate ı̂n plane diferite. Dacă EF ⊥
(ABC), F ∈ (AB), BG ⊥ DE, G ∈ (AE), EF ∩ BG = {H}, GM ⊥ DE, M ∈
(DE), AB ∩ BE = {I}, DI ∩ BM = {P }:
a) demonstraţi că AH ⊥ CE;
b) determinaţi ∠((BDE), (EF P ));
c) dacă AE, AB, BE sunt numere naturale consecutive şi EF = 12, aflaţi distanţa
dintre AC şi EP .
Dorina Zaharia

5. Fie n un număr natural nenul şi a1 , a2 , ..., an numere ı̂ntregi astfel ı̂ncât a1 + a2 +
... + an = 15k, k ∈ Z. Arătaţi că a51 + a52 + ... + a5n se divide prin 15.
Marian Teler

8.3 Clasa a IX-a


1. Fie numerele naturale nenule a1 , a2 , .., an+1 şi k, astfel ı̂ncât
a1 ≤ a2 ≤ ... ≤ an+1 .
Demonstraţi inegalitatea
       
a2 − a1 a3 − a2 an+1 − an an+1 − a1
+ + ··· + +n−1≥ .
k k k k

Manea Cosmin, Petrică Dragoş, Piteşti


99

2. Demonstraţi că pentru orice număr real x > 0 şi orice ı̂ntreg n ∈ N∗ are loc
inegalitatea
n √ r
X 2k − 1 n
2
< .
k=1
x + k x

Dan Nedeianu, Dr. Tr. Severin

3. Determinaţi şirurile (an )n≥1 şi (bn )n≥1 , cu an ∈ {−1, 1} şi bn ∈ N, pentru fiecare
n ∈ N∗ , şi cu proprietatea

n(n + 1)
a1 b21 + a2 b22 + ... + an b2n = an
2
pentru orice n ∈ N∗ .
Romanţa şi Ioan Ghiţă, Blaj
1 + an
4. Se consideră un şir (an )n≥1 de numere reale, definit prin an+1 = , pentru
1 + a2n
orice n ∈ N∗ . Arătaţi că dacă a1 ∈ (0, 2), atunci |an+1 − 1| < 21n , pentru orice
n ∈ N∗ .
Lucian Dragomir, Oţelu-Roşu

5. a) Dacă a, b ∈ (0, ∞) şi ab = 1, arătaţi că

a2 b2
+ ≥ 1.
a+1 b+1

b) Fie a1 , a2 , ..., an ∈ (0, ∞), n ∈ N, n ≥ 2, astfel ı̂ncât a1 a2 ...an = 1. Dacă a1 + a2 +


... + an = s, arătaţi că

a21 a22 a2n


+ + ... + ≥ 1.
s + 1 − a1 s + 1 − a2 s + 1 − an

Traian Tămı̂ian, Carei, Satu Mare

8.4 Clasa a X-a


1. Fie a, b ∈ (1, ∞) astfel ı̂ncât a · b = a + b. Să se rezolve ı̂n R ecuaţia:

x[(b − 1)ax + (a − 1)bx − x − ab + 2] = (ax − 1)(bx − 1).

Traian Tămı̂ian, Carei, Satu Mare


100

2. (i) Într-un reper cartezian xOy se consideră punctele distincte

A(cos x, sin x), B(cos y, sin y), C(cos z, sin z), x, y, z ∈ R.

Să se arate că


4S 2 = (sin(x − y) + sin(y − z) + sin(z − x))2 ,
unde S este aria triunghiului ABC.
(ii) Arătaţi că dacă a, b, c ∈ R cu a + b + c = 0, atunci

3 3
| sin a + sin b + sin c| ≤ .
2

Dan Ştefan Marinescu, Viorel Cornea, Hunedoara

3. Să se determine toate numerele naturale strict pozitive n astfel ı̂ncât sistemul de
ecuaţii 
x + y + z = 3


x2 + y 2 + z 2 = 3

xn + y n + z n = 3

să aibă unica soluţie x = y = z = 1 ı̂n mulţimea numerelor complexe.


Cosmin Pohoaţă, CNI Tudor Vianu, Bucureşti
Capitolul 9

Probleme avute ı̂n atenţia comisiei, 2010

9.1 Clasa a VII-a


1. Fie ABCD un paralelogram de arie 1 ı̂n care AD ≤ AB ≤ BD < AC. Arătaţi că

paralelogramul poate fi acoperit cu un dreptunghi de arie 3.
Vasile Pop, Cluj-Napoca

2. Fie un triunghi ABC cu AB ≤ BC. Fie M mijlocul laturii BC, N piciorul bisec-
toarei din B, O intersecţia dreptelor AM şi AB, şi P intersecţia dreptelor CO şi
AB. Arătaţi că 4AP ≤ AB + BC.
Gheorghe Molea

3. Un triunghi are laturile a, b, c şi perimetrul egal cu 3. Arătaţi că:


(a) lungimile ı̂nălţimilor nu sunt exprimate prin numere naturale;
√ √ √
(b) dacă a + b − c + b + c − a + c + a − b = 3, atunci triunghiul este echilat-
eral.
Adriana şi Lucian Dragomir, Oţelu Roşu

9.2 Clasa a VIII-a



1. Tetraedrul ABCD are volumul 12 1
, ∠ABC = 60◦ şi AD + BC + AB 3 = 3.
Calculaţi lungimile muchiilor tetraedrului.
Dan Nedeianu, Drobeta Turnu Severin

2. Fie tetraedrul regulat V ABC de latură a şi punctele M , N pe muchiile V B, V C,


astfel ca V M = a3 şi V N = a6 . Dacă Q este proiecţia lui V pe planul (AM N ), iar O

101
102

este proiecţia lui V pe planul (ABC), calculaţi lungimile segmentelor V O si V Q,


precum şi o funcţie trigonometrică a unghiului dintre dreptele V Q şi V O.
Gheorghe Iurea, Iaşi

3. Fie piramida triunghiulară regulată V ABC cu latura bazei a. Punctele M şi N


sunt mijloacele laturilor AB, respectiv AC, iar O este centrul bazei. Notăm E
intersecţia dreptelor M N şi AO. Ştiind că unghiului dintre planul (DM N ) şi
planul bazei este de 30◦ , calculaţi distanţa de la O la AD şi sinusul unghiului
planelor (BED) şi (CED).
Gheorghe Molea

9.3 Clasa a IX-a


1. Să se arate că numerele reale a, b satisfac relaţia

(x + y + z)3 + a(x + y + z)(xy + yz + zx) + bxyz ≥ 0, ∀x, y, z ≥ 0

dacă şi numai dacă există r ≥ 0, s ≥ 0 astfel ı̂ncât a = −4 + r, b = 9 − 9r + s.


Dumitru Barac, Sibiu

2. Se consideră un triunghi ABC şi un punct M ı̂n interiorul său. Se notează cu


D, E, F proiecţiile lui M pe laturile BC, CA respectiv AB şi cu r lungimea razei
cercului ı̂nscris ı̂n triunghi. Să se arate că dacă
BC CA AB AB + BC + CA
+ + = ,
MD ME MF r
atunci dreptele AD, BE, CF sunt concurente.
Lucian Dragomir, Oţelu-Roşu

3. În interiorul triunghiului ABC, cercul exı̂nscris C(Ia ) corespunzător laturii BC


este tangent la dreptele BC, CA, respectiv AB ı̂n D, E, F . Bisectoarea unghiului
BIa C intersectează BC ı̂n M , iar AM intersectează EF ı̂n P . Să se arate că DP
este bisectoarea unghiului F DE.
Virgil Nicula, Bucureşti

4. Să se arate că pentru orice numere reale x, y, z, t are loc


p p
x2 + xy + y 2 )(z 2 + zt + t2 ) + (y 2 − yz + z 2 )(x2 − xt + t2 ) ≥ (x + z)(y + t).

Să se precizeze ı̂n ce condiţii are loc egalitatea.


Călin Burduşel, Târgovişte
103

5. Să se arate că dacă a, b, c sunt numere reale pozitive cu proprietatea că abc = 1,
atunci  
3 3 3 b c a
(1 + a )(1 + b )(1 + c ) ≥ 2 1 + + + .
a b c

Dan Nedeianu, Dr. Tr. Severin

6. Arătaţi că ı̂n orice triunghi neechilateral există o bisectoare strict mai mare decât
√ √
de 3/2 ori latura opusă şi una strict mai mică decât 3/2 ori latura opusa.
G. René, Otopeni

9.4 Clasa a X-a


1. Dacă a, b, c ∈ C sunt toate de modul 1, arătaţi că

|a − b|2 + |a − c|2 − |b − c|2 ≥ −1.

Dan Nedeianu, Dr. Tr. Severin

2. Fie a, b, c ∈ (0, 1) şi x, y, z ∈ (0, ∞) astfel ı̂ncât a = (bc)x , b = (ca)y , c = (ab)z . Să se
arate că
1 1 1
+ + ≤ 1.
x+y+2 y+z+2 z+x+2
Cezar şi Tudorel Lupu, Constanţa

3. Fie m, p ∈ N mai mari sau egale cu 3. Determinaţi cel mai mic număr natural n,
astfel ı̂ncât orice submulţime cu n elemente a mulţimii {1, 2, 3, ..., pm} să conţină
două numere a căror sumă să se dividă cu p.
Marin Ionescu, Marian Teler
n
4. Considerăm şirul definit prin x1 = 1 şi xn+1 = 1 + , pentru orice n ∈ N∗ .
xn
Determinaţi valorile lui n pentru care xn este număr natural.
Gheorghe Iurea, Iaşi
104
Partea II

Soluţii

105
Capitolul 10

Probleme avute ı̂n atenţia comisiei, 2002

10.1 Clasa a VII-a


1. Soluţiile sunt abc ∈ {203, 255, 426, 501, 500, 581, 828, 930}.
r
x+y
q
2. Să observăm că 0, xx(y) + 0, yy(x) = . Atunci x + y ∈ {1, 9}, de unde
9
obţinem soluţiile (x, y) ∈ {(0, 1), (0, 9), (1, 8), (2, 7), (3, 6), (4, 5)}.

3. Să observăm că 5|81250 − 1 ⇒ 5|32000 − 1 ⇒ 5|32001 − 3, deci 32001 dă restul 3 la
ı̂mpărţirea prin 5, deci şi 52002 · n2002 + 32001 are restul 3 la ı̂mpărţirea prin 5, deci
nu poate fi pătrat perfect.
1 3 2001 3 5 2001
4. Fie b = · · ... · şi c = · · ... · . Să observăm că b < a < c, ab = 1/2001
2 4 2002 4 6 2002
şi ac = 2/2002 = 1/1001, deci 1/2001 < a2 < 1/1001, de unde concluzia.

5. a) Din teorema lui Menelaus ı̂n triunghiul ACCn cu transversala Dn − D − B,


avem
AD CB Cn Dn Cn Dn ACn
· · =1⇒ =n+1⇒ = n + 2.
DC BCn Dn A Dn A ADn

b) Din teorema lui Menelaus ı̂n triunghiul BDn Cn cu transversala A−D−C avem

BD Dn A Cn C
· · = 1.
DDn ACn CB

6. Inecuaţia se scrie (a − 4)x < 2a − 3. Dacă a > 4, aceasta se scrie echivalent x <
2a − 3 2a − 3
, având ca soluţii toate numerele naturale mai mici decât . Dacă a <
a−4 a−4
2a − 3
4, atunci inecuaţia se scrie x > , având ca soluţii toate numerele naturale
a−4
107
108

2a − 3
mai mari decât , deci are o infinitate de soluţii, şi evident numârul maxim
a−4
posibil de soluţii.

7. Ecuaţia se mai scrie :



2
√ 25  
2
√ 25 
2 2x − 5 2x + + 2 3y − 5 3y + =0⇔
4 4
√ 5 √ 5 2 5 5
2 2x − + 2 3y − = 0 ⇔ x = √ ,y = √
2 2 2 2 2 3
8. Vom folosi coordonate. Fie A(xA , yA ), B(xB , yB ), C(xC , yC ). Rezultă imediat că :

M ((1 − k)xA + kxB , (1 − k)yA + kyB )

N ((1 − k)xB + kxC , (1 − k)yB + kyC )


P ((1 − k)xC + kxA , (1 − k)yC + kyA )

De asemenea se arată uşor că centrul de greutate al triunghiului ABC are coor-
donatele : x + x + x y + y + y
A B C A B C
G , )
3 3
Deoarece :

[(1 − k)xA + kxB ] + [(1 − k)xB + kxC ] + [(1 − k)xC + kxA ] = xA + xB + xC

şi

[(1 − k)yA + kyB ] + [(1 − k)yB + kyC ] + [(1 − k)yC + kyA ] = yA + yB + yC

rezultă că ABC şi M N P au acelaşi centru de greutate.

9. Să presupunem mai ı̂ntâi că AC⊥BD. Fie {X} = AC ∩ BD. Atunci din teorema
lui Pitagora rezultă că AB 2 + CD2 = AX 2 + BX 2 + CX 2 + DX 2 = BC 2 + AD2 .
Acum să presupunem că AB 2 +CD2 = BC 2 +AD2 . Fie AX⊥BD, CY ⊥BD, X, Y ∈
(BD). Din teorema lui Pitagora avem AB 2 − AD2 = BX 2 − DX 2 şi CD2 − BC 2 =
BY 2 −DY 2 , deci BX 2 −DX 2 = BY 2 −DY 2 ⇔ BD(BX−DX) = BD(BY −DY ) ⇔
BX = BY ⇔ X = Y , de unde rezultă că AC⊥BD.

10. Vom demonstra că latura maximă a unui astfel de triunghi este 10/ cos 15◦ . Să
demonstrăm mai ı̂ntâi că nu poate fi mai mare de atât. Ducem prin vârfurile
triunghiului paralele la laturile triunghiului astfel ı̂ncât triunghiul să fie “ı̂nscris”
ı̂ntr-un dreptunghi.
109

Cele două unghiuri marcate pe figură au suma 90 − 60 = 30, deci unul din-
tre ele (notat cu x) va fi cel mult 15◦ . Atunci cos x ≥ cos 15. Cum l = L/ cos x
(unde l este latura triunghiului şi L este latura corespunzatoare a dreptunghiu-
lui), l ≤ 10/ cos 15. Este clar că egalitatea o obţinem dacă facem ca dreptunghiul să
coincidă cu pătratul şi ca cele două unghiuri să fie egale. Acest lucru este simplu
de făcut: plecăm de la pătrat şi construim cele două laturi ale triunghiului care au
ca extremitate un vârf al patratului astfel ı̂ncât ele să formeze unghiuri de 15◦ cu
laturile pătratului. Atunci egalitatea este obţinută. Cum 10, 3 < 10/ cos 15 < 10, 4,
problema este rezolvată.

11. Dacă x ≥ 1, atunci 1 − x ≤ 0 şi x2002 > 0, deci 1 = x2002 (1 − x) ≤ 0, fals. Dacă
x ∈ Q, fie x = a/b cu (a, b) = 1. Atunci din enunţ avem a2002 b = a2003 + b2003 , deci
b|a2003 , dar cum (a, b) = 1, rezultă cu b = 1 sau b = −1. Atunci ±a2002 = a2003 ± 1,
deci a|1 şi a = 1 sau a = −1. Rezultă x ∈ {−1, 1}. Acestea două nu sunt soluţii,
deci x ∈ R \ Q.

12. Pentru partea din stânga aducem la acelaşi numitor.

a bc
1< + 2 ⇔ (b + c)a2 < a3 + bc(b + c) ⇔
b+c a

⇔ a3 > b(a2 − c2 ) + c(a2 − b2 ) ⇔ a3 > b3 + c3


Acum vom ridica la pătrat ı̂n scopul de a folosi că a2 = b2 + c2 . Avem:

a3 > b3 + c3 ⇔ a6 > (b3 + c3 )2 ⇔ (b2 + c2 )3 > (b3 + c3 )2 ⇔


2
⇔ b6 + 3b4 c2 + 3b2 c4 + c6 > b6 + 2b3 c3 + c ⇔ b4 c2 + b2 c4 > b3 c3 .
3
2
Însă din inegalitatea mediilor avem b4 c2 + b2 c4 ≥ 2b3 c3 > b3 c3 , ceea ce trebuia să
3
demonstrăm.
Pentru partea a doua notăm x = b/a, y = c/a. Evident, x2 + y 2 = 1. Rescriem
inegalitatea sub forma:

1 1+ 2 2 √ 2 √
+ xy ≤ ⇔ + 1 + 2xy ≤ 2 + 2 ⇔ + (x + y)2 ≤ 2 + 2.
x+y 2 x+y x+y
r
x2 + y 2 √
Fie s = x + y. Din inegalitatea mediilor (M A − M P ), s = x + y ≤ 2 = 2
2
2 √
şi cum x, y < 1, s = x+y > x2 +y 2 = 1. Avem de demonstrat că +s2 ≤ 2+ 2 ⇔
√ s
√ 2 √
( 2 − s)( − s − 2) ≤ 0. Primul factor este evident pozitiv. Să demonstrăm
s
110

2 √ √ √ √ √
că al doilea este negativ. − s − 2 ≤ 0 ⇔ 2 − s2 − 2s ⇔ s2 + 2s ≥ 2,
s
evident adevărat deoarece s > 1.

13. Să observăm mai ı̂ntâi că AM 2 + BM 2 = (2r)2 = AB 2 , deci m(∠AM B) = 90◦ .
Atunci, din teorema lui Van Aubel, M P 2 AB 2 = AM 2 P B 2 +M B 2 AP 2 , deci M P 2 4r2 =

2r(2r − 1)2 + 2r(2r − 1) = 4r2 (2r − 1), de unde M P = 2r − 1 şi cum M P = P1 P2 ,

rezultă că P1 P2 = 2r − 1.

14. Răspuns: 3 · 29 . Bila cu numărul 1 poate fi pusă ı̂n orice urnă. Apoi, fiecare bilă
cu numărul 2, 3, 4, . . . , 8 sau 9 poate fi pusă ı̂n 2 urne.

15. Să observăm că a3 + 3a = b3 + 14b = c3 − 30c. Rezultă că c > a > b. Adunând
relaţiile din enunţ avem a2 +b2 +c2 = ab+bc+ca+13, deci (c−a)2 +(a−b)2 +(c−b)2 =
26. Singura combinaţie posibilă este 1, 3, 4. Cum c − b este cel mai mare, rezultă
că c − b = 4. Acum avem două cazuri. În primul, a − b = 3, c − b = 1, nu avem
soluţii. În al doilea, a − b = 1, c − b = 3, de asemenea nu avem soluţii.

10.2 Clasa a VIII-a


2000
1. Fie x = 55 . Să presupunem că numerele nu sunt prime ı̂ntre ele şi fie p un
divizor prim comun al lor. Atunci p|x + 1|x25 + 1, şi p|x25 + 26 deci p|25 ⇒ p = 5,
imposibil deoarece numerele nu se divid cu 5.
b−c b−c
2. a) Avem că c = ta + (1 − t)b ⇔ t(a − b) + b = c ⇔ t = şi cum ∈ [0, 1],
b−a b−a
problema este rezolvată.
b) O posibilitate chiar mai simplă decât să folosim punctul a) este următoarea:
Fie a + b = c + d = 2x. Atunci există t > s > 0 astfel ı̂ncât a = x + t, b = x − t, c =
x ± s, d = xms. Atunci ab < cd ⇔ x2 − t2 < x2 − s2 , adevărat.

3. Relaţia x + y + z = 3a − 1 devine prin ridicare la pătrat şi utilizarea celei de-a


doua relaţii:
x2 + y 2 + z 2 + 2(3a2 − x − y) = 9a2 − 6a + 1
sau
3(a − 1)2 = (x − 1)2 + (y − 1)2 + z 2 .
Pe de altă parte, din ipoteză, avem (x − 1) + (y − 1) + z = 3a − 3 şi inegalitatea
lui Cauchy implică

9(a − 1)2 = 3[(x − 1)2 + (y − 1)2 + z 2 ] ≥ [(x − 1) + (y − 1) + z]2 = 9(a − 1)2 .

Cum egalitatea are loc doar dacă x − 1 = y − 1 = z rezultă x = a, y = a, z = a − 1.


111

4. Este cunoscută identitatea :

a3 + b3 + c3 − 3abc = (a + b + c)(a2 + b2 + c2 − ab − ac − bc).

Notastfel ı̂ncât nd y = ab + bc + ca, observăm că x ≥ y şi avem :


p √ √
a3 + b3 + c3 − 3abc = x + 2y · x − y · x − y ≤
 √x + 2y + √x − y + √x − y 3  (x + 2y) + (x − y) + (x − y) 3
r

≤ = x x.
3 3
5. Din inegalitatea mediilor avem
 3
a+b+b+c+c+a 8
(a + b)(b + c)(c + a) ≤ = (a + b + c)3 .
3 27
Ramâne de arătat că
 6
8 a+b+c 8
(a + b + c)3 ≤ ⇔ ≤ a + b + c.
27 2 3
8
Dar din inegalitatea mediilor avem (a + b + c)3 ≥ 27abc = 27 ⇒ a + b + c ≥ 3 > .
3
r √
a+1 2 2 2 2
6. Avem = 1+ ≥ 2 > √ . Scriind analoagele şi ı̂nmulţind
a−1 a−1 a√− 1 a
(a + 1)(b + 1)(c + 1) 8·2 2 √
avem > √ = 8 abc > 8, unde am folosit că a, b, c > 1.
(a − 1)(b − 1)(c − 1) abc
a+2
7. 1) Fie = k 2 ⇒ a = 3k 2 − 2. În plus, k trebuie să fie impar. Atunci k ∈ {1, 3}.
3
Rezultă că a ∈ {1, 25}.
2) Să obsevăm că p(b2 ) = p(b2 +1) = . . . = p((b+1)2 −1) = b2 şi p((b+1)2 ) = (b+1)2 .
Atunci 2002 = b2 · 2b + (b + 1)2 , ceea ce nu este posibil pentru b natural.
 
n n(n − 1) n(n − 3)
8. Fie d numărul diagonalelor. Atunci d + n = = ⇒d= .
2 2 2
n(n − 3) n 2
Atunci = ⇒ a = . Aunci n ∈ {4, 5}. Pentru n = 4, a = 2 şi
2 a n−3
pentru n = 5, a = 1.

9. Să notăm cu ABCDA0 B 0 C 0 D0 paralelipipedul dreptunghic din problemă. Atunci


1 1 1 1
AM 2 + M C 02 ≥ AC 02 = , BM 2 + M D02 ≥ BD02 = ,
2 2 2 2
1 1 1 1
CM 2 + M A02 ≥ A0 C 2 = , DM 2 + M B 02 ≥ DB 02 =
2 2 2 2
112

1
(pentru că a2 + b2 ≥ (a + b2 )).
2
Prin adunare avem AM + BM + CM + DM 2 + A0 M 2 + B 0 M 2 + C 0 M 2 +
2 2 2

D0 M 2 ≥ 2, egalitatea atingându-se atunci când M este centru, deci minimul este


2. Pentru maxim lucrăm asemănător. Toate diagonalele mari (de lungime 1) se
văd sub unghiuri obtuze (sau eventual drepte). Dacă ı̂n triunghul OXY avem
m(∠XOY ) ≥ 90◦ , atunci OX 2 + OY 2 ≤ XY 2 . Atunci AM 2 + M C 02 ≤ AC 02 = 1.
Scriind analoagele şi adunând obţinem că AM 2 + BM 2 + CM 2 + DM 2 + A0 M 2 +
B 0 M 2 +C 0 M 2 +D0 M 2 ≤ 4, egalitatea atingându-se când M este unul dintre vâfuri.

10. Trebuie să aplicăm de mai multe ori teorema lui Pitagora. Pentru ı̂nceput, avem
CD2 = AD2 + AC 2 , CE 2 = BE 2 + BC 2 şi CD2 + CE 2 = DE 2 . Atunci AC 2 +
BC 2 + AD2 + BE 2 = DE 2 . Să construim dreptunghiul BADD0 . Atunci DE 2 =
DD02 + D0 E 2 = AB 2 + (AD + BE)2 . Atunci, din aceste relaţii avem AC 2 + BC 2 +
AD2 + BE 2 = AB 2 + (AD + BE)2 = AB 2 + AD2 + BE 2 + 2AD · BE, de unde
rezultă că AC 2 + BC 2 = AB 2 + 2AD · BE, dar AC 2 + BC 2 − AB 2 = 2AC ·
BC cos α, unde α = ∠BCA, deci 2AD · BE = 2AC · BC cos α ⇒ AD · BE = AC ·
BC cos α ⇒ AC 2 · BC 2 (1−sin α2 ) = AD2 ·BE 2 , de unde rezultă că AC ·BC sin α =
√ 1 p
AC 2 BC 2 − AD2 BE 2 , adică Aria(ABC) = · AC 2 · BC 2 − AD2 · BE 2 .
2
11. Din teorema lui Menelaus ı̂n triunghiul AN D cu transversala Q − F − C obţinem
AQ 1 AR 1
că = . Apoi, din teorema lui Ceva ı̂n triunghiul ACD, obţinem = .
QD 2 RC 2
Fie BE ∩ AC = {R0 }. Lucrând analog ı̂n triunghiul ABC obţinem că R = R0 şi
AP 1
= .
PB 2
a) Observăm că P R⊥BC, RQ⊥CD ⇒ (P RQ)⊥(BCD). În particular, 4P RQ ∼
1 1
4BCD şi cum raportul de asemănare este rezultă că raportul ariilor este .
3 9
BD MN BD BD
b) Să observăm că M N = , EF = = , PQ = , deci 12(P Q +
2 2 4 3
EF + M N ) = 13BD.

12. Observăm că m4 − m2 + 1 = (m2 − 1)2 + m2 şi n4 − n2 + 1 = (n2 − 1)2 + n2 , deci


conform identităţii lui Lagrange, expresia din enunţ se scrie ca
2 2
(m4 −m2 +1)(n4 −n2 +1) = (m2 − 1)(n2 − 1) + mn + (m2 − 1)n − (n2 − 1)m .

10.3 Clasa a IX-a


1. Fie s = x + y şi p = xy. Atunci :
x3 − xy + y 3 = (x + y)3 − 3xy(x + y) − xy = s3 − 3sp − p,
113

deci ecuaţia devine :

2s3 − 3s2
2s3 − 6sp + 2p = 3s2 − 6p ⇔ =p∈Z (*)
6s − 4
şi avem :

5s2 5s2 30s2 10s


s2 − ∈Z⇒ ∈Z⇒ ∈Z⇒ ∈ Z ⇒ 3s−2|10(3s−2)+20
6s − 4 6s − 4 6s − 4 3s − 2
deci 3s − 2|20. Din (*) rezultă că s este par. Deducem acum imediat că s ∈
{−6, 0, 2, 4}. Rămâne doar s = 0 sau s = 4. Dacă s = 0, este clar că p = 0 şi
deci x = y = 0. Dacă s = 4, avem şi p = 4 din (*) şi deci x = y = 2. Aşadar
perechile cerute sunt (0, 0) şi (2, 2).

2. Avem că a3 + b3 + c3 = 3abc + (a + b + c)[(a + b + c)2 − 3(ab + bc + ca)] = 3abc + 1 −


3(ab + bc + ca) şi a2 + b2 + c2 = (a + b + c)2 − 2(ab + bc + ca) = 1 − 2(ab + bc + ca)
şi astfel inegalitatea devine

4(ab + bc + ca) ≤ 1 + 9abc ⇔ (1 − 2a)(1 − 2b)(1 − 2c) ≤ abc.

Evident nu se poate ca două dintre numerele a, b, c să fie mai mari ca 1/2. Dacă
ı̂nsă unul din numerele a, b, c este mai mare ca 1/2, atunci inegalitatea de mai
sus este evidentă. Dacă a, b, c ≤ 1/2 atunci putem aplica inegalitatea mediilor
 2
1 − 2a + 1 − 2b
şi obţinem (1 − 2a)(1 − 2b) ≤ = c2 şi analoagele, iar prin
2
ı̂nmulţire şi extragere de radical vom obţine tocmai inegalitatea de demonstrat.

3. Putem presupune că a1 < a2 < . . . < an . Vom demonstra afirmaţia prin inducţie
după n. Pentru n = 1 este clar. Presupunem inegalitatea adevărată pentru n şi
demonstram că este adevărată şi pentru n + 1. Avem aşadar că

(a1 + a2 + . . . + an )2 n3 − n
a21 + a22 + . . . + a2n + a2n+1 ≥ + + a2n+1 ≥
n 12
(a1 + a2 + . . . + an + an+1 )2 (n + 1)3 − (n + 1)
≥ + .
n+1 12
Această inegalitate este echivalentă cu

(a1 + a2 + . . . + an )2 (a1 + a2 + . . . + an + an+1 )2 n(n + 1)


+ a2n+1 ≥ + ⇔
n n+1 4
⇔ (n + 1)(a1 + a2 + . . . + an )2 + n(n + 1)a2n+1 ≥
 2
2 2 n(n + 1)
≥ n(a1 + a2 + . . . + an ) + 2n(a1 + a2 + . . . + an )an+1 + nan+1 + ⇔
2
114
 2
2 n(n + 1)
⇔ (nan+1 − a1 − a2 − . . . − an ) ≥ .
2
Din presupunerea a1 < a2 < . . . < an < an+1 avem că an+1 −ai ≥ n+1−i, i = 1, n,
n(n + 1)
de unde nan+1 − a1 − a2 − . . . − an ≥ , adică tocmai inegalitatea de mai
2
sus.

4. Deoarece avem relaţia cos2 X + cos2 Y + cos2 Z + 2 cos X cos Y cos Z = 1 pentru
orice unghiuri X, Y, Z ∈ (0, π), cu X + Y + Z = π, rezultă că putem găsi
A, B, C ∈ (0, π) cu A + B + C = π astfel ı̂ncât a = cos A, b = cos B, c = cos C. În
aceste condiţii inegalitatea de demonstrat devine

cos A + cos B + cos C


cos A cos B + cos B cos C + cos C cos A ≤ .
2
(x + y + z)2
Dar, din inegalitatea cunoscută xy + yz + zx ≤ , avem că
3
(cos A + cos B + cos C)2
cos A cos B + cos B cos C + cos C cos A ≤ =
3
3
(cos A + cos B + cos C)(cos A + cos B + cos C) (cos A + cos B + cos C)
= ≤ 2 =
3 3
cos A + cos B + cos C
= ,
2
3
unde am folosit cos A + cos B + cos C ≤ .
2
y −→ −−→
5. Soluţia 1: Împărţim relaţia cu x 6= 0 şi notăm t = . Luăm egalitatea |OA+tOB| =
x
−−→ −→ −→ −−→ −→ −−→ −−→
|OB+tOC|. Notăm OB 0 = tOB şi OC 0 = tOC şi apoi OA+tOB = OA+OB 0 = OE
−−→ −→ −−→ −−→ −→
şi analog OB + tOC = OB + OC 0 = OF . Deducem că OE = OF , şi imediat
∆OEB 0 ≡ ∆OF C 0 , de unde ∠EOB = ∠F OC. Rezultă imediat şi că ∠AOE =
∠OF B de unde ∆AEO ≡ ∆BF O, şi deci ∠AOE = ∠BOF . Adunând această
relaţie cu ∠EOB = ∠F OC, obţinem că ∠AOB = ∠BOC, de unde ∆AOB ≡
∆BOC şi ı̂n fine AB = BC. Analog se obţine că AB = BC = CD = DA, deci
patrulaterul inscriptibil ABCD este un pătrat.
Soluţia 2: Avem
−→ −−→ −→ −−→ −→ −−→ −→ −−→
|xOA + y OB|2 = (xOA + y OB) · (xOA + y OB) = x2 |OA|2 + y 2 |OB|+
−→ −→ −→ −−→
+2xy OA · OA = x2 R2 + y 2 R2 + 2xy OA · OB
115

(punctul desemnează produsul scalar, iar R este raza cercului). Prin urmare dacă
ridicăm la pătrat relaţiile din enunţ vom obţine (xy 6= 0)
−→ −−→ −−→ −→ −→ −−→ −−→ −→
OA · OB = OB · OC = OC · OD = OD · OA.

Prima egalitate se mai scrie


−−→ −→ −→ −−→ −→
OB · (OA − OC) = 0 ⇔ OB · CA = 0 ⇔ OB ⊥ AC.

Analog vom obţine OD ⊥ AC, OC ⊥ BD şi OA ⊥ BD. Rezultă că ABCD este un
patrulater ale cărui diagonale sunt perpendiculare şi se intersectează ı̂n centrul
cercului, deci este pătrat.

6. a) Pentru ca o parabolă să intersecteze axele de coordonate ı̂n trei puncte dis-
tincte, trebuie ca ∆ = b2 − 4ac > 0 şi ca cx1 x2 6= 0, unde x1 , x2 sunt cele două
rădăcini reale distincte ale ecuaţiei ax2 + bx + c = 0. Cele trei puncte vor fi
A(x1 , 0), B(x2 , 0), C(0, c).
Pentru ca ∆ABC să fie echilateral trebuie ca AB = BC = CA, sau (x1 − x2 )2 =
x21 + c2 = x22 + c2 . Din ultima relaţie obţinem că x1 = −x2 şi că√
x1 = x2 , imposibil,

−b + ∆ −b − ∆
pentru că ar rezulta ∆ = 0. Aşadar avem x1 = −x2 , sau =− ,
2a 2a
de unde rezultă b = 0. Înlocuind x1 = −x2 ı̂n relaţia (x1 − x2 )2 = x21 + c2 , avem
c
că 3x21 = −3x1 x2 = c2 ⇔ c2 + 3 = 0de unde c = 0 sau ac = −3. Dacă c = 0
a
am obţine cx1 x2 = 0, contradicţie. Aşadar avem că ac = −3 şi ecuaţia generală a
3 3
parabolelor echilatere este y = ax2 − , cu a ∈ R, sau, renotând y = ae x2 − ,
a ae
ae ∈ R.
Pentru parabolele pitagorice avem mai multe cazuri. Dacă AB 2 = BC 2 + CA2 ,
c
atunci obţinem că (x1 − x2 )2 = x21 + x22 + 2c2 ⇔ x1 x2 + c2 = 0 ⇔ c2 + = 0, de unde
a
c = 0 sau ac = −1. Cazul c = 0 nu convine ca mai sus, deci o clasă de parabole
1 1
pitagorice este dată de y = ax2 +bx− , a, b ∈ R, sau, renotând y = ap x2 +bp x− ,
a ap
2 2 2 2 2 2 2 2
ap , bp ∈ R. Dacă avem BC = AB + CA , atunci x2 + c = x1 + c + (x1 − x2 ) ⇔
x1 (x1 − x2 ) = 0. Dar am văzut că nu putem avea nici x1 = 0, nici x1 = x2 . Analog,
dacă CA2 = BC 2 + AB 2 .
b) Evident nu se poate ca cele două puncte să fie ambele pe Ox deoarece ar rezulta
3 1
că ecuaţiile sunt echivalente, ceea ce este imposibil. Deci trebuie ca = , de
ae ap

ae 3
unde ap = . Mai trebuie să mai avem un punct ı̂n comun, deci fie =
p 23 ae
3(−bp + bp + 4)
, de unde rezultă bp . Analog se tratează şi celelalte cazuri.
ae
116

7. Să ı̂ncepem prin a ”elimina” exponenţii, folosind inegalitatea Cauchy - Schwartz:


 5 5 5 2
 √ √ √  2 2
(ab) + (bc) + (ca)
4 4 4 ≤ ab + bc + ca (a b + b2 c2 + c2 a2 ).

Cum
√ √ √ a+b b+c c+a
bc + ca ≤
ab + + + = 1,
2 2 2
tot ce a mai rămas este să arătăm că:
1
a2 b 2 + b 2 c 2 + c 2 a2 < . (*)
16
Vom prezenta două soluţii pentru (*).
Soluţia 1: Să presupunem fără a restrânge generalitatea că c ≥ a ≥ b. Cum c +
1
(a + b) = 1, din inegalitatea mediilor avem (¸a + b) ≤ . Pe de altă parte, cum
4
c ≥ a ≥ b, avem 2abc2 > abc2 ≥ a2 b2 . Deci:
1
≥ c2 (a + b)2 = c2 a2 + c2 b2 + 2abc2 > a2 b2 + b2 c2 + c2 a2 ,
16
şi (*) este demonstrată.
Soluţia 2: Este o soluţie mai complicată. Se poate demonstra exact ca ı̂n soluţia
problemei 3 că avem
9abc + 1
ab + bc + ca ≤ . (1)
4
1
Notăm x = abc. Din inegalitatea mediilor avem x ≤ . Atunci, folosind (1),
27
rezultă că
81x2 + 18x + 1 1
a2 b2 + b2 c2 + c2 a2 = (ab + bc + ca)2 − 2x ≤ − 2x < ,
16 16
1
căci x ≤ .
27

10.4 Clasa a X-a


 z n z 2iπ
n n n
1. Avem că z ∈ R ⇔ z = z ⇔ = 1, de unde = εi , unde εi = cos +
z z n
2iπ 1+z
i sin , cu i ∈ {0, 1, 2, . . . , n − 1}. Analog, din (1 + z)n ∈ R, obţinem = εj ,
n 1+z
1+z z
cu j ∈ {0, 1, 2, . . . , n − 1}. Dacă i = j, atunci = ⇔ z = z, adică z ∈ R,
1+z z
z
contradicţie cu ipoteza, deci i 6= j. Scoţând din relaţia z = εi z pe z = şi
εi
1+z εi εj − εi
introducându-l ı̂n relaţia = εj , obţinem z = , pentru orice i, j ∈
1+z εi − εj
{0, 1, 2, . . . , n − 1} cu i 6= j.
117

2. Dacă un xk = 0, atunci xi = 0, i = 1, n, deci putem considera xi > 0, i = 1, n.


Vom demonstra că (1 + x1 )(1 + x2 ) . . . (1 + xn )(x1 + x2 + . . . + xn ) ≥ 2n2 x1 x2 . . . xn ,
şi vom decide asupra cazului de egalitate. Aplicând inegalitatea lui Huygens

(1 + x1 )(1 + x2 ) . . . (1 + xn ) ≥ (1 + n x1 x2 . . . xn )n şi inegalitatea mediilor x1 + x2 +
√ √
. . . + xn ≥ n n x1 x2 . . . xn , inegalitatea de mai sus devine (1 + n x1 x2 . . . xn )n ·

n n x1 x2 . . . xn ≥ 2n2 x1 x2 . . . xn . Împărţind această inegalitate cu x1 x2 . . . xn şi
1
notând x = √ n x x ...x
, trebuie să mai demonstrăm că (1 + x)n ≥ 2nx ⇔ x + 1 ≥
1 2 n

n 1 1 1
2nx. Dar aplicând inegalitatea mediilor avem că x+ + +. . .+ ≥
s n−1 n−1 n−1
n−1


1 n 1
nn x n−1
≥ 2nx ⇔ 1 + ≥ 2, adevărată conform inegalităţii
(n − 1) n−1
lui Bernoulli. Ultima inegalitate se transformă ı̂n egalitate doar dacă n = 2 şi apoi
obţinem că x1 = x2 = 1.

3. Fie X = {a1 , . . . , an } cu ai ∈ C. Din enunţ rezultă că |XX| ≤ n, ı̂nsă obţinem


imediat |XX| ≥ n ⇒ XX = {a}X. Presupunem că {|ak |} are cel puţin două
elemente. Fie atunci i şi j astfel ı̂ncât |ai | să fie minim şi |aj | să fie maxim. Evident
|ai | < |aj |. Mulţimile {ai }X şi {aj }X au fiecare câte n elemente şi sunt incluse ı̂n
XX, deci {ai }X = {aj }X = XX. Însă |ai |2 ∈ {ai }X şi totuşi |ai |2 este mai mic
strict decât orice element din {ai }X. Contradicţie. Deci toate elementele din X
au acelaşi modul a. Pentru cazul ı̂n care a = 0 ⇒ X = {0}. Presupunem a 6= 0.

Putem presupune (eventual renotând a indicii) că 0 < arg(a1 ) < arg(a2 ) < . . . <
arg(an ) < 2π. Pentru orice i, j ∈ {1, .  avem că {ai }X = {aj }X ⇔ {ai ak |1 ≤
. . , n} 
aj aj
k ≤ n} = {aj ak |1 ≤ k ≤ n} ⇔ X = X. Alegem i şi j astfel ı̂ncât arg
ai ai
să fie minim şi 
pozitiv
 (evidentj =  i + 1; folosim convenţia an+1  = a1 ). Din
aj aj aj
arg(ak+1 ) ≥ arg ak şi X = X ⇒ arg(ak+1 ) = arg ak , dar |ak+1 | =
ai ai ai
aj aj
|ak | = 1 ⇒ ak+1 = ak . De aici rezultă că ak = bεk , unde b ∈ C∗ şi εnk = 1.
ai ai
Din condiţia XX = {a}X rezultă imediat că abε1 ∈ XX, deci abε1 = b2 εk εi ⇒ a =
bεm (ε1 = 1). Rezultatul este că pentru a ∈ C∗ fixat mulţimea X = {aεk }.

4. Luăm originea ı̂n O şi notăm cu A, B, C imaginile ı̂n planul complex ale nu-
merelor a, b, c. Dacă H este ortocentrul triunghiului ABC, avem că h = a + b + c.
OP kh
Fie P un punct pe OH pentru care = k de unde p = , unde p este
PH k+1
1
afixul punctului P . De aici AP = |p − a| = | − a + kb + kc| şi analoagele şi
k+1
118

inegalitaatea de demonstrat devine

AP + BP + CP ≤ 3R.

Dacă definim funcţia f : [OH] → R, f (P ) = AP + BP + CP este suma a trei


funcţii concave, deci este concavă de unde deducem că ı̂şi atinge maximul ı̂n
capetele intervalului [OH]. Dar

AO + BO + CO = 3R

şi
3
AH + BH + CH = 2R(cos A + cos B + cos C) ≤ 2R · = 3R,
2
şi deci f (P ) ≤ 3R, ∀P ∈ [OH].

5. Se poate demonstra prin inducţie că pentru orice m ∈ N∗ şi x, y, z ∈ (0, ∞) are loc
inegalitatea
m−1
2m 2m 2m (x2 + y 2 + z 2 )2
x +y +z ≥ ,
32m−1 −1
care se poate deduce şi din inegalitatea lui Hőlder. Aplicând această inegalitate
m−1
S22
pentru x = |a + b|, y = |b + c|, z = |c + a|, obţinem că S2m ≥ 2m−1 −1 . Avem acum
3
identitatea |a + b|2 + |b + c|2 + |c + a|2 = |a|2 + |b|2 + |c|2 + |a + b + c|2 , care se
verifică uşor pentru orice a, b, c ∈ C. Dar din ipoteză avem că |a| = |b| = |c| = 1,
deci S2 = |a + b|2 + |b + c|2 + |c + a|2 = 3 + |a + b + c|2 ≥ 3. Am obţinut că
m−1
S22
S2 ≥ 3, care ı̂mpreună cu inegalitatea S2m ≥ 2m−1 −1
, ne dau S2m ≥ 3, ∀m ∈ N∗ ,
3
cu egalitate doar pentru a + b + c = 0. Dacă există p ∈ N∗ pentru care S2p ≤ 3,
rezultă că S2p = 3, deci se realizează egalitatea, de unde a + b + c = 0. Dar acum
Sn = |a + b|n + |b + c|n + |c + a|n = |a|n + |b|n + |c|n = 3, ∀n ∈ N∗ .

6. Vom folosi următoarea lemă:


Fie triunghiul XY Z şi T ∈ (Y Z). Fie Y1 ∈ (XY ), Z1 ∈ (XZ) şi T1 ∈ (XT ). Fie y =
Y Y1 ZZ1 T T1
, z= şi t = . Atunci Y1 , Z1 şi T1 sunt coliniare ⇔ tY Z = yT Z + zT Y.
XY1 XZ 0 XT1
Demonstraţia lemei: Presupunem {T1 } = Y1 Z1 ∩ XT. Este destul să demonstrăm
TZ YT 1
că tY Z = yT Z + zT Y. Avem XT = XY + XZ şi XT1 = XT =
YZ YZ t+1
TZ YT (y + 1)T Z (z + 1)Y T
XY + XZ = XY1 + XZ1 Însă XT1 =
(t + 1)Y Z (t + 1)Y Z (t + 1)Y Z (t + 1)Y Z
T1 Z1 Y1 T1 (y + 1)T Z
XY1 + XZ1 , care ı̂mpreună cu relaţia anterioară ne dau =
Y1 Z1 Y1 Z 1 (t + 1)Y Z
T1 Z1 (z + 1)Y T Y1 T1 (y + 1)T Z (z + 1)Y T
şi = ⇒ + = 1 ⇒ tY Z = yT Z + zT Y .
Y1 Z1 (t + 1)Y Z Y1 Z 1 (t + 1)Y Z (t + 1)Y Z
119

Revenind la problemă, presupunem că K ∈ (M N P ). Fie {A1 } = AG ∪ BC şi


A1 A2
{A2 } = DA1 ∪ N P . Aplicând lema ı̂n triunghiul 4BCD obţinem BC = =
A2D
CP BN A1 A2 CP BN
BA1 + CA1 ⇒ 2 = + = n + p. Aplicând acum lema ı̂n
PD ND A2 D PD ND
GK A1 A2 AM A1 A2
triunghiul 4DAA1 obţinem AA1 = AG + GA1 ⇒ 3k = 2 +
KD A2 D MD A2 D
AM
= m + n + p.
MD
(m + n + p)
Cum K este determinant unic rezultă că valoarea lui k = se obţine
3
numai pentru K ∈ (M N P ).

7. Scriem relaţia de recurenţă sub forma a2n+1 = 6−2a2n . Vom căuta să găsim expresia
generală a lui an . Avem că:

a2n = 6 − 2a2n−1
a2n−1 = 6 − 2a2n−2 · (−2)
..
.
a23 = 6 − 2a22 · (−2)n−3
a22 = 6 − 2a21 · (−2)n−2

Adunând aceste relaţii obţinem că a2n = 6(1 + (−2) + (−2)2 + . . . + (−2)n−2 ) +
(−2)n−1 a21 = (−2)n +2+(−2)n−1 a21 = (−2)n−1 (a21 −2)+2. Acum, dacă presupunem
că a21 > 2, vom obţine pentru un n suficient de mare că a22n+1 = −22n+1 (a21 −2)+2 <
0, absurd. Analog obţinem o contradicţie dacă a21 < 2. Avem aşadar că a21 = 2,

de unde a2n = 2, ∀n ∈ N∗ , de unde obţinem că an = 2, ∀n ∈ N∗ , deoarece
an > 0, ∀n ∈ N∗ .

8. Inegalitatea dată se mai scrie (1 − ab − ab + 1) + (1 − bc − bc + 1) + (1 − ca − ca + 1) >


8 ⇔ 1 > 3 + ab + ab + bc + bc + ca + ca ⇔ 1 > (a + b + c)(a + b + c) ⇔ 1 > |a + b + c|.
Dacă vom considera punctele A(a), B(b), C(c), atunci numărul complex a + b + c
reprezintă ortocentrul H al triunghiului ABC. Condiţia 1 > |a + b + c| spune
că ortocentrul se află ı̂n interiorul cercului circumscris triunghiului, deci ABC
este ascuţitunghic. Fie acum M mijlocul segmentului [BC]. Avem că |b + c| =
2OM = 2R cos A = 2 cos A şi analoagele şi inegalitatea de demonstrat devine
1
cos A cos B cos C ≤ , fapt cunoscut pentru triunghiul ascuţitunghic.
8
9. Dacă notăm a, b, c lungimile laturilor triunghiului ABC, obţinem imediat din
relaţiile din ipoteză că BA1 = B1 A = p − c, CA1 = AC1 = p − b, BC1 =
CB1 = p − a. Notând acum cu zA , zB , zC afixele vârfurilor triunghiului ABC, şi
120

cu zA1 , zB1 , zC1 afixele punctelor corespunzătoare obţinem că

(p − b)zB + (p − c)zC (p − c)zC + (p − a)zA (p − a)zA + (p − b)zB


zA1 = , zB1 = , zC1 = .
a b c

Acum avem că


1
SA1 B1 C1 = Im(zA1 zB1 + zB1 zC1 + zC1 zA1 ) =
2
1 X  (p − b)zB + (p − c)zC (p − c)zC + (p − a)zA 
= Im · =
2 a b
1 X
= Im ((p − b)(p − c)(b + c)zB zC + a(p − b)(p − c)zB zC ) =
2abc
1 X X 
= Im 2(p − a)(p − b)(p − c)zB zC + a(p − b)(p − c)(zB zC + zB zC ) =
2abc
2(p − a)(p − b)(p − c) 1 2(p − a)(p − b)(p − c)
= · Im(zA zB +zB zC +zC zA ) = SABC =
abc 2 abc
(a + b − c)(b + c − a)(c + a − b) SABC
= SABC ≤ ,
4abc 4
unde am folosit, pe rând faptul că |z| ∈ R, ∀z ∈ C, xy + yx ∈ R, ∀x, y ∈ C şi ı̂n
fine inegalitatea cunoscută (a + b − c)(b + c − a)(c + a − b) ≤ abc.
Capitolul 11

Probleme avute ı̂n atenţia comisiei, 2003

11.1 Clasa a VII-a


1. Deoarece x3 + y 3 = (x + y)(x2 + y 2 − xy), x + y|x3 + y 3 deci x + y|2x+y , de unde
x + y = 2k , x2 + y 2 − xy = 1 + 2x+y−k , şi 22k − 3xy = 1 + 2x+y−k , adică 22k − 3xy − 1 =
2x+y−k , ceea ce ı̂nseamnă că 3|2x+y−k , deoarece 3|22k − 1, fals.
a+1 b+2 c+3 3
2. Fie = = . Atunci ∈ N ⇒ k ∈ {1, 3}. Dacă k = 1, a = b = c = 1,
2 3 4 k
fals. Dacă k = 3, obţinem soluţia a = 5, b = 7, c = 9.

3. Fie Aj Aj = 100 (unitatea de măsură este cm). Atunci Ai Aj + Ai+1 Aj+2 + . . . +


Ai−1 Aj = 100, de unde (i + 1) + (i + 2) + . . . + j = 100 ⇒ (j − i)(i + j + 1) =
200. Pentru a reduce din numărul de cazuri trebuie să ţinem cont de faptul că
0 < j − i < j + i + 1 ≤ 50 şi că j − i şi j + i + 1 au parităţi diferite. Avem cazurile
Cazul I. j − i = 8 şi j + i + 1 = 25. Rezultă că j = 16 şi i = 8.
Cazul II. j − i = 5 şi j + i + 1 = 40. Rezultă că j = 22 şi i = 17.

4. Fie a ∈ Z un număr cu proprietatea din enunţ şi x o soluţie ı̂ntreagă a inecuaţiei


√ √ √ √
|x − 2| + |x − 3| < a cu proprietatea că x ≤ 1. Atunci avem 2 + 3 − 2x < a ⇔
3−a 3−a
3 − 2x ≤ a ⇔ x ≥ , deci ≤ x ≤ 1. Cum există cel mult două numere
2 2
3−a
ı̂ntregi x cu această proprietate, rezultă că > −a ⇒ a ≤ 4. Cum inecuaţia
2
are soluţii, rezultă că a > 0, deci a ≥ 1. Analizând cazurile a ∈ {1, 2, 3, 4} obţinem
singura soluţie a = 2.

5. Dacă t = 0, rezultă xy + yz + zx = 0, deci două dintre numerele x, y, z sunt nule.


Dacă t ≥ 1, atunci xya(x + y + z) ≤ xy + yz + zx. Notăm xy = a, yz = b, zx = c.
Atunci ab + bc + ca ≤ a + b + c. Dacă unul dintre numerele a, b, c este nul, atunci

121
122

şi unul dintre numerele x, y, z va fi nul. Rezultă că xy + yz + zx = 0, deci două


dintre numerele x, y, z vor fi nule. Dacă a, b, c ≥ 1, atunci a + b + c = ab + bc + ca ≥
a + b + c, deci inegalităţile devin egalităţi, de unde a = b = c = 1. Obţinem
soluţiile (x, y, z, t) ∈ {(1, 1, 1, 1), (u, 0, 0, v), (0, u, 0, v), (0, 0, u, v)}.

6. Avem n = 50(7a + b) = k 2 , de unde 50|k 2 ⇒ 10|k. Însă k 2 = 50(7a + b) < 502 , deci
k < 50. Avem cazurile k ∈ {10, 20, 30, 40}.
Cazul I. k = 10. Atunci 7a + b = 2, deci a = 0, b = 2.
Cazul II. k = 20. Atunci 7a + b = 2, deci a = 1, b = 1.
Cazul III. k = 30. Atunci 7a + b = 18, deci a = 2, b = 4.
Cazul VI. k = 40. Atunci 7a + b = 32, deci a = 4, b = 4.
b
7. Dacă a > 0, pentru d suficient de mare ecuaţia ax+ +c = d ⇔ ax2 +(c−d)x+b = 0
x
are soluţie reală nenulă. Alegând d neı̂ntreg, obţinem o contradicţie. Dacă a < 0,
b
analog. Rezultă că a = 0. Dacă b 6= 0, fie d 6= c, d 6= e. Atunci ecuaţia + c = d are
x
soluţie reală nenulă, contradicţie. Rezultă b = 0 şi alegând x real nenul oarecare
obţinem şi c ∈ Z.
1 1 1
8. Evident −3 ≤ + + ≤ 3. Deoarece această sumă este nenulă avem 6 cazuri
x y z
posibile:
1 1 1
Cazul I. + + = 3. Atunci x = y = z = 1.
x y z
1 1 1 1 1
Cazul II. + + = 2. Să presupunem x ≥ y ≥ z. Dacă z < 0, rezultă + > 2,
x y z x y
imposibil, deci z > 0. Dacă z = 1, rezultă uşor că x = y = 2. Dacă z ≥ 2, atunci
1 1 1 3
2 = + + ≤ , imposibil.
x y z 2
1 1 1 1 1
Cazul III. + + = 1. Dacă z < 0, rezultă + > 1, deci x = 1 sau y = 1,
x y z x y
1 1
dar x ≥ y, deci y = 1. Atunci + = 0 ⇒ x = −z. Rămâne z ≥ 1. Dacă
x z
1 1 1 1 1
z = 1, atunci + = 0, imposibil. Dacă z = 2, atunci + = . Rezultă
x y x y 2
xy = 2x + 2y ⇔ (x − 2)(y − 2) = 4 şi cum x ≥ y, rezultă (x, y) ∈ {(4, 4), (6, 3)}.
1 1 1 1 1 1
Dacă z ≥ 3, rezultă 1 = + + ≥ + + = 1, deci toate inegalităţile devin
x y z 3 3 3
egalităţi, deci x = y = z = 3.
1 1 1
Cazul IV. + + = −1. Se ı̂nmulţeşte cu −1 şi se reduce la Cazul III.
x y z
123

1 1 1
Cazul V. + + = −2. Se ı̂nmulţeşte cu −1 şi se reduce la Cazul II.
x y z
1 1 1
Cazul VI. + + = −3. Se ı̂nmulţeşte cu −1 şi se reduce la Cazul I.
x y z
s 2  2
24 5 5
9. Observăm că A = 7n + − < 7n + = 7n, 025. Pe de altă
200 200 200
 2  2
2 35 24 3
parte A > 7n, 024 deoarece (7n) + n > 7n + = 7n + =
100 1000 125
42n 9
(7n)2 + + , adevărat. Rezultă că primele 3 zecimale după virgulă ale lui
125 15625
A sunt 024.

10. Din condiţie rezultă că b + c|2bc. Analizând toate cazurile obţinem soluţiile:
abc ∈ {111, 222, 326, 333, 436, 444, 555, 362, 463, 666, 777, 888, 999}

11. Fie P şi Q proiecţiile punctului N pe AB şi AC. Atunci BP = BD, DC = CQ şi
AP = AQ, deci AB + BD = AC + CD.

12. Fie M intersecţia dreptelor AB şi CD şi P simetricul lui A faţă de M . Din puterea
punctului faţă de cerc, M C 2 = M A · M B = M D2 , deci M C = M D şi M C ·
M D = AM · BM = BM · P M , deci BCP D este inscriptibil, de unde 180◦ −
∠BCD = ∠DP C = ∠DAC (1). Pe de altă parte unghiurile O1 OO2 şi CAD au
laturile perpendiculare, deci 180◦ − ∠O1 OO2 = ∠DAC (2). Din (1) şi (2) rezultă
că ∠BCD = ∠O1 OO2 .

13. Fie a lungimea laturii pătratului. Concluzia este echivalentă cu AM + CN > a.


Însă AM = a · tg(90 − 2x) şi CN = a · tg x, unde x = m(CDN
\ ). Atunci concluzia
este echivalentă cu tg x + tg(90◦ − 2x) > 1(x < 45◦ ), adevărat.

14. Notăm {K} = DC ∩ BE. Perpendicularele din A pe CD şi BE intersectează


CD, BE, BE, CD ı̂n P, M, Q, respectiv N . Rezultă că patrulaterele AP BD
şi AQCE sunt inscriptibile, deci ∠BP D = 45◦ şi ∠CQE = 45◦ , ⇒ ∠BP C =
∠BQC = 135◦ ⇒ patrulaterul BP QC este inscriptibil, deci ∠KP Q = ∠KBC.
Pe de altă parte ı̂n triunghiul AM N, M Q şi N P sunt ı̂nălţimi, deci patrulaterul
M N P Q este inscriptibil ⇒ ∠KP Q = ∠KM N (2) şi AK ⊥ M N (3). Din (1) şi
(2) rezultă că M N ||BC şi combinat cu (3) rezultă că AK ⊥ BC, ceea ce trebuia să
demonstăm.
BN BC CP BC BN + CP BC BC
15. Să observăm că = şi = , deci = + ≥
AM BM AM CM AM BM MC
4BC 1 1 4
= 4 (am folosit + ≥ ). Într-adevăr, acesta este minimul
BM + M C x y x+y
deoarece valoarea 4 se poate atinge când M este mijlocul laturii BC.
124

11.2 Clasa a VIII-a


1. Din inegalitatea Cauchy-Schwartz avem:

(a + b)3 (b + c)3 (c + a)3


 
+ + ((a + b)c + (b + c)a + (c + a)b) ≥
c a b

≥ ((a + b)2 + (b + c)2 + (c + a)2 )2 .


2
Rămâne să demonstrăm că ((a + b)2 + (b + c)2 + (c + a)2 ) ≥ 8(a2 +b2 +c2 ) ((a + b)c + (b + c)a +
echivalent cu (a2 + b2 + c2 + ab + bc + ca)2 ≥ 4(a2 + b2 + c2 )(ab + bc + ca), care
rezultă din inegalitatea mediilor.

.
2. Mai ı̂ntâi să observăm că a7 + b7 = (a + b)(a6 − a5 b + . . . + b6 )..a + b. Fie S =
.
17 +27 +37 +. . .+10007 . Atunci 2S = (17 +10007 )+(27 +9997 )+. . .+(10007 +17 )..1001.
Pe de altă parte, mai putem scrie că 2S = (17 + 9997 ) + (27 + 9987 ) + . . . + (9997 +
.
17 ) + 2 · 10007 ..1000. Rezultă că 1001000|2S ⇒ 500500|S.

p−1 k(p − 1)
3. Avem p{x} = x + [x] ⇔ p(x − [x]) = x + [x] ⇔ x = [x]. Atunci x =
p+1 p
cu k ı̂ntreg. Toate numere de această formă verifică.
X X  X
4. a) Din inegalitatea lui Cebâşev 3 a2003 ≥ a3 a2000 . Pe de altă parte,
X 2
X X X a1000 X
observăm că a3 ≥ 3abc şi a2000 = (a1000 )2 ≥ , deci 3 a2003 ≥
X 2 3
abc a1000 .
16
X 3k ≥ 3X⇒ k 
b) Dacă a = b = c = 1, rezultă 315 . Să arătăm
≥X 15
X că k = 3 verifică.
Ca la punctul a), avem 3 a2003 ≥ a3 a2000 şi a3 ≥ 3abc. Rămâne
X X 16
să demonstrăm că 315 a2000 ≥ a125 . Notăm a125 = x, b125 = y, c125 = z.
X X 16
Mai trebuie să demonstrăm că 315 x16 ≥ x . Cea mai simplă variantă
este să aplicăm direct inegalitatea lui Hőlder. Dacă nuX ştiţi inegalitatea X
lui Holder,
15 16 14
aplicaţi Cauchy-Buniakovski de mai multe ori: 3 x = 3 ·3 (x8 )2 ≥
X 2 X 16
314 x8 ≥ . . . ≥ x .

5. Fie O centrul rombului şi M mijlocul segmentului A0 C 0 . Condiţia se scrie ∠BM D =


∠A0 OC 0 ⇒ ∠OA0 M = ∠BM O. Atunci 4A0 M O ≡ 4BOM (C.U.) ⇒ A0 M =
BO ⇒ AO = BO, deci ABCD este pătrat.
125

6. Această problemă este o aplicaţie a problemei biliardului. Să presupunem că A


şi B se află ı̂n primul cadran. Fie B 0 şi B 00 simetricele punctului B faţă de axele
Ox şi Oy. Din inegalitatea triunghiului rezultă că min (M A + M B) = AB 00 şi
M ∈Oy
0
min (N A + N B) = AB . Condiţia este ı̂ndeplinită dacă şi numai dacă A aparţine
N ∈Ox
mediatoarei segmentului BB 0 , echivalent cu AO⊥BB 0 ⇔ ∠AOy ≡ ∠BOx ⇔
a d
tg∠AOy = tg ∠BOx. Aceasta se scrie echivalent = ⇔ ac = bd.
b c
7. a) Să lucrăm mai ı̂ntâi ı̂n planul (ABC). Să observăm că M este centrul de greu-
tate al triunghiului ABE. Atunci AB = 6M N = 2N E ⇒ m(∠AEB) = 90◦ .
Cum AN = N E = N B, rezultă că N este proiecţia lui V pe planul (ABC). Cum
d(V, AE) = d(V, BE), rezultă că d(N, AE) = d(N, BE), deci triunghiul ABE este
isoscel cu AE = BE (deci chiar dreptunghic isoscel). În particular, rezultă că
N E⊥AB, deci (V N E)⊥AB, de unde V E⊥AB.
b) Teoremă. În orice tetraedru V ABC cu V A⊥V B⊥V C⊥V A, este valabilă relaţia:

σ(4V AB)2 + σ(4V BC)2 + σ(4V CA)2 = σ(4ABC)2 .

Demonstraţie. Se găseşte ı̂n orice manual de clasa a VIII-a.


Revenind la problemă, să observăm că tetraedrul N V CD are proprietatea de mai
sus, deci concluzia este demonstrată.

11.3 Clasa a IX-a


1. Folosind inegalitatea Cauchy - Schwartz, obţinem:
 a2 b2 c 2  a2 b 2 c 2
+ + (c + a + b) ≥ (a + b + c)2 , de unde + + ≥ a + b + c. (1)
c a b c a b
Din ipoteză, ab + bc + ca ≥ a2 c + b2 a + c2 b. De aici şi din inegalitatea Cauchy -
Schwartz, rezultă
√ √ √ √ √ √ 2
(ab + ac + bc)2 = a2 c c + b 2 a a + c 2 b b ≤

≤ (a2 c + b2 a + c2 b)(a + b + c) ≤ (ab + bc + ca)(a + b + c),


de unde

ab + bc + ca ≥ a + b + c. (2)
Din (1) şi (2), totul este clar.
126

2. Scriem inegalitatea a doua sub forma


r r r
1 1 1
x − + y − + z − ≥ 3.
3 3 3
Folosind acum inegalitatea mediilor avem că
s  s  s 
1 1 1 x+y+z+2 4+2
3≤ x− ·1+ y− ·1+ z− ·1≤ ≤ = 3,
3 3 3 2 2

4
deci are loc egalitatea ı̂n inegalitatea mediilor, de unde x = y = z = .
3
 
16 16 48x
3. Dacă x < 0, atunci x + ≤ −8 şi deci x + ≤ −8. Dar 2 ≥ −8 şi
  x x x +9
48x 16 48x
deci 2 ≥ −8, deci unica posibilitate este x + = 2 = −8, de unde
x +9  x  x +9
16 16 48x
x = −4. Dacă x > 0 atunci x + ≥ 8 de unde x + ≥ 8 şi 2 ≤ 8, deci
  x x x +9
48x 16 48x
2
≤ 8 şi din nou singura posibilitate este x + = 2 = 8, de unde
x +9 x x +9
x = 4. Aşadar soluţiile ecuaţiei sunt x ∈ {−4, 4}.

4. Soluţia 1: Scriem sistemul sub forma


2003


 x2 = 2004 −

 x1
2003


2004 −


 x3 =

 x2
..

. .
2003


xn = 2004 −





 xn−1

 2003
 x1 =
 2004 −
xn
2003
Considerăm acum funcţia f : R → R, f (x) = 2004 − . Sistemul se reduce
x
la ecuaţii de forma f (n) (xi ) = xi , ∀i = 1, n, unde am notat f (n) compusa de n ori
a lui f . Dar funcţia f este crescătoare, deci dacă am presupune f (xi ) > xi , am
obţine imediat f (n) (xi ) > xi , contradicţie. Analog obţinem o contradicţie dacă
presupunem f (xi ) < xi . Aşadar f (xi ) = xi , ∀i = 1, n, de unde xi = 1, ∀i = 1, n,
sau xi = 2003, ∀i = 1, n.
Soluţia 2: Fie ai = xi − 1, i = 1, n şi p = 2003. Avem
2003 pai
xi+1 + = 2004 ⇔ ai+1 = .
xi ai + 1
127

Deci
pa1 pa2 p 2 a1
a2 = , a3 = = .
a1 + 1 a2 + 1 (p + 1)a1 + 1
Prin inducţie obţinem imediat că

pk−1 a1
ak = , k = 2, n
(1 + p + . . . + pk−2 )a1 + 1

(convenim ca an+1 = a1 ). Deci avem

pn−1 a1
a1 = ⇔ a1 ∈ {0, p − 1}.
(1 + p + . . . + pn−2 )a1 + 1

Dacă a1 = 0 ⇒ ai = 0, i = 1, n. Deci ı̂n acest caz obţinem soluţia x1 = . . . = xn =


1. Dacă a1 = p − 1, atunci

pk−1 (p − 1)
ak = = p − 1, k = 2, n.
(1 + p + . . . + pk−2 )(p − 1) + 1

Deci obţinem soluţia x1 = . . . = xn = 2003. Aşadar sistemul are două soluţii.


n
[  
5. Presupunem că Ai = R/Q, deci evident ai ∈ R/Q i = 1, n . Cum an −
i=1
a1 , . . . , an − an−1 ∈ R/Q, deducem că an − a1 ∈ Ai1 , . . . , an − an−1 ∈ Ain−1 ,
unde i1 , . . . , in−1 ∈ {1, 2, . . . , n}. Dacă avem ij = ik pentru indicii j, k, atunci
(an − aj − aij ) − (an − ak − aik ) ∈ Q, de unde

aj − ak ∈ Q. (1)

Cum diferenţa oricăror două numere distincte din secvenţa a1 , . . . , an este număr
iraţional, rezultă că j = k. Deci i1 6= . . . 6= in−1 6= i1 . Cum ai ∈ R/Q, este clar că
ij 6= n pentru j = 1, n − 1. Aşadar {i1 , . . . , in−1 } = {1, 2, . . . , n − 1} şi deci

n−1
X
(an − aj − aij ) ∈ Q ⇒ (n − 1)an − 2(a1 + . . . + an−1 ) ∈ Q ⇒
j=1

⇒ (n + 1)an − 2(a1 + . . . + an ) ∈ Q.

Analog obţinem
(n + 1)a1 − 2(a1 + . . . + an ) ∈ Q,

deci an − a1 ∈ Q, fals.
128

6. Dacă n = 1, cel mai mare poate fi pe rând oricare dintre cele două numere. Pentru
n ≥ 2, vom demonstra că
a b c 3
+ + ≤ .
na + b nb + c nc + a n+1
Dacă ı̂nmulţim inegalitatea cu n şi scădem 3 din ambii membrii, aceasta devine
b c a 3
+ + ≥ .
na + b nb + c nc + a n+1
Dacă amplificăm fiecare raport cu b, c şi respectiv a şi aplicăm inegalitatea Cauchy-
Schwartz avem că
b2 c2 a2 (a + b + c)2 3
2
+ 2
+ 2
≥ 2 2 2
≥ .
nab + b nbc + c nca + a n(ab + bc + ca) + a + b + c n+1
Ultima inegalitate este echivalentă cu

(n + 1)(a2 + b2 + c2 ) + (2n + 2)(ab + bc + ca) ≥ 3n(ab + bc + ca) + 3(a2 + b2 + c2 ) ⇔

⇔ (n − 2)(a2 + b2 + c2 − ab + bc + ca) ≥ 0,
adevărată.

7. Aplicând inegalitatea mediilor, avem că


2
(m + n)2 (x + y)2

mx + ny + my + nx
(mx + ny)(my + nx) ≤ = ,
2 4
de unde
X x4 X 4x4 1
≥ 2 2
≥ ,
(mx + ny)(my + nx) (m + n) (x + y) 3(m + n)2
X  x2  2 1
deci trebuie să demonstrăm că ≥ . Acum aplicând inegalitatea
x+y 12
Cauchy-Schwartz avem că
X 2 2
x
X  x 2 2 x+y 1 X x2 1
≥ ≥ ⇒ ≥ .
x+y 3 12 x+y 2
Această ultimă inegalitate este adevărată pentru că, dacă aplicăm din nou inegal-
itatea Cauchy-Schwartz avem că
X x2 (x + y + z)2 1
≥ = .
x+y 2(x + y + z) 2
129

8. Aplicând inegalitatea Cauchy-Schwartz, avem că

n
!2 n
!2 n
X X X
n
xi xi xi
X x2i
≥ n i=1 n = i=1
n = i=1
.
i=1
xi + yi X X X 2
xi + yi 2 xi
i=1 i=1 i=1

9. Ridicând la pătrat expresia din enunţ obţinem că


r

q
x + x + x + x + x2 + x + . . . + x2 + x < x2 + 2x + 1 ⇔
2 2

s r

q
⇔ x2 +x+ x2 + x + x2 + x + . . . + x2 + x < x + 1,

de unde se vede că inegalitatea este aceeaşi pentru orice n ∈ N∗ , şi ı̂n particular şi

pentru n = 1, deci inegalitatea de demonstrat devine x2 + x < x + 1 ⇔ x2 + x <
x2 + 2x + 1 ⇔ x + 1 > 0, adevărată deoarece x > 0.

10. Din condiţiile de existenţă ale radicalilor rezultă că numerele x, y, z au acelaşi
semn. Presupunem mai ı̂ntâi că x, y, z < 0. Atunci dacă notăm a = −x, b =
−y, c = −z, a, b, c > 0, sistemul devine:
 √ 6 √
 2√bc − √a = 13
3

2√
6
ca − 3 b = 13 .
 6 √
2 ab − 3 c = 13

Scăzând ecuaţiile sistemului două câte două, obţinem că


 √ 6 √ √6 √ √
 (√b − √c)( √b + √c + 2 √a) = 0
6 6 6

(6c− √ 6
a)( 6 c + √
6
a + 2 6 b) = 0 .
 √ √ √
( 6 a − 6 b)( 6 a + 6 b + 2 6 c) = 0

Singura posibilitate este a = b = c de unde obţinem soluţia x = y = z = −133 .


Dacă x, y, z > 0, scăzând la fel ca mai sus ecuaţiile sistemului, obţinem că
 √ √ √ √ √
6 y − 6 z)( 6 y + 6 z − 2 6 x) = 0
 (√ √ √ √ √
( 6 z − 6 x)( 6 z + 6 x − 2 6 y) = 0 .
 √ √ √ √ √
( 6 x − 6 y)( 6 x + 6 y − 2 6 z) = 0
 3
13
Şi acum singura posibilitate este ca x = y = z, de unde x = y = z = .
3
130

11. Fie (a, b, c, d) o 4 - uplă pentru care se atinge maximul (acesta există, căci avem cel
mult (n−1)4 soluţii ale ecuaţiei a+b+c+d = n). Dacă a > 1, 4 - upla (a−1, b, c+1, d)
verifică (a − 1) + b + (c + 1) + d = n şi (a − 1)b + b(c + 1) + (c + 1)d = ab + bc +
cd + d > ab + bc + cd, fals. Deci a = 1. Dacă d > 1 obţinem analog contradicţie
cu maximalitatea 4-uplei (a, b, c, d), considerând 4 - upla (a, b + 1, c, d − 1). Deci
a = d = 1 şi avem b + c = n − 2 şi ab + bc + cd = n − 2 + bc. Dar maximul lui bc
n 2
se atinge pentru b = c sau |b − c| = 1. Deci maximul lui bc este − 1 pentru n
(n − 3)(n − 1) n 2 2
par şi pentru n impar. Răspunsul este − 1 + n − 2 pentru n
2 2
(n − 3)(n − 1)
par şi + n − 2 pentru n impar.
2
12. Din f (x + 2003) ≤ f (x) + 2003 rezultă prin inducţie că f (x + 2003n) ≤ f (x) +
2003n pentru x ı̂ntreg şi n natural. Din f (x + 1987) ≥ f (x) + 1987 obţinem prin
inducţie după n că f (x + 1987n) ≥ f (x) + 1987n pentru x ı̂ntreg şi n natural. Cum
(2003, 1987) = 1, rezultă că există a, b naturale astfel ı̂ncât 2003a = 1987b + 1. Fie
x ı̂ntreg. Avem
f (x) + 2003a ≥ f (x + 2003a) = f (x + 1 + 1987b) ≥ f (x + 1) + 1987b,
deci f (x + 1) ≤ f (x) + 1. Aşadar f (x + 1) ≤ f (x) + 1 pentru x ı̂ntreg. Deci
f (x) + 1987 ≤ f (x + 1987) ≤ f (x + 1986) + 1 ≤ . . . ≤ f (x) + 1987.
Avem egalitate dacă şi numai dacă f (x + 1) = f (x) + 1 pentru x ı̂ntreg. De aici
rezultă imediat că f (n) = f (0) + n pentru n ∈ Z. Deci soluţiile sunt de forma
f (x) = x + a, a ∈ Z.

13. Desigur, dacă toate numerele sunt ≥ 1, sau toate sunt ≤ 1, datorită condiţiei
abc = 1, rezultă că ele sunt chiar egale cu 1, iar inegalitatea se verifică banal ca
egalitate. Ne rămâne aşadar să considerăm doar cazul când două dintre numere
sunt ≤ 1 şi al treilea este ≥ 1 (sau invers); graţie simetriei putem presupune că
a şi b sunt numerele la fel situate faţă de 1, c fiind de cealaltă parte a sa. Atunci
1 1 1
se verifică uşor că inegalitatea ab + + 2 ≥ a + + b + este adevărată, fiind
ab a b
echivalentă cu (ab + 1) (a − 1) (b − 1) ≥ 0. Prin urmare (ţinând seama de ipoteza
abc = 1) are loc ab + c + 2 ≥ a + b + ac + bc. Acum mai trebuie doar să adunăm pe
aceasta cu inegalităţile simple a2 +b2 ≥ 2ab şi c2 +1 ≥ 2c şi ajungem la inegalitatea
din enunţ.
BA1 CB1 AC1
14. Notăm = = = k. Dacă aplicăm teorema lui Menelaos ı̂n tri-
A1 C B1 A C1 B
unghiul ABB1 , obţinem că
BA2 AC1 B1 C BA2 k+1
· · =1⇔ = ,
A2 B1 C1 B AC A2 B1 k2
131

de unde
−−→ k2 −−→ k + 1 −−→
OA2 = 2
OB + 2 OB1 , pentru orice punct O din plan.
k +k+1 k +k+1
−−→ 1 −→ k −→
Dar acum avem că OB1 = OC + OA, pentru orice punct O din plan,
k+1 k+1
care ı̂mpreună cu relaţia de mai sus ne dau

−−→ k2 −−→ k −→ 1 −→
OA2 = 2
OB + 2 OA + 2 OC,
k +k+1 k +k+1 k +k+1
−−→ k2 −→ k −−→ 1 −→
şi analog obţinem şi că OB2 =2
OC + 2 OB + 2 OA, şi
k +k+1 k +k+1 k +k+1
−−→ k2 −→ k −→ 1 −−→
OC2 = 2 OA + 2 OC + 2 OB.
k +k+1 k +k+1 k +k+1
−−→ −−→ −−→ −→ −−→ −→
a) Acum rezultă imediat că OA2 + OB2 + OC2 = OA+ OB+ OC, de unde rezultă că
−−→ −→
3OG2 = 3OG ⇔ G = G2 , unde G şi G2 sunt centrele de greutate ale triunghiurilor
ABC şi respectiv A2 B2 C2 .
−−→ −−→ −−→ k2 −→ 1 −→ k2 −−→
b) Avem că AA2 + BB2 + CC2 = 2
AB + 2
AC + 2
BC +
k +k+1 k +k+1 k +k+1
2 2
1 −→ k −→ 1 −−→ k −1  −→ −−→ −→
BA+ 2 CA+ 2 CB = 2 AB + BC + CA =
k2 + k + 1 k +k+1 k +k+1 k +k+1

− −−→ −−→ −−→
0 , de unde rezultă că vectorii AA2 , BB2 , CC2 pot fi laturile unui triunghi.
DP
15. Fie = k şi atunci avem că
PE
−→ 1 −−→ k −−→
OP = OD + OE.
1+k k+1
Din ipoteză avem acum că

−→ −−→ −−→ −→ 1 −−→ k −−→ −−→


−−→ OA + OD + OE −−→ OA + 1 + k OD + k + 1 OE + OB
OG1 = , OG2 = ,
3 3

−→ −−→ −→ −→ 1 −−→ k −−→ −→


−−→ OA + OB + OC −−→ OA + OD + OE + OC
OG3 = , OG4 = 1 + k k + 1 ,
3 3
pentru orice punct O din plan. Condiţia ca G1 G2 G3 G4 să fie paralelogram este
−−→ −−→ −−→ −−→ −−→ −−→
ca OG1 + OG3 = OG2 + OG4 , sau, folosind relaţiile de mai sus, OD + OE =
2 −−→ 2k −−→ −−→
OD + OE, de unde rezultă imediat că k = 1, deoarece vectorii OD şi
1+k k+1
−−→
OE nu sunt coliniari. Aşadar G1 G2 G3 G4 este paralelogram dacă şi numai dacă P
este mijlocul segmentului [DE].
132

11.4 Clasa a X-a


1. Fie xi , i = 1, 2003 rădăcinile polinomului. Avem, din relaţiile lui Viéte, că

2003 2003
X X 1
xi = −a2002 , x1 x2 . . . xn = a1 , x1 x2 . . . xn = −a0 .
i=1 i=1 i
x

2003 2003
X X 1
xi + x1 x2 . . . xn
a2002 − a1 i=1 i=1 i
x
Atunci ∈R⇔ ∈ R. Dar acum
1 − a0 1 + x1 x2 . . . xn
2003 2003 2003 2003
X X 1 X 1 1 X
x i + x 1 x 2 . . . xn + xi
i=1 i=1 i
x i=1 i
x x1 x2 . . . xn i=1
= =
1 + x1 x2 . . . xn 1
1+
x 1 x 2 . . . xn

2003 2003
X X 1
x i + x 1 x 2 . . . xn
i=1 i=1 i
x
= ,
1 + x1 x2 . . . xn
1
unde am folosit xi = , ∀i = 1, 2003, deoarece |xi | = 1, ∀i = 1, 2003. Deci
xi
a2002 − a1
∈ R.
1 − a0

2. Se observă imediat că A1 nu are nici un element. Pentru n ≥ 2, scriem ecuaţia sub
forma  x  x  x  x
1 2 n n+2
+ + ... + =1+ .
n+1 n+1 n+1 n+1
Studiind acum graficele funcţiilor exponenţiale de bază subunitară şi respec-
tiv de bază supraunitară, observăm că o ecuaţie de forma f (x) = g(x), unde
f este o sumă de funcţii exponenţiale de bază subunitară şi g este o sumă de
funcţii exponenţiale de bază  supraunitară,
x  are o  singură soluţie reală
x pozitivă.
x 
1 2 n
Aşadar, dacă alegem f (x) = + + ... + şi g(x) =
 x n + 1 n + 1 n + 1
n+2
1+ , obţinem că mulţimile An , n ≥ 2 au un singur element. Deoarece
n+1
am stabilit că A1 este vidă, rezultă că B are 2001 elemente.
b
x
3. Se poate demonstra că funcţia f : (0, ∞)
√ → R, f (x) = a + a , unde
x
√ a > 1 şi
b > 0 este strict descrescătoare pe (0, b) şi strict crescătoare pe ( b, ∞). Mai
133
 
b
mult, avem şi f (x) = f . Din aceste observaţii rezultă că o ecuaţie de forma
x
b
f (x) = c are ori două soluţii x0 şi , ori nici o soluţie.
x0
În cazul nostru, vom lua a = 5, b = log5 9 şi se observă imediat că soluţiile ecuaţiei
log5 9 log5 9
5x + 5 x = 28 sunt x0 = 2 şi = log5 3. Aşadar avem că x ∈ {2, log5 3}.
2
4. Pentru x < 0, avem că [2x ] = [3x ] = [6x ] = 0.
Observăm că x = 0 şi x = 1 nu sunt soluţii.
Pentru x > 1, [6x ] − [2x ] − [3x ] = (2x − 1)(3x − 1) + [6x ] − 6x + 2x − [2x ] + 3x − [3x ] > 1,
unde am folosit (2x − 1)(3x − 1) > (2 − 1)(3 − 1) = 2, [6x ] + 1 > 6x , 3x ≥ [3x ] şi
2x ≥ [2x ]. Aşadar nu avem soluţii.
Dacă x ∈ (0, 1), atunci [2x ] = 1 şi ecuaţia devine 1 + [3x ] = [6x ]. Avem că [3x ] ∈
{1, 2}. Dacă [3x ] = 1, atunci trebuie ca [6x ] = 2. Din cele două condiţii, obţinem
că x ∈ [0, log3 2) ∩ [log6 2, log6 3) = [log6 2, log6 3), deoarece log3 2 > log6 3. Analog
dacă [3x ] = 2, atunci [6x ] = 3 şi x ∈ [log3 2, 1) ∩ [log6 3, log6 4) = [log3 2, log6 4).
Aşadar soluţia finală este x ∈ (−∞, 0) ∪ [log6 2, log6 3) ∪ [log3 2, log6 4).

5. Împărţind ecuaţia prin 4x 8x , se obţine:


 x  x  x   x  x  x  x 
2 3 4 5 6 7 8
+ + + + + =
4 4 4 8 8 8 8
 x  x   x  x  x 
4 5 8 9 10
= + + + .
4 4 8 8 8
Partea dreaptă este un produs de două funcţii strict descrescătoare şi pozitive,
deci este o funcţie strict descrescătoare, pe când partea stângă este o funcţie strict
crescătoare. Aşadar ecuaţia are soluţie unică, şi aceasta este cuprinsă ı̂n intervalul
(0, 1).
p(p + 1) (p + 1)(p + 2)
6. Observăm că m + p + 1 = +p+1 = ⇒ şirul este crescător.
2∗ 2
Fie (bn )n∈N∗ bp = a p(p+1) , ∀p ∈ N . Evident bp+1 − bp = p(p + 1). (sunt (p + 1) diferite
2
p(p + 1)(p + 2)
de p). De aici se arată uşor prin inducţie că bp+1 = + 1. (1)
3
k(k + 1) (k + 1)(k + 2)
Pentru un n ∈ N∗ oarecare, fie k ∈ N∗ astfel ı̂ncât ≤n≤ .
2 2
Un astfel de k există ı̂ntotdeauna. Cum şirul (an )n∈N∗ , este strict crescător ⇒ bk ≤
an < bn+1 .
 
k(k + 1)
Din cele de mai sus rezultă că an = bk + (an − a k(k+1) ) = bk + k n − . (2)
2 2
134
 
(n − 1)n(n + 1) k(k + 1)
Din (1) şi (2) ⇒ an = +1+k n− .
3 2
a) Pentru n = 2003 ⇒ k = 62 ⇒
 
2002 · 2003 · 2004 62 · 63
⇒ a2003 = + 1 + 62 2003 − = 2678687109.
3 2
b) a153 = b17 = 1938 şi a157 = 2003.

7. Vom demonstra că x2n + yn2 + xn yn = 3n−1 , ∀n ∈ N, n ≥ 2. Pentru aceasta, fie ε ∈


C\R, astfel ı̂ncât ε3 = −1. Notăm zn = C1n − C4n + C7n − . . .. Dacă dezvoltăm după
formula binomului lui Newton şi ţinem cont că ε3 = −1 observăm că: (1 + ε)n =
xn + εzn + ε2 yn şi că (1 − ε2 )n = xn − ε2 zn − εyn , ∀n ∈ N, n ≥ 2. În contiunare
vom demonstra că xn + yn = zn ⇔ C0n − C3n + C6n − . . . + C2n − C5n + C8n + . . . =
C1n − C4n + C7n − . . . ⇔ C0n + C2n + C6n + . . . = C1n + C3n + C5n . . ., evident adevărată.
Acum (1+ε)n (1−ε2 )n = (xn +εzn +ε2 )(xn −ε2 zn −εyn ) ⇔ 3n = x2n +yn2 −xn yn +zn2 +
zn xn +zn yn (1), unde am folosit relaţia ε2 −ε+1 = 0 şi respectiv ε3 = −1. Înlocuind
zn = xn + yn ı̂n relaţia (1) obţinem tocmai că x2n + yn2 + xn yn = 3n−1 , ∀n ∈ N, n ≥ 2.
3
Dar acum (x2n + yn2 ) ≥ x2n + yn2 + xn yn = 3n−1 ⇔ x2n + yn2 ≥ 2 · 3n−2 , ∀n ∈ N, n ≥ 2.
2
8. Aplicând inegalitatea Cauchy-Schwartz, obţinem că
X 2 X 2
X log2 c logab c logab c 9
ab
≥ = ≥ ,
a+b 2(a + b + c) 2 8
X 3 X log c 3
deci mai rămâne să demonstrăm că logab c ≥ ⇔ ≥ , unde
2 log a + log b 2
logritmii se iau ı̂ntr-o bază subunitară oarecare. Ultima inegalitate este acum
evidentă dacă luăm x = log a, y = log b şi z = log c ı̂n inegalitatea cunoscută
X z 3
≥ .
x+y 2
9. Evident ecuaţia nu are soluţii negative, deci vom presupune că x > 0. Aplicând
de două ori inegalitatea mediilor obţinem că
q √
1 1 1
p4 3+ 1 + 1 + 1 4 4 3 1 1 1

x3
4 34 x · x · x · x
x 4
3 + 3x + 3x + 3x 4 3 x x x 4 34
4= ≥ = = = 4,
3 3 3 3
deci se realizează egalitatea ı̂n inegalitatea mediilor, de unde x = 1 este unica
soluţie a ecuaţiei.

10. Să presupunem că z > 1. Din ecuaţia a doua obţinem că y > 1 şi din a treia că
x > 1. Avem acum că 7x + 7y + 7z > 21, contradicţie cu prima ecuaţie. Analog
obţinem o contradicţie dacă presupunem că z < 1. Aşadar z = 1 şi x = y = z = 1
este unica soluţie a sistemului.
135

11. Soluţia 1: Deoarece A ∪ B = M şi A ∩ B = ∅, avem că B = M \A. În timp ce


A parcurge mulţimea submulţimilor lui M , M \A parcurge aceeaşi mulţime, deci
suma de calculat devine X
2 m(A).
A⊆M

Evident, A = ∅ are ca element maxim pe 0. Pentru un k ∈ {1, 2, . . . , n}, căutăm


numărul de submulţimi ale lui M care au ca element maxim pe k. Se observă
imediat că numărul submulţimilor de i, i = 1, k − 1 elemente ale lui M care au
ca element maxim pe k este Cik−1 , adică toate submulţimile de i elemente ale
mulţimii {1, 2, . . . , k−1}, cărora li se adaugă elementul {k}. Aşadar numărul total
al submulţimilor mulţimii M care au ca element maxim pe k este 2k−1 . Deducem
de aici că suma de calculat devine
n
X n · 2n+2 − (n + 1) · 2n+1 + 2
k · 2k = 2
= (n − 1) · 2n+1 + 2.
k=1
(2 − 1)

Soluţia 2: Observăm că dacă A ∪ B = M,XA ∩ B = ∅ atunci m(A) = n sau


m(B) = n. Deci suma este egală cu 2n + 2 (n + m(B)), suma luându-se după
A,B
perechile (A, B) cu A ∪ B = M, A ∩ B = ∅, m(A) = n, B 6= ∅. Dar, dacă notăm
cu xi numărul de perechi (A, B) cu proprietăţile:
a) m(B) = i, m(A) = n;
b) A ∪ B = M, A ∩ B = ∅;
n−1
X
suma va fi egală cu (n + i)xi . Să calculăm xi . Numerele 1, 2, . . . , i − 1 se pot
i=1
alege ı̂n 2i−1 moduri şi aceste alegeri determină perechea (A, B). Deci xi = 2i−1 ,
adică numărul cerut este
n−1
X
2n + 2 (n + i)2i−1 = (n − 1)2n+1 + 2.
i=1

12. Notăm x = a3 , y = b3 , z = c3 şi problema devine x, y, z > 0, x + y + z = 1 şi


X √
3 √ X
log √
3x 3 3 yz > 0 ⇔ logx 3yz > 0.

În continuare vom demonstra că 3yz < 1. Presupunem prin absurd că am avea
1
yz > . Dar acum, din inegalitatea mediilor, rezultă că:
3
r
y+z √ 1 2 2
≥ yz > ⇔ 1 − x > √ ⇔ x < 1 − √ < 0,
2 3 3 3
fals, deci 3yz < 1 de unde logx 3yz > 0 şi prin ı̂nsumare obţinem inegalitatea
dorită.
136

13. Să presupunem că x ≥ y ≥ z. Avem ı̂n acest caz că logy z ≤ 1, logx y ≤ 1 şi
logz x ≥ 1. De aici obţinem că x + y + z = xlogy z + y logz x + z logx y ≤ x + z + y logz x ⇔
y ≤ y logz x ⇔ logz x ≥ 1, care ı̂mpreună cu presupunerea făcută ne dau x = z şi
apoi x = y = z. Aşadar soluţia este (x, y, z) = (t, t, t), cu t ∈ (1, ∞) oarecare.
z+z
14. Aplicând formula Re(z) = , inegalitatea devine:
2
2(|z1 |2 + |z1 |2 + |z1 |2 ) ≥ z1 z2 + z2 z3 + z3 z1 + z1 z2 + z2 z3 + z3 z1 ⇔

⇔ |z1 − z2 |2 + |z2 − z3 |2 + |z3 − z1 |2 ≥ 0.


Egalitatea are loc dacă z1 = z2 , z2 = z3 , z3 = z1 , adică z1 = z2 = z3 ∈ R.

15. Este evident că x = 0 şi x = 1 sunt soluţii. Vom demonstra că sunt unice. Pentru
x < 0 nu avem soluţii, deci putem presupune x > 0. Fie f (x) = 6x − 5x . Dacă
x ∈ (1, ∞), atunci f (x2 ) > f (x), deoarece f este strict crescătoare pe (0, ∞). Dar
acum avem că 6x − 5x > 5x − 4x , ∀x > 1, deci ecuaţia nu are soluţii pentru x > 1.
Dacă x < 1, atunci f (x2 ) < f (x), şi din 6x − 5x < 5x − 4x , ∀x < 1, obţinem că
ecuaţia nu are soluţii nici pentru x < 1. Aşadar x = 0 şi x = 1 sunt singurele
soluţii.
Mai trebuie să demonstrăm că 6x −5x > 5x −4x , ∀x > 1 şi 6x −5x < 5x −4x , ∀x < 1.
Funcţia g : (0, ∞) → R, g(t) = tx este convexă pentru x > 1 şi  concavă
x pentru
ax + b x a+b
x < 1. Aşadar, pentru a, b pozitive oarecare, avem că ≥ , pentru
x 2 2
ax + b x

a+b
x > 1, şi ≤ , pentru x < 1, cu egalitate doar pentru a = b sau
2 2
x = 1 sau x = 0. Dacă luăm a = 6 şi b = 4 ı̂n inegalităţile de mai sus obţinem
tocmai inegalităţile dorite.

2 −1 + 7 i
16. a) Din ecuaţia caracteristică x +x+2 = 0, obţinem soluţiile x1 = , x2 =
√ 2
−1 − 7 i
, şi apoi folosind a1 = 1 şi a2 = −1, rezultă imediat că
2
√ !n √ !n !
i −1 − 7 i −1 + 7 i
xn = √ − .
7 2 2
√ √
b) Avem că (−1 + 7 i)n = an + 7 i bn , unde an , bn ∈ Z. Se poate demonstra!prin
√ n
−1 + 7 i
inducţie că 2n an şi 2n bn , deci vor exista cn , dn ∈ Z, astfel ı̂ncât =
2
√ !n
√ −1 − 7 i √
cn + 7 i dn , de unde avem şi = cn − 7 i dn , deci xn = 2dn ∈
2
137
√ !n √ !n
−1 + 7 i −1 − 7 i
Z. Se observă acum că dacă luăm yn = + , atunci
2 2
yn = 2cn ∈ Z şi se verifică uşor că yn2 + 7x2n = 2n+2 , pentru orice n ∈ N. De aici
c2n +7d2n = 2n , ∀n ∈ N. Aşadar soluţiile ecuaţiei date sunt (x, y) = (cn , dn ), ∀n ∈ N.
n#
"
n#
"

2
X 2
X
17. Avem că (Ckn − Ck−1 2
n ) = (Ckn )2 + (Ck−1 2 k k−1
n ) − 2 Cn Cn .
k=0 k=0

Cazul 1: Pentru n = 2t avem ı̂n continuare

n#
"

2
X n
X t
X
(Ckn )2 + (Ck−1
n )
2
− 2 Ckn Cnk−1 = (Ckn )2 − 2 Ckn Cn−k+1
n =
k=0 k=0 k=0
n
X n
X
= Ckn Cn−k
n − Ckn Cnn−k+1 = Cn2n − Cn+1
2n .
k=0 k=0

Cazul 2: Pentru n = 2t + 1 avem ı̂n continuare

n#
"

2
X n
X t
X
(Ckn )2 + (Ck−1 k k−1
n ) − 2 Cn Cn = (Ckn )2 − (Ct+1 2
2t+1 ) − 2 Ckn Cn−k+1
n =
k=0 k=0 k=0

n n
!
X X
Ckn Cn−k
n −(Ct+1 2
2t+1 ) − Ckn Cn+1−k
n − Ct+1 t+1
2t+1 C2t+1 = Cn2n − Cn+1
2n .
k=0 k=1

Deci
n#
"

2
X 1
(Ckn − Ck−1 2 n n+1
n ) = C2n − C2n = Cn .
k=0
n + 1 2n

18. Soluţia 1: Ştim că dacă scrierea ı̂n baza 10 a unui număr A este a1 a2 . . . an atunci
A ≡ an − an+1 + . . . + (−1)n+1 a1 (mod 11).
Fie M = {n|n are 10n
! + 1 cifre şi nu conţine cifra 0 ı̂n reprezentarea ı̂n baza 10}.
X X
Atunci S = m mod 11 = (m mod 11) = s1 − s2 + . . . + s10n+1 = s1 , unde
m∈M m∈M
si este suma cifrelor de pe poziţia i (de la dreapta la stânga) a numărului m, când
138

m parcurge M . Evident s1 = s2 = . . . = s10n+1 = (1 + 2 + . . . + 9)910n ≡ 45(910 )n ≡


1(mod 11). Deci restul lui S la 11 este 1.
Soluţia 2: Considerăm perechile ordonate de numere de 10n + 1 cifre (x, y) asrfel
ı̂ncât dacă x are pe poziţia i (de la dreapta la stânga) cifra i atunci m (ı̂ntre 1 şi
9) are pe poziţia i cifra 10 − m. Pentru o pereche (x, y) avem x + y = 1 . . . 10 ≡
910n+1 − 1
−1(mod 11). Există perechi de numere căci numărul 55 . . . 5 ≡ 5(mod 11)
2
este singurul care nu are pereche.
910n+1 − 1 9−1
Deci restul lui S la 11 este (−1) + 5 = (−1) = −4 + 5 = 1. Putem
2 2
efectua ı̂mpărţirea la 2 deoarece (11, 2) = 1.
   
z2 + z3 s
19. a) Notăm s = z1 +z2 +z3 şi observăm că A =A − 1 şi analoagele.
z1 z1
s s s
Dacă notăm m = , n = , p = , triunghiul M N P , M (m), N (n), P (p), va fi
z1 z2 z3
translatatul triunghiului ABC de număr complex 1. Dacă notăm cu O respectiv
K centrele cercurilor circumscrise celor două triunghiuri, avem că k = o − 1. Dar
s s
acum |m| = = = |n|, şi analog |n| = |p|, deci |m| = |n| = |p|, şi apoi k = 0
z1 z2
de unde o = −1. Aşadar afixul centrului cercului circumscris triunghiului ABC
este −1.
   
s s s
b) Pentru ca ABC să fie echilateral trebuie ca −1+ − 1 ε+ − 1 ε2 = 0
    z 1 z2 z 3
s s 2 s
sau − 1 + −1 ε + − 1 ε = 0, unde ε3 = 1, ε ∈ C \ R. Vom trata doar
z1 z2 z3 
ε2
  
s s s 1 ε
primul caz. Avem că −1+ −1 ε+ − 1 ε2 = 0 ⇔ + + =
z1 z2 z3 z1 z2 z3
0 ⇔ z1 + z2 ε2 + z3 ε = 0, unde am folosit 1 + ε + ε2 = 0, |z1 | = |z2 | = |z3 | şi
s 6= 0. Aşadar triunghiul ABC este echilateral dacă şi numai dacă triunghiul cu
vârfurile de afixe z1 , z2 , z3 este echilateral.

20. Fie B 0 şi respectiv C 0 proiecţiile punctului M pe laturile CA respectiv AB. Avem
că dc = ta sin M \ AC 0 şi db = ta sin M\ AB 0 , de unde db + dc = ta (sin M \ AC 0 +
sin M\ AB 0 ). Dar acum, aplicând inegalitatea lui Jensen pentru funcţia concavă f :
h πi M
\ AC 0 + M
\ AB 0
0, → [0, 1], f (x) = sin x, obţinem că sin M
\ AC 0 +sin M
\ AB 0 ≤ 2 sin =
2 2
A A
2 sin . Aceasta, ı̂mpreună cu relaţia de mai sus ne dau db + dc ≤ 2ta sin , care
2 2
ı̂mpreună cu analoagele ne dau inegalitatea de demonstrat.

21. Considerăm mulţimile :


M1 = {z ∈ {z1 , z2 , . . . , zn }| Re(z) ≥ 0, Im(z) ≥ 0}
139

M2 = {z ∈ {z1 , z2 , . . . , zn }| Re(z) ≥ 0, Im(z) < 0}


M3 = {z ∈ {z1 , z2 , . . . , zn }| Re(z) < 0, Im(z) ≥ 0}
M4 = {z ∈ {z1 , z2 , . . . , zn }| Re(z) < 0, Im(z) < 0}.
X
Observăm imediat că dacă notăm cu Si = z , atunci:
z∈Mi
X
|Si | = |z|.
z∈Mi

Cum mulţimile M1 , M2 , M3 , M4 partiţionează {z1 , z2 , . . . , zn }, avem că (∃)i astfel


1
ı̂ncât Si ≥ . În acest caz, alegem numerele din Mi şi demonstraţia se ı̂ncheie.
4
22. Cunoaştem că σ(XY Z) > σ(X 0 Y 0 Z 0 ), dacă X 0 , Y 0 şi Z 0 sunt proiecţiile punctelor
X, Y, Z pe un plan α, neparalel cu (XY Z).
Direct: Fie ABCD tetraedru şi fie E proiecţia punctului A pe (BCD). Unu, două
sau trei dintre triunghiurile EBC, EBD şi ECD acoperă suprafaţa triunghiului
BCD (ı̂n funcţie de poziţia punctului E faţă de triunghiul BCD) deci σ(ABC) +
σ(ABD) + σ(ACD) > σ(BCD). Analog se demonstreză şi celelalte inegalităţi.
Reciproc: Fie (din simetrie) a ≥ b ≥ c ≥ d şi construim triunghiul BCD echilateral
de arie a. Căutăm un punct A cu distanţele x, y, z respective la dreptele CD, BD
şi BC astfel ı̂ncât x · CD = 2b, y · BD = 2c şi z · BC = 2d. Considerăm cilindrele
M, N şi P (infinite) construite ı̂n jurul dreptelor CD, BD şi respectiv BC de rază
x, y şi respectiv z.
Aceste trei cilindre intersectează planul (BCD) ı̂n trei perechi de drepte a1 k
a4 , a2 k a5 şi a3 k a6 pentru cilindrele M, N şi respectiv P . Presupunem dreapta
a1 (paralelă cu CD) şi punctul B de părţi diferite ale dreptei CD. Analog pentru
a2 şi a3 . Fie Aij = ai ∩ aj , ∀i, j ∈ {1, . . . , 6}, i < j.
Cilindrele M şi N se intersectează după două elipse care intersectează planul
(BCD) ı̂n punctele A12 şi A45 şi respectiv A24 şi A15 . Vom arăta că puncele A12 şi
A45 sunt de o parte şi de alta a dreptei a6 şi de aceeaşi parte a dreptei a3 . Din acest
lucru va rezulta că suprafaţele celor trei cilindre M, N şi P au un punct comun
A cu distanţele la dreptele CD, BD şi BC respectiv x, y şi z. Condiţia de mai sus
este echivalentă cu
h l√3 √
l 3 i √
l 3
x∈ − (y + z), + (y + z) şi − x ≤ − (y + z)
2 2 2

(l este latura triunghiului echilateral BCD). Deoarece 4a = l2 3, xl = 2b, yl = 2c
şi zl = 2d, ı̂nmulţind relaţia cu l obţinem
h i
2b ∈ 2a − (2c + 2d), 2a + (2c + 2d) şi − 2b ≤ 2a − (2c + 2d)
140

condiţii adevărate din ipoteză şi din a ≥ b ≥ c ≥ d.

23. Dacă notăm z = −x−y, inegalitatea devine | cos x|+| cos y|+| cos z| ≥ 1. Deoarece
x + y + z = 0, avem că 1 = | cos(x + y + z)| = | cos x cos(y + z) − sin x sin(y + z)| ≤
| cos x|·| cos(y+z)|+| sin x|·| sin(y+z)| ≤ | cos x|+| sin(y+z)| = | cos x|+| sin y cos z+
sin z cos y| ≤ | cos x| + | sin y| · | cos z| + | sin z| · | cos y| ≤ | cos x| + | cos y| + | cos z|.
Capitolul 12

Probleme avute ı̂n atenţia comisiei, 2004

12.1 Clasa a VII-a


1. Soluţia problemei este n = 5.
Să numerotăm vârfurile octogonului ı̂ordine, de la 1 la 8.
Observăm că n = 4 nu este bun, deoarece putem colora vârfurile 1, 2, 4 şi 5 nici
un triunghi isoscel roşu nu se formează. Mai trebuie dovedit că n = 5 convine,
deci că oricum am colora ı̂roşu cinci vârfuri ale poligonului, există un triunghi
isoscel cu toate vârfurile roşii.
Presupunem că se pot colora cinci vâfuri ı̂roşu fără să se formeze vreun triunghi
isoscel cu toate vârfurile roşii. Datorită simetriei, putem considera că vârful 1
ste colorat. Atunci, conform presupunerii făcute, din fiecare pereche (2,8), (3,7)
şi (4,6) cel mult un vârf este colorat, deci obligatoriu este colora vârful (altfel ar
fi roâii cel mult patru vârfuri!). Dar atunci nu poate fi colorat nici 3 (ar apărea
triunghiul isoscel 135), nici 7 (ar fi 175), deci din perechea (3,7) nu se poate alege
nici un vârf, Rămân atunci cel mult patru vârfuri colorate, contradicţie.

2. Răspuns: 310 perechi.


Fie A ⊂ B ⊂ X o pereche de sumbulţimi ca ı̂enunţ. Fiecare element din X va fi
colorat cu alb, roşu sau mov după cum el aparţine mulţimii A, mulţimii B − A,
rexpectiv mulţimii X − B; astfel fiecărei perechi (A, B) ca ı̂enunţ ı̂i asociem o
colorare (bine definită) cu trei culori a elementelor lui X. Şi reciproc, plecând de la
o asemenea colorare şi notând A, R, respectiv M mulţimile formate cu elementele
din X colorate cu alb, roşu respectiv mov avem o pereche de submulţimi A şi
B = A ∪ R astfel că A ⊆ B ⊆ X. (Se consideră şi colorările ı̂care nu există
elemente de o anume culoare - sau de două culori). Dar numărul acestor colorări
se găseşte uşor: fiecare element poate primi una din cele trei culori (adică are trei

141
142

posibilităţi), deci ansamblul celor zece elemente poate fi colorat ı̂3 · 3 · . . . · 3 = 310
moduri (numărul facotrilor este egal cu cel al elementelor din X).
Observaţie. Dacă X are n elemente, evident că problema similară are răspunsul
3n . Ba chiar putem găsi numărul ansamblurilor (A1 , A2 , . . . , Ak ) de mulţimi astfel
ı̂ncât
A1 ⊆ A2 ⊆ . . . ⊆ Ak ⊆ X;
anume, aceasta va fi (k + 1)n .

3. Avem a = 10n − 1, unde n = 2003, prin urmare,

ab = 10n b − b = b1 b2 . . . bk 00 . . . 0} − b1 b2 . . . bk ,
| {z
n cifre

unde b = b1 b2 . . . bk este scrierea lui b ı̂baza zece; cum b ≤ a, avem k ≤ n. De


aceea, după scădere, vom obţine

ab = b1 b2 . . . bk 99 . . . 9} 8c1 c2 . . . ck ,
| {z
n−k−1 cifre

unde ci + b + i = 9 pentru 1 ≤ i ≤ k − 1, iar ck + b + k = 10. Astfel se poate


observa că suma cifrelor numărului ab este 9n adică, ı̂cazul concret al enunţului
nostru este 9 · 2003. Evident, acest număr nu este pătrat perfect. Cititorul este
invitat să observe că nu am puat ı̂considerare chiar toate cazurile, dar că suma
cifrelor rămâne tot 9n şi pentru k = n.

4. Din relaţia bc > a rezultă că b şi c au acelaşi semn (a fiind număr natural). Apoi,
din ac + b > 3a ≥ 0, rezultă că acest acelaşi semn nu poate fi minus (altimteri
ac + b ar fi negativ), deci a şi b vor fi, şi ele, numere naturale. Ba chiar sunt
numere naturale nenule, căci bc > a ≥ 0 ⇒ bc > 0. Prin urmare

ab + c ≥ 2a + c > 2a.

Din cele de mai sus rezultă că mai avem de verificat doar ı̂cazul b = 1. Atunci
prima de relaţiile din ipoteză devine c > a şi implică a + c > 2a, deci, iar, ab + c >
2a.

5. Din enunţ rezultă că cuma cifrelor numărului dat este un număr de forma 3k + 2,
deci numărul ı̂nsuşi este de această formă (se ştie că orice număr natural dă la
ı̂mpărţirea cu 3 acekaşi rest ca şi suma cifrelor scrierii sale ı̂baza zece). Cum
pătratul oricărui număr ı̂ntreg este ori divizibil cu 3, ori de forma 3k + 1, e clar

că numărul considerat a nu poate fi pătrat perfect, ı̂consecinţă a este iraţional.
Faptul că numărul cifrelor lui a este 2004 nu este esenţial.
143

6. Un calcul simplu arată că

11a + 19b
a, (ba) + b, (ab) + a, b(a) = 2a + b + ,
90

deci pentru ca numărul dat să se poată scrie sub formă zecimală finită trebuie ca
fracţia să se poată simplifica prin 9. Altfel spus, trebuie ca

11a + 19b = 9(a + 2b) + (2a + b)

să fie divizibil cu 9, deci 2a + b trebuie să fie divizibil cu 9. Cum a este cifră
diferită de 0 şi de 9, se obţin imediat soluţiile, anume perechea (a, b) trebuie să fie
una dintre următoarele:

(1, 7), (2, 5), (3, 3), (4, 1), (5, 8), (6, 6), (7, 4), (8, 2).

7. a) Fie abc un număr de trei cifre, deoarece

cba − abc = 99(c − a) = 9 · 11(c − a),

nu poate fi pătrat perfect decât pentru c = a (altfel are factorul 11, dar nu şi
pe 112 ), răspunsul la prima ı̂ntrebare este simplu: n = 0 este sigurul număr cu
proprietatea cerută.
b) Trebuie să existe un număr de patru cifre abcd astfel ı̂ncât

n3 = dcba − abcd = 9[111(d − a) + 10(c − b)]

şi atunci numărul din paranteza pătrată va fi obligatoriu de forma 3k 3 , k fiind un


număr natural. Putem considera d > a (d = a nu dă soluţii, iar d < a ar conduce la
diferenţă negativă). Deci practic să găsim x ∈ {1, 2, . . . , 9} şi y ∈ {−9, −8, . . . , 8, 9}
astfel ca 111x + 10y să fie de forma 3k 3 . Atunci numărul căutat va fi n = 3k.
Numerele de forma 3k 3 care nu depăşesc pe 999 + 90 sunt: 3, 24, 108, 192, 375,
648, 1029. Primele două sunt prea mici. Ca să avem 111x + 10y = 108 ar trebuie
ca x să fie 8, deci nu se poate găsi y. Pentru 192, x trebuie să fie 2, deci găsim
y = −3 şi o primă soluţie n = 12, pentru care 123 = 3141 − 1414. Pentru 375 ar fi
x = 5, dar 555 este prea departe de 375 (cel mai mic y edte −9). La fel nu putem
ajunge de la 888 la 648. Mai devreme 1029 pentru care x = 9, deci y = 3. Astfel
mai obţinem soluţia n = 21 pentru care, de exemplu, 213 = 9520 − 259; dar ı̂enunţ
nu se acceptă asemenea soluţii (şi , ı̂mod necesar, dacă x = 9, avem d = 9 şi a = 0)
deci singura soluţie este n = 12 (ı̂afară de cea evidentă, n = 0).
144


1
8. Fie a, b numere naturale astfel ı̂ncât x = a + √ ∈ N; atunci
b
r
1 a
x2 = a + + 2
b b
este tot un număr natural, deci
r  
a 1 2 1
= x −a−
b 2 b
este număr raţional. Fie atunci
r
a
= r ∈ Q+ ⇒ a = r 2 b
b
deci
√ 1 rb + 1
a+ √ = √ ;
b B

cum acesta este număr raţional
√ şi la fel sunt r şi b, rezultă b ∈ Q. Dar b este
număr natural, deci dacă b este raţional, atunci el este chiar natural, adică b este
pătrat perfect. Atunci
√ 1
a=x− √
b
este număr raţional, deci natural (pentru că a este natural) şi soluţia se ı̂ncheie.

9. Numărul N se termină cu cifra 3, deci nu este pătrat perfect, deci N este iraţional.

10. Se constată că [AB este bisectoarea unghiului ∠DAC. Într-adevăr,

m(∠DAB) = m(∠ABC) − m(∠ADC) = m(∠BAC),

conform proprietăţii unghiului exterior şi relaţiei din enunţ. Cum şi [DP este bi-
sectoarea unghiului ∠ADC, rezultă că, ı̂triiunghiul ADC, P este centrul cercului
ı̂nscris iar [CP este a treia bisectoare. Conform teoremei bisectoarei vom avea
atunci egalităţile
AQ AC BD AD
= şi = .
QD CD BC AC
Folosim teorema lui Menelaus pentru triunghiul ACD cu transversala BQT ; obţinem
BD AQ T C
· · = 1,
BC QD T A
de unde, ı̂nlocuind rapoartele de mai sus,
TC CD
= .
TA AD
145

Conform reciprocei teoremei bisectoarei, asta conduce la concluzia că [DT este
bisectoarea exterioară a lui ∠ADC, deci este perpendiculară pe bisectoarea inte-
rioară a acestui unghi, adică T D ⊥ DP .

11. a) Primul punct este valabil chiar dacă H nu este neapărat ortocentrul triunghi-
ului, ci doar punctul de intersecţie a trei ceviene concurente, AM, BN şi CP .
Avem, cu teorema lui Menalaus pentru triunghiul ABH şi transversala N T P
TH P B NH
= ·
TA P A NB
şi, cu aceeaşi teoremă (triunghiul AHN şi transversala BM C)
MH BH CN
= · ;
MA N B AC
egalitatea celor două rapoarte revine atunci la
P B NH BH CN
· = · ,
P A NB N B AC
care se mai scrie
BH P A CN
· · = 1.
N H P B AC
Dar aceasta nu este decât teorema lui Menelaus pentru triunghiul ABN cu transver-
sala P HC şi demonstraţia se ı̂ncheie.
b) Acum folosim teorema lui Menelaus ı̂triunghiul ABH şi transversala U QM :
AU QB M H
· · = 1;
U B QH M A
dar QB = QH, deci avem relaţia
AU MA
= .
UB MH
Analog obţinem şi
AV MA
=
CV MH
AU AC
deci = , ceea ce arată (reciproca teoremei lui Thales) că U V k BC; atunci
UB CV
una din ı̂nălţimile triunghiului U AV are ca dreaptă suport pe AT . Pe de altă
parte, folosind punctul a), avem
AU MA TA
= = ,
UB MH TH
deci (iar cu reciproca teoremei kui Thales, acum ı̂triunghiul ABH) U T k BH; dar
BH ⊥ AC, deci U T ⊥ AC, adică U T este ı̂nălţime ı̂triunghiul AU V şi concluzia
rezultă imediat.
146

12. Să observăm mai ı̂ntâ că, atunci când punctul P este variabil pe latura [AB] a
dreptunghiului, minimul sumei din enunţ este atins când P coincide cu mijlocul
A1 al lui [AB]. Într-adevăr, pentru P ∈ [AB] avem P A + P B = a (constant) deci
minimul sumei P A + . . . + P D se realizează odată cu cel al sumei P C + P D; dar
P C + P D ≥ A1 C + A1 D, oricare ar fi P ∈ [AB]
(chiar oricare ar fi P ∈ AB), aceasta fiind suficient să considerăm simetricul D0 al
lui D faţă de A şi să observăm că
P C + P D = P C + P D 0 ≥ D 0 C = A1 C + A1 D
conform inegalităţii triunghiului; desigur, egalitatea se realizează numai când
P = A1 .
Într-un mod cu totul asemănător se observă că suma ı̂3i atinge minimul pe latura
[CD] ı̂mijlocul C1 al acesteia; prin urmare, avem
r
a2 √
PA + PB + PC + PD ≥ a + 2 + b2 = a + a2 + 4b2
4
pentru orice punct P ∈ [AB] ∪ [CD], egalitatea având loc numai pentru P ∈
{A1 , C1 }. La fel vedem că, pentru P variabil pe une din laturile [AD] sau [BC],
avem √
P A + P B + P C + P D ≥ b + b2 + 4a2 ,
cu egalitate doar pentru P = B1 (mijlocul lui [BC]) sau P = D1 (mijlocul lui
[DA]). Se dă ı̂nsă că a > b, de aceea
√ √
a + a2 + 4b2 < b + b2 + 4a2 ,

deci minimul sumei P A + . . . + P D pe conturul poligonului este a + a2 + 4b2 şi
se realizează ı̂două pucte: mijlocalele segmentelor [AB] şi [CD].
Ar mai fi de demonstrat ultima inegalitate; o rescriem ı̂forma
√ √
a − b < b2 + 4a2 − a2 + 4b2
sau, după raţionalizare,
3(a2 − b2 )
a−b< √ √ ;
a2 + 4b2 + b2 + 4a2
cum a − b > 0, ne mai rămâne doar să arătăm că
√ √
a2 + 4b2 + a2 + 4b2 < 3(a + b),
ceea ce rezultă din
√ √
a2 + 4b2 < a + 2b şi b2 + 4a2 < b + 2a.
147

13. Aici e o mică grşeală ı̂enunţ: din CM ⊥ DN rezultă CM şi CM ≡ DN (CM ⊥


DN ), cu, scrie acolo, e aiurea). De fapt, totul e greşit, pentru că ı̂cazul unui
romb ABCD ca ı̂enunţ (nu neapărat pătrat) nu putem găsi puncte M ∈ (DA)
şi N ∈ (BC) astfel ı̂ncât CM ⊥ DN deci implicaţia din enunţ este adevărată (şi
paralelogramul poate să nu fie pătrat).
Totuşi, o mică schimbare ne permite să obţinem concluzia. Cum BC ≥ CD,
putem alege punctele M ∈ [DA] şi N ∈ [BC] astfel ı̂ncât DM = CN = DC =
AB; atunci DM N C este romb, deci CM ⊥ DN . Conform ipotezei, CM ≡ DN ,
deci patrulaterul DM N C este chiar pătrat, ceea ce implică faptul că ABCD este
dreptunghi. Dacă am avea ı̂nsă chiar BC > DC s-ar putea găsi punctele M1 ∈
(AM ) şi N1 ∈ (CN ) pentru care CM1 ⊥ DN1 , dar CM1 > DN1 , deci ABD trebuie
să fie pătrat (dacă dăm voie lui M să parcurgă segmentul ı̂nchis [DA], etc).

14. Să notăm cu d, e, f lungiile laturilor [EF ], [DF ], respectiv [DE] ale triunghi-
ului DEF şi cu s semiperimetrul său. Se ştie că ı̂orice triunghi ı̂nălţimile sunt
bisectoarele unghiurilor triunghiului ortic, deci A, B, C sunt centrele cercurilor
exı̂nscrise triunghiului DEF (de exemplu, [F A şi (EA sunt bisectoarele unghi-
urilor exterioare din E şi din F , etc); şi iar se ştie că (demonstraţi!)

F M = DP = s − d, M E = DN = s − e, EP = F N = s − f

şi prima parte este rezolvată.


Pentru punctul b) observăm că
F M EP DN
· · = 1,
ME P D NF
conform celor arătate mai sus, deci (pe baza reciprocei teoremei lui Ceva) dreptele
DM, EN şi F P sunt concurente.

15. a) Desigur, AB k CD şi ABCD este trapez. Apoi avem

m(∠ACD) = m(∠ADE) + m(∠EDC) = m(∠DCE) + m(∠ECB) = m(∠DCB),

deci trapezu; ABCD este isoscel.


b) Pentru partea a doua folosim asemănarea triunghiurilor isoscele DAE şi CDE,
din care obţinem
DC DE
= ,
AD AE
adică
b c
= ;
c a−b
sigur că această egalitate este echivalentă cu aceea care trebuie demonstrată.
148

16. Datorită teoremei lui Menelaus putemscrie

A0 B B 0 C C 0 A
· · = 1,
A0 C B 0 A C 0 B
de aceea cele două relaţii de la punctul a) o implică pe cea dată la punctul b),
prin urmare ipoteza trebuie să cuprindă numai concurenţa dreptelor şi cele două
relaţii de la a).
Presupunând că acestea au loc se vede imediat că trunghiurile BAC şi BA0 C 0 sunt
asemenea şi, la fel 4CAB ∼ 4CA0 B 0 (cazulLUL); atunci avem m(∠C 0 A0 B) =
m(∠B 0 A0 C) (amble fiind egale cu m(∠BAC)), deci [AA0 este bisectoarea unghiu-
lui C 0 A0 B 0 .
Să ducem paralela prin A la BC, care intersectează pe A0 B 0 şi A0 C 0 ı̂D, respectiv
E. Din teorema fundamentală a asemănării avem atunci egalităţile

AD AB 0 AE AC 0
= şi =
A0 C BC A0 B C 0B0
care conduc la
AE A0 B B 0 C C 0 A
= 0 · 0 · 0 = 1,
AD AC BA C B
deci la AE = AD, conform teoremei lui Menelaus. Prin urmare, ı̂triunghiul
A0 DE, A0 A este şi bisectoare, şi mediană; atunci triunghiul este isoscel (cu A0 D =
A0 E) şi A0 A este, de asemenea, ı̂nălţime. Adică A0 A ⊥ DE, deci (cum DE k BC)
A0 A ⊥ BC.
Evident, ı̂mod cu totul asemănător, se demonstrează căBB 0 ⊥ AC şi CCC 0 ⊥
AB, de unde concluzia că patrulaterele AB 0 M C 0 , BA0 M C 0 şi CA0 M B 0 sunt in-
scriptibile este imediată.

17. Şi de astă dată vom rezolva mai ı̂ntâi punctul b), apoi pe a).
Fie [CM ], M ∈ [BD] ı̂nălţimea (şi totodată mediana) triunghiului isoscel CBD;
cum M este mijlocul lui [BD], avem BM = 3, 5 şi OM = 0, 5. Exprimând ı̂două
moduri diferite pe CM (cu teroema lui Pitagora) avem
√ √
BC 2 − 3, 52 = (2 3)2 − 0, 52 ⇒ BC = 2 6;

bineı̂nţeles, avem şi CD = 2 6. Acum, ı̂triunghiul COD cunoaştem toate la-
turile, deci putemfolosi teorema cosinusului pentru a calcula

OC 2 + OD2 − CD2 1
cos(∠COD) = = √ ,
2OC · OD 4 3
149

1 1
ceea ce ı̂nseamnă că şi cos ∠AOB = √ şi cos ∠AOD = − √ . Atunci, tot cu
4 3 4 3 √
teorema cosinusului (ı̂triunghiurile AOB şi AOD) obţinem AB = 3 2 şi AD =
√ AB OB
4 2 (acum punctul b) este rezolvat). Prin urmare = , deci (reciproca
AD OD
bisectoarei) [AO este bisectoarea unghiului ∠BAD.

18. Enunţul conţine o greşeală: unghiul ∠AM C trebuie să fie congruent cu ∠CM B
(nu cu ∠CM D, cu care nici nu are cum să fie congruent!).
Să considerămı̂ntâi că ABCD este dreptunghi şi că triunghiul BM N construit ca
ı̂enunţ este echilateral şi să arătăm că ABCD este pătrat. Vrem adică că arătăm
că AB = AD.
Se obţine imediat că, m(∠BM N ) = m(∠CM N ) = 60◦ . Atunci rezultă şi m(∠CM D) =
60◦ , deci, deoarece m(∠CM A) = m(∠CM B) = 120◦ , avem şi m(∠AM D) = 60◦ ,
precum şi m(∠AM B) = 120◦ . Cu teorema cosinusului ı̂triunghiurile AM B şi
AM D calculăm atunci

AB 2 = M A2 + M B 2 + M A · M B şi AD2 = M A2 + M D2 − M A · M D,

vedem astfel că egalitatea AB = AD revine la

M A(M B + M D) = (M D − M B)(M B + M D)

deci, pănâ la urmă, la M D = M A + M B, cum ı̂nsă triunghiul BM N este echilat-


eral, asta e totuna cu M D = M A + M N , deci avem de demonstrat că M D = AN .
Fie x măsura unghiului ∠N BC; rezultă imediat că măsura unghiului ∠N AB este
30◦ − x, ı̂particular că x < 30◦ . De asemenea, se calculează uşr măsurile unghi-
urilor triunghiului DM C: vem m(∠DCM ) = 90◦ − x, m(∠DM C) = 60◦ , deci
m(∠M DC) = x + 30◦ . Prin urmare m(∠M DC) > 2x şi atunci există un punct
(unic determinat) N ∈ (AC) astfel ı̂ncât m(∠N DC) = 2x. Rezultă atunci că
m(∠DN C) = 90◦ − x, deci că triunghiul DN C este isoscel şi DN = DC ⇒ DN =
AB. Atunci triunghiurile DN M şi ABN sunt congruente (cazul ULU), de unde
rezultă că DM = AN şi demonstraţia se ı̂ncheie.
Dacă presupunem că ABCD este pătrat e mai uşor. Întâi se observă că punctul
M trebuie astfel ales pe diagonala [BD] ı̂ncât m(∠M CB) = 15◦ . Într-adevăr, dacă
notăm m(∠M CB) = u, vom avea şi m(∠M AB) = u, apoi m(∠CM B) = 135◦ −u şi
m(∠CM A) = 2u + 90◦ ; cum aceste două măsuri trebuie să fie egale se obţine, cum
am spus, u = 15◦ şi mai departe se arată uâor că triughiul BM N este echilateral
(demonstraţi!).
150

12.2 Clasa a VIII-a

12.3 Clasa a IX-a


1. Avem
k k
X X ni nj X ni k−1X k−1
S= f (x) ≡ + = (k − 1) = ni ≡ (mod 2)
x∈A 1≤i<j≤k
2 2 i=1
2 2 i=1 2

k−1
deci 2 | ⇒ 4 | k − 1.
2
2. Avem
√ √ √ √ √ √
1 1 1+ a 1+ b 1+ a 1+ b 1 1 a b
√ + √ = + = + = + + + .
1− a 1− b 1−a 1−b b a a b b a
1 a
Dar + ≥ 4 deoarece
a b
 
1 1 a b a b
(a + b) + ≥4⇔2+ + ≥4⇔ + ≥2
a b b a b a
√ √
a b √
adevărat şi + ≥ 2 2 deoarece
b a
√ √ s
√ √
a b a b 1 1 √
+ ≥2 = 2√
4
≥ 2 r = 2 2.
b a b a ab 4 1

4
Rezultă că √ √
1 1 1 1 a b √
√ + √ = + + + ≥4+2 2
1− a 1− b a b b a
1 √
cu egalitate pentru a = b = , deci minimul expresiei din enunţ este 4 + 2 2.
2
3. Voi rezolva direct punctul b). Fie m ∈ N. ŞTim că
    h i
n n+1 n2
+ + = n. (1)
p q r

Pentru n0 = n + pqr obţinem că


     
n + pqr n + pqr + 1 n + pqr + 2
+ + = n + pqr (2)
p q r
151

dar      
n + pqr n + pqr + 1 n + pqr + 2
+ + =
p q r
     
n n+1 n+2
= + + + pq + qr + pr. (3)
p q r
Din (1), (2) şi (3) rezultă că pq + qr + rp = pqr deci
1 1 1
+ + = 1. (∗)
p q r
1 1 1
Pentru a putea folsi (∗) vom rezolva ı̂N ecuaţia + + = 1. Să impunem
a b c
a
ordinea a ≤ b ≤ c. Atunci ≤ 1 ⇒ a ≤ 3. Dacă a = 3, rezultă că a = b = c = 3.
3
1 1 1
Dacă a = 2, + = , de unde obţinem (b, c) ∈ {(4, 4), (3, 6)}. Rezultă că soluţii
b c 2
sunt (a, b, c) ∈ {(3, 3, 3), (2, 4, 4), (2, 3, 6)}. Analizând toate cele 10 cazuri posibile
pentru (p, q, r) (cele 3 cazuri plus permutări), găsim imediat contraexemple ı̂toate
cazurile cu exceptţia cazurilor p = q = r = 3 şi p = r = 4, q = 2. Să vedem acum
că acestea verifică ı̂ntradevăr. Cazul p = q = r = 3 este identitatea lui Hermite
pentru n = 3. Pentru p = r = 4, q = 2 aplicăm identitatea lui Hermite,cazul 2, de
două ori astfel
 n   n 
hni n + 1 n + 2 +1 
n + 1
 h i 
n n + 1

+ + 2
=   +  2  + = + = n.
4 2 4 2 2 2 2 2

4. Avem că
Xa+b X X1  X X1 X X1 
≥2 a+ −3 ⇔ a −3≥2 a+ −3
c a a a

deci este suficient să demonstrăm că


X  X 1 
a−2 −2 ≥1
a
X X1
care rezultă imediat din a ≥ 3 şi − 2 ≥ 1 care, la rândul lor, rezultă
a
imediat din inegalitatea mediilor.

5. Pentru x = a, y = b rezultă că

|g(a) − g(b)| ≥ |f (a) − f (b)|

dar g(a), g(b) ∈ |f (a), f (b)| deci {g(a), g(b)} = {f (a), f (b)}/
152

Cazul 1. g(a) = f (a), g(b) = f (b). Fie x ∈ (a, b), oarecare. Atunci

|g(x) − g(a)| ≥ |f (x) − f (a)|

şi
|g(a) − g(y)| ≥ |f (a) − f (y)|
şi prin adunare rezultă că

|g(x) − g(a)||g(a) − g(y)| + |g(a) − g(y)| ≥ f (y) − f (x) = g(y) − g(x).

Dar cum g(x) ∈ [g(a), g(b)],

|g(x) − g(a)| + |g(a) − g(y)| = g(y) − g(x)

deci toate inegalităţile devin egalităţi. În particular,

|g(x) − g(a)| = |f (x) − f (a)|

şi cum g(a) ≤ g(x), f (a) ≤ f (x) rezultă că g(x)−g(a) = f (x)−f (a) ⇒ g(x) = f (x).
Rezultă că g(x) = f (x), oricare ar fi x ∈ [a, b].
Cazul 2. g(a) = f (b), g(b) = f (a). Se tratează asemănător cu primul caz şi obţinem
g(x) = f (b) + f (a) − f (x).
 
n=1
6. Dacă a > 1, există M astfel ı̂ncât f n + ≥ n2 + n − 1 pentru orice n ≥
n
M (acest lucru rezultă imediat din faptul că pentru orice polinom p ∈ R[x] cu
coeficientul dominan pozitiv, există o constantă c astfel ı̂ncât f (x) > 0, oricare
ar fi x > c). Printr-un argument asemăntător, obţinem că şi cazul a < 1 este
imposibil, deci a = 1. Rezultă că

f (x) = x2 + bx + c.
   
1 2 n−1
Fie n cu f n + > n − n + 1 şi f n + < n2 + n − 1. Prin calcul, devin
n n

1 2 b c 1
3
+ +b+ 2 + +1− >0
n n n n n
şi
1 (n − 1)2 b(n − 1) c
1− + + b + + < 0.
n n3 n2 n
Prin trecere la limită, ele devin b ≥ −1 şi b ≤ −1, deci b = −1. Rezultă că

f (x) = x2 − x + c.
153

Procedăm ca mai dus, şi inegalităţile


   
1 2 n−1
f n+ > n − n + 1 şi f n + < n2 + n − 1
n n

devin acum
 2
1 1 1 n−1
2
+ + 1 + c > 0 şi − 2 + + + c < 0.
n n n n

Din a doua rezultă direct că c < 2, iar din prima, cu trecere la limită rezultă că
c ≥ −1. Cum c = f (0) ∈ Z, rezultă că c ∈ {−1, 0, 1}. În toate cele 3 cazuri condiţia
este realizată, deci funcţile sunt f (x) = x2 −x−1, f (x) = x2 −x, f (x) = x2 −x+1.

7. Este clar că f este injectivă. Dacă f ar fi descrescătoare, atunci ar fi chiar strict de-
screscătoare, deci am avea un şir strict descrescător de numere naturale, imposi-
bil. Deci f este (chiar strict) crescător. Înlocuind ı̂relaţie n cu f (n) − f (2) (n > 2),
rezultă că
f (f (f (n) − f (2))) = f (n) ⇒ f (f (n) − f (2) = n.
Atunci f ia toate valorile 3, 4, . . .. Cum f este strict crescătoare rezultă că preimag-
inile acestor numere sunt numere naturale consecutive, deci

f (k) = 3, f (k + 1) = 4, . . .

Avem f (1) = 1 ⇒ f (2) = 0 deci f (1) 6= 1.


Avem f (1) = 2 ⇒ f (2) = 1 + f (2) deci f (1) 6= 2.
Rezultă că f (1) ≥ 3, deci f (k) = 3 ≤ f (1) ⇒ k ≤ 1 ⇒ k = 1, deci f (n) = n + 2,
oricare ar fi n ∈ N∗ , funcţie care verifiă cerinţa ı̂mod evident.

8. Să rezolvăm mai ı̂ntâi punctul a). Cum f (i1 ), f (i2 ), . . . , f (i3n ) sunt distincte două
câte două, rezultă că

3n(3n + 1)
f (i1 ) + f (i2 ) + . . . + f (i3n ) ≥
2
cu egalitate când

{f (i1 ), f (i2 ), . . . , f (i3n )} = {1, 2, . . . , 3n}.

La fel, cum f (f (i1 )), f (f (i2 )), . . . , f (f (i3n )) sunt distincte două câte două, rezultă
că
3n(3n + 1
f (f (i1 )) + . . . + f (f (i3n )) ≥
2
154

cu egalitate când

{f (f (i1 )), f (f (i2 )), . . . , f (f (i3n ))} = {1, 2, . . . , 3n}.

Prin adunare,

f (i1 ) + f (i2 ) + . . . + f (i3n ) + f (f (i1 )) + . . . + f (f (i3n )) ≥ 3n(3n + 1)

dar ı̂problemă ni se spune că avem egalitate, deci {f (i1 ), f (i2 ), . . . , f (i3n )} = {1, 2, . . . , 2n}
şi {f (f (i1 )), f (f (i2 )), . . . , f (f (i3n ))} = {1, 2, . . . , 3n}. Vom folosi notaţia [k] =
{1, 2, . . . , k}.Atunci f (f ([3n])) = [3n] şi f ([3n]) = [3n], deci f (f ([3n])) = f ([3n])
şi din injectivitate, rezultă că f ([3n]) = [3n], oricare ar fi n ∈ N∗ . Rezultă că

f ({3k + 1, 3k + 2, 3k + 3}) = {3k + 1, 3k + 2, 3k + 3}

oricare ar fi k ∈ N. De fapt, toate funţiile injective care transformă mulţimile


{3k + 1, 3k + 2, 3k + 3} ı̂ele ı̂nsele au proprietatea din enunţ, deoarece putem alege
ı̂ntotdeauna ij = j, deci punctul a) este rezolvat. Punctul b) este trivial. Putem
alege chiar funcţia identitate.

9. Din CBS avem X X a X 2


abc(c + a) ≥ a
bc(c + a)
deci este suficient să se demonstreze că
X 4 X X 4 X X 3
2 a ≥ 27 abc(c + a) ⇔ a ≥ 27abc a⇔ a ≥ 27abc

care este chiar inegalitatea mediilor.

10. Avem două cazuri: f (0) = 9, f (1) = f (5), f (1) = 9 şi f (1) = 1, f (0) =
f (4), f (5) = 9. Rezolvând cele două sisteme de 3 ecuaţii cu 3 necunoscute,
obţinem soluţiile f (x) = x2 − 6x + 9 şi f (x) = x2 − 4x + 4. Ambele funcţii verifică,
deci acestea sunt soluţiile problemei.

11. Pentru omogenitatea inegalităţii, vom face substituţia t = 1!. Atunci, folosind
ı̂nmulţirea ciclică, inegalitatea devine
Y x rY
1 x 1
≤ √
4
⇔ 4 ≤ √
t + ax (1 + abcd t + ax 1 + 4 abcd
care este echivalentă cu
r rY
4
Y t ax
+ 4
le1
t + ax t + ax
care rezultă din inegalitatea mediilor aplicată pentru cei doi termeni.
155
√ √
12. Este suficient să demonstrez că a(b3 + c2 ) ≥ abc(b2 + c2 ), deoarece scriind
analoagele (prin permutări circulare ale variabilelor) şi adunând obţinem chiar
inegalitatea din enunţ. Avem
√ 3 √ √
a(b + c3 ) ≥ abc(b2 + c2 ) ⇔ (b3 c3 ) ≥ bc(b2 + c2 )

care rezultă din 2(b3 +c3 ) ≥ (b+c)(b2 +c2 ) ⇔ (b+c)(b−c)2 ≥ 0, evident adevărată.
1
13. Prin adunare avem (x4 + 2x3 − x) + (y 4 + 2y 3 − y) = − . Dar cei doi termeni se pot
8
1
scrie ca x +2x −x = (x +x) −(x +x) ≥ − şi y +2y 3 −y = (y 2 +y)2 −(y 2 +y) ≥
4 3 2 2 2 4
4
1
− , deci ambele inegalităţi devin egalităţi. Rezultă că x şi y sunt rădăcini ale
4 ( √ √ )
1 −1 − 3 −1 + 3
ecuaţiei x2 + x = , dar cum x 6= y ⇒ {x, y} = , . Analizând
2 2 2
√ √
−1 + 3 −1 − 3
cele două cazuri, vedem că soluţia este x = , y= .
2 2
14. Din inegalitatea mediilor avem
√ √ √ √ (a + b + c)2 (a + b + c)2 (a + b + c)2
abc( a + b + c) + + + ≥
3 3 3
s√ √
√ √
4 abc( a + b + c)(a + b + c)6
≥4 .
27
Deci trebuie să demonstrăm că
s√ √
√ √
4 abc( a + b + c)(a + b + c)6 p
4 ≥ 4 3abc(a + b + c) ⇔
27
√ √ √ 3
⇔ ( a + b + c)(a + b + c)4 ≥ 23abc 2 ,
care din nou rezultă din inegalitatea mediilor.

15. Nu ştiu să fac cazul a, b, c > 0 (care nu este cel cu cazul egalitate), ı̂care inegalitatea
se reduce la ceva oribil ı̂ntr-o variabilă.

16. Din Cauchy-Schwartz, avem


X X a2 X 2 X a2 a+b+c
(b + c) ≥ a ⇒ ≥ .
b+c b+c 2
Atunci,
X a 1 X a2 1 a+b+c 1X 1
= ≥ =
bc(b + c) abc b+c abc 2 2 ab
156

X 1
≥ 3. Aceasta rezultă tot din Cauchy:
deci este suficient să demonstră că
ab
X 1 X a X 1 2 X a 2 X 1 Xa
≥ ≥ ⇒ ≥ ≥ 3.
ab b a b ab b
√ √
17. Fie yn = (p + q)n + (p − q)n . Voi demonstra că yn este natural par, oricare
ar fi n ∈ N. Se vede uşor că yn+2 = 2pyn+1 + (p2 − q)yn , dar cum y0 şi y1 sunt
pare, rezultă imediat prin inducţie că yn este par, oricare ar fi n ∈ N. Dar cum

−1 < (p − q)n < 1, rezultă că

√ n yn , n ≡ 1 (mod 2)
xn = [(p + q)] =
yn − 1, n ≡ 0 (mod 2)

deci {n|2|xn } = 2N+1 şi {n|2|xn +1} = 2N, şi cum ı̂ntre mulţimea numerelor pare
şi mulţimea numerelor impare există evident o bijecţie, problema este rezolvată.

18. Răspuns: S = {3, 4, 5, 6, . . .}. Evident, 1 6 S. Să demonstrăm acum că 2 ∈


/ S.
Fie a = b = 1. Să presupunem prin absurd că există x1 , x2 ∈ R astfel ı̂ncât
1 1
x1 + x 2 = + = 1. Atunci
x2 x2
1 1 x1 + x2 1
1= + = = ⇒ x1 x2 = 1
x1 x2 x1 x2 x1 x2
dar
1 = (x1 + x2 )2 ≥ 4x1 x2
/ S. Acum urmează pasul esenţial: 3 ∈ S. Fie a, b ∈ R∗ . Trebuie
imposibil, deci 2 ∈
1 1 1
să demonstrăm că există x1 , x2 , x3 astfel ı̂ncât x1 + x2 + x3 = a şi + + = b.
x1 x2 x3
Presupunem x1 , x2 , x3 găsite şi prelucrăm cele două egalităţi pentru a vedea cum
trebuie să alegem x1 , x2 , x3 . Avem
1 1 1
+ + =b
x1 x2 a − x1 − x2
sau
(x1 + x2 )[a − (x1 + x2 )] + x1 x2 = bx1 x2 [a − (x1 + x2 )].
Notăm s = x1 + x2 , p = x1 x2 . Atunci ultima relaţie devine

s(a − s) + p = bp(a − s) ⇔ s2 − s(bp + a) + abp − p = 0.

Această ecuaţie de gradul 2 are discriminantul

∆ = (bp + a)2 − 4(abp − p) = (bp − a)2 + 4p.


157

Fie f (x) = (bx − a)2 + 4x. Cum a 6= 0, (bx − a)2 nu se anulează ı̂0, deci f (0) > 0.
Atunci, din continuitate, există x < 0 astfel ı̂ncât f (x) > 0. Acum este momentul
să construim numerele x1 , x2 , x3 . Alegem p = x (p < 0 astfel ı̂ncât f (p) > 0).
Atunci, ecuaţia s3 − s(bp + a) + abp − p = 0 are soluţie.
Lemă. Dacă p < 0 şi s ∈ R, oarecare, atunci există x1 , x2 astfel ı̂ncât x1 + x2 =
s, x1 x2 = p.
p
Demonstraţie. Este suficient să arătăm că există x1 ∈ R astfel ı̂ncât x1 + =s⇔
x1
x21 − sx1 + p = 0, dar ∆0 = s2 − 4p > 0 deci acest lucru este adevărat şi lema
este demonstrată. Aşadar, fie x1 , x2 astfel ı̂ncât x1 + x2 = s, x1 x2 = p. Acum nu
ne mai rămâne decât să alegem x3 = a − x1 − x2 , deci am demonstrat că 3 ∈ S.
Acum să demonstrăm că orice n ≥ 3 are proprietatea cerută. Am văzut că n = 3
b n−3
funcţionează. Fie n ≥ 4. Fie a, b ∈ R∗ , oarecare. Fie x 6= , . Aplicăm
n−3 a
n−3 0
cazul 3 pentru a0 = a− , b = b−(n−3)x, care ne spune că există x1 , x2 , x3 ∈ R
x
n−3 1 1 1
astfel ı̂ncât x1 + x2 + x3 = a − , + + = b − (n − 3)x. Atunci putem
x x1 x2 x3
alege x1 , x2 , x3 , x, x, . . . , x, care verifică ı̂mod evident şi problema este rezolvată.
| {z }
n−3

19. Fie A0 mijlocul arcului BC al cercului circumscris triunghiului ABC. Este clar că
D ∈ (AA0 ) şi că OA0 ⊥ DH. Din acest ultim fapt, rezultă că A0 se află pe dreapta
y = −x (deoarece şi O se află pe această dreaptă). Fie A0 (−a, a). Dar AH ⊥ DH,
deci
yA − 4 4−2
· = −1 ⇒ xA + yA = 5.
xA − 1 1 − (−1)
D ∈ (AA0 ) se poate exprima ca

−a + 1 xA + 1
=
a−2 yA − 2

şi cu xA + yA = 5, obţinem A(4a − 5, −4a + 10). Dar A, A0 ∈ (ABC) ⇒ OA = OA0 .


Atunci,  2  2  2
1 1 1
4a − 5 − + −4a + 10 + =2 a+ .
2 2 2
 
7
Rezolvând, obţinem a ∈ , 2 . În ambele cazuri obţinem soluţii.
3

20. Fie C1 , C2 ∈ (AB), A1 , A2 ∈ (BC) şi B1 , B2 ∈ (CA) astfel ı̂ncât AC1 = BC2 =
1 1 1
AB, BA1 = CA2 = BC şi CB1 = AB2 = CA, Atunci A ∈
1+k 1+k 1+k
0 0
(A1 A2 ), B ∈ (B1 B2 ) şi C (C1 C2 ). Vom folosi următoarea:
158

Lemă. Fie triunghiul ABC şi M ∈ (BC). Atunci AM ≤ max{AB, AC}


Demonstraţie. Dacă ∠AM B ≤ 90◦ , atunci AM ≤ AB ≤ max{AB, AC}, iar dacă
∠AM C ≥ 90◦ , atunci AM ≤ AC ≤ max{C 0 B2 , C 0 B1 }. Rezultă că

B 0 C 0 ≤ max{B2 C1 , B2 C2 , B1 C2 , B1 C1 }.

Fie M − max{AB, BC, CA}. Voi demonstra că

k
max{B2 C1 , B2 C2 , B1 C2 , B1 C1 } ≤ M.
k+1
1 k k k k
Evident, B2 C1 = BC ≤ BC ≤ M şi B1 C2 = BC ≤ M.
k+1 k+1 k+1 k+1 k+1
În plus,
1 k−1 k
B1 C1 < B1 B2 + B2 C1 = BC + AC ≤ M
k+1 k+1 k+1
şi
1 k−1 k
B2 C2 < B2 C1 + C1 C2 = BC + AB ≤ M.
k+1 k+1 k+1
k
Rezultă că B 0 C 0 ≤ M . Scriind şi analoagele, rezultă că
k+1
k
max{A0 B 0 , B 0 C 0 , C 0 A0 } ≤ max{AB, BC, CA},
k+1
c.c.t.d.
AM BN CP
21. Nu voi folosi decât că = = = k. Fie R şi R1 , razele cercurilor cir-
MB NC PA
cumscrise triunghiurilor ABC şi respectiv M N P . Conform teoremei lui Stewart
ı̂triunghiul AOB,

OM 2 · AB = OA2 · M B + OB 2 · M A − AM · M B · AB ⇔
k
⇔ cR12 = cR2 − 2
c3
(k + 1)
deci
(k + 1)2 2
c2 = (R − R12 ).
k
Scriind astfel ı̂ncât analoagele, rezultă că a = b = c, adică triunghiul este echilat-
eral.

22. Fie I = int(P ) şi X = {M ∈ I | s(M ) = k}. trebuie să demonstrăm că X = I.
Vom ı̂ncepe cu:
159

AM
Lema 1. Fie M ∈ (AB) astfel ı̂ncât = p şi o dreaptă oarecare d. Atunci
MB
d(A, d) + pd(B, d)
d(M, d) = .
1+p

Demonstraţie. Putem presupune că d(A, d) ≤ d(B, d). Fie d0 paralela prin A la d.
Din asemănare,
p p
d(M, d0 ) = d(B, d0 ) ⇒ d(M, d0 ) + d(d, d0 ) = d(B, d0 ) + d(d, d0 )
p+1 p+1
dar
p 1 p
d(B, d0 ) + d(d, d0 ) = d(A, d) + d(B, d)
p+1 p+1 p+1
de unde urmează concluzia lemei. Să revenim la problemă. Acum urmează pasul
cel mai important.
Lema 2. Dacă A, B ∈ X, atunci AB ∩ I ⊆ X.
Demonstraţie. Fie M ∈ AB. Trebuie să demonstrăm că M ∈ X. Să luăm ı̂ntâi
AM
cazul M ∈ (AB). Fie p = . Fie L mulţimea laturilor poligonului. Din lema 1,
MB
avem
X X 1 p 1 p
s(M ) = d(M, l) = d(A, l) + d(B, l) = s(A) + s(B)
l∈L i∈L
p + 1 p + 1 p + 1 p + 1

dar S(A) = s(B) = k, deci s(M ) = k, adică M ∈ X. Celelalte două cazuri se


tratează asemănător.
Să revenim la problemă. Fie M ∈ I, oarecare. Fie o dreaptă d care intersectează
laturile triunghiului ABC ı̂două puncte, P şi Q. Din lema 2 rezultă că P, Q ∈ X
şi de aici, tot din lema 2, M ∈ X, deci problema este rezolvată.

23. Fie patrualterul ABCD şi punctul M astfel ı̂ncât orice dreaptă care trece prin M
ı̂mparte patrulaterul ı̂două poligoane de arii egale. Este clar că M ∈ int(ABCD).
Fie S mulţimea semidreptelor (ı̂nchise) cu originea ı̂M . Pentru fiecare s ∈ S, fie
s ∈ S, semidreapta opusă lui s şi D(s) punctul de intersecţie dintre s şi fron-
tiera patrulaterului. Definim funcţia f : S → R prin f (s) = M D(s) − M D(s).
Este clar că f este continuă, şi cum f (s) = −f (s), ia atât valori negative, cât şi
valori pozitive, deci există s ∈ S astfel ı̂ncât f (s) = 0. Cu alte cuvinte, există o
dreaptă d care trece prin M astfel ı̂ncât M P = M Q, unde P şi Q sunt punctele de
intersecţie ale dreptei d cu frontiera patrulaterului. Fie d0 6= d o altă dreaptă care
intersectează aceleaşi laturi ale patrulaterului ca şi d, ı̂punctele P 0 şi Q0 . Aplicând
ipoteza pentru d şi d0 , obţinem că [P M P 0 ] = [QM Q0 ], deci
P M · P 0 M · sin P M P 0 QM · Q0 M · sin QM Q0
=
2 2
160

dar M P = M Q şi ∠P M P 0 = ∠QM Q0 , deci M P 0 = M Q0 , de unde rezultă că


P P 0 Q0 Q este paralelogram, deci cele două laturi ale patrulaterului suntparalele,
să zicem AB k CD. Am văzut că o dreaptă care trece prin M şi intersectează
bazele, are proprietatea că segmentul din ea din interiorul trapezului ı̂l are pe
M mijloc, deci M se află pe linia mijlocie. Aplicând ipoteza pentru linia mi-
jlocie, rezultă că patrulaterul este paralelogram. Se observă uşor că toate paralel-
ogramele au proprietatea cerută (datorită simetriei centrale).

24. Conform relaţiei lui Leibnitz, avem


X X
GM 2 = |Xi |GG2i + Gi M 2
M ∈X M ∈Xi

pentru orice i ∈ {1, 2, . . . , n}. Prin ı̂nsumare,


n X n
!
X X X
GM 2 = |Xi |GG2i + Gi M 2
i=1 M ∈Xi i=1 M ∈Xi

adică
n n n
!2
X X X
S =s+ |Xi |GG2i ≥ s + n GG2i ≥ s + GGi = s + d2 .
i=1 i=1 i=1

25. Dacă triunghiul este echilateral, putem alege mijloacele laturilor, şi cerinţa este
ı̂ndeplinită. Să rezolvăm acum reciproca. Cum ı̂orice triunghi, centrul de greu-
tate se află pe segmentul determinat de ortocentrul astfel ı̂ncât centrul cercului
circumscris, la o trime din distanţă dinspre centrul cercului circumscris spre or-
tocentru, cele două triunghiuri trebuie să aibă şi acelaşi centru al cercului circum-
scris. Esenţialul este următoarea:
Lemă. Dacă triunghiurile ABC şi M N P au acelaşi centru de greutate şi M ∈
(AB), N ∈ (BC) şi P ∈ (CA), atunci
AM BN CP
= = .
MB NC PA

AM BN CP
Demonstrţie. Fie = k1 , = k2 , = k3 . Vom alege originea B şi fie
MB NC PA
−→ −−→ −−→ 1 −−→ k2
BA = ~u şi BC = vecv. Oţinem imediat că BM = ~u , BN = ~v şi
1+k+1 1 + k2
−−→ k3~u + ~v
BP = . Atunci,
k3 + 1
~0 + −→ −−→ −−→ −−→ −−→
BA + BC = BM + BN + BP ⇔
161
   
1 k3 k2 1
⇔ ~u + ~v = ~u + + ~v +
1 + k + 1 1 + k3 1 + k2 1 + k3
şi cum vectorii ~u şi ~v sunt lineari independenţi, rezultă că
1 k3
+ = 1 ⇒ k1 = k3
1 + k1 1 + k3
şi
k2 1
+ = 1 ⇒ k2 = k3 .
1 + k2 1 + k3
Deci k1 = k2 = k3 , c.c.t.d. Să revenim la problemă. Am demonstrat că
AM BN CP
= = = k.
MB NC PA
Fie R şi R1 , razele cercurilor circumscrise triunghiurilor ABC şi respectiv M N P .
Din teorema lui Stewart ı̂AOB.

OM 2 · AB = OA2 · M B + OB 2 · M A − AM · M B · AB ⇔
k
⇔ cR12 = cR2 − 2
c3
(k + 1)
deci
(k + 1)2 2
c2 = (R − R12 ).
k
Scriind şi analoagele, rezultă că a = b = c, adică triunghiul este echilateral.

26. Să facem mai ı̂ntâi implicaţia inversă. Fie {O} = AC ∩ BD. Dacă alegem M = O,
atunci

AC 2 + BD2 ≤ 4A · OC + 4BO · OD ≤ (AO + OC)2 + (BO + OD)2 = AC 2 + BD2 .

Rezultă că toate inegalităţile devin egalităţi, deci AO = OC, BO = OD, adică
ABCD este paralelogram. Să alegem acum M = A. Rezultă că

4AB · AD ≥ AC 2 + BD2 = 2AB 2 + 2AD2

deci AB = AD, adică ABCD este romb. Să rezolvăm acum şi directa. Fie ABCD
romb. VOm folosi numere complexe. Fie {O} = AC ∩ BD. Alegem AC să fie axa
imaginară şi BD axa reală. Fie A(ia), B(b), C(−ia), D(−b) cu a, b > 0 şi M (z).
Atunci, inegalitatea devine

4|z − ia||z + ia| + 4|z − b||z + b| ≥ 4a2 + 4b2 ⇔ |z + a2 | + |z − b2 | ≥ a2 + b2

care rezultă imediat din inegalitatea modului.


162

12.4 Clasa a X-a


1. f = g ◦ h ⇒ h este injectivă ⇒ dacă n < m funcţia f nu poate fi scrisă ca f = g ◦ h.
Dacă n ≥ m funcţia h poate fi aleasă ı̂n(n − 1) . . . (n − m + 1) moduri iar funcţia
g este unic determinată ı̂punctele din mulţimea B ∩ Im h iar restul punctelor din
domeniul de definiţie este o funcţie oarecare deci valorile ei ı̂acele puncte pot fi
alese ı̂(n − m)p moduri.
Deci funcţia f poate fi scrisă ca f = g ◦ h = n(n − 1) . . . (n − m + 1) · (n − m)p .
k
2. Şirul (an )n≥0 este progresie geometrică ⇒ ak = a! 0 · r , unde r este raţia progresiei
m
X n   n
X n   n
X n  
⇒ ak = a0 · r k = a0 rk = a0 (r + 1)n este pătrat perfect
k=0
k k=0
k k=0
k
pentru orice n.
Pentru n par ⇒ a0 este pătrat perfect, a0 = x2 ⇒ (1+r)n este pătrat perfect pentru
orice n.
Pentru n impar ⇒ r + 1 este pătrat perfect, r + 1 = q 2 .
Deci an = x2 (q 2 − 1)n .

3. Vom folosi inegalitatea mediilor: pentru două numere reale pozitive a şi b avem
a+b √
că ≥ ab.
2
1 1
Presupunem prin absurd că ecuaţia 5x +6 x +7x+ x = 58 admite cel puţin o soluţie
ı̂mulţimea numerelor reale.
1 1
Dacă x ≤ 0 ⇒ 5x ≤ 1, 6 x ≤ 1, 7x+ x ≤⇒ ecuaţia nu are soluţii ı̂acest caz.
1 1
Dacă x > 0 ⇒ 58 = 5x + 6 x + 7x+ x = 58 nu admite soluţii ı̂mulţimea numerelor
reale.

4. Vom folosi umrătoarea identitate


     
n n n+1
+ = .
k k+1 k+1
n   n  
X p+k X p+k
an = = =
k
k=1 k=1
p
       
p+1 p+1 p+2 p+n
= + + + ... + −1=
p+1 p p p
 
p+n+1 (p + 2)(p + 3) . . . (p + n + 1)
= −1= − 1.
p+1 n!
163

p−1
Noi vom aplica formula obţinută pentru an doar când 1 ≤ n ≤ deci putem
2
să lucrăm cu an ı̂inelul Zp şi av4m
(p + 2)(p + 3) . . . (p + n + 1)
an = − 1 = 2 · 3 · . . . · (n + 1) · (n!)−1 − 1 =
n!
= (n + 1)! · (n!)−1 − 1 = n + 1 − 1 = n
p−1
adică an ≡ n (mod p) pentru 1 ≤ n ≤ .
2
Astfel că
   
p−1 p−1 p−1 p−1 p−1
2 2 · +1 · 2 +1
X
2
X
2 2 2 2 (p − 1)p(p + 1)
ak = n = = ,
k=1 k=1
6 24
p−1
2
X
dar p ≥ 5 deci (p, 24) = 1 ⇒ a2k se divide prin p.
k=1

5. Vom arăta că x, y, z se află simultan ı̂unul din următoarele intervale: (−∞, 0],
(0, 1], (1, ∞). Presupunem că unul dintre ele se află ı̂primul interval, de ex. x ⇒
1 − ax
≤ 0 ⇒ toate celelalte fracţii sunt mai mici decât 0 şi rezultă că toate cele-
1−a
1 − ax
lalte numere sunt mai mici decât 0. În al treilea caz ⇒ ≥ 1 ⇒ toate
1−a
celelalte sunt mai mari decât 1. Dacă nici unul dintre x, y, z nu aparţin primului
sau ultimului interval, atunci ele sunt toate ıal doilea interval. Acum vom arăta
1 − ax 1 − ax
că x ı̂primul şi al treilea interval ≤ x iar pentru al doilea ≥ x, din
1−a 1−a
care scriind ı̂ncă două relaţii asemănătoare pentru celelalte două necunoscute şi
adunându-le obţinem imposibilitatea egalităţii din enunţ cu excepţia cazurilor
1 − ax
când are loc egalitatea = x. Studiem comportarea faţă de 1 a funcţiei
1−a
ax + x(1 − a). Derivând de două ori această funcţie obţinem că ea este convexă.
În 1 şi 0 valoarea funcţiei este aceeaşi 1. Deci ı̂ntre 0 şi 1 funcţia ia valori mai mici
decât 1, iar ı̂rest valori mai mari decât 1, ceea ce se rescrie ca inegalităţile formu-
late mai sus. Egalitatea are loc doar ı̂0 sau 1. Se obţin singurele soluţii (0,0,0) şi
(1,1,1).

6. Fie a > 1 cazul a < 1 tratându-se analog.


Dacă x > 1 ⇒ ax > a ⇒ loga y < 0 ⇒ y < 1 ⇒ ay < a ⇒ loga z > 0 ⇒ z > 1 ⇒
az > a ⇒ loga x < 0 ⇒ x < 1 contradicţie.
Dacă x < 1 ⇒ ax < a ⇒ loga y > 0 ⇒ y > 1 ⇒ ay > a ⇒ loga z < 0 ⇒ z < 1 ⇒
az < a ⇒ loga x > 0 ⇒ x > 1 contradicţie.
164

Deci x = 1 ⇒ ax = a ⇒ loga y = 0 ⇒ y = 1 ⇒ ay = a ⇒ loga z = 0 ⇒ z = 1 ⇒


x = y = z = 1 soluţie unică.

7. Desenând cercul de rază 1 cu centrul ı̂origine să ”aplicăm” numărul complex


(vectorul) z1 ı̂punctul A(z = 1). Obţinem un punct B care conform ipotezei este
tot pe cerc. Să ”aplicăm” z2 ı̂A, şi obţinem un al treilea punct C, care conform
√ se scrie√abc = 2(a + b) ⇒
ipotezei este tot pe cerc. Cu notaţiile obişnuite relaţia (b)
S · 4R = 2(a + b) = 2ab sin C = 2(a + b) ⇒ ab ≥ 2 ab ⇒ ab ≥ 4, unde S
este aria triunghiului, iar relaţiile de mai sus au loc dacă a, b 6= 0. Dar evident
ultima relaţie implică că AC, BC să fi diametre ı̂cerc şi pentru ca inegalitatea să
π
aibă loc mai este nevoie şi ca sin C = 1, deci C = , imposibil. Atunci a sau b
2
este 0 de unde rezultă uşor că şi celălalt este tot 0, de unde singurele soluţii sunt
z1 = z2 = 0.

8. Problemă greşită. Enunţul modifiat este:


2x−1 (ax + bx ) + x(ax + bx ) = (a + b)x + x(a + b).
Vom trata problema pe intervale. Luăm separat intervalele (−∞, 0], (0, 1], (1, ∞).
Vom arăta că ı̂primul şi ultimul caz primul termen din partea stângă este mai
mare sau egal cu primul din partea dreaptă, iar al doilea din partea stângă este
mai mare sau egal cu al doilea din partea dreaptă, iar pe al doilea interval relaţiile
sunt invers. Va rezulta că egalitatea este posibilă doar pentru x = 0, 1. Funcţia
f (y) = y x este convexă pentru x ı̂primul sau ultimul interval şi concavă pentru al
ax + b x
doilea interval, de aici şi din inegalitatea lui Jensen rezultă ≥ (respectiv
 x 2
a+b
≤) . Pentru al doilea termen lucrurile sunt şi mai simple.
2
9. Fie z = |z|(cos α + i sin α) cu α ∈ (0, 2π).
|1 + z 2n+1 | ≤ 1 = |1 + |z|2n+1 (cos(2n + 1)α + i sin(2n + 1)α| ≤ 1
⇒ 2|x|2n+1 cos(2n + 1)α + |z|4n+2 ≤ 1 ⇒ cos(2n + 1)α < 0 pentru orice n număr
natural.  
π 3π
cos α < 0 ⇒ α ∈ , .
2 2
 π
Fie α = π + ε, cu ε ∈ R, ε ∈ 0, ⇒ cos(2n + 1)α = cos(π + (2n + 1)ε) < 0.
2
π π
Evident există n astfel ı̂ncât (2n + 1)ε şi (2n + 3)ε > .
2 2  

Dar cos(π + (2n + 1)ε) < 0 şi cos(π + (2n + 3)ε) < 0 ⇒ π + (2n + 1)ε ∈ π, şi
  2
5π 7π 5π 3π π
π + (2n + 3)ε ∈ , ⇒ 2ε > − ⇒ ε > contradicţie.
2 2 2 2 2
165

În
 mod analog se ajunge la o contradicţie ı̂cazul ı̂care α = π − ε, cu ε ∈ R, ε ∈
π
0, .
2
Deci α = π ⇒ z = −|z| ⇒ |1 − |z|2n+1 ≤ 1.
Dacă |z| > 1 există n astfel ı̂ncât |z|2n+1 > 2 ⇒ contradicţie.
Deci mulţimea soluţiilor este S = {z | z ∈ R, z ∈ [−1, 0]}.

10. Problemă greşită.Enunţul modificat este (1−sin x)(1−sin y)(1−sin z) ≥ sin x sin y sin z.
 
1
Vom folosi inegalitatea lui Jensen pentru funcţisa ln − 1 . Pentru aceasta
sin x
ı̂mpărţim inegalitatea cu sin x sin y sin z şi logaritmăm rezultatul obţinut. Funcţia
care a fost obţinută are adoua derivată
 şi pozitivă
 deci esteconvexă. Aplicând
X 1 1 X 1
Jensen, minorăm ln − 1 cu 3 ln − 3 . Dar din Cauchy
sin x 3 sin x
X 1 X 3
avem sin x sin ≥ 9, din Jensen obţinem rezultatul clasic sin x ≤
X 1 sin x 2
deci avem ≥ 6 care 1nlocuit mai sus dă rezultatul dorit. Egalitatea se
sin x
π
atinge când x = y = z = .
6
11. Fie G centrul de greutate al tetraedrului, ha ı̂nălţimea din A a tetraedrului, S aria
feţelor tetraedrului, V volumul tetraedrului şi AG ∩ (BCD) = A1 .
Înălţimea tetraedrului din A şi perpendiculara din M pe planul (BCD) sunt par-
d(G, (BCD)) A1 G 1 ha
alele, coplanare ⇒ = = ⇒ d(G, (BCD)) = .
ha A1 A 4 4
3V
Dar 3V = ha · S ⇒ d(G, (BCD)) = .
4S
3V
Analog se demonstrează că = d(G, (ABC)) = d(G, (DAB)) = d(G, (CDA)) =
4S
d(G, (BCD)) ⇒ G este şi centrul sferei ı̂nscrise ı̂tetraedrul ABCD.
Deci ı̂ntr-un tetraedrul echifacial centrul sferei ı̂nscrise coincide cu centrul de
greutate.

12. Fie ABCD tetraedrul.


Cum muchiile opuse sunt două câte două perpendiculare ⇒ AB 2 +CD2 = AC 2 +
BD2 = AD2 + BC 2 .
Fie AB = x cea mai mică muchie şi r > 0 raţia progresiei muchiilor ⇒ CD este
cea mai mare muchie. Deci celelalte muchii se află ı̂mulţimea {x + r, x + 2r, x +
3r, x + 4r} şi suma pătratelor a două dintre ele este egală cu suma pătratelor
celorlalte două ⇒ x2 + (x + 5r)2 = (x + r)2 (x + 4r)2 = (x + 2r)2 + (x + 3r)2 ⇒ r = 0
tetraedrul ABCD este regulat.
166

13. Fie ABCD tetraedrul M, N, P mijloacele muchiilor AB, AC respectiv AD, r


raza sferei ı̂nscrise ı̂tetraedru, V volumul şi aria [XY Z] aria triunghiului XY Z.
d(A, (BCD))
Evident (M N P ) k (BCD), (BCD) = .
2
” ⇒ ” Tetraedrul este median ⇒ sfera ı̂nsrisă ı̂tetraedru este tangentă planelor
paralele (M N P ) şi (BCD) ⇒ d((M N P ), (BCD)) = 2r ⇒ d(A, (BCD)) = 4r.
3V
Dar 3V = d(A, (BCD)) · aria[BCD] ⇒ aria[BCD] = .
4r
În mod analog se demonstrează că aria[ABC] = aria[BCD] = aria[CDA] =
3V
aria[DAB] = ⇒ tetraedrul ABCD este echifacial.
4r
” ⇐ ” Tetraedrul ABCD este median ⇒ ariile feţelor sunt egale ⇒ ı̂nălţimile
tetraedrului sunt egale.
1
Este cunoscut că raza sferei ı̂nscrise ı̂tetraedrul echifacial este din ı̂nălţimea
4
acestuia.
Deci 2r = d((M N P ), (BCD)), iar (M N P ) k (BCD) şi cum sfera ı̂nscrisă este
tangentă planului (BCD) ⇒ sfera ı̂nscrisă este tangentă şi planului (M N P ).
Deci tetraedrul este median.

14. Fie vol(XY ZT ) volumul tetraedrului XY ZT , iar V volumul tetraedrului ABCD.


Ducem perpendicularele din A şi M pe planul (BCD). Cele două drepte sunt
paralele, coplanare ⇒

M A0 d(M, (BCD)) vol(M BCD) AM V − vol(M BCD)


0
= = ⇒ 0
= .
AA d(A, (BCD)) V AA V

Aplicăm inegalitatea lui Cauchy-Buniakowski-Schwartz:


r !2 X 2
X
2
X 1 X 1 AM
AM ≥ AM 2 · = =
AA02 AA02 AA0

X 2  2
V − vol(M BCD) 4V − V X X 1
= = ⇒ AM 2 ≥ 9.
V V AA02

15. Problemă greşită. Enunţul nu poate fi reconstituit.

16. Fie MA , MB , MC , MD mijloacele segmentelor AHA , N HB , CHC , DHD respectiv.


Fie S = P A2 + P B 2 + P C 2 + P D2 − 2P H 2 ,
167

Este cunoscută Relaţia lui Sylvester pentru triunghiul BCD:


−→ −−−→
−−−→ −−→ −→ −−→−−−→ OA + OHA 1 −→ −−→ −→ −−→
OHA = OB + OC + ODOMA = OA + OB + OC + OD .
2 2
În mod analog se demonstează că:

−−−→ −−−→ −−−→ −−−→ 1 −→ −−→ −→ −−→


OMA = OMB = OMC = OMD = (OA + OB + OC + OD) ⇒
2
⇒ MA = MB = MC = MD
⇒ dreptele AHA , BHB , CHC , DHD sunt concurente ı̂ntr-un punct notat cu H şi
−−→ 1 −→ −−→ −→ −−→
OH = (OA + OB + OC + OD).
2
Cum H este mijlocul segmentului ABA ⇒

2P A2 + 2P BA2 − AHA2
2P B 2 = ⇒ S = P B 2 + P C 2 + P D2 + AHA2 − P HA2 .
2
Din relaţia lui Stewart se obţine uşor că pentru orice punct P din plan şi tri-
unghiul BCD cu centrul de greutate G, avem

P B 2 + P C 2 + P D2 = GB 2 + GC 2 + GD2 + 2P G2 ⇒

⇒ S = AHA2 + GB 2 + GC 2 + GD2 + 3P G2 − P HA2 .


GO 1
Dar G ∈ (OHA ) şi = . Aplicând relaţia lui Stewart pentru triunghiul
GHA 2
P OHA şi punctul G pe OHA avem că

2
3P G2 − P HA2 = 2P OA
2
· ⇒
3
⇒ S = AHA2 + GB 2 + GC 2 + GD2 + 2P O2 − OHA2 ⇒ S
este constantă.
168
Capitolul 13

Probleme avute ı̂n atenţia comisiei, 2005

13.1 Clasa a VII-a


1. Metoda I. Fie numărul abc care conţine numai cifre impare, deci a, b, c ∈ {1, 3, 5, 7, 9}.
Rezultă că sunt 53 = 125 astfel de numere. Fiecare cifră apare de acelaşi număr
de ori, deci de 125 : 5 = 25 de ori pe poziţia unităţilor, tot de 25 de ori pe poziţia
zecilor şi la fel de 25 de ori pe poziţia sutelor.
Suma tuturor acestor numere va fi:
Si = 25(1 + 3 + 5 + 7 + 9)(102 + 10 + 1) = 69375.

Fie acum numărul def format numai cu cifre pare, deci din mulţimea {0, 2, 4, 6, 8}.
Cum d 6= 0 sunt 4 · 5 · 5 = 100 astfel de numere. Pe poziţia unităţilor şi zecilor
cele 5 cifre pare apar cu aceeaşi frecvenţă, deci fiecare cifră apare de 20 de ori la
unităţi şi tot de 20 de ori la zeci. Pe poziţia sutelor apar numai cele 4 cifre nenule,
deci fiecare apare de 100 : 4 = 25 de ori.
Suma tuturor numerelor ce au numai cifre pare va fi:
Sp = 20(0 + 2 + 4 + 6 + 8)(10 + 1) + 25(2 + 4 + 6 + 8) · 102 = 54400.
Suma tuturor numerelor de trei cifre fiind:
999 · 1000 99 · 100
S= − = 494550
2 2
suma cifrelor cerute, ce au atât cifre pare cât şi cifre impare va fi:
S − Si − Sp = 494550 − 69375 − 54400 = 370775.

Metoda II. Suma numerelor de trei cifre care au numai cifre impare cuprinse ı̂ntre
100 şi 200 se face prin calcul direct:
111 + 113 + 115 + 117 + 119 + 131 + 133 + . . . + 197 + 199 = 3875.

169
170

Se observă că la fiecare creştere cu 2 a sutelor suma celor 25 de numere creşte cu


200 · 25 = 5000, astfel suma dintre 300 şi 400 va fi 3875 + 5000, suma dintre 500 şi
600 va fi 3875 + 2 · 5000, etc, astfel suma numerelor de trei cifre (cu cifre impare)
Si va fi:

Si = 3875 · 5 + 5000 + 2 · 5000 + 3 · 5000 + 4 · 5000 = 69375.

Analog, suma numerelor cu cifre exclusiv pare cuprinse ı̂ntre 200 şi 300 se face
uşor: 200 + 202 + 204 + 206 + 208 + 220 + 222 + . . . + 286 + 288 = 6100.
Exact ca şi anterior se observă că suma numerelor cu cifre pare cuprinse ı̂ntre 400
şi 500 va fi cu 200 · 25 mai mare, cea dintre 600 şi 700 cu 2 · 5000 = 10000 mai mare
şi cea dintre 800 şi 900 cu 3 · 5000 = 15000 mai mare.
Suma numerelor cu cifre pare Sp va fi:

Sp = 6100 · 4 + 5000 + 2 · 5000 + 3 · 5000 = 54400.

Ţinând seama că suma numerelor de 3 cifre este S = 494550 rezultă că suma
numerelor de trei cifre ce au atât cifre pare cât şi cifre impare este:

S − Si − Sp = 494550 − 69375 − 54400 = 370775.

2
2. Se duc toate cele C2005 mediatoare ale segmentelor formate cu cele 2005 puncte
date. Se ia un punct O care să nu fie pe nici una din mediatoarele respective. Se
notează punctele ı̂ncepând cu cel mai apropiat, astfel că

OA1 < OA2 < OA3 < . . . < OA2004 < OA2005 .

(Se observă că OAi 6= OAj , pentru că O nu poate fi pe mediatoarea segmentului
Ai Aj ). Cercul de rază OA1003 va conţine ı̂n interior punctele A1 , A2 , . . . , A1002 , deci
1002 puncte, iar ı̂n exterior A1004 , A1005 , . . . , A2005 , adică tot 1002 puncte.
Observaţie: condiţia ca oricare trei puncte să fie necoliniare nu este necesară,
demonstraţia de mai sus rămâne valabilă.

3. Se scade 1 din ambii membri şi avem

2(n − 1)ab + n − 1 = 2005 sau (n − 1)(2ab + 1) = 5 · 401.

Numărul ab fiind de două cifre 2ab + 1 > 5 şi rămâne 2ab + 1 = 401, care este
prim. Rezultă ab = 200 care nu convine. Problema nu are soluţie.
Observaţie: Modificând uşor enunţul avem egalitatea

(n − 1)ab2 + n = 2006 ⇒ (n − 1)(ab2 + 1) = 5 · 401


171

de unde ab2 = 400, problema este tot fără soluţie. O altă modificare ar putea fi:
(n − 1)ab2 − n = 2004 care duce la (n − 1)(ab2 − 1) = 5 · 401 de unde ab = 40 şi
n = 6.

4. Se observă că
. .
a + 4b + 3c − d..13 ⇔ 13(a + b + c + d) − (a + 4b + 3c − d)..13
. .
⇔ 12a + 9b + 10c + d..13 ⇔ 76 · 13a + 12a + 7 · 13b + 9b + 10c + d..13
. .
⇔ 1000a + 100b + 10c + d..13 ⇔ abcd..13.

.
Condiţia se reduce la abcd..13, condiţie ı̂ndeplinită de toate numerele cuprinse
ı̂ntre 1000 şi 9999 care se ı̂mpart la 13, adică 693 numere.
Cum a 6= 5, eliminăm cei 77 multipli de 13 din intervalul [5000, 6000]. În final,
vom avea 693 − 77 = 616 numere care ı̂ndeplinesc condiţiile problemei.

0 0 0
5. Fie P Q ∩ BC = {M }. Atunci ^B BC ≡ ^B BA (pentru că BB este bisectoare) şi
0 0 0 0
^B BA ≡ ^BB M (alterne interne). Rezultă ^B BC ≡ ^BB M de unde

B 0 M = BM. (1)

Avem P M k AB ⇒
AP = BM (2)
0
PQ DP CM BM
şi P M k AB ⇒ = = = , de unde
AB DA CB AB
P Q = B 0 M. (3)

Din (1), (2) şi (3) rezultă AP = P Q deci P AQ este isoscel.


Observaţie: La analiza iniţială a problemei este util de considerat cazul particular
când ABCD este dreptunghi, deci m(^AP Q) = 90◦ , ceea ce ne arată că vârful
triunghiului isoscel P AQ nu poate fi decât ı̂n P .

6. a) 4ABN ≡ 4BCM de unde m(^ABN ) = m(^BCM ) = x, m(^AN B) =


m(^BM C) = y şi x + y = 90◦ . Rezultă ^M QB) = 180◦ − (x + y) = 90◦ deci
CM ⊥ BN .
b) m(^N QC)+m(^N DC) = 180◦ , deci N DCQ este patrulater inscriptibil rezultă
m(^DQC) = m(^DN C), dar din 4DCN = 4ABN (cc) avem m(^DN C) = y.
Cum m(^DCQ) = 90◦ − x = y, rezultă m(^DQC) = m(^DCQ) = y deci DQ =
DC = AB.
172

c) Dacă E este mijlocul lui QC, DE ⊥ QC, pentru că DQ = DC. Rezultă DP k
N B, deci EP este linie mijlocie ı̂n 4BCQ şi P este mijlocul lui BC. Dacă P este
mijlocul lui BC, 4CDP = 4ABN şi m(^CDP ) = x, de unde m(^DEC) =
180◦ − (x + y) = 90◦ şi DE este ı̂nălţime ı̂n 4QDC isoscel, deci E este mijlocul lui
QC.

d) Dacă E este mijlocul segmentului QC rezultă că P este mijlocul lui BC(c).
Dreapta N P care uneşte mijloacele lui AD şi BC trece prin O. Dacă N, O, P sunt
coliniare rezultă că P este mijlocul lui BC şi din (c) avem că E este mijlocul lui
QC.

7. Folosim metoda reducerii la absurd. Presupunem că AB şi CD nu sunt par-


alele. Ducem prin D o paralelă la AB care taie pe AC ı̂n M şi pe EF ı̂n N .
AE OA
Din asemănarea triunghiurilor 4AEO ∼ 4M N O avem = şi din
MN OM
AB OA
4AOB ∼ 4M OD, = de unde
MD OM

AE MN
= . (1)
AB MD

Dar din enunţ avem


AE CF
= . (2)
AB CD
CF MN
Din (1) şi (2) rezultă că = de unde CM k N F sau AC k EF absurd.
CD MD
Rezultă că presupunerea că AB şi CD nu ar fi paralele este falsă şi AB k CD.
1
Deoarece r = , rezultă că EF k AD. Cum punctele E, O, F sunt coliniare,
2
ı̂nseamnă că O este mijlocul segmentelor [BD] şi [AC], deoarece [ED] este linie
mijlocie ı̂triunghiurile ABD şi respectiv ABC. Deci patrulaterul ABCD este par-
alelogram.

Observaţie. Este evident faptul că ipoteza suplimentară este abundentă. În
1
demonstraţia dată am folosit numai r = . Lăsăm ca temă cititorului să demon-
2
AD + BC
streze concluzia folosind numai ipoteza suplimentară EF = .
2

32005 −1
8. Se calculează a = 2
şi b = 22005 − 2501 . Rezultă că:

x = 2a − b + 22004 · 32004 = 32005 − 1 − 22005 + 2501 + 62004 ,


173

de unde, folosind şi relaţia (a + b)n = Ma + bn , obţinem:

x = (35 )401 − (25 )401 + (63 )668 + 2501 − 1 =


= 243401 − 32401 + (211 + 5)668 + 2501 − 1 =
= 243401 − 32401 + 211 · k + 5668 + 2501 − 1 =
= 243401 − 32401 + 211 · k + (54 )167 + (23 )167 .

. . .
Însă 243401 − 32401 ..243 − 32 deci 243401 − 32401 ..211 şi (54 )167 + (23 )167 ..54 + 23 deci
. .
(54 )167 + (23 )167 ..633..211. Aşadar 2a − b + 22004 · 32004 = 211q − 1 = 211q + 210 deci
0

restul este 210.

9. a) Considerăm două cazuri n = 2k şi n = 2k + 1


I) n = 2k
√ √ √ √ √
En (x) = |x − 1| + |x − 2| + . . . + |x − k| + |x − k + 1| + . . . + |x − 2k. Folosim
inegalitatea |x − A| + |x − B| ≥ |A − B| şi avem
√ √ √ √ √ √
En (x) ≥ 2k − 1 + 2k − 1 − 2 + . . . + k + 1 − k =
√ √ √ √ √ √
= 1 + 2 + . . . + 2k − 2( 1 + 2 + . . . + k) = S1 − 2S2 .
Pornim de la inegalitatea
2 a3 − b 3
b< · <a
3 a2 − b 2
valabilă dacă b < a. Demonstraţia se face prin calcul direct.
√ √
Facem a = p + 1 şi b = p şi rezultă

√ 2 p p p
p < ( (p + 1)3 − p3 ) < p + 1
3

Se face p = 1, 2, ..., k − 1 şi se adună inegalităţile. Rezultă


2 √ 3 2 √ √
( k − 1) + 1 < S2 < ( k 3 − 1) + k
3 3
şi apoi
2 √ 3 2 p √
( 8k − 1) + 1 < S1 < ( (2k)3 − 1) + 2k
3 3
Avem
2k 4k √ 1 4k √ 4 √
> En (x) ≥ S1 − 2S2 > S1 min − 2S2 max = 2k + − k+ −2 k
3 3 3 3 3
Inegalitatea este valabilă numai pentru k = 1 şi k = 2.
174

În aceste cazuri se verifică direct soluţiile x, observând că x < 0 şi x > m nu
convin ı̂n mod evident.
Pentru n = 2, x = 1 este soluţie pentru a) şi b).
Pentru n = 4, x = 1, 2, 3, 4 nu sunt soluţii.
Analog
II. n = 2k + 1 duce la o inegalitate adevărată pentru n = 1 şin = 3.
Pentru n = 3, x = 1, 2, 3 nu sunt soluţii.
Rămâne n = 1, caz ı̂n care
1
|x − 1| < ⇒x=1
3
b) Concluziile de la a) rămân valabile, avem n = 1
1
|x − 1| < ⇒ x = 1.
4

10. Determinăm minimul expresiei:


100a + 10b + c 99a + 9b + a + b + c 1 99a + 9b
E= = = + (1)
11(a + b + c) 11(a + b + c) 11 11(a + b + c)
Fracţia este minimă pentru c maxim, deci c = 9
1 9 11a + b 1 9 10a − 9 + a + b + 9 1 9 9 10a − 9
E= + · = + · = + + ·
11 11 a + b + 9 11 11 a+b+9 11 11 11 a + b + 9
a, b, c 6= 0, deci E este minim pentru b maxim, b = 9
10 9 10a − 9 10 9 10a + 180 − 189 10 90 9 189
E= + · = + · = + − · .
11 11 a + 18 11 11 a + 18 11 11 11 a + 18
E este minim pentru cazul a minim, deci a = 1
199 199
Rezultă Emin = = . Determinăm maximul expresiei. Din (1)
19 + 99 + 91 209
rezultă c minim, deci c = 1.
1 9 11a + b 1 9 10a − 1 + a + b + 1 10 9 10a − 1
E= + · = + · = + ·
11 11 a + b + 1 11 11 a+b+1 11 11 a + b + 1
E este maxim pentru b minim, deci b = 1
10 9 10a − 1 10 9 10a + 20 − 21 10 90 9 · 21 1
E= + · = + · = + − ·
11 11 a + 2 11 11 a+2 11 11 11 a+2
911 911
E maxim pentru a + 2 maxim, a = 9. Rezultă Emax = 91+11+19
= 121
.
175

11. Notăm cu a şi b numerele de pe cele două bile. Vom evalua numărul cazurilor
.
favorabile, adică numărul cazurilor ı̂n care a · b..12. În funcţie de forma lui a dis-
tingem 6 situaţii:
.
Situaţia I: a prim cu 12 (17 numere) deci b..12 (4 numere), rezultă 17 · 4 = 68 cazuri.
. .
Situaţia II: a..2, dar a nu se divide la 3 şi a nu se divide la 4 (9 numere) deci b..6 (8
numere), rezultă 9 · 8 = 72cazuri.
. .
Situaţia III: a..3, dar a nu se divide la 2 (8 numere) deci b..4 (12 numere), rezultă
8 · 12 = 96 cazuri.
. .
Situaţia IV: a..4, dar a nu se divide la 3 (8 numere) deci b..3 (16 numere), rezultă
8 · 16 = 128 cazuri.
. .
Situaţia V: a..6 dar a nu se divide la 12 (4 numere) deci b..2 (25 numere) rezultă
4·25−4 = 96 cazuri. S-au scos cele 4 cazuri când a = b, adică (6; 6), (18; 18), (30; 30)
şi (42; 42).
.
Situaţia VI: a..12 (4 numere) deci b oarecare (50 numere), rezultă 4 · 50 − 4 = 196
cazuri. S-au scos cele 4 cazuri când a = b, cazuri imposibile (12; 12), (24; 24), (36; 36), (48; 48).
Avem aşadar 68 + 72 + 96 + 128 + 96 + 196 = 656 cazuri favorabile. Dacă nu se ţine
seama de ordinea a, b rezultă 656 : 2 = 328 cazuri favorabile. Numărul cazurilor
2 328
posibile va fi C50 = 25 · 49 = 1225 cazuri. Probabilitatea va fi = 26, 75 . . . %
1225

12. a) Implicaţia directă (fig. 1). Presupunem că

AM = BN. (1)

Din 4AP M ∼ 4CP D ⇒


AP AM
= . (2)
PC CD
Din 4BQN ∼ 4DQC ⇒
NQ BN
= . (3)
QC CD
AP NQ
Din (1), (2), (3) rezultă
= deci P Q k AB.
PC QC
La fel din 4ASN ∼ 4CSD avem

AS AN
= . (4)
SC CD
Din 4M RB ∼ 4CRD rezultă

MR MB
= . (5)
RC CD
176

AS MR
Din (4) şi (5) dacă se ţine seama că AN = M B rezultă = deci SR k AB.
SC RC
Aşadar P QRS este trapez cu laturile paralele cu AB şi CD. Reciproc, dacă P QRS
este trapez, să arătăm că AM = BN .
Vom nota (fig. 2) AM = x şi BN = y, CD = b şi AB = B. Presupunem că
P Q k SR şi notăm AC ∩ BD = {O}, CM ∩ P Q = {E}, DN ∩ P Q = {F }. Din
SR k P Q avem
OS OR
= , (6)
SP RQ
Aplicând teorema lui Menelaus 4OP Q şi transversalei CE:

CO P E QR
· · = 1. (7)
CP EQ RO

precum şi transversalei DF :

DO QF SP
· · = 1. (8)
DQ F P OS

Ţinând seama de (7), (8) şi (6) rezultă

DO CO PE PF
· = · . (9)
CO DQ EQ F Q

Se exprimă succesiv

b b b b
DO = DB, CO = CA, CP = CA, DQ = DB
b+B b+B b+x b+y
de unde
DO CP b+y
· = . (10)
CO DQ b+x

Exprimând ariile ı̂n două feluri avem

PE CP sin ^P CE x AC sin ^P CE
= şi = ,
EQ CQ sin ^QCE M N CN sin ^QCE
PE CP CN x b b+y x b+y x
de unde = · · = · · = · .
EQ AC CQ M N b+x b MN b + x MN
PF b + y MN
Analog = · deci
FQ b+x y
 2
PE PF b+y x
· = · . (11)
EQ F Q b+x y
177

Înlocuind ı̂n (9) expresiile (10) şi (11) avem


 2
b+y b+y x
= ·
b+x b+x y
b+y y
de unde = sau x = y deci AM = N B.
b+x x
b) P QRS trapez, rezultă x = y şi P Q k SR k AB, deci condiţia este CS = AP . Dar
CS b AP x
= şi = . Din egalarea expresiilor rezultăx2 − Bx + b2 = 0
AC  B + b− x AC b + x
2 2
r
B B B B2
sau x − = − b2 , de unde condiţia B ≥ 2b şi x − = = b2 .
2 2 2 4
Cele două soluţii duc la acelaşi trapez, pentru că x1 + x2 = B.

BC 2 0
13. Soluţia 1. = . Notăm BC = 2x, BN = N C = x, CD = 5x. Fie AA ⊥
0
DC 5 0 0 0
CD, AA = BC = 2x, 4AA D isoscel, A D = 2x, deci CA = 3x.
Notăm DN ∩ AB = {M } 4M BN ≡ 4DCN (CU ), deci M B = CD = 5x AC =
√ √ √
BC 2 + AB 2 = 4x2 + 9x2 = x 13
Aplicăm teorema lui Menelaus pentru 4ABC şi transversala M F :
M A BN CF 8x x CF CF 5 CF 5
· · =1⇒ · · =1⇒ = ⇒ =
MB NC F A 5x x F A FA 8 AC 13
5 BC 2
deci CF = √ x şi cos ^BCA = =√ .
13 AC 13
Se află N F cu teorema cosinusului ı̂n 4N CF :

NF = N C 2 + CF 2 − 2N C · CF · cos ^BCA =
s r
25 5 2 2
= x2 + x2 − 2x √ x √ = 3 x.
13 13 13 13

Din acelaşi triunghi se exprimă


18 2
N F 2 + CF 2 − N C 2 13
x + 25 x2 − x2 1
cos ^N F C = = q 13 =√
2N F · CF 2
2 · 3 13 · √513 x2 2

deci m(^N F C) = 45◦ iar m(^AF N ) = 180◦ − 45◦ = 135◦ .


Soluţia 2. Vârfurile A, B, C, D ale trapezului aparţin unei reţele de pătrate
de latură l = BN . Considerăm paralelogramul N CAA0 , atunci triunghiul N A0 D
este, evident dreptunghic isoscel (?!). Înseamnă că m(∠A0 N, N D) = m(∠AC, N D) =
45◦ . Deci m(∠AF N ) < 135◦ .
178

14. Se roteşte triunghiul ABC ı̂n jurul punctului B, ı̂n sens invers orar. A ajunge ı̂n
0 0 0 0
A , C ajunge ı̂n A şi M ı̂n M . m(^M BM ) = 60◦ , M B = M 0 B, deci 4M BB este
0 0
echilateral şi M M = M B, M A = M C.
0 0
Relaţia dată M A2 +M B 2 = M C 2 devine M A2 +M M 2 = M A2 , de unde rezultă că
0 0 0
4M AM este dreptunghic ı̂n M , deci m(^AM B) = m(^AM M )+m(^BM M ) =
90◦ + 60◦ = 150◦ .
0 0
Dacă considerăm că m(^AM B) = 150◦ , rezultă m(^AM M ) = 150◦ −m(^BM M ) =
0 0
150◦ − 60◦ = 90◦ , deci M A2 + M M 2 = M A2 sau M A2 + M B 2 = M C 2 .

15. Numărul se scrie succesiv


r
r 102k − 1 10k − 1 2 p 2k
44 . . . 4 − 88 . . . 8 = 4 − 8 = 10 − 1 − 2 · 10k + 2 =
| {z } | {z } 9 9 3
2k cifre k cifre
2 2
= · (10k − 1) = · (10 − 1) · (10k−1 + 10k−2 + . . . + 1) =
3 3
= 6 · (10k−1 + 10k−2 + . . . + 1) = |66 {z
. . . 6}.
k cifre

Numărul este natural şi divizibil cu 3. Se observă că problema este valabilă şi
pentru k = 1.

TB TQ TA TQ
16. Din M N k P Q rezultă = (1). Din N P k QM avem = (2). Din
TN TC TN TE
TB TE
raportul relaţiilor (1) şi (2) avem: = , de unde BE k AC, deci ABEC este
TA TC
trapez.

PQ PT
17. Din asemănarea triunghiurilor N T S şi P T Q avem = (1). Din asemănarea
NS NT
MN NT
triunghiurilor M N T şi RT P avem = (2). Înmulţind relaţiile (1) şi (2),
PR PT
ţinând seama că M N = P Q rezultă P Q2 = P R · N S.

AB
18. a) Din EM = = AM rezultă ^M EA ≡ ^M AE ≡ ^DAE adică AD k M N (1)
2
AB
Din EM = = BM avem ^M EB ≡ ^M BE ≡ ^CBE adică BC k M N (2)
2
Din (1) şi (2) rezultă AD k BC, deci ABCD este trapez
b) Avem M E k BC şi N F k BC, deci punctele M, E, N, F sunt coliniare,
aflându-se pe linia mijlocie a trapezului. Urmează că
|(AB + CD) − (AD + BC)
EF = |M N − (EM + F N )| = .
2
179

19. Din teorema bisectoarei ı̂n 4ABD şi din teorema lui Thales avem:

AB BM AP
= = . (1)
AD MD PD
Din teorema bisectoarei ı̂n 4ADC şi din teorema lui Thales avem:

AC NC AP
= = . (2)
AD ND PD
Din (1) şi (2) avem AB = AC, deci ABC este isoscel.

20. Soluţia 1. Fie O centrul cercului. Din motive de simetrie, O, E şi F sunt coliniare.
Notăm OF ∩ CD = {M } şi OF ∩ AB = {N }
AB
Pentru a simplifica expunerea soluţiei, notăm cu R raza cercului, AN = =a
2
CD
şi DM = = b.
2
√ √ √
Se calculează succesiv ON = R2 − a2 , OM = R ≥2 −b2 , M N = R2 − b2 −

R 2 − a2 √ √
MF b MF b b · ( R 2 − b 2 − R 2 − a2 )
Din M D k N A avem = de unde = şi M F = .
NF a MN a−b a−b
Rezultă √ √
a R 2 − b 2 − b R 2 − a2
OF = OM + M F = . (1)
a−b
√ √
NE a a · ( R 2 − b 2 − R 2 − a2 )
Din AB k CD avem = şi N E = de unde
MN a+b a+b
√ √
b R 2 − a2 + a R 2 − b 2
OE = ON + N E = . (2)
a+b
Din (1) şi (2) avem

a2 · (R2 − b2 ) − b2 · (R2 − a2 )
OF · OE = = R2
a2 − b 2
de unde rezultă că triunghiul OT1 F este dreptunghic ı̂n T1 , deci F T1 este tangentă
la cerc.
_
Soluţia 2. Fie F T10 tangenta la cerc T10 ∈ AD. Atunci F T102 = F A · F D. Avem:
_ _
_ m(AD) + m(BC)
m(∠AOD) = m(AD) = = m(∠AED), deci patrulaterul AEOD
2
este inscriptibil. Prin urmare F A · F D = F E · F O. Atunci F T 02 = F E · F O şi,
conform reciprocei teoremei catetei, alicată ı̂DF T 0 O, avem T10 E ⊥ F O. Urmează
că T1 = T10 şi problema este rezolvată.
180

21. Tangenta comuna interioară B1 C1 intersectează cercul C1 ı̂n Q şi C2 ı̂n P . Folosim
proprietatea tangentele duse dintr-un punct la un cerc sunt egale şi rezultă:

B1 T2 = B1 P = x, C1 U1 = C1 Q = y, apoi
B1 T1 = B1 Q sau T1 T2 − x = x + P Q (1) şi
C1 U2 = C1 P sau U1 U2 − y = y + P Q (2)

Din T1 T2 = U1 U2 şi din relaţiile (1) şi (2) avem x = y. Rezultă succesiv:

T1 T2 = T1 B1 + x = B1 Q + x = B1 Q + y = B1 C1
U1 U2 = y + C1 U2 = y + C1 P = x + C1 P = B1 C1

Avem T1 T2 + U1 U2 = 2B1 C1 . Analog se demonstrează

T2 T3 + U2 U3 = 2B2 C2

şi aşa mai departe


Tn−1 Tn + Un−1 Un = 2Bn−1 Cn−1 .

Prin adunarea relaţiilor de mai sus avem

T1 Tn + U1 Un = 2(B1 C1 + B2 C2 + . . . + Bn−1 Cn−1 ).

22. Notăm xk = k − ak ≥ 0, de unde k = ak + xk


Inegalitatea devine

a1 · a2 · . . . · a100 + x1 · x2 · . . . · x100 < (a1 + x1 ) · (a2 + x2 ) · . . . · (a100 + x100 ).

Produsul din membrul drept conţine 2100 termeni pozitivi dintre care 2 sunt chiar
cei doi termeni din membrul stâng.
Observaţie. a1 = 1, a2 = 2, . . . , a100 = 100 satisface condiţiile din enunţ şi duce la
egalitate.

0 0
23. Metoda I. Fie AA ı̂nălţimea triunghiului ABC, N M ∩ AA = {T }, N M ∩ AC =
0 0 0
{N }. Atunci AN = 2c cos A, AT = AN cos(901◦ −C) = c2 cos A
sin C
= R cos A. Fie
0 0 0
P mijlocul segmentului BC. Avem: ^BOP ≡ ^A ⇒ OP = R cos A(1) şi
4BN M ≡ 4AT M (^N BM ≡ ^T AM alterne interne; BM = M A şi ^N M B ≡
^T M A opuse la vârf) de unde BN = AT = R cos A (2).
0 0 0
Din (1) şi (2) ⇒ BN = OP , deci BN OP dreptunghi şi N P = OB = R.
0 0
Cum N P = R ⇒ N P = N P , deci P ≡ P şi P este mijlocul laturii BC.
181

0
Metoda II. Fie P mijlocul segmentului BC şi E punctul diametral opus lui A.
EC k BH (ambele fiind perpendiculare pe AC) (1)
EB k CH (ambele fiind perpendiculare pe AB) (2).
0
Din (1) şi 2 rezultă că BHCE este paralelogram, deci E, P , H sunt coliniare şi
0 0 0 0
EP = P H. Înseamnă că OP este linie mijlocie ı̂n triunghiul AEH şi OP = AH 2
.
AH 0 0
Dar 2 = AT pentru că M N este linie mijlocie ı̂n triunghiul ABB . Aşadar
0
OP = AT . Dar AT = BN , pentru că 4AM T ≡ 4BM N (U LU ) Rezultă că
0 0 0
OP = BN , deci BN OP este dreptunghi, N P = OB = R, deci P este identic cu
0
P .
Metoda III. Fie O centrul cercului circumscris triunghiului ABC şi D mijlocul
laturii [BC]. Avem OD ⊥ BC şi OM ⊥ AB decu patrulaterul BDOM este in-
scriptibil. Pe de altă parte, M D k AC, deci M D ⊥ N E adică patrularerul M N BD
este inscriptibil. Avem:

180◦ = m(∠M N B) + m(∠BDM ) = m(∠M N B) + m(∠M OB)

şi deci patrulaterul M N BO este inscriptibil. Atunci m(∠BM O) = m(∠BN O) =


90◦ . Înseamnă că patrulaterul BDON este dreptunghic, deci N D = OB. Cum
toate implicaţiile sunt echivalente, rezultă concluzia.

24. Soluţia 1. AEDF este paralelogram. AD ∩ EF = {O}, EO = OF (1), ^EAO ≡


^F AO (2).
Din (1) şi (2) ⇒ AE = AF deci AEDF este romb. Notăm AE = AF = F D =
DE = x, SAEDF = x2 sin A.
Inegalitatea se scrie:
1 x AC
x2 sin A ≤ AB · AC sau sin A ≤ . (1)
4 AB 4x
x FD CD AC AC BC
Rezultă = = (2) iar = = (3).
AB AB BC x ED BD
Înlocuind ı̂n (1) relaţiile (2) şi (3), avem

CD 1 BC
sin A ≤ sau 4 · BD · CD sin A ≤ BC 2 .
BC 4 BD
Dar BC 2 = (BD+CD)2 = BD2 +CD2 +2·BD·CD ≥ 4BD·CD ≥ 4BD·CD·sin A
aşa ı̂ncât relaţia (1) este demonstrată.
2S
Soluţia 2. Fie h = d(D, AC) = d(D, AB) şi S = SABC . Avem h = . Dacă
AC + AB
EC AC AB
y = d(E, BC), avem y = ·h = ·h. Analog x = d(F, BC) = ·h. Analog
DC BC BC
182

AB
x = d(F, BC) = · h.
BC
1 AB 1 DC · AC
S1 = SBF D + SDEC = BD · ·h+ · ·h=
2 BC 2 BC
 
1 AB 1 AC
= · h AB · + · AC · =
2 AB + AC 2 AB + AC
1 AB 2 + AC 2 AB 2 + AC 2 1
= ·h· =S· ≥ S · .
2 (AB + AC) (AB + AC)2 2
S S AB · AC · sin A AB · AC
SAF DE = S − S1 ≤ S − = = ≤ .
2 2 4 4

25. a) Produsul cifrelor lui 2005 fiind nul, perechea magică ar fi (0, 2005) care nu se
consideră acceptabilă. Căutăm o pereche magică de forma (2005, b), deci pro-
dusul cifrelor lui b ar fi 5 · 401, dar 401 fiind prim, nu poate fi scris ca un produs
de cifre, deci 2005 nu este prietenos.
b) Considerând inacceptabilă soluţia (0, 200 . . . 0}5), rămâne să căutăm perechea
| {z
de n ori
(200 . . . 0}5, b). Produsul cifrelor lui b trebuie să fie
| {z
de n ori

200 . . . 0}5 = 5 · 4 00
| {z . . . 10}
| {z
de n ori de n−1 ori

Cum 400 . . . 0}1 nu se divide la 2, 3, 4, 5, 6, 8, 9, singura posibilitate ar fi să se scrie


| {z
de n−1 ori
ca produs de cifre 7. Ar urma ca 400 . . . 0}1 = 7k sau 400 . . . 00 = 7k − 1 imposibil,
| {z
de n−1 ori
pentru că membrul drept se divide la 3, iar cel stâng nu.

26. a) Presupunem problema rezolvată, fie M ∈ AB şi N ∈ AC astfel ı̂ncât AM +


M N +N A = AB sau M N +N A = M B. Prelungim segmentul M N cu N P = AN ,
de unde M P = M B. Triunghiul AN P fiind isoscel, m(^N AP ) = m(^N P A) = x
iar din triunghiul BM P isoscel, avem m(^M BP ) = m(^M P B) = y.
Rezultă m(^AM N ) = 2y şi m(^AN M ) = 2x, de unde x + y = 90◦ − A2 . Înseamnă
că m(^AP B) = x + y = 90◦ − A2 =constant. Punctul P se află pe arcul capabil de
unghiul 90 − A2 care trece prin punctele A şi B. (Din construcţia acestui arc rezultă
că centrul O al lui se află pe bisectoarea unghiului A). Cum M P = M B, P se află
şi pe cercul cu centrul ı̂n M şi de rază M B.
În concluzie, P se află la intersecţia acestui cerc cu cercul cu centrul O. Cele două
cercuri trec prin punctul comun B, deci P este unic. Rezultă N la intersecţia lui
M P cu AC.
183

Observaţie. Este evident că problema nu funcţionează decât dacă AM < M B.


0
b) Vom face demonstraţia pentru M ∈ AB. Fie M ∈ (AB), astfel ı̂ncât BM =
0 0 0
2BM . Fie B ∈ (AB astfel ca AB = 2AB.
0
Se reia demonstraţia de la punctul a) cu condiţia AM +M N +N A = AB şi se află
0
P la intersecţia cercului cu centrul ı̂n O0 cu cercul cu centrul ı̂n M şi rază M B .
0 0
Se iau acum M (mijlocul segmentului M B) şi N mijlocul segmentului N C. Rezultă:
0 0 0 0 0
AM + M N + N A > AM + M N + N A = AB = 2 · AB.

13.2 Clasa a VIII-a


a
1. (a) Fie N N 0 ⊥AM, N 0 ∈ AM . Atunci M N 0 = N C şi N C = N 0 A = . Rezultă că
√ 2
a 5 √
NB = NM = şi cum M B = a 2 putem uşor calcula aria ∆N M B isoscel;
2 √
a2 6
obţinem că AN M B = . Exprimăm volumul tetraedrului ABM N ı̂n două
4
moduri. Fie CP ⊥AB; deoarece CP ⊥AM (AM ⊥ (ABC) , CP ⊂ (ABC)), atunci
CP ⊥ (ABM ). Însă N Ck k (ABM ), deci d (N, (ABM )) = d (C, (ABM )) = CP .
Aşadar:
AABM · CP AN M B · d (A, (M BN ))
VABM N = = ,
3 3
prin urmare
a2 a 3
√ √
AABM · CP 2
· 2 a 2
d (A, (M BN )) = = √ = .
AN M B a2 6 2
4

(b) Observăm că N CN 0 M este paralelogram ([N C] şi [M N 0 ] sunt paralele


 şi con-

0
gruente), deci [N M ] şi [CN ] sunt paralele şi congruente. Astfel, m M\ N, BC =
  √
m N \ 0 CB . Triunghiul N 0 BC este isoscel cu N 0 C = N 0 B = a 5/2 şi cu BC = a;
  √
0
cu uşurinţă obţinem că sin N CB = 2 5/5.
\

2. (a) Evident că M este mijlocul lui [BC] şi N este mijlocul lui [AC]; atunci [M N ]
a 
este linie mijlocie ı̂n triunghiul ABC, deci M N kAB, M N = . Rezultă că m DM, \ AB =
    2
m DM N , iar cos DM N se calculează uşor din ∆DM N isoscel cu DM =
\ \
√   √
DN = a 3/2; obţinem cos DM \ N = 3/6.

(b) Dacă M N ∩ OC = {Q}, atunci OQ⊥M N . Cum DO⊥ (ABC), din teorema
celor trei perpendiculare rezultă că DQ⊥M N . Dreapta M N este dreapta comună
184
   
a planelor (ABC) şi (DM N ),deci m (DM N\) , (ABC) = = m DQO \ .Tangenta
 
unghiului o aflăm din triunghiul DOQ cu m O b = 900 ; obţinem că

a 6 √
\ = DO =
 
3
tg DQO √ = 4 2.
OQ 1 a 3
·
6 2

(c)

Calculând ı̂n două moduri volumul tetraedrului DM N C obţinem că d (C, (DM N )) =
a 22
22
.

3. Cu notaţia 2x − 1 = y ⇔ x = (y + 1) /2 din prima inegalitate obţinem f (y) ≤


2y − 1, ∀y ∈ R. Apoi, cu notaţia x + 2 = z ⇔ x = z − 2 a doua inegalitate devine
f (z) ≤ 2z − 1, ∀z ∈ R. Din aceste două relaţii rezultă că f (x) = 2x − 1, ∀x ∈ R.
Atunci

(f (x − 1))2 + (f (3 + x))2 + 16x = (2x − 3)2 + (2x + 5)2 + 16x


= 2 (2x + 3)2 + 16 ≥ 16,

deci valoarea minimă a expresiei date este 16, atinsă pentru x = −3/2; apoi

(f (2005))2 + 4f (1005) − 17 = 40092 + 4 · 2009 − 17


= 40092 + 2 · 4009 + 1
= 40102 .

4. Evident că CD0 ∩ (M N P ) = CD0 ∩ N P şi, cum S ∈ [CD0 , iar ∆C 0 CD0 este isoscel,
ı̂n mod necesar vom avea că P C 0 < N C 0 . (1)
Fie N 0 =pr(ABC) N, M 0 =pr(BCC 0 ) M. Atunci ∆AN 0 C =pr(ABC) ∆M N P şi M 0 C 0 P =pr(BCC 0 ) ∆M N
Folosind formula care dă aria proiecţiei, obţinem
a
AAN 0 C = AM N P · cos α ⇒ cos α = · N C 0 . (2)
2AM N P
a
AM 0 C 0 P = AM N P · cos β ⇒ cos β = · P C 0 . (3)
2AM N P
(Am notat cu a muchia cubului, iar cu α, β unghiurile pe care planul (M N P ) le
face cu planele (ABC), respectiv (BCC 0 )).
Din relaţiile (1) , (2) şi (3) obţinem cos β < cos α, prin urmare β > α, concluzia
dorită.
Să remarcăm că rezultatul are loc ı̂n condiţii mai puţin restrictive decât cele ale
problemei; nu este nevoie ca D0 să fie mijlocul lui [CS], ci este destul ca C 0 N >
C 0P !
185

5. (a) Fiindcă d(D, (ABB 0 )) =d(D, (ABA0 )), vom calcula distanţa cerută exprimând
ı̂n două moduri volumul tetraedrului A0 ABD:
1 1
VAA0 BD = AABD · A0 O = AA0 AB · d (D, (ABA0 )) ,
3 3
de unde
A0 O · AABD
d (D, (ABA0 )) = .
AA0 AB
Ştim că AC = 16, deci AO = 8 şi din ∆AOB obţinem că OB = 6. Apoi, din

∆AOA0 găsim că A0 O = 6, iar din ∆A0 OB că A0 B = 6 2. Aria triunghiului isoscel
√ √
A0 AB cu AB = AA0 = 10, A0 B = 6 2 va fi 6 41cm2 , iar AABD = (AO · BD) /2 =
48cm2 . Atunci √
0 6 · 48 48 41
d (D, (ABA )) = √ = cm.
6 41 41
(b) Dreapta comună celor două plane este AB. Fie OP ⊥AB; dinteorema
 celortrei
0 \ 0 0 P O . În
perpendiculare obţinem că A P ⊥AB, deci m (ABC) , (ABB ) = m A \
OA · OB 24
∆OAB dreptunghic, găsim că OP = = şi atunci
AB 5
0P O =
A0 O 5
tgA
\ = .
OP 4
(c) Cum [AB] şi [C 0 D0 ] sunt paralele şi congruente, rezultă că ABC 0 D0 este par-
alelogram, deci BC 0 kAD0 . Însă AD0 ⊂ (ACD0 ) şi atunci avem că BC 0 k (ACD0 ),
de unde rezultă că d(BC 0 , AC) =d(BC 0 , (ACD0 )) = =d(B, (ACD0 )). Pentru a cal-
cula această din urmă distanţă, exprimăm ı̂n două moduri volumul tetraedrului
D0ABC :
1 1
VD0 ABC = AABC · d (D0 , (ABC)) = AD0 AC · d (B, (D0 AC)) ,
3 3
de unde
AABC · d (D0 , (ABC))
d (B, (D0 AC)) = .
AD0 AC
AC · OB
Avem că d(D0 , (ABC)) =d(A0 , (ABC)) = A0 O = 6, iar AABC = = = 48.
√ √ 2
Din motive de simetrie, D0 C = A0 B = 6 2, iar AD0 = AB 0 = 82 ( calcul imediat
ı̂n rombul ABB 0 A0 ). Aria ∆D0 AB o calculăm cu formula lui Heron; găsim că

3 83
AD0 AC = .
2

0 192 83
Atunci d(B, (D AC)) = cm.
83
186

6. Din inegalitatea mediilor obţinem



r 
x  x x
2 x − a2 = 2 a −a ≤a+ −a=
a a a
şi analoagele, de unde, prin sumare, găsim
√ p √ 1 x y z 
x − a2 + y − b2 + z − c 2 ≤ + + .
2 a b c
x y z
Ipoteza ne arată că are loc egalitatea, deci − a = a, − b = b, − c = c, adică
a b c
x = 2a2 , y = 2b2 , z = 2c2 . Rezultă că x + y + z = 2 (a2 + b2 + c2 ) ≤ 2 · 3 = 6. Este
clar că egalitatea se atinge pentru a = b = c = 1, x = y = z = 2.

7. Considerând egalitatea
1 1 1 1 1
+ + ... + = − ,
k (k + 1) (k + 1) (k + 2) (k + n − 1) (k + n) k k+n
pentru fiecare n ≥ 2 obţinem soluţia

x1 = k (k + 1) , x2 = (k + 1) (k + 2) , ..., xn−1 = (k + n − 1) (k + n) ,
xn = k + n, y = k,

unde k ∈ N∗ este oarecare. Aşadar ecuaţia dată are o infinitate de soluţii.


Alte soluţii se pot obţine plecând de la egalitatea
1 1 1 1
+ 2 + ... + n−1 = 1 − n−1
2 2 2 2
şi considerând, pentru fiecare k ∈ N∗ ,

x1 = 2k, x2 = 22 k, ..., xn−1 = 2n−1 k, xn = 2n−1 k, y = k.

8. Avem evident că ∆AM Q ≡ ∆BN M (L.U.L.), deci M Q = M N . În ∆M N Q isoscel


mediana M O va fi şi ı̂nălţime: N Q⊥M O. Analog N Q⊥P O şi de aici rezultă că
N Q⊥ (M OP ), dar numai dacă M, O, P nu sunt coliniare! Observăm că M, O, P
sunt coliniare dacă şi numai dacă M, N, P, Q sunt coplanare. Conform teoremei
lui Menelaus ı̂n spaţiu, aceasta se ı̂ntâmplă când:
AM BN CP DQ
· · · = 1 ⇔ AM = M B.
M B N C P D QA
Prin urmare, dacă M, N, P, Q sunt mijloacele laturilor corespunzătoare, problema
nu are sens.
187

9. Din teorema lui Ceva aplicată ı̂n ∆A0 B 0 C 0 obţinem că


A0 E CD B 0 M
· · = 1,
EC DB 0 M A0
CE CD
şi, cum A0 M = M B 0 , deducem că 0
= . Reciproca teoremei lui Thales
0 0
EA DB 0
arată că DEkA B , deci DEkAB. Rezultă că AEDB este trapez, de unde concurenţa
dreptelor AE, BD. Astfel, dreptele AE, BD, CC0 sunt trei drepte necoplanare,
două câte două concurente; de aici rezultă că toate trei sunt concurente.

10. (a) Cum M N kAC şi N P kBC, rezultă că (M N P ) k (ABC). Fie A0, B0, C0 mijloacele
segmentelor [AM ] , [CN ] , [BP ]; atunci este evident că (M N P ) k k (M 0 N 0 P 0 ) k (ABC),
iar cele trei plane sunt şi echidistante.
Linia mijlocie a unui trapez conţine mijloacele diagonalelor acestuia, prin urmare
X ∈ [M 0 N 0 ] , Y ∈ [N 0 P 0 ] , Z ∈ [P 0 M 0 ], deci (XY Z) = (M 0 N 0 P 0 ) şi de aici rezultă
concluzia.

(b) Fie U mijlocul lui [BN ]; atunci U XkAB( linie mijlocie ı̂n ∆N AB), U Y kBC(
segmentul ce uneşte mijloacele diagonalelor
 unui trapez este inclus
 ı̂n linia mi-
jlocie), de unde rezultă că m XU
\ Y ∈ {600 , 1200 }, iar m XY
\ U ∈ {450 , 1350 } .
Cele două unghiuri nu pot fi ambele
 obtuze
 şi atunci
 rămân
 de studiat trei cazuri;
singurul care convine va fi m XU \ Y = 1200 , m XY \ U = 450 ( pentru celelalte
două calculele sunt asemănătoare, dar conduc la contradicţii!).
Fie x = M D, y = AD, a = AB = BC = CA. Atunci:
 
ax a BC − P N a x
MP = = P N = M N, XU = , Y U = = 1−
y 2 2 2 y
şi, din teorema sinusurilor, obţinem:

XU YU x 3− 3
= ⇔ = ,
sin 450 sin 150 y 2
MD √
de unde, folosind proporţiile derivate, deducem că = 3.
DA

11. (a) Avem

f (x) = 2k ⇔ 2k−1 < x ≤ 2k . (1)


g (x) = 2l ⇔ 2l ≤ x < 2l+1 . (2)

Atunci f (x) = g (x) ⇔ x = 2m , x ∈ A ⇔ x ∈ {20 , 21 , ..., 210 }. Rezultă că mulţimea


{x ∈ A |f (x) = g (x)} are 11 elemente.
188

(b) Folosind relaţiile (1) şi (2) deducem că

1 1 1 1 1 1 981 981
+ + ... + = 1 + + + ... + + 11 = 6 + 11 .
f (1) f (2) f (2005) |2 2 {z 2} 2 2
de 10 ori

981
Însă ' 0, 48, de unde concluzia.
211
p p
12. Notăm x = OA, y = OB, z = OC; atunci AB = x2 + y 2 , BC = y 2 + z 2 , AC =

z 2 + x2 . Din inegalitatea dintre media armonică şi media pătratică avem
r   √
2 x2 + y 2 1 1 1 2
1 1 ≤ ⇔ + ≥p , ∀x, y > 0
x
+ y
2 2 x y x2 + y 2

şi analoagele, care, prin sumare, conduc la concluzia problemei. Egalitatea are
loc pentru OA = OB = OC.

13. Pentru a = 4, b = 20k 2 + 8k, k ∈ N, expresia dată este

a + b + ab = 4 + 20k 2 + 8k + 80k 2 + 32k = 100k 2 + 40k + 4 = (10k + 2)2 ,

deci pentru o infinitate de valori pare ale lui b se obţine un pătrat perfect.

14. Poliedrul considerat are 8 feţe: paralelogramele AA”B0B”, B0B”CC”, CC”D0D”,


AA”D0D” şi triunghiurile AB”D”, CB”D”, A”B0D0, C”B0D0. Pentru calculul
volumului acestui poliedru, vom scădea din volumul cubului volumele tetrae-
drelor A0A”B0D0 şi C0D0B0C”( care sunt egale), precum şi volumele piramidelor
patrulatere ABB”D”D şi CBB”D”D( care sunt din nou egale).
Cum A0 B 0 ⊥ (A0 A00 D0 ), atunci

1 1 9 9
VAA00 D0 B 0 = · A0 B 0 · AA0 A00 D0 = · 3 · = cm3 .
3 3 4 4
Apoi, fiindcă AO⊥BD şi AO⊥BB 00 (BB 00 ⊥ (ABC) , AO ⊂ (ABC)), rezultă că AO⊥ (BD00 D),
deci √ √
1 1 3 2 9 2 9
VABB 00 D00 D = · AO · ABB 00 D00 D = · · = cm3 .
3 3 2 2 2
Volumul poliedrului dat va fi

9 9
V = 27 − 2 · − 2 · = 13, 5cm3 .
4 2
189

b
15. Din prima inegalitate, pentru t = x − , obţinem că
a
a b
f (t) ≤ t2 + , ∀t ∈ R.
b a
b
Din a doua inegalitate, pentru t = x + , deducem că
a
a b
f (t) ≥ t2 + , ∀t ∈ R
b a
a b
şi ı̂n consecinţă vom avea că f (x) = x2 + , ∀x ∈ R.
b a

16. (a) Din inegalitatea mediilor, (a + b + c)3 ≥ 27abc, ∀a, b, c ≥ 0, deducem că (a + b + c)3 ≥
27 (a + b + c) şi, cum a + b + c > 0, atunci

(a + b + c)3 ≥ 27 ⇔ a + b + c ≥ 3 3.
√ √ √ 
Cum tripletul (a, b, c) = 3, 3, 3 verifică ipoteza, valoarea minimă a sumei

este 3 3.

(b) Cum abc 6= 0, relaţia dată revine la


1 1 1
+ + = 1,
ab bc ca
1 1 1
deci, ı̂n primă instanţă, la rezolvarea ı̂n Z a ecuaţiei
+ + = 1.
x y z
Datorită simetriei putem presupune |x| ≤ |y| ≤ |z|. Dacă |x| ≥ 3, atunci

1 1 1 1 1 1 1 1 1
+ + ≤ + + ≤ + + = 1,
x y z |x| |y| |z| 3 3 3

cu egalitate doar dacă x = y = z = 3. În acest caz ab = bc = ca = 3, (abc)2 = 27,


imposibil ı̂n Z. Rămân de studiat situaţiile x ∈ {±1, ±2}.
1 1
i) Dacă x = 1, atunci + = 0, deci y = −z. Rezultă că ab = 1, ac = −bc, de unde
y z
a = −b, imposibil.
1 1 z
ii) Dacă x = −1, atunci + = 2, y = ∈ Z, deci
y z 2z − 1
2z − 1 | z ⇔ 2z − 1 | 2z − (2z − 1) ⇔ 2z − 1 ∈ {−1, 1} ⇔ z ∈ {0, 1} .

Convine doar când z = 1, pentru care y = 1, de unde ab = −1, ac = bc = 1, prin


urmare (abc)2 = −1, imposibil.
190

1 1 1
iii) Dacă x = 2, atunci + = şi, procedând la fel ca ı̂n cazul precedent,
y z 2
obţinem că z ∈ {−2, 0, 1, 3, 4, 6}. Convin situaţiile (y, z) ∈ {(1, −2) ; (3, 6) ; (4, 4)},
prin urmare
(ab, ac, bc) ∈ {(2, 1, −2) ; (2, 3, 6) ; (2, 4, 4)} .
Atunci (abc)2 ∈ {−4, 36, 32} şi convine doar situaţia (abc)2 = 36, de unde abc ∈
{−6, 6}. Soluţiile vor fi (a, b, c) ∈ {(−3, −2, −1) ; (3, 2, 1)} precum şi cele care se
obţin prin permutări.
1 1 3
iv) Dacă x = −2, atunci + = şi, procedând la fel ca ı̂n cazurile precedente,
y z 2
obţinem că z ∈ {0, 1, 2}. De aici (y, z) = (1, 2) şi nu vom găsi noi soluţii.
În concluzie,

(a, b, c) ∈ {(1, 2, 3) ; (1, 3, 2) ; (2, 1, 3) ; (2, 3, 1) ; (3, 1, 2) ; (3, 2, 1) ;


(−1, −2, −3) ; (−1, −3, −2) ; (−2, −1, −3) ; (−2, −3, −1) ;
(−3, −1, −2) ; (−3, −2, −1)} .

17. Cum exponentul 2a2 − 1 este un număr impar, avem descompunerea


 2 2

xa = (a + 1) a2a −2 − a2a −3 + ... − a + 1 .

În mod evident, cei doi factori ai acestui produs sunt strict mai mari decât 1.Vom
2 2
arăta că a + 1 şi ya = a2a −2 − a2a −3 + ... − a + 1 sunt numere prime ı̂ntre ele.
Observăm că
 2   2 
ya − 2a2 − 1 = a2a −2 − 1 − a2a −3 + 1 + ... − (a + 1) + (1 − 1)


= M (a + 1)

şi atunci

ya = M (a + 1) + 2a2 − 1
= M (a + 1) + 2 a2 − 1 + 1,


de unde
ya = M (a + 1) + 1,
prin urmare (ya , a + 1) = 1 şi rezolvarea este ı̂ncheiată.

18. Notăm xy = a, x + y = b; relaţia din enunţ devine

(a − p)2 = b2 − 2a ⇔ a2 − 2ap + p2 = b2 − 2a ⇔
⇔ a2 − 2a (p − 1) + (p − 1)2 + 2p − 1 = b2 ,
191

de unde

b2 − (a − p + 1)2 = 2p − 1 ⇔
(b − a + p − 1) (b + a − p + 1) = 2p − 1.

Însă 2p − 1 este număr prim, deci rămân de studiat 4 cazuri:


i) b − a + p − 1 = 1, b + a − p + 1 = 2p − 1;
ii) b − a + p − 1 = 2p − 1, b + a − p + 1 = 1;
iii) b − a + p − 1 = −1, b + a − p + 1 = −2p + 1;
iv) b − a + p − 1 = −2p + 1, b + a − p + 1 = −1.
După studiul acestor 4 cazuri, ţinând seama şi de faptul că p este număr prim,
obţinem că perechile căutate sunt {(0, p) ; (p, 0)} când p şi 2p − 1 sunt simultan
prime şi p 6= 7, şi {(0, 7) ; (7, 0) ; (3, 4) ; (4, 3)} dacă p = 7.

19. Dacă A, B, HA , HB sunt conciclice, evident că sunt şi necoplanare.


Reciproc, fie A, B, HA , HB coplanare şi fie {T } = BHA ∩CD, {T 0 } = = AHB ∩CD;
atunci T, T 0 ∈ (ABHA ) ∩ CD, prin urmare T = T 0 .
Avem   ∆HAT C ∼ ∆DT B(
 asemănarea  ambele
 triunghiuri sunt dreptunghice, iar
0
m H A T C = m T BD = 90 − m D ), rezultă că
\ \ b

HA T TC
= ⇒ HA T · T B = T C · T D.
DT TB

Analog se demonstrează că HB T · T A = T C · T D, prin urmare HA T · T B =


= HB T ·T A. Aplicând reciproca puterii punctului faţă de cerc, rezultă că punctele
A, B, HA , HB sunt conciclice.

20. (a) Dacă A0 , B 0 , C 0 , D0 s-ar afla ı̂n acelaşi semispaţiu faţă de planul (ABC), condiţia
necesară şi suficientă pentru ca ele să fie coplanare ar fi AA0 + +CC 0 = BB 0 +DD0 .
Însă AA0 + CC 0 ≥ CC 0 = 100 > 60 = BB 0 + DD0 , de unde rezultă că această
situaţie este imposibilă. Prin urmare A0 se află ı̂n semispaţiul opus celui ı̂n care se
află B 0 , C 0 , D0 , iar ı̂n acest caz condiţia de coplanaritate a punctelor A0 , B 0 , C 0 , D0
este |CC 0 − AA0 | = BB 0 + DD0 , de unde obţinem că AA0 = 40cm.

(b) Dacă BC ∩B 0 C 0 = {M } , CD∩C 0 D0 = {N }, atunci (ABC)∩(A0 B 0 C 0 ) = = M N .


Folosind asemănările ∆M BB 0 ∼ ∆M CC 0 şi ∆N DD0 ∼ ∆N CC 0 obţinem că
5 5
CM = · BC = 60cm şi CN = · BC = 80cm.
4 3
192

Rezultă că
CM · CN CM · CN
d (C, M N ) = =√ = 48cm.
MN CM 2 + CN 2
21. Pentru x = y = 0, găsim f (0) = 0. Apoi, făcând y = 0, obţinem că
f x2 = f 2 (x) , ∀x ∈ Z (1) ,


de unde rezultă că f 2 (−x) = f (−x)2 = f (x2 ) = f 2 (x) , ∀x ∈ Z, prin urmare




f (−x) = ±f (x) , ∀x ∈ Z.
Însă f este strict crescătoare şi atunci f (−x) = −f (x) , ∀x ∈ Z, adică f este o
funcţie impară, este suficient deci să determinăm f pe N.
Facem ı̂n relaţia (1) x = 1 şi atunci f (1) = f 2 (1), adică f (1) ∈ {0, 1}. Cum
f (0) = 0 şi f este strict crescătoare, nu putem avea f (1) = 0, deci f (1) = 1.
Luând ı̂n ecuaţia iniţială x = y = 1, deducem că f (2) = 2 şi din relaţia (1) se
n n
demonstrează inductiv că f 22 = 22 , ∀n ∈ N. (2)
n n+1
Pentru k ∈ N∗ , există unic n ∈ N pentru care 22 ≤ k < 22 şi atunci, folosind (2)
n n+1
şi stricta monotonie, urmează că 22 ≤ f (k) < 22 . În aceste condiţii, apelând
ı̂ncă o dată la stricta monotonie, obţinem că f (k) = k, ∀k ∈ N; ı̂nsă f este impară,
de unde rezultă că f (x) = x, ∀x ∈ Z.

22. (a) Relativ la congruenţa modulo 5, orice pătrat perfect are una din formele M5, M5+
1 sau M5 + 4; atunci 3b2 poate fi M5, M5 + 3 sau M5 + 2. Singurul caz ı̂n care
5 | a2 + 3b2 este acela ı̂n care 5 | a şi 5 | b.
(b) Avem
x3 − y 3 = 2005 x2 − y 2 ⇔ (x − y) x2 + xy + y 2 − 2005x − 2005y = 0
 

şi observăm că perechile (x, x) , x ∈ N, sunt soluţii ale ecuaţiei.


Pentru x 6= y, avem de rezolvat ecuaţia
x2 + xy + y 2 = 2005 (x + y) ⇔ (2x + y)2 + 3y 2 = 4 · 2005 (x + y) .
Fie 2x + y = a, y = b şi atunci ecuaţia devine a2 + 3b2 = 2 · 2005 (a + b).
Conform celor demonstrate la punctul a), avem ı̂n mod necesar că a = = 5p, b =
5q, adică
25p2 + 3 · 25q 2 = 2 · 2005 · 5 (p + q) ⇔ p2 + 3q 2 = 2 · 401.

Atunci p şi q au aceeaşi paritate (suma fiind pară). Dacă p, q sunt ambele pare, 4 |
p2 + 3q 2 ⇔ 4 | 2 · 401, imposibil, deci nu obţinem soluţii. Dacă p, q sunt ambele
.
impare, atunci p2 , q 2 sunt de forma M4 + 1, deci p2 + 3q 2 ..4 şi iarăşi nu obţinem
soluţii. Rămân doar soluţiile {(x, x) , x ∈ N}.
193

13.3 Clasa a IX-a


1 1 1
1. Să demonstrăm că tripletele (a, b, c)şi ( , , ) sunt la fel ordonate.Aceasta
ma mb mc
reiese din : 4(m2a − m2b ) = 2b2 + 2c2 − a2 − 2c2 − 2a2 + b2 = 3(b2 − a2 ).
1 1
ma ≤ mb ⇒ a ≥ b, ≥ .
ma mb
Folosind Cebâşev avem
X a X X 1 
3 ≥( a) (1)
ma ma
.
X 1 9 1 √ 2  1q X 3 X 2
X q
≥P , ma = 2b + 2c2 − a2 ≤ 3 (2b2 + 2c2 − a2 ) = a
ma ma 2 2 2
X 1 6
⇒ ≥ pP (2)
ma a2
Din (1) şi (2) obţinem inegalitatea cerută.(Matei Vlad)

2. . Din inegalitatea mediilor


p 2 8
3
(a + b)(b + c)(c + a) ≤ (a + b + c) ⇒ (a + b)(b + c)(c + a) ≤ (a + b + c)3 (1)
3 27
.Functia f (x) = x3 este convexă pentru x > 0
3
(a + b + c)3

1 3 3 3 a+b+c
⇒ (a + b + c ) ≥ ⇔ a3 + b 3 + c 3 ≥ (2)
3 3 9
. Din (1) si (2) avem
8
(a + b)(b + c)(c + a) ≤ (a3 + b3 + c3 )
3
. În acest caz avem egalitate ,deci a=b=c=2.(Matei Vlad)

3. Conform Ceva: AN · CM · BP =√ CN · BM ·√P A. Dar BC √· AB · CA = (AN +


N C)(CM + BM )(AP + BP ) ≥ 2 AN · N C · 2 CM · BM · 2 AP · BP = 8 · N A ·
CM · BP (Ilie Livia)

4. p
Folosind faptul ca triunghiurile sunt dreptunghice si aplicând CBS obţinem: a1 a2 =
(b21 + b22 )(c21 + c22 ) ≥ b1 b2 + c1 c2 . Egalitatea se obţine pentru
b1 b2
=
c1 c2
adică atunci când triunghiurile sunt asemenea.(Ilie Livia)
194

−d
5. Substituim x cu ,dacă c 6= 0. Dacă c=0, obţinem |ax + b| ≤ |d|.Luăm x =
c
d−b
+ m, m · d > 0 şi obţinem |d + m| ≤ |d| ,contradicţie. Obţinem
c
−ad −ad
+b ≤0⇒ + b = 0 ⇒ ad = bc.
c c
a c
Putem scrie această relaţie sub forma = . Înlocuind x cu
b d
a c
x = y + = y + ⇒ |ay| ≤ |cy|
b d
şi alegând y diferit de 0 obţinem |a| ≤ |c|.(Matei Vlad)

6. a1 > a2 > ... > an ≡ q(mod2005) ⇒ a1 ≥ (n − 1)2005 + q, a2 ≥ (n − 2)2005 +


q, ..., an = q.Ecuaţia a1 x1 + a2 x2 + ... + an xn = 2005 devine: 2005((n − 1)x1 + (n −
2)x2 + ... + xn−1 ) + q(x1 + x2 + ... + xn ) ≤ 2005 ⇔ q(x1 + x2 + ... + xn ) ≤ 0, dar
x1 + x2 + ... + xn ≥ 0, q ≥ 0.Avem egalitate de unde

a1 = (n − 1)2005 + q, a2 = (n − 2)2005 + q, ..., an = q.

(Matei Vlad)

7. Aplicând CBS obţinem:


a3 c a3 c
X  X X X  X X
2 2
( abc(a+c)) ≥ ( a c) ⇔ 2bac ( a) ≥ ( a2 c)2
b(c + a) b(c + a)
a3 c
X  X  X 
1 a 2
⇔2 ≥ (1).
b(c + a) a b
Tot din CBS obţinem:
X a  X 1  X 1 2

b ab a
(2). De asemenea mai avem
X X a  X Xa X X X X1 X 1
( ab) ≥( a)2 , ≤ a⇒ ab ≥ a⇔ ≥
b b a ab
(3). Din (2)şi (3) se obţine
Xa X1

b a
şi folosind ı̂n (1) obţinem
X a3 c 1 X a  3
≥ ≥
b(a + c) 2 b 2
(Matei Vlad)
195

y z x
8. Dacă abc = 1 ⇒ ∃x, y, z > 0 astfel incât a = , b = ; c = . Inegalitatea
x  y z
x3 x3
X
X 3
( xyz(y+z)) ≥ ( x2 )2 (1).Din
P P
devine ≥ .Din CBS avem
yz(y + z) 2 yz(y + z)
inegalitatea mediilor avem
 3
x+y+z X 2 X 4
xyz ≤ ⇒ 2xyz( x) ≤ ( x) (2).
3 27
X 1 X 2
Tot din CBS avem x2 ≥ ( x) (3).Folosind (2) şi (3) in relaţia (1) obţinem
3
inegalitatea cerută.(Matei Vlad)

X  X
a X
9. Din Cauchy-Buniakowski- Schwartz avem √ ( a b + c) ≥ ( a)2 .(1)
b+c
Tot din CBS avem
X √ X X X √ X X 2
( a b + c)2 ≤ ( a)( a(b+c)) ⇔ ( a b + c)2 ≤ 2( a)( ab) ≤ (a+b+c)3 (2)
3

Înlocuind in (1) avem


r r
X a 3 X a 3
√ ≥ (a + b + c) ⇒ √ ≥
b+c 2 b+c 2

.(Matei Vlad)

10. a)Lemă:Dacă H este ortocentrul triunghiului 4ABC şi O centrul cercului circum-
−−→ −−→ −−→ −−→
scris ,atunci pentru orice punct M din spaţiu avem M H = M A + M B + M C −
−−→
2M O.
Demonstratie: Fie Q punctul diametral opus lui A. Din QC⊥AC, QB⊥AB ⇒
−−→ −−→ −−→ −−→
HBQC paralelogram ⇒ M H = M B + M C − M Q (1). O mijlocul lui AQ ⇒
−−→ −−→
−−→ MA + MQ −−→ −−→ −−→
MO = ⇒ M Q = 2M O − M A. Înlocuind in (1) obtinem relatia
2 −−→ −−→
ceruta. Fie P un punct arbitrar in plan. Folosind lema obtinem:P A1 = P M +
−−→ −→ −→ −−→ −→ −→ −→ −−→ −−→ −−→
P B + P C − 2P O, P B1 = P M + P C + P A − 2P O) ⇒ P A1 − P B1 = P B −
−→
P A, deci A1 B1 kBA, AB = A1 B1 , analog obctinem BC = B1C1, CA = C1 A1 .
Din M A1 ⊥BC; BCkB1 C1 ⇒ M A1 ⊥B1 C1 , analog obţinem M B1 ⊥A1C1, de unde
rezultă că M este ortocentrul 4A1 B1 C1 .
−→ −−−→
b)Din AB = A1 B1 ⇒ ABA1 B1 paralelogram,deci AA1 şi BB1 se intersectează
la mijloc ,analog BB1 şi CC1 .Să demonstrăm acum că AHM A1 este paralelo-
−−→ −−→ −→ −−→ −→ −−→ −→
gram.Avem P H + P M = P A+ P B + P C + P M −2P O,folosind lema.De asemenea
−→ −−→ −→ −−→ −→ −−→ −→ −−→ −−→ −→ −−→
se obţine P A+P A1 = P A+P B+P C+P M −−2P O ⇒ P H+P M = P A+P A1 ,deci
196

AHM A1 paralelogram.Fie R mijlocul lui HM.Cum in două triunghiuri omotetice


T T
liniile lui Euler sunt paralele,avem că GG1 OO1 HM = {R}, deoarece tri-
unghiurile sunt congruente.(Matei Vlad)

11.

12. Dacă amplificăm fiecare număr ai cu t2·3·...·(i+1) , ı̂n partea dreaptă t va apărea la
puterea 1, iar ı̂n parte stângă va apărea la puterea
2 + (2 · 3) + . . . + (2 · 3 · . . . + k + 1)
.
32
Dacă t nu se simplfică din ambele părţi ale inegalităţii, vom putea alege o valoare
a sa pentru care inegalitatea să nu mai fie adevărată. Deci 32 = 2 + (2 · 3) +
. . . + (2 · 3 · . . . · (k + 1)). De aici rezultă k = 3. Înainte de a demonstra că
2·3·...·(i+1)
inegalitatea
r este adevărată pentru k = 3 amplificăm fiecare număr ai cu t
1
cu t = 24 . În urma acestei amplifcări se va obţine a3 = 1. Inegalitea devine
a3
p √ √ √ √
a1 + 3 a2 + 1 ≥ 32 a1 a2 ⇔ a1 + 3 a2 + 1 ≥ 16 a1 a2 Distingem 4 cazuri:
Cazul 1:
√ √ √ √
a1 ≤ 1, a2 ≤ 1 ⇒ 16 a1 a2 ≤ 1, a1 + 3 a2 + 1 ≥ 1 ⇒ a1 + 3 a2 + 1 ≥ 16 a1 a2

Cazul 2:
√ √ √
a1 ≤ 1, a2 ≥ 1 ⇒ a1 a2 ≤ 3 a2 ≤ a1 + 3 a2 + 1
16

Cazul 3:
√ √
3
a1 ≥ 1, a2 ≤ 1 ⇒ 16
a1 a2 ≤ a1 ≤ a1 + a2 + 1

Cazul 4: √ √
√ 8 a1 + 8 a2 √
3
a1 ≥ 1, a2 ≥ 1 ⇒ 16
a1 a2 ≤ ≤ a1 + a2 + 1
2
(Ilie Livia)

13. MN,EF şi PC concurente⇔PC trece prin mijlocul lui MF.Aceasta implicaţie se
T
verifică uşor aplicând teorema lui Ceva .Fie acum P N BC = {R}.Deoarece
BM CN AP BM n CN p AP m
· · = 1 ⇒ ∃m, n, p = ; = ; = .
MC NA P B MC m NA n PB p
Cu teorema lui Menelaus in 4ABCpentru tranversala N-P-R avem:
AN CR P B n CR p CR m
· · =1⇔ · · =1⇒ = .
N C RB P A p RB m RB n
197

Presupunem acum m 6= n şi m > n.Avem


CR m CB m − n BM n BM m−n RB m+n
= ⇔ = , = ⇒ = ⇔ = (1)
RB n RB n BC m+n RB m+n RM 2m
. Cu teorema fundamentală a asemănării in 4RQM, P BkM Q avem
RB PB PB m+n 2m
= ⇔ = ((1)) ⇒ M Q = P B(2).
RM MQ MQ 2m m+n
T
Fie {O} = M Q P C.Cu teorema fundamentală a asemănării in4BP C, M OkP B
MO MC MO m m
= ⇔ = ⇒ MO = P B(3).
PB CB PB m+n m+n
Din (2) şi (3) rezultă că PC trece prin mijlocul lui MQ.(Matei Vlad)

14. (a)
5
f (x) = − 2x − 4
1−x

(b) Să presupunem prin absurd că A ar avea centru de simetrie. Fie acesta punc-
tul de coordonate D(a, b).Translatăm ı̂ntreaga mulţime, astfel ı̂ncât punctul D să
ajungă ı̂n origine. Vom avea o nouă funcţie g care va avea ca grafic mulţimea
translatată g(x − a) = f (x) − b ⇒
5
g(x) = − 4 − 2x − 2a − b
1−x−a
g trebuie să fie o funcţie impară
5 −5
⇒ g(x) = −g(−x) ⇒ − 4 − 2x − 2a − b = + 4 − 2x + 2a + b ⇒
1−x−a 1+x−a
5 5 10 − 10a
+ = 8+4a+2b ⇒ = 8+4a+2b ∀x ∈ R−{1} ⇒
1−x−a 1+x−a (1 − a)2 − x2
absurd.
(c) Distanţa dintre S şi M când M parcurge mulţimea A este de forma:
v !2 r
u

r q
u
tx2 + 2x + 5 2 2
25 2
25
= x + 4x + 2 + 20 = 5x + 2 + 20 ≥ 20 + 2 125 =
x x x
p √
20 + 10 5 Aceasta este lungimea minimă a segmentului şi se obţine pentru
25 √
5x2 =
4
⇒ x = ± 5
x2
(Ilie Livia)
198

15.

16.
BD c2
= 2 ⇒
DC b
−→ 2 −→ 2
−−→ AB · b AC · c
⇒ AD = 2 2
+ 2
b +c b + c2
Teorema transversalei⇒
BM CD CN BD
· + · =1⇒
AM BC AN BC
BM b2 CN c2
⇒ · 2 + · =1⇒
AM b + c2 AN b2 + c2
BM + AM b2 CN + AN c2
⇒ · 2 + · =2⇒
AM b + c2 AN b2 + c 2
c b2 b c2
⇒ · + · =2⇒
AM b2 + c2 AN b2 + c2
b c b2 + c 2
⇒ + =2· ≥4
AM AN bc
constant.(Ilie Livia)

17. ab + bc + ca + 2abc = 1. Notăm ab = x2 ; bc = y 2 ; ca = z 2 . Ecuaţia devine x2 +


y 2 + z 2 + 2xyz = 1, ∆ = 4(x2 y 2h− x2i− y 2 + 1) = 4(1 − x2 )(1 − y 2 ).Cum 0 ≤
π
x ≤ 1, 0 ≤ y ≤ 1 ⇒ că ∃ α, β ∈ 0; ,unice astfel incât x = cos α; y = cos β ⇒
2
∆ = 4 sin2 α sin2 β.Soluţiile sunt z1 = sin α sin β − cos α cos β, z2 = − sin α sin β −
cos α cos β ⇔ z1 = − cos(α + β), z2 = − cos(α − β); z2 < 0,deci nu poate fi soluţie.
Rămâne z1 = − cos(α + β) = cos(π − α − β) = cos γ. Inegalitatea devine

3
cos α + cos β + cos γ ≤ , α + β + γ = π
2
 
α+β+γ
.Funcţia cos x este concavă pe [0; π] ⇒ cos α + cos β + cos γ ≤ 3 cos =
3
3
.(Matei Vlad)
2
T T T
18. Notăm BM AC = {R}, AM BC = {S}, RS AL = {J}.
Cazul I: M ∈ [CD]
Conform Ceva:
RC AD BS AR BS
· · =1⇒ = ⇒ SRkAB ⇒ JS ⊥ AC ⇒ 4JAS
AR DB SC RC SC
199

isos. ⇒ JR = RS
RC RS JR LR
= = = ⇒ LCkAB ⇒
AC AB AB LB
punctele L se află pe paralela dusă prin C la AB.
Cazul II: D ∈ [CM ]
Se demonstrează uşor căRSkABşi că triunghiul AJR este isoscel ⇒ RS = RJ ⇒

CB AB AB BL
⇒ = = = ⇒ RSkCL ⇒ ABkCL.
CS RS RJ AL

Cazul III: C ∈ [DM ]


Din nou se demonstrează uşor aplicând Ceva in triunghiul ABM că RSkAB Şi
imediat rezultă că RS = SJ
RC RS SJ LS
= = = ⇒ CLkRS ⇒ CLkAB.
BC AB AB BL
Vom demonstra că orice punct L0 de pe paralela dusă prin C la AB are propri-
etatea din enunţ şi, prin urmare, acesta este locul geometric căutat. L0 B DC =
T

{M }, L0 B AC = {R0 }, AM BC = {S 0 }, RS AL0 = {J 0 } ⇒ R0 S 0 kAB ⇒


T T T

R0 S” ⊥ AC. Cum LCBA este trapez, iar R intersecţia diagonalelor⇒ J 0 R0 =


R0 S 0 ⇒ 4JAS isos.⇒ ∠LAC = ∠CAS ⇒ LA şi AS sunt simetrice faţă de AC.(Ilie
Livia)

19. Aplicând teorema sinusurilor in triunghiul 4M BC obctinem

a BM a
= ⇒ BM =
sin 70 sin 30 2 sin 70
.Aplicând-o in triunghiul 4AN C obţinem

b AN b
= ⇒ AN = .
sin70 sin 30 2 sin 70
De asemenea avem
c sin 20 c sin 20 c c
BP = AP = = = = .
sin 40 2 sin 20 cos 20 2 cos 20 2 sin 70
Cu teorema cosinusului in 4AN P ⇒ P N 2 = AP 2 + AN 2 − 2AP · AN cos(A +
1 1
60) ⇔ P N 2 = 2 (c2 + b2 − bc cos(A + 60)) = (2b2 + 2c2 − 2bc cos A +
4 sin 70 8 sin2 70
√ 1 2 2 2 2 2
√ 1
2 3bc sin A) = 2 (2b + 2c − b − c + a + 4S 3) = 2 (a2 + b2 + c2 +
√ 8 sin 70 8 sin 70
4S 3)(1). În triunghiul 4BP M avem P M 2 = BP 2 + BM 2 − 2BP · BM cos(B +
200

1 1
60) ⇔ P M 2 = 2 (a2 + c2 − 2ac cos(B + 60)) = 2
(2a2 + 2c2 − 2ac cos B +
4 sin 70 8sin 70
√ 1 2 2 2 2 2
√ 1
2 3ac sin B) = (2a + 2c − a − c + b + 4S 3) = (a2 + b2 + c2 +
√ 2
8sin 70 8 sin2 70
4S 3)(2). Să calculăm acum MN.Avem relaţiile:

a b
CM = sin 70, CN = sin 70
sin 40 sin 40

,din teorema sinusurilor.Cu teorema cosinusului in 4CM N obţinem: M N 2 =


sin2 40 2 2 sin2 40 2 2

(a + b − 2ab cos(C + 60)) = (2a + 2b − 2ab cos C + 2 3bc sin C) =
sin2 70 2 sin2 70
sin2 40 2 2 2 2 2
√ sin2 40 2 2 2

(2a + 2b − a − b + c + 4S 3) = (a + b + c + 4S 3).(3) Din
2 sin2 70 2 sin2 70
(1) şi(2),avem că triunghiul este isoscel,iar din (2) şi (3)obţinem

MN sin 2∠P M N
= 2 sin 40 = 2 cos 50 = = 2 cos ∠P M N ⇒
PN sin ∠P M N

⇒ ∠P M N = 50◦ ⇒ ∠M P N = 80◦ .

(Matei Vlad)

20. Se duce S piciorul perpendicularei din I pe ı̂nălţimea din A.Fie R piciorul ı̂nălţimii
din A şi T piciorul perpendicularei din A pe BC. Cum SRT I este paralelogram,
RS = T I ⇒ ha = R + r ⇔ r = RS ⇔ R = AS. ∠BAS = 90 − ∠B =
∠OAC ⇒ ∠SAI = ∠IAO. Relaţia e chivalenta cu AS = AO ⇔ IO⊥AO(am
folosit IS⊥AS)⇔ ∠ON A = 90 − ∠OAN = ∠B ⇔ BM N C inscriptibil.
Fie D piciorul bisectoarei din A a triunghiului ABC. Triunghiurile AM N şi ACB
sunt asemenea, iar dreapta AD este bisectoare in amandoua⇒

MN AI b+c
= =
BC AD a+b+c

A doua parte a egalităţii rezultă aplicând teorema bisectoarei ı̂n triunghiul ABCşi
Menelaus in triunghiul ADB.Se notează cu E intersecţia lui CI cu AB.

BC ID AE AI b+c b b+c AI b+c


· · =1⇒ = · = ⇒ =
DC AI BE ID b a a AD a+b+c

Din relaţia de mai sus rezultă

a a+b+c a 1 1 1
= =1+ ⇒ = +
MN b+c b+c MN b+c a

(Ilie Livia)
201

21. 2005 ≡ 5 (mod 1000), deci este suficient sâ aflâm ultimele trei cifre ale lui 52005 .
52 ≡ 1 (mod 8) ⇒ 52004 ≡ 1 (mod 8) ⇒ 52005 ≡ 5 (mod 8). Deoarece 125|52005 ⇒
ultimele trei cifre ale numarului pot fi doar

125, 250, 375, 500, 625, 750, 875, 000.

Verificăm congruenţa modulo 8 şi obţinem ca singură soluţie 125. (Ilie Livia)
1 1
22. an+1 bn+1 = an bn + + 1.(1) Pentru n=0 avem a1 b1 = a0 b0 + + 1.
4an bn 4a0 b0
1
Din inegalitatea mediilor avem a0 b0 + ≥ 1 ⇒ a1 b1 ≥ 2(2). Din (1) se
4a0 b0
obţine an+1 bn+1 > an bn + 1. Continuând acest procedeu obţinem prin inducţie
an+1 bn+1 > a1 b1 + n, ∀n ≥ 1. Folosind (2) obţinem an+1 bn+1 > n + 2, ∀n ≥ 1.

Cum max2 {an+1 , bn+1 } > an+1 bn+1 ⇒ max{an+1 , bn+1 } > n + 2. Pentru n=2003
se obţine rezultatul cerut.(Matei Vlad) Se duce S piciorul perpendicularei din I
pe ı̂nălţimea din A.Fie R piciorul ı̂nălţimii din A şi T piciorul perpendicularei din
A pe BC. Cum SRT I este paralelogram, RS = T I ⇒ ha = R + r ⇔ r = RS ⇔
R = AS. ∠BAS = 90 − ∠B = ∠OAC ⇒ ∠SAI = ∠IAO. Relaţia e chivalenta
cu AS = AO ⇔ IO⊥AO(am folosit IS⊥AS)⇔ ∠ON A = 90 − ∠OAN = ∠B ⇔
BM N C inscriptibil.
Fie D piciorul bisectoarei din A a triunghiului ABC. Triunghiurile AM N şi ACB
sunt asemenea, iar dreapta AD este bisectoare in amandoua⇒
MN AI b+c
= =
BC AD a+b+c
A doua parte a egalităţii rezultă aplicând teorema bisectoarei ı̂n triunghiul ABCşi
Menelaus in triunghiul ADB.Se notează cu E intersecţia lui CI cu AB.
BC ID AE AI b+c b b+c AI b+c
· · =1⇒ = · = ⇒ =
DC AI BE ID b a a AD a+b+c
Din relaţia de mai sus rezultă
a a+b+c a 1 1 1
= =1+ ⇒ = +
MN b+c b+c MN b+c a
(Ilie Livia)

23. Fie a3 , (a + 1)3 , (a + 2)3 , (a + 3)3 cele patru cuburi conţinute in progresie. Sa ob-
servăm că (a + 1)3 − a3 = q1 r, (a + 2)3 − (a + 1)3 = q2 r,r raţia progresiei,q1 , q2 ∈
Z∗ .Aceasta se mai scrie sub forma ((a + 1)3 − a3 , (a + 2)3 − (a + 1)3 ) = r(q1 , q2 ) ⇔
(3a2 + 3a + 1; 3a2 + 9a + 7) = r(q1 , q2 )(1). Fie d ∈ Z∗ , d|3a2 + 3a + 1, d|3a2 + 9a + 7 ⇔
d|3a2 +9a+7−3a2 −3a−1 ⇒ d|6a+6, (d, 6) = 1 ⇒ d|a+1; d|3a(a+1)+1 ⇒ d|1 ⇔
202

d = 1.Din d=1 şi(1)⇒ r(q1, q2) = 1, r > 0 ⇒ r = 1, deci progresia conţine toate
numerele de la a1 , a1 < 0 ⇒ progresia conţine toate numerele naturale,implicit şi
toate cuburile numerelor naturale.(Matei Vlad)

24.
T T
25. (a)Fie M F AB = {P }, M F AC = {R}.Să observăm că AE⊥P R,(AE bis.
∠BAC 4AP R isoscel cu AP=AR(1).Din(1)mai avem ∠AP M = ∠M RA ⇔ ∠M BP =
BP
∠M RC.∠M BP = ∠M RC; ∠M BP = ∠M CR ⇒ 4M BP ≡ 4M RC ⇒ =
RC
PM T QS BQ
(2). Fie acum AQ P R = {S}.Din teorema transversalei avem: = ·
MR   SA BC
RC QC BP QS 1 (BP + RC PS
+ · ⇒ = .(3) Poziţia lui S pe(PR)este: =
CA BC P A SA 2 PA SR
AP sin ∠P AS sin ∠P AS BQ AB sin ∠P AS
= (4).În triunghiul ABC avem = ⇔
AR sin ∠RAS sin ∠RAS QC AC sin ∠RAS
sin ∠P AS AC PS AC PS AC PE
= . Înlocuind in (4) obţinem = ⇔ = , =
sin ∠RAS AB SR AB PR AC + AB P R
1 PS 2AC
⇒ = . Presupunem fără a restrânge generalitatea AB < AC.Ultima
2 PE AC + AB
SE
relaţie devine: = AC − ABAC + AB(5). Din (1) avem că E este şi mijlocul
PE
BP RF RF BP RP AB + AC
lui PR.Relatţia (2) devine = ⇔ = , = ⇒
RC FP PR BP + RC RS AB
RF BP (AB + AC) SF BP · AC − CR · AB BP (CR + AP ) − CR(BR + AP )
= ⇒ = = ⇔
RS (BP + RC)AB SR (AB + AC)BP (AB + AC)BP
SF AP (BP − CR) AP (AC − AB) SE SR
= = (6). Din(5) şi (6) avem · =
SR (AB + AC)BP (AB + AC)BP SF P E
BP SR 2SR 2BP SE RC + RB
, = = ⇒ = (7). Din (7) si (3) obţinem
AP P E PS BP + RC SF 2AP
QS ES
= ⇒ EQkAF.
SA SF
(b) Fie T=pr.(M,bis.ext).Avem că MTAE dreptunghi deci AT = M E, AT kM E, AT =
EF ⇔ AT kEF ⇒ AT F E paralelogram ⇒ AF kkET, EQkAF ⇒ Q, E, T col-
iniare.(Matei Vlad)
AD c AD c b2
26. Din teorema bisectoarei = ⇔ = ⇒ AD = ,din b=c.BC=AC+2AD,devine
DC a b c+a b+a
2b2
+ b = a ⇔ 2b2 + b2 + ba = a2 + ba ⇒ a2 = 3b2 = 2b2 − 2b2 cos A,din teorema
b+a
1 2π
cosinusului ⇒ cos A = − ,deci A = .(Matei Vlad)
2 3
27.
CN AN SAN P AP AN
=k⇔ = 1 − k. = · = k(1 − k) ⇒ SAN P = k(1 − k)S.
CA CA S AB AC
203

Scriind relaţiile analoage se obţine SAN P = SBP M = SCM N = k(1−k)S.(1)Folosind


abc
formula 4R = ⇔ abc = 4RS,relaţia(1) şi RAN P = RBP M = RCM N obţinem
S
:AN · AP · P N = BP · BM · P M = M C · CN · M N.(2) Cu notaţii uzuale in
triunghi,obţinem cu teorema cosinusului in 4AP N :P N 2 = k 2 c2 + (k − 1)2 b2 +
2k(k − 1)bc cos A = k 2 c2 + (k − 1)2 b2 + k(k − 1)(b2 + c2 − a2 ) = k(2k − 1)c2 + (k −
1)(2k − 1)b2 − k(k − 1)a2 .Analog se obţin relatiile P M 2 = k(2k − 1)a2 + (k − 1)(2k −
1)c2 −k(k −1)b2 , M N 2 = k(2k −1)b2 +(k −1)(2k −1)a2 −k(k −1)c2 .Ridicând relaţia
(2) la pătrat şi inlocuind pentru primele două produse avem b2 (k(2k − 1)c2 + (k −
1)(2k − 1)b2 − k(k − 1)a2 ) = a2 (k(2k − 1)a2 + (k − 1)(2k − 1)c2 − k(k − 1)b2 ) ⇔
k(2k − 1)b2 c2 + (k − 1)(2k − 1)b4 = k(2k − 1)a4 + (k − 1)(2k − 1)a2 c2 (3).Luând şi
celelalte produse şi grupândule câte două se mai obţin relaţiile k(2k − 1)c2 a2 +
(k − 1)(2k − 1)c4 = k(2k − 1)b4 + (k − 1)(2k − 1)a2 b2 (4)k(2k − 1)b2 a2 + (k − 1)(2k −
1)a4 = k(2k 4
X − 1)c + (k −X 1)(2k − 1)b2 c2 (5) Sumând relaţiile (3),(4),(5) se obţine
(2k − 1) a4 = (2k − 1) b2 c2 .Avem două cazuri :
1 PABC S
1.k = ⇒ PAN P = PBP M = PCM N = (P-semiperimetru),r = ⇒ cu(1) ⇒
2 2 P
rAN P = rBP M = rCM N .
1 X X
2.k 6= ⇒ a4 = b2 c2 ⇒ 4ABC echilateral⇒ P M = P N = M N =
√ 2 √
a 3k 2 − 3k + 1 ⇒ 2PAN P = 2PBP M = 2PCM N = a(1 + 3k 2 − 3k + 1) ⇒ rAN P =
rBP M = rCM N , folosind(1).(Matei Vlad)

28.
p p p p
29. x2 + 2pn x ∈ Q ⇔ x2 + 2pn x ∈ N; x ∈ Z∗ , p, n ∈ N∗ . x2 + 2pn x = x(x + 2pn ).Avem
două cazuri:x este pătrat perfect sau opusul unuia şi x nu este pătrat perfect.
Cazul 1.x = q 2 , x + 2pn = r2 (1) sau x = −q 2 , x + 2pn = −r2 , q, r ∈ N∗ .Vom
trata doar unul din acestea celălalt rezolvânduse la fel. (1) se mai scrie 2pn =
r2 − q 2 = (r − q)(r + q), r − q şi r + q au aceeaşsi paritate csi cum 2|(r − q)(r + q) ⇒
4|(r − q)(r + q) ⇒ 2|pn ,p este prim ⇒ p = 2(p 6= 2 ⇔ nu avem soluţii). Avem
ecuaţia 2n+1 = (r − q)(r + q) ⇒ r − q = 2k , r + q = 2n+1−k ⇒ q = 2n−k − 2k−1 ⇒
x = 22n−2k + 22k− 2 − 2n .Observâand că k1 + k2 = n + 1 ⇔ x1 = x2 ,obţinem in
n+1
total soluţii.În a doua situaţie x = −(22n−2k + 22k−2 + 2n ) şi cu aceeaşi
2    
n+1 n+1
observaţie avem soluţii.Obţinem in total pentru card(M 2) = 2 .
2 2
Cazul2. ∃d, d ∈ N∗ , d|x, d|x n
p + 2p ⇔ d|2p . ∗
n

2a.d = 2 ⇒ x = 2x1 ⇒ x1 (x1 + pn ) ∈ N , (x1 , x1 + pn ) = 1,cu aceleaşi notaţii


avem x1 = q12 , x1 + pn = r12 saux1 = −q12 , x1 + pn = −r12 .Îl vom analiza pe
primul,analog celălalt. x1 = q12 ; x1 +pn = r12 ⇒ r12 −q12 = pn , (r1 −q1 )(r1 +q1 ) = pn ⇒
pn−k1 − pk1 p2n−2k1 + p2k1 − 2pn
r1 − q1 = pk1 , r1 + q1 = pn−k1 , q1 = ⇔x= . Dar tre-
2 4
204

p2n−2k1 + p2k1 − 2pn p2n−2k1 + p2k1 + 2pn


 
n
buie să avem (x1 , x1 + p ) = 1 ⇒ ; =1
4 4
,de unde p2k1 = 1 sau p2n−2k1 = 1 de unde k1 = 0 sau k1 = n.Pentru p=2
p2n + 1 − 2pn
avem k1 = 1 sau k1 = n − 1.Soluţie x = , x = −p2n − 1 − 2pn 2
2
sau x = 22n−3 + 2 − 2n , x = −22n−3 − 2 − 2n
2b.d|pn , d > 1 ⇒ d ∈ {p, p2 , ...pn }.Fie d = pa , a ∈ {1, a
p2, ..., n}deci x = p m, (m, p) =
n−a ∗
1, (m, 2) = 1. Rămâne la a găsi m astfel incât m(m + 2p ) ∈ N ⇒ ∃c, d
m = c2 ; m + 2pn−a = d2 ⇒ d2 − c2 = 2pn−a d − c şi d + c au aceeaşi pariate ⇒
p = 2,altfel nu avem soluţii.Acesta
  este chiar cazul1 d.p.d.v al soluţiilor.Obţinem
n+1
in final card(M2 ) = 2 + 2 sau card(Mp ) = 2, p > 2.(Matei Vlad)
2
30. ” ⇒ ” BE ⊥ AC, AD ⊥ BC ⇒ ABDE inscriptibil ⇒ ∠ESA = ∠EAD =
∠EBC = ∠BQD ⇒ ESDQ inscriptibil ⇒ ∠ADE = ∠BQS = ∠ABE(1) Dar
∠EAR = ∠P AB = 45 ⇒ ∠RAD = 45−∠DAC = 45−∠EBC = ∠P BR ⇒ P ARB
inscriptibil ⇒ ∠AP R = ∠ABE(2). Din (1) şi (2)⇒ ∠RP D = ∠RQD ⇒ P RDQ
inscriptibil.
” ⇐ ” P RSQ inscriptibil ⇒ ∠QP D = ∠QRS ⇒ ∠P DQ = 180 − 2∠QP D =
180 − 2∠ERS = ∠RES ⇒ ESDQ inscriptibil⇒ ∠EAS = ∠ESA = ∠BQR =
∠QBD ⇒ EDBA inscriptibil⇒ BE ⊥ AC(Ilie Livia)

31. Să consideram un reper cartezian şi O originea acestui sistem.Fie A(a1 ; a2 ); B(b1 ; b2 )

două
q puncte cu proprietatea
q AB = 2 şi α = ∠OAB.Să observăm că: OA =
a21 + a22 , OB = b21 + b22 (1). Din inegalitatea triunghiului avem |OA − OB| ≤

AB ⇔ |OA−OB| ≤ 2(2).Din(1) şi modul cum a fost definită M avem OA, OB ∈
N ⇒ |OA − OB| ∈ N.Folosind in(2)⇒ OA = OB sau OA = OB + 1,presupunând
fără a restrânge generalitatea OA > OB.
Cazul1.OA=OB.Din teorema cosinusului avem: AB 2 = OA2 + OB 2 − 2OA ·
OB cos α ⇔ AB 2 = 2OA2 − 2OA2 cos α = 2OA2 (1 − cos α) ⇒ 2 = 2OA2 (1 −
1 1
cos α) ⇒ 1 − cos α = 2
⇒ cos α = 1 − 2
, OA = x, x ∈ N∗ ⇒ α =
  OA OA
1
arccos 1 − 2 , x ∈ N∗ .
x
Cazul2.OA=OB+1.Din teorema cosinusului avem: AB 2 = OA2 + OB 2 − 2OA ·
OB cos α ⇒ AB 2 = 2OA2 − 2OA + 1 − 2OA2 cos α + 2OA cos α ⇒ 1 = 2OA2 (1 −
1
cos α) − 2OA(1 − cos α) ⇒ 1 = (2x2 − 2x)(1 − cos α) ⇔ 1 − cos α = 2 ⇒
  2x − 2x
1
α = arccos 1 − 2 , x ∈ N∗ .(Matei Vlad)
2x − 2x
b−a b−a
32. Pentru orice x, y ∈ [a; b]; , |x − y| ≤ , atunci |f (x) − f (y)| ≤ |x − y| ≤ ,şi
2 2
205

b−a
problema este rezolvată . Pentru orice x, y ∈ [a; b], |x−y| > ,deoarece |−α| =
2
b−a
|α|,putem presupune a ≤ x < y ≤ b ⇒ |x − y| = y − x > (1).|f (x) − f (y)| =
2
|f (x)−f (a)+f (b)−f (y)| ≤ |f (x)−f (a)|+|f (b)−f (y)||f (x)−f (a)|+|f (b)−f (y)| ≤
b−a b−a
|x − a| + |b − y| = x − y + b − a, y − x ≥ (1) ⇒ |f (x) − f (y)| ≤ b − a − =
2 2
b−a
.(Matei Vlad)
2

13.4 Clasa a X-a


1. Pentru ca

√
Apn = n − p + 1,
p


trebuie sa avem

(n − p + 1)p ≤ Apn < (n − p + 2)p

şi inlocuind Apn , cautam p pentru care

(n − p + 1)p ≤ (n − p + 1) (n − p + 2) ... (n − 1) n < (n − p + 2)p .

Vom demonstra ca singurele solutii sunt p ∈ {1, 2, 3}. Acestea verifica trivial
inegalitatile. Acum, pentru p ≥ 4, avem

(n − p + 1) (n − p + 2) ... (n − 1) n >
(n − p + 1) (n − p + 2) (n − p + 3) (n − p + 4) (n − p + 2) (n − p + 2) ... (n − p + 2) >
(p−4) ori
4 p−4 p
> (n − p + 2) (n − p + 2) = (n − p + 2) . Fals

Asadar singurele solutii sunt pentru p ∈ {1, 2, 3}.

2. Gresit

3. Sa eliminam pentru inceput cazurile triviale:


Daca n = 0, evident nu avem solutii: 2 < 8.
Daca n = 1, avem
206

log2 (2x2 + 2) + logx2 +1 (2x2 + 2) = 8 ⇔


log2 (x2 + 1) + 1 + logx2 +1 (2) + 1 = 8 ⇔
log2 (x2 + 1) + logx2 +1 (2) = 6.

Notam log2 (x2 + 1) = a. Rezolvand

1
a+ a
= 6 ⇔ a2 − 6a + 1 = 0,


obtinem solutiile a ∈ {3 ± 2 2}, deci

√ p √
x2 = 23±2 2
− 1 ⇔ x = ± 23±2 2 − 1.

Acum, daca n ≥ 2, vom utiliza inegalitatea

an +bn a+b n

2
≥ 2
.

Avem
n
a+ a1 +2
n 
8 = logn2 (2x2 + 2) + lognx2 +1 (2x2 + 2) = (a + 1)n + 1
a
+1 ≥2 2

n
2 2+2
2
≥ 2 · 22 = 8.

Cum am ajuns 8 ≥ 8, trebuie sa avem egalitate peste tot, adica a = a1 = 1 si n = 2.


Asadar mai avem doar solutie pentru n = 2 :a = 1 ⇔ log2 (x2 + 1) = 1 ⇔ x = 1.

4. Daca x ar fi negativ, cum a, b > 1, am avea

a
2 > abx + b x = ab + ba > a + b > 2. Fals

a
Deci cautam x > 0. Fie f (x) = abx + b x . Avem

a a 3
f 00 = abx b ln a + 2 x
x3
b a ln b + b x xa4 ln2 b > 0,

rezulta f este convexa, deci ecuatia noastra nu poate avea mai mult de 2 solutii.
a
Se observa ca x = 1 si x = logba b verifica. Ar mai ramane de vazut cazul cand cei
2 ”x” sunt egali, obtinandu-se

loga ba
b
= 1 ⇔ loga ba = b ⇔ ba = ab = m.
207

Dar atunci, din inegalitatea mediilor,

1
p 1 √
mx + m x ≥ 2 mx+ x ≥ 2 m2 = 2m,

deci in acest caz, unica solutie este x = 1.

5. Fie a = x + yi si b = z + ti. Avem a + b real, deci t = −y si, de asemenea, a2 + b2


real, deci 2xy = 2zy. Se disting 2 cazuri:
i) y = 0, adica a si b reale. Sa eliminam cazurile cand a sau b este −1, 0, sau 1.
Daca a = −1 => b ∈ {1, 0, −1};
Daca a = 0 => b ∈ {1, −1};
Daca a = −1 => b ∈ {1, 0, −1}.
, k ∈ N ∗.

Acum, ne uitam la subsirul a2k + b2k k

Daca |a| , |b| < 1, atunci sirul tinde la 0(atingand evident o infinitate de valori
reale);
Daca unul dintre |a| , |b| este supraunitar, iar celalalt subunitar, sirul tinde la in-
finit;
Daca |a| , |b| > 1, din nou sirul tinde la infinit si am demonstrat contradictia in
toate cazurile.
ii) x = z, deci a = x + yi si b = x − yi. Putem considera y > 0, deoarece y = 0 a
fost deja analizat si daca y este negativ practic a si b permuta. Avem sirul

(x + yi)n + (x − yi)n = [r (cos θ + i sin θ)]n + [r (cos θ − i sin θ)]n =


= rn (cos nθ + i sin nθ) + rn (cos nθ + i sin nθ) = 2rn cos nθ.

Cum r > 0, iar 2rn cos nθ trebuie sa ia numai 2 valori(de asemenea cos nθ marginit
la [−1, 1]), din considerente asemenea i), rezulta r = 1. Cautam θ in [0, 2π) pentru
care cos nθ ia doar 2 valori. Daca θ = kπ, unde k irational, atunci cos nθ pentru n
natural este dens pe (−1, 1) si deci nu ia doar 2 valori. Fie k = pq , (p, q) = 1. Sa
observam sirul pentru n = 1, 2, 3:cos pq π; cos 2p
q
π; cos 3p
q
π. Cel putin 2 dintre aceste
3 valori trebuie sa fie egale.
n o
a). Daca cos q π = cos q π => q ∈ 2 − q , 2 + q , 4 − q => pq = 2 − 2p
p 2p 2p p p p
q
, pq =
4 − 2p
q
=> pq = 23 , sau pq = 34 => a = cos 2π
3
+ i sin 2π
3
, b = cos 2π
3
− i sin 2π
3
; sau
4π 4π 4π 4π
a = cos 3 + i sin 3 , b = cos 3 − i sin 3 .
208
n o
b). Daca cos pq π = cos 3p
q
π => 3p
q
∈ 2 − pq , 2 + pq , 4 − pq , 4 + pq , 6 − pq => p
q
=
3p p 3p p 3p p 3p p 1 p p 3
2− q
, q= − 2, = 6 −
q q
sau = 4 −
q
, q
=> = q q
= 1, =,
2 q
sau
q 2
,
p π π π π
q
= 1 => a = cos 2 + i sin 2 , b = cos 2 − i sin 2 ; a = cos π + i sin π, b = cos π − i sin π;
a = cos 3π
2
+ i sin 3π
2
, b = cos 3π
2
− i sin 3π
2
; sau a = cos π + i sin π, b = cos π − i sin π.
n o
2p 3p 3p 2p 2p 2p 2p 2p
c). Daca cos q π = cos q π => q ∈ 2 − q , 2 + q , 4 − q , 4 + q , 6 − q =>
3p
q
= 2 − 2pq
, 3p
q
= 4 − 2p q
, sau 3p
q
= 6 − 2p q
=> pq = 25 , pq = 45 , sau pq = 65 =>
a = cos 2π
5
+ i sin 2π
5
, b = cos 2π
5
− i sin 2π
5
; a = cos 4π
5
+ i sin 4π
5
, b = cos 4π
5
− i sin 4π
5
;
6π 6π 6π 6π
sau a = cos 5 + i sin 5 , b = cos 5 − i sin 5 .
Evaluand aceste solutii pentru n = 4, 5(sub forma 2 cos npπ q
), solutiile care convin
sunt a = cos π + i sin π, b = cos π − i sin π; a = cos 3 + i sin 3 , b = cos 2π
2π 2π
3
− i sin 2π
3
;
4π 4π 4π 4π
a = cos 3 + i sin 3 , b = cos 3 − i sin 3 , ultimele doua alaturandu-se solutiilor
reale gasite(a = cos π + i sin π, b = cos π − i sin π este de asemenea solutie reala).

6. Cum avem expresia log4 x, trebuie sa avem x > 0. Daca x > 4, avem [log4 x] ≥ 1
si deci

(−3)[log4 x] + x[log4 x] > x[log4 x] − 3[log4 x] ≥ x − 3 > 4 − 3 = 1. Fals

Asadar, 0 < x ≤ 4. Considerand cazul x = 4, obtinem solutie deoarece (−3)[log4 4] +


4[log4 4] = −3 + 4 = 1. Daca 0 < x < 4, [log4 x] = 0 si ecuatia devine

1 = (−3)0 + x0 = 2. Fals.

Concluzionand, singura solutie este x = 1.

7. a) daca se presupune ca toti coeficientii sunt diferiti vom avea o sg valoare si


anume n, iar in caz contrar avem mai mult de 3.
b) considerand n = 2, daca b < a < 2b, cel mai mare coeficient va fi cel de rang 1
pt ambele polinoame, deci k1 = 1 ≤ 1 = k2 , dar b < a fals. [obs. pt n impar este
adevarat]

8. Sa privim problema sintetic. Avem A, B, C, M conciclice. Fie H ortocentrul


triunghiului ABC si D mijlocul segmentului HM . Vom demonstra ca D este
centrul de simetrie cautat. Pentru aceasta, este de ajuns sa demonstram ca HA
este simetricul lui A fata de D, analog rezultand ca triunghiurile ABC si A1 B1 C1
sunt congruente si se obtin unul din celalalt printr-o simetrie centrala. Cautam sa
aratam ca AM HA H este paralelogram. Avem AH||HA M , deoarece ambele sunt
perpendiculare pe BC si de asemenea,
209

AH = 2R |cos A| = 2R |cos CM B| = HA M,

deci AM HA H este paralelogram si demonstratia se incheie.

9. a) Sa calculam x6n+1 − x6n . Este de ajuns sa numaram elementele lui F6n+1 ce sunt
alcatuite numai din 2 si 3(celelalte sunt elemente ale lui F6n carora li s-a adaugat
un 1.) Trebuie sa numaram solutiile naturale ale ecuatiei 6n + 1 = 2x + 3y. y
trebuie sa fie impar si ia valori de la 1 la 2n − 1, deci un total de n solutii, deci
x6n+1 − x6n = n. In mod analog se analizeaza si se obtine

x6n+2 − x6n+1 = n + 1;
x6n+3 − x6n+2 = n + 1;
x6n+4 − x6n+3 = n + 1;
x6n+5 − x6n+4 = n + 1;
x6n+6 − x6n+5 = n + 2,

deci x6n+1 , x6n+2 , x6n+3 , x6n+4 , x6n+5 alcatuiesc o progresie aritmetica de ratie n+1.
b) De asemenea, adunand cele 6 relatii, avem

x6n+6 − x6n = 6n + 6.

Vom demonstra prin inductie ca

x6n = 3n2 + 3n + 1.

Pentru n = 1, se verifica usor ca x6 = 7, F6 fiind

{(1, 1, 1, 1, 1, 1) ; (1, 1, 1, 1, 2) ; (1, 1, 1, 3) ; (1, 1, 2, 2) ; (1, 2, 3) ; (2, 2, 2) ; (3, 3)}.

Presupunand ca x6k = 3k 2 + 3k + 1, avem

x6k+6 = (3k 2 + 3k + 1)+6k +6 = 3k 2 +6k +3+3k +3+1 = 3 (k + 1)2 +3 (k + 1)+1,

ceea ce trebuia demonstrat.


210
Capitolul 14

Probleme avute ı̂n atenţia comisiei, 2006

14.1 Clasa a VII-a


1. Din teorema bisectoarei,
BD c BD c ac
= ⇔ = ⇔ BD =
DC b a b+c b+c
1 b+c 1 a+b 1 a+c
deci = . Analog, = , iar = , deci
CA · BD abc AB · CE abc BC · AF abc
relaţia de demonstrat revine la
2(a + b + c) 1 4p 1
= ⇔ = .
abc r·R abc r·R
Această din urmă egalitate este evident adevărată, ţinând seama de binecunos-
S abc
cutele r = , R = , unde S este aria 4ABC.
p 4S
2. Fie {E} = CD ∩ AB, BM ⊥ CE, M ∈ CE, {N } = BM ∩ AC. Atunci
m(∠CEA) = 180◦ −110◦ −10◦ = 60◦ şi observăm că CM este ı̂nălţime şi bisectoare
ı̂4CBN , deci 4CBN este isoscel, iar m(∠CBN ) = m(∠CN B) = 80◦ . Obţinem
că m(∠EBM ) = 80◦ − 50◦ = 30◦ şi cum evident că 4EM B ≡ 4EM N , atunci
m(∠EN M ) = 30◦ , apoi m(∠CN E) = 80◦ − 30◦ = 50◦ . Triunghiurile 4EBD şi
4EBN au m(∠DEB) = m(∠N EB) = 120◦ , m(∠DBE) = m(∠N BE) = 30◦ şi
[BE] comună, deci DE = N E şi cum AE = AE, m(∠DEA) = m(∠N EA) = 60◦ ,
deducem că 4DAE ≡ 4N AE. Astfel, m(∠ADE) = 50◦ şi deci m(∠ADB) =
50◦ + 30◦ = 80◦ . Mai avem că m(∠BDC) = 180◦ − 20◦ − 10◦ = 150◦ , prin urmare
m(∠ADC) = 360◦ − 150◦ − 80◦ = 130◦ .

3. Există trei perechi de triunghiuri opuse: (4ABC, 4DEF ); (4BCD, 4EF A);
(4CDE, 4F AB); notăm cu G1 , G2 , . . . , G6 centrele de greutate ale celor 6 tri-
unghiuri, ı̂ordinea ı̂care le-am scris anterior şi cu M, N, P, Q, R, S mijloacele

211
212

RG4 RG2 1
laturilor [AB], [BC], . . . , [F A]. Cum = = , atunci G2 G4 k AD şi
G4 A G2 D 2
G2 G4 RG4 1 G1 G3 1
= = . Analog, G1 G3 k AD şi = , prin urmare segmentele
AD RA 3 AD 3
[G2 G4 ] şi [G1 G3 ] sunt paralele şi congruente. Deducem că G1 G3 G2 G4 este parale-
logram şi atunci [G1 G2 ] şi [G3 G4 ] se ı̂njumătăţesc. Analog obţinem că [G1 , G2 ] şi
[G5 G6 ] se ı̂njumătăţesc, de unde concluzia problemei.

AN BQ CM
4. Fie {Q} = AP ∩ BC; conform teoremei lui Ceva, avem că · · = 1,
N B QC M A
BQ m
deci = . Folosind relaţia lui Van Aubel sau aplicând de două ori teorema
QC n
AP AN AM
lui Menelaus, obţinem că = + = m + n = 2. Atunci
PQ NB MC

AAN P AAN P AABQ AN · AP · sin(∠BAQ) AB · BQ · sin(∠B)


= · = · =
AABC AABQ AABC AB · AQ · sin(∠BAQ) AB · BC · sin(∠B)

AN AP BQ n 2 m 2mn mn
= · · = · · = = .
AB AQ BC n+1 3 m+n 3(n + 1)(m + n) 3(n + 1)
AAM P mn
Analog, = , prin urmare
AABC 3(m + 1)

AAM P N mn(m + n + 2) 4mn


= = .
AABC 3(m + 1)(n + 1) 3(m + 1)(n + 1)

4
Rămâne să demonstrăm că ≥ 1 ceea ce, după calcule, revine la
(m + 1)(n + 1)
 2
m+n
mn ≤ 1. Însă, din inegalitatea mediilor, mn ≤ = 1, ceea ce ı̂ncheie
2
rezolvarea.

5. Notăm cu M, N, P, Q, R mijloacele laturilor [AB], [BC], [CD], [DE], res- pectiv


[EA] şi cu GM , GN , GP , GQ , GR centrele de greutate ale triunghiurilor 4CDE, 4DEA, 4EAB
respectiv 4BCD.
QGN QGM 1 GM GN 1
Cum = = , ı̂nseamnă că GM GN k AC şi = . Apoi,
GN A GM C 2 AC 3
MN 1
deoarece [M N ] este linie mijlocie ı̂4ABC, atunci M N k AC şi = . De-
AC 2
GM GN 2
ducem că GM GN k M N şi = . Dacă {X} = M GM ∩ N GN , obţinem că
MN 3
GM X GN X 2
= = . Considerând intersecţiile lui M GM cu P GP , QGQ , RGR ,
XM XN 3
găsim că toate aceste puncte coincid cu X, de unde concluzia problemei.
213

6. Avem de-a face cu o variantă a problemei biliardului. Fie A0 simetricul lui A faţă
de OX, iar B0 simetricul lui B faţă de OY şi notăm cu M, N intersecţiile lui A0B0
cu [OX, respectiv [OY . Avem că BN + N M + M A = B0N + N M + M A0 = A0B0.
Dacă M 0 ∈ [OX şi N 0 ∈ [OY , atunci AM 0+M 0N 0+N 0B = A0M 0+M 0N 0+N 0B0 ≥
A0B0, deci M, N sunt punctele ce se cer a fi determinate.

7. Avem că
a4 + b4 + c4 + d4 = (a4 + b4 − 2a2 b2 ) + (c2 + d2 − 2c2 d2 ) + (a2 b2 + c2 d2 − 2abcd)
+(a2 b2 + c2 d2 − 2abcd) + 4abcd
= (a2 − b2 )2 + (c2 + d2 )2 + (ab − cd)2 + (ab − cd)2 + (2k)2 ,
unde k 2 = abcd.

8. Ne ocupăm de cazul general. Vrem să determinăm n, n ≥ 2, numere naturale,


distincte x1 , x2 , . . . , xn astfel ı̂ncât
x1 + x2 + . . . + xn √ n
ma = , mg = n x1 x2 . . . xn şi mh =
n 1 1 1
+ + ... +
x1 x2 xn
să fie naturale.
Alegem x1 , x2 , . . . , xn astfel ı̂ncât mg să fie număr natural. Deoarece 1 + 3 + 5 +
. . . + (2n − 1) = n2 alegem x1 = ab, x2 = ab3 , x3 = ab5 , . . . , xn = ab2n−1 cu
a, b ∈ N∗ , b 6= 1. Astfel x1 , x2 , . . . , xn sunt distincte şi mg = abn ∈ N.
nab2n−1
Pentru aceste valori, mh = şi pentru ca mh să fie număr
1 + b2 + b4 + . . . + b2n−2
natural considerăm a un multiplu al lui 1 + b + . . . + b2n−2 .
2

Alegem a = 1+b2 +b4 +. . .+b2n−2 . Pentru noile valori ale numerelor x1 , x2 , . . . , xn


a(b + b3 + . . . + bn−1 )
avem ma = şi considerând, de exemplu b = n deducem că
n
şi ma ∈ N. În concluzie numerele: x1 = n(1 + n2 + n4 + . . . + n2n−2 ), x2 =
n3 (1 + n2 + n4 + . . . + n2n−2 ), . . . , xn = n2n−1 (1 + n2 + n4 + . . . + n2n−2 ) verifică
cerinţele problemei.
Pentru n = 3 obţinem x1 = 3 · 91, x2 = 33 · 91, x3 = 35 · 91, care reprezintă o soluţie
a problemei. Evident, problema are o infinitate de soluţii!

9. Dacă două dintre numere sunt egale, să zicem că y = z, atunci

x(y − z) y(z − x) z(x − y) y(y − x) y(x − y)


+ + =0+ + = 0.
y+z z+x x+y x+y x+y
Reciproc, avem:
x(y − z) y(z − x) z(x − y)
+ + =
y+z z+x x+y
214

x2 y + xyz − x2 z − xz 2 + y 2 z + yz 2 − xy 2 − xyz z(x − y)


= + =
(y + z)(x + z) x+y
(x − y)(xy − z 2 − xz − yz) z(x − y)
= + =
(y + z)(x + z) x+y
x−y
(x + y)(xy − z 2 − xz − yz) + z(y + z)(x + z) =
 
=
(x + y)(y + z)(x + z)
x−y
= (x2 y + xy 2 − x2 z − xyz − y 2 z + z 3 ) =
(x + y)(y + z)(x + z)
(x − y)(y − z)(x − z)(x + y + z)
= .
(x + y)(y + z)(x + z)
Dacă această expresie este egală cu 0, cum x + y + z 6= 0, ı̂nseamnă că se anulează
una dintre paranteze x − y, y − z sau x − z, deci cel puţin două dintre numere vor
fi egale.

10. Fie nk numărul elevilor care au câştigat exact k meciuri, 0 ≤ k ≤ 3. Se ştie că
n0 = 0, iar n1 + n2 + n3 = 20. În fiecare din cele trei zile se dispută câte 10 meciuri,
deci ı̂total 30 de meciuri. Numărând aceste meciuri funcţie de ı̂nvingători, avem
că 1·n1 +2·n2 +3·n3 = 30; numărându-le funcţie de ı̂nvinşi, obţinem că 2·n1 +1·n2 +
0 · n3 = 30. Scăzând membru cu membru cele două egalităţi, −n1 + n2 + 3n3 = 0,
deci n2 + 3n3 = n1 . Înlocuind ı̂n1 + n2 + n3 = 20, găsim că n2 + 2n3 = 10. Cum
turneul se termină cu un singur câştigător, convine doar situaţia n3 = 1, n2 =
8, n1 = 11.

14.2 Clasa a VIII-a


1. Din m, n ∈ N şi m > n > 3 rezultă că m ≥ 5 şi n ≥ 4, prin urmare,

m2 − 4n ≥ 5m − 4m = m > 0

adică discriminantul ecuaţiei, ∆ = m2 − 4n este pozitiv şi avem două soluţii reale
şi distincte: √ √
m + m2 − 4n m − m2 − 4n
x1 = , x2 = . (1)
2 2
Atunci
p p
m2 + m2 (m2 − 4n) m2 − m2 (m2 − 4n)
mx1 = , mx2 = . (2)
2 2

Trecem la justificarea celor două implicaţii.


” ⇒ ”. Din faptul că x1 , x2 ∈ N, ţinând cont de (2), avem [mx1 ] + [mx2 ] = m2 ceea
215

ce justifică implicaţia directă.


” ⇐ ”. Dacă, prin absurd, x1 , x2 ∈ N, atucni se deduce imediat că m2 − 4n nu este
2 2
pătrat perfect. Dar nici pm (m − 4n) nu este pătrat perfect. Prin urmare, există
k ∈ N astfel ı̂ncât k < m2 (m2 − 4n) < k + 1.
În continuare, apar patru situaţii:
   
m=2 m = 2s + 1 m = 2s m = 2s + 1
I. ; II. ; III. ; IV.
k = 2t k = 2t k = 2t + 1 k = 2t + 1

s, t ∈ N.
O analizăm pe prima; celelalte se tratează analog.

m=2
I.
k = 2t
Avem p
2t < m2 (m2 − 4n) < 2t + 1 (3)
şi imediat p
−2t − 2 < − m2 (m2 − 4n) < −2t. (4)
Atunci din (3) şi (4) rezultă
1
2s2 + t < mx1 < 2s2 + t + ⇒ [mx1 ] = 2s2 + t
2
1
2s2 − t − < mx2 < 2s2 − t ⇒ [mx2 ] = 2s2 − t − 1.
2
Deducem astfel că [mx1 ]+[mx2 ] = 4m2 −1 care nu este pătrat perfect; contradicţie.
Cazurile II, III, IV conduc, de asemenea, la contradicţii.

Prin urmare, presupunerea făcută este falsă şi urmeaxă ca m2 − 4n este număr
natural. Mai mult, el are aceeaşi paritate cu m (se verifică uşor!) şi de aici,
rădăcinile x1 , x2 sunt numere naturale.

2. Adunăm 3 la ambii membrii ai inegalităţii de demonstrat şi, prin calcule simple,


avem echivalenteţe
Xb+c (a2 + b2 + c2 )(ab + bc + ca
≥3+ ⇔
a abc(a + b + c)
 
(a + b + c)2 − 2(ab + bc + ca) (ab + bc + ca)
 
1 1 1
⇔ (a + b + c) + + ≥ 6+ ⇔
a b c abc(a + b + c)
2(ab + bc + ca)2
   
1 1 1 1 1 1
⇔ (a + b + c) + + ≥ 6 + (a + b + c) + + − ⇔
a b c a b c abc(a + b + c)
216

2(ab + bc + ca)2
⇔ ≥ 6 ⇔ (ab + bc + ca)2 ≥ abc(a + b + c) ⇔
abc(a + b + c)
⇔ (ab)2 + (bc)2 + (ca)2 ≥ (ab)(bc) + (bc)(ca) + (ca)(ab).
Ultima inegalitate este evidentă. Cazul de egalitate are loc dacă şi numai dacă
a = b = c.
a[x]
3. Mai ı̂ntâi, din a[x] = b{x} rezultă că {x} = ∈ Q, deci soluţiile ecuaţiei (dacă
b
există) sunt raţionale. Căutăm aşadar soluţii de forma
p
x = m + , m, p, q ∈ N, p < q, (p, q) = 1.
q

Înlocuind ı̂a[x] = b{x}, obţinem


p
am = b · . (1)
q
Din ipoteză ştim că a nu este divizor al lui b. Atunci din (1) rezultă că a este un
divizor al lui p, deci
p = as, s ∈ N∗ . (2)
Tot din (1), ţinând cont că (p, q) = 1, avem q divizor al lui b, adică

b = qt, t ∈ N. (3)

Înlocuim ı̂(1) şi obţinem


m = st (4)
şi forma lui x este
ast
x = st + . (5)
b
ast ast
Se verifică imediat că < 1 şi apoi că x = st + verifică ecuaţia dată.
b b
În concluzie, mulţimea soluţiilor ecuaţiei date este
 
 a
S = x = st 1 + s, t ∈ N .
b

4. Arătăm, mai ı̂ntâi că A ∩ B conţine o infinitate de elemente. Pentru aceasta, tre-
1 1 1 1 1
buie să arătam că sistemul + = + + = λ (λ ∈ R) are o infinitate de
a b x y z
soluţii.
Mai exact, trebuie să arătăm că sistemul generează o infinitate de valori λ. Ob-
 2
1 1 5
servăm că + = , prin urmare, putem alege a = 9k, b = 16k (k ∈ R∗ )
9 16 12
217
 2
1 1 1 5
şi vom avea + + .
a b k 12
 2
1 1 1 1 1 1 5
Căutăm acum x, y, z astfel ı̂ncât + + = + = .
x y z 9k 16k k 12
1 1 1
Să ı̂ncercăm cu alegerea y = 16k, + = .
x z 16k
Rezolvând a doua ecuaţie, găsim, de exemplu, x = 36k, z = 12k. Am obţinut
astfel 1 = 9k, b = 16k (a 6= b) x = 36k, y = 16k, z = 12k (x > y > z) astfel ı̂ncât
 2
1 1 1 1 1 1 5
+ = + + = · .
a b x y z k 12
s 2


1 5
Prin urmare, A ∩ B conţine elemente de forma λ= · , adică o infin-
k 12
tate de valori.
5
Punând acum k = t2 , t = N∗ , deducem că toate elementele de forma se află
12t
ı̂A ∩ B, deci o infinitate de valori raţionale.
Pentru a găsi ı̂A ∩ B o infinitate de valori iraţionale, este suficient să alegem
5
k = 2s2 , s = N∗ şi avem √ ∈ A ∩ B.
12s 2
5. Folosind inegalităţile a2 + b2 ≥ 2ab, b2 + c2 ≥ 2bc, c2 + a2 = 2ca, avem
X a2 X a2 X a2
= ≤ ≤
3a2 + b2 + 2ac 2a2 + a2 + b2 + 2ac 2a2 + 2ab + 2ac
1X a 1 1
≤ = ·1= .
2 a+b+c 2 2
Egalul se atinge dacă si numai dacă a = b = c.

6. Folosind inegalitatea dintre media aritmetică şi media armonică:


x 1 + x2 + . . . + xn n
≥ , oricare ar fi x1 , x2 , . . . , xn > 0,
n 1 1 1
+ + ... +
x1 x2 xn
 
1 1 1
rezultă că (x1 + x2 + . . . + xn ) + + ... + ≥ n2 deci ţinând cont de
  x1 x2 xn
1
ipoteză, 3n 1 + ≥ n2 . (1)
4n
Din (1), avem echivalenţa 4n2 − 12n − 3 ≤ 0 ⇔ (2n − 3)2 ≤ 12 şi ţinând cont că n
este natural nenul obţinem n ∈ {1, 2, 3}.
1 1
Pentru n = 1 relaţiile din ipoteză se scriu x1 = 3 şi = 1 + , imposibil.
x1 4
218

1 1 9
Pentru n = 2, aceleaşi relaţii devin x1 +x2 = 6 şi + = şi este, de asemenea,
x1 x2 8
imposibil (se verifică uşor).
1 1 1 13
Pentru n = 3, avem x1 + x2 + x3 = 9 şi + + = . Ţinând cont de simetria
x1 x2 x3 12
relaţiilor, e suficient să găsim soluţii (x1 , x2 , x3 ) cu x1 ≤ x2 ≤ x3 , apoi vom pune
şi permutările acestora.
13 1 1 1 3 36
Putem scrie = + + ≤ ⇒ x1 ≤ ⇒ x1 ∈ {1, 2}.
12 x1 x2 x3 x1 13
1 1 1
Dacă x1 = 1, atunci x2 + x3 = 8 şi + = de unde x! x2 = 96, imposibil,
x2 x3 12
deoarece din x2 + x3 = 8 obţinem x1 , x2 < 8.
Dacă x1 = 2, atunci

 x2 + x3 = 7 
x2 + x3 = 7
1 1 7 ⇔ ⇔
 + = x2 x3 = 12
x 2 x3 12
 
x2 + x 3 = 7 x2 + x3 = 7
⇔ ⇔
x2 (7 − x2 ) = 12 (x2 − 3)(x2 − 4) = 0

x2 = 3
cu soluţia .
x3 = 4
În concluzie, soluţiile iiţiale sunt
     
 x 1 = 2  x1 = 2  x1 = 3  x1 = 3  x1 = 4  x1 = 4
x =3 ; x =4 ; x =2 ; x =4 ; x =2 ; x =3
 2  2  2  2  2  2
x3 = 4 x3 = 3 x3 = 4 x3 = 2 x3 = 3 x3 = 2

(x + y + z)2
8. Folosind inegalitatea x2 + y 2 + z 2 ≥ , oricare ar fi x, y, z ∈ R, deducem
3
t4
că t3 ≥ echivalentă cu t(t − 3) ≤ 0 de unde, ţinând cont de semnul trinomului
3
de gradul 2, avem t ∈ [0, 3]. Cum t ∈ Z, obţinem t ∈ {0, 1, 2, 3}.
Aplicăm situaţiile apărute.
I. Dacă t = 0, atunci, din x2 + y 2 + z 2 = 0 găsim x = y = z = 0 care verifică şi
cealaltă ecuaţie a sistemului.
II. Dacă t = 1, atunci
 
x+y+z =1 2x + 2y + 2z = 2
2 2 2 ⇔ .
x +y +z =1 x2 + y 2 + z 2 = 1

Scăzând membru cu membru ultimele două ecuaţii, deducem că

x2 − 2x + y 2 − 2y + z 2 − 2z = −1 ⇔ (x − 1)2 + (y − 1)2 + (z − 1)2 = 2.


219

De aici, obţinem situaţiile


2 2 2
  
 (x − 1) = 1  (x − 1) = 1  (x − 1) = 0
(y − 1)2 = 1 ; (y − 1)2 = 0 ; (y − 1)2 = 1
(z − 1)2 = 0 (z − 1)2 = 1 (z − 1)2 = 1
  

fiecare situaţie generând câte 4 soluţii. Convenabile sunt doar (1, 0, 0); (0, 1, 0);
(0, 0, 1).

x+y+z =4
III. Dacă t = 2, atunci şi, ı̂mod asemănător, avem
x2 + y 2 + z 2 = 8

(x − 1)2 + (y − 1)2 + (z − 1)2 = 3

de unde (x−1)2 = (y −1)2 = (z −1)2 = 1 cu soluţiile convenabile (2, 2, 0); (2, 0, 2);
(0, 2, 2).

x+y+z =9
IV. Dacă t = 3, atunci .
x2 + y 2 + z 2 = 27
Obţinem imediat (x − 1)2 + (y − 1)2 + (z − 1)2 = 9 cu situaţiile
2 2 2
  
 (x − 1) = 9  (x − 1) = 0  (x − 1) = 0
(y − 1)2 = 0 ; (y − 1)2 = 9 ; (y − 1)2 = 0
(z − 1)2 = 0 (z − 1)2 = 0 (z − 1)2 = 9
  

2 2 2
  
 (x − 1) = 4  (x − 1) = 4  (x − 1) = 1
(y − 1)2 = 4 ; (y − 1)2 = 1 ; (y − 1)2 = 4
(z − 1)2 = 1 (z − 1)2 = 4 (z − 1)2 = 4
  

Pentru nici una din situaţiile obţinute nu se verifică x + y + z = 9, adică nu mai


obţinem soluţii noi.
În concluzie, sistemul iniţial admite soluţiile (x, y, z, t) date de

(0, 0, 0, 0); (1, 0, 0, 1); (0, 1, 0, 1); (0, 0, 1, 1); (2, 2, 0, 2); (2, 0, 2, 2); (0, 2, 2, 2).

14.3 Clasa a IX-a


1. Fie n ∈ N, n ≥ 2 şi a1 , a2 , . . . , a2n numere reale strict pozitive astfel ı̂ncât a1 + a2 +
· · · + a2n = s. Demonstraţi inegalitatea

a1 an an+1 a2n
+ ··· + + + ··· + ≥ 1.
s + an+1 − a1 s + a2n − an s + a1 − an+1 s + an − a2n

Soluţie. Din Cauchy-Schwarz,


220

X X 2
LHS · ai (s + an+i − ai ) ≥ ai ,

şi cum ai (s + an+i − ai ) = s2 + ni=1 (2ai an+i − a2i − a2n+i ) ≤ s2 avem LHS · s2 ≥
P P

LHS · ai (s + an+i − ai ) ≥ s2 şi astfel concluzia. Egalitatea are loc pentru ai = an+i ,
P

i = 1, . . . , n.

2. Fie a, b, c numere reale strict pozitive astfel ı̂ncât a2 + b2 + c2 = 3. Să se arate că
pentru orice numere reale strict pozitive x, y, z avem

x y z √ √ √
+ + ≥ xy + yz + zx.
a b c
Soluţie. Din Hölder,
 x y z 2
+ + (a2 + b2 + c2 ) ≥ (x2/3 + y 2/3 + z 2/3 )3 ,
a b c
√ √ √
şi mai rămâne de arătat că (x2/3 + y 2/3 + z 2/3 )3 ≥ 3( xy + yz + zx)2 . Notăm
u = x1/6 , v = y 1/6 , w = z 1/6 şi inegalitatea revine la
X 3 X 2
u4 ≥ 3 u3 v 3 .

Avem acum următoarele:

u12 + v 12 + w12 + 3u4 v 4 w4 ≥ u8 v 4 + u8 w4 + v 8 w4 + v 8 u4 + w8 u4 + w8 v 4 (Schur)

u8 v 4 + u8 w4 + u4 v 4 w4 ≥ 3u20/3 v 8/3 w8/3 , etc. (AM − GM)

3u20/3 v 8/3 w8/3 + · · · ≥ 3u6 v 3 w3 + · · · (Muirhead)

1, 5u8 v 4 + 1, 5u4 v 8 + · · · ≥ 3u6 v 6 + 3v 6 w6 + 3w6 u6 (AM − GM)

1, 5u8 v 4 + 1, 5u8 w4 + · · · ≥ 3u6 v 3 w3 + 3v 6 w3 u3 + 3w6 u3 v 3 . (Muirhead)

Însumând toate aceste inegalităţi se obţine concluzia.

3. Fie a, b, c numere pozitive astfel ı̂ncât abc = 1. Demonstraţi că

X 1 1
≤ .
a2 2
+ 2b + 3 2
221

Soluţie. Din a2 + 2b2 + 3 = (a2 + b2 ) + (b2 + 1) + 2 ≥ 2ab + 2b + 2 şi analoagele,

X 1 X 1 1
2 2
≤ = ,
a + 2b + 3 2(ab + b + 1) 2

cum 1/(ab + b + 1) + 1/(bc + c + 1) + 1/(ca + a + 1) = ac/(acab + acb + ac) +


a/(abc + ac + a) + 1/(ac + a + 1) = ac/(a + 1 + ac) + a/(1 + ac + a) + 1/(ac + a + 1) = 1,
folosind egalitatea din ipoteză. Egalitate avem pentru a = b = c = 1.

4. O funcţie bijectivă f : {1, 2, . . . , n} → {1, 2, . . . , n} are proprietatea P dacă relaţia

 
f (1) + · · · + f (k)
g(k) = f (1) + · · · + f (k) − n ,
n

pentru orice k = 1, 2, . . . , n defineşte o funcţie bijectivă g : {1, 2, . . . , n} →


{0, 1, . . . , n − 1}. Arătaţi că există o astfel de funcţie dacă şi numai dacă n este par.

Soluţie. Este clar că g(k) ∈ {0, . . . , n − 1} şi g e bijecţie dacă şi numai dacă
{f (1) + · · · + f (k) : k = 1, . . . , n} e un sistem de reprezentanţi distincţi (mod n).
Pentru n = 1, f (1) = 1 şi totul este clar. Pentru n = 2k + 1, k ≥ 1 fie există i ∈ {2, . . . , n}
astfel ı̂ncât f (i) = n şi atunci f (1) + · · · + f (i − 1) ≡ f (1) + · · · + f (i)(mod n), fie f (1) = n
iar f (1)+· · ·+f (n) = 1+· · ·+n = n· n+1 2
≡ 0(mod n) deci f (1) ≡ f (1)+· · ·+f (n)(mod n).
În concluzie o astfel de bijecţie f nu poate exista pentru n > 1 impar
Pentru n = 2k, k ≥ 1 alegem f (1) = n, f (2) = 1, f (3) = n−2, f (4) = 3, f (5) = n−
4, . . . , f (n−2) = n−3, f (n−1) = 2, f (n) = n−1 şi atunci resturile f (1)+· · ·+f (k)(mod n)
vor fi 0, 1, n − 1, 2, n − 2, . . . , n−2
2
, n+2
2
, n2 .

5. Spunem că o funcţie f : [0, ∞) → [0, ∞) are proprietatea P dacă

f (xf (y 2 )) = f (y)f (f (x2 )),

pentru orice x, y ∈ [0, ∞).


(a) Să se arate că există o infinitate de funcţii cu proprietatea P.
(b) Să se arate că există o unică funcţie cu proprietatea P a cărei imagine conţine
un interval deschis centrat ı̂n 1.

?
222

6. Rezolvaţi ı̂n numere ı̂ntregi x, y ecuaţia

q p
x = y 2 − y 2 + x.
p
Soluţie. Avem x ≥ 0; pentru x = 0, y 2 = y 2 + x = |y| deci y ∈ {−1, p 0, 1}, iar
2 2
pentru x = 1 avem doar y = 0. Presupunem xp> 1 şi avem x = y − y 2 + x <
y 2 − |y| ≤ y 2 deci |y| > x şi apoi x2 ≥ (x + 1)2 − y 2 + x deci y 2 + x ≥ 4x2 + 4x + 1 >
4x2 + x ⇒ |y| > 2x. Acum y 2 + x < y 2 + x2 ≤ (|y| + x)2 deci x2 > y 2 − |y| − x >
2x(2x − 1) − x = 4x2 − 3x > (2x − 1)2 deci x > 2x − 1, contradicţie. Deci singurele
soluţii sunt cele indicate la ı̂nceput.

7. Să se determine


 
1 1
min n ∈ N < { n} < .
100 10

Soluţie. Fie n = a2 + k, a, k ∈ N cu 0 ≤ k ≤ 2a. Presupunem că n este soluţie,


√ √ √ √ √
astfel ı̂ncât k 6= 0. Atunci { n} = n − b nc = a2 + k − a = k/(a + a2 + k). Dar

aceasta implică 1/10 > k/(a + a2 + k) > 1/(2a + 1), de unde mai departe 2a + 1 > 10
şi a ≥ 5. Deci n ≥ 26, ı̂nsă se verifică imediat că n = 26 este o soluţie, de unde minimul
căutat 26.

?
1 1
8. Fie a, b, c trei numere reale strict pozitive astfel ı̂ncât a + b + c ≥ a
+ b
+ 1c . Să se
arate că

3 2
a+b+c≥ + .
a + b + c abc

Soluţie. Din ipoteză abc(a+b+c) ≥ ab+bc+ca deci (ab+bc+ca)2 ≥ 3abc(a+b+c) ≥


3(ab + bc + ca) şi astfel ab + bc + ca ≥ 3 deci (a + b + c)2 ≥ 3(ab + bc + ca) ≥ 9, de unde
1
3
(a+b+c) ≥ 3/(a+b+c). Rămâne de arătat 23 (a+b+c) ≥ 2/abc, adică abc(a+b+c) ≥ 3,
ı̂nsă am stabilit că abc(a + b + c) ≥ ab + bc + ca ≥ 3, ceea ce ı̂ncheie demonstraţia.

?
P1
9. Să se arate că dacă a, b, c sunt numere reale strict pozitive astfel ı̂ncât a
≤ 3,
atunci
223

X a2 + 1
√ ≥ 6.
a2 − a + 1

Soluţie. Fie x = 1/a, y = 1/b, z = 1/c şi considerăm funcţia

t2 + 1
f : (0, ∞) → R, f (t) = √ .
t t2 − t + 1

Inegalitatea revine la f (x) + f (y) + f (z) ≥ 6 pentru x + y + z ≤ 3. Calculăm acum

−t3 − 2t2 + 3t − 2 4t5 + 11t4 − 32t3 + 35t2 − 20t + 8


f 0 (t) = , f 00 (t) =
2t2 (t2 − t + 1)3/2 4t3 (t2 − t + 1)5/2

şi considerăm polinoamele de la numărătorul fiecărei expresii (şi cele care toto-
dată le determină semnul).
Pentru g(t) = t3 + 2t2 − 3t + 2 avem g 0 (t) = 3t2 + 4t − 3 cu g 0 ≤ 0 pe (0, x+ ] şi
0

g ≥ 0 pe [x+ , ∞), unde x+ = ( 13 − 2)/3 ∈ (0, 5; 0, 6). Deci ∀t, g(t) ≥ g(x+ ) > 0 după
cum se poate uşor estima. În concluzie f 0 (t) ≤ 0 pe (0, ∞) deci f este descrescătoare.
Pentru h(t) = 4t5 +11t4 −32t3 +35t2 −20t+8 avem h000 (t) ≥ 0 pentru 10t2 +11t−8 ≥
0, ceea ce se ı̂ntâmplă pentru t ≥ 1/2. Deci h00 are un minim ı̂n 1/2 care este 17 > 0.
Rezultă h0 strict crescătoare pe [0, ∞), h0 (0) = −20, h0 (x) = 0 ı̂n x0 ∈ (0, 62; 0, 63) (se
arată că h0 (0, 62) < 0 < h0 (0, 63)). Deci h are un minim ı̂n x0 , şi astfel ∀t, h(t) ≥ h(x0 ) > 0
ı̂nlocuind pe x0 cu 0, 62 ı̂n monoamele cu coeficient nenegativ şi cu 0, 63 ı̂n cele cu coe-
ficient negativ. Rezultă f 00 ≥ 0 deci f e convexă.
Soluţia se ı̂ncheie observând că

x + y + z0
 
f (x) + f (y) + f (z) ≥ f (x) + f (y) + f (z 0 ) ≥ 3f = 3f (1) = 6,
3

unde z 0 = 3 − x − y ≥ z, aplicând f (z) ≥ f (z 0 ) şi apoi Jensen.

10. Fie a, b, c ∈ R astfel ı̂ncât |ax2 + bx + c| ≤ 1, (∀)x ∈ [−1, 1]. Să se arate că dacă
α ∈ [0, 1], atunci

α(1 + α)|b| + (1 − α2 )|c| ≤ 1 + α2 .

Când are loc egalitatea?

?
224

11. Să se determine coordonatele vârfurilor triunghiului ABC cunoscând picioarele


36 26
, 17 ; E 21 , 12 ; F 65 , 85 .
  
ı̂nălţimilor D 17
Soluţie. Fie S suprafaţa lui 4DEF şi d, e, f drepte cu d ∩ S = {D}, e ∩ S = {E},
f ∩ S = {F } astfel ı̂ncât ∠(F D, d) = ∠(d, ED) şi celelalte. Pentru orice soluţie 4ABC
se poate deduce ∠ADE = ∠ADF ⇒ ∠EDC = ∠BDC deci BC = d şi celelalte şi apoi
{A} = e ∩ f , etc., deci singura soluţie va fi 4ABC determinat.
1 7
Avem pantele mF D = − 13 , mDE = 11 , mEF = 11
7
26
. Fie ecuaţia lui d : y = 17 +
36

m x − 17 . Relaţia tg∠(F D, d) = tg∠(d, ED) revine la

1 7
− 13 −m m − 11
1 = 7
1 − m · 13 1 + m · 11

cu unica soluţie convenabilă m = −4. La fel, e : y = 12 +m0 x − 12 cu ∠(DE, e) =




∠(e, F E) duce la m0 = −1 şi f : y = 58 +m00 x − 65 cu ∠(f, DF ) = ∠(EF, f ) are ca unică




soluţie pe m00 = 21 . Astfel A ∈ e∩f, A(x, y) ⇒ 21 − x − 12 = 58 + 12 x − 65 , 1−x = 1+x/2


 

deci x = 0, A(0, 1) . La fel, pentru B ∈ f ∩d, B(x, y) avem 85 + 12 x − 56 = 26 36


 
17
−4 x − 17
,
1+x/2 = 10−4x deci x = 2, B(2, 2) . În final C ∈ d∩e, C(x, y) duce la 10−4x = 1−x ⇒
x = 3, C(3, −2) . Se verifică imediat că triunghiul astfel obţinut satisface ipoteza.

12. Se consideră triunghiul ascuţitunghic ABC având ortocentrul H şi ı̂nălţimile


AM, BN, CP . Fie Q şi R mijloacele segmentelor BH respectiv CH şi notăm U =
M Q ∩ AB, V = M R ∩ AC, T = AH ∩ P N . Să se arate că:
(a) M
MA
H
= TT H
A
;
(b) T este ortocentrul triunghiului U AV .
Soluţie. (a) Presupunem cunoscute lungimile elementelor importante din configuraţie.
Din teorema transversalei,

TM BM CN CM BP c · cos B a · cos C b · cos C a · cos B


= · + · = · + · ,
TA BC N A BC P A a c · cos A a b · cos A
TM
TA
= 2 coscos
B cos C
A
TA
deci M A
= cos A+2cos A
cos B cos C
iar TT H
A
= AH
TA
−1 = −1+ M2RAcos A
cos A
(cos A+
a2 bc MH 2R cos B cos C a2 bc
2 cos B cos C) = −1+ 4S 2 (cos A+2 cos B cos C). Acum M A = 2S/a
= 4S 2 cos B cos C
2
şi egalitatea revine la 1 = a4Sbc2 (cos A + cos B cos C). Însă AM = 2S/a = AH + HM =
2
2R cos A + 2R cos B cos C implică cos A + cos B cos C = a4S2 bc şi astfel relaţia cerută.
(b) Din simetrie este suficient să arătăm U T ⊥ AV , sau U T k BN adică UP B
U
= PT
TN
.
Din nou teorema transversalei (ı̂n 4P BC) ne arată că
225

HQ P H CM CH P U
1= = · + ·
QB P C MB P C UB

de unde P C = 2R cos C · UP B U
+ 2R cos Ac cos
cos B·b cos C
B
şi astfel UP B
U
= c·2R2Scos C − b cos
c
A
=
16S 2 2 2
a −b −c 2 PU
2 2 2 4
P P
2c2 (−c2 +a2 +b2 )
+ 2c2 şi folosind 16S = 2 a b − a se obţine ı̂n final U B =
b2 (a2 +c2 −b2 ) PH AC NT PT
c2 (a2 +b2 −c2 )
. Acum Menelaus ı̂n 4AP C ne arată că HC
· AN
· TP
= 1 de unde TN
=
PH b2 (a2 +c2 −b2 )
HC AN
· AC = coscos A cos B
C
· b
c cos A
= b cos B
c cos C
= c2 (a2 +b2 −c2 )
.

13. Considerăm un triunghi ABC, punctul M al laturii BC pentru care M


MC
B
= c(b+c)
b2
şi punctul N al segmentului AM pentru care ∠BN M = A. Să se arate că ∠CN M = A2 .
Soluţie (Virgil Nicula). Fie S ∈ (AB), M S k AC şi R ∈ (AC), M R k AD unde
SA MC b2 SA b2
D ∈ (BC) este piciorul bisectoarei din A. Avem SB =M B
= bc+c 2 deci AB = b2 +bc+c2 şi
b2 c2
astfel AS · AB = b2 +bc+c 2 . Cum ∠SBN = ∠BN M − ∠BAN = A − ∠BAN = ∠M AC =

∠N M S (am folosit ipoteza şi M S k AC) avem SBM N inscriptibil şi deci AN · AM =
b2 c2
AS · AB = b2 +bc+c 2.
MC b2 ab2 DC b ab
Acum BC = b2 +bc+c 2 ⇒ M C = b2 +bc+c2 iar BC = b+c ⇒ DC = b+c şi deci

ab ab2 abc2
DM = |DC − M C| = b+c
− b2 +bc+c2
= (b+c)(b2 +bc+c2 )
. Acum M R k AD conduce la
AR 2 b 2 c2
AC
= DM
DC
c
= b2 +bc+c2 şi deci AR · AC = AN · AM = AR · AC,
b2 +bc+c2
. În concluzie
deci M N RC este inscriptibil şi astfel ∠CN M = ∠CRM = ∠CAD = A2 , ceea ce trebuia
demonstrat.

14. Fie ABC un triunghi şi I centrul cercului ı̂nscris. Fie A1 , B1 , C1 respectiv centrele
cercurilor ı̂nscrise ı̂n triunghiurile IBC, ICA, IAB. Arătaţi că AA1 , BB1 , CC1 sunt
concurente.
Soluţie. Fie {A0 } = AA1 ∩ BC şi dreapta XA1 Y k BC unde X ∈ (AB) şi Y ∈
0
(AC). Rezultă BA CA0
= XA 1
Y A1
. Din teorema sinusurilor ı̂n 4BA1 X, sin ∠XBA XA1
1
BA1
= sin ∠BXA 1
BA1
iar cum sin ∠BXA1 = sin B (din XA1 k BC), XA1 = sin 3B 4
· sin B
. Analog Y A1 =
0
sin 3C
4
CA1
· sin . Concluzionăm BA
C  CA0 
= sin 3B/4
sin 3C/4
sin C
· sin B
· BA
CA1
1
. Dar BA
CA1
1
= sinsin C/4
B/4
. Rezultă că
Q BA0 Q sin 3B/4 sin C sin C/4
CA0
= · ·
sin 3C/4 sin B sin B/4
= 1 şi din reciproca teoremei Ceva AA1 , BB1 , CC1
vor fi concurente.

15. Într-un campionat de fotbal se ı̂nscriu 18 echipe. Fiecare echipă poate să joace
cel mult un meci cu o altă echipă. Spunem că campionatul este bun dacă la sfârşitul
său oricum am alege 12 echipe din cele 18, există cel puţin 6 meciuri disputate ı̂ntre ele.
226

Determinaţi numărul minim posibil de meciuri care se desfăşoară ı̂ntr-un cam-


pionat bun.
Soluţie. În termeni de grafuri, se cere n minim astfel ı̂ncât să existe un G = (V, E)
cu |V | = 18 şi |E| = n astfel ı̂ncât oricare ar fi A ⊂ E cu |A| = 12, A induce un subgraf
cu cel puţin şase muchii.
Pentru n = 18 luăm G o reuniune disjunctă de 6 triunghiuri (C3 ) şi astfel alegând
din x triunghiuri trei vârfuri, din y două şi din z unul singur, x+y+z ≤ 6 şi 3x+2y+z =
12, se obţine un subgraf indus cu 3x + y = 12 − (y + z) ≥ 6 muchii şi totul este clar.
Arătăm că pentru n ≤ 17, oricare ar fi G există A ⊂ E cu |A| = 12 care in-
duce un subgraf cu cel mult 5 muchii. Fie A ⊂ E, |A| = 12 care induce un subgraf cu
numărul minim m de muchii. Pentru m < 6 am terminat, şi vom presupune m ≥ 6.
Dacă m ≥ 7, va exista v ∈ A cu gradul ı̂n A cel puţin 2. Din principiul cutiei printre
cele 6 vârfuri din V \A va exista unul legat de cel mult un alt vârf din A (ne-au mai
rămas cel mult 10 legături ı̂ntre un vârf din V \A şi unul din A), să zicem w. Luăm
acum A0 = (A\{v}) ∪ {w} şi observăm că numărul de muchii ı̂ntre membrii A\{v}
a rămas constant, ı̂n timp ce avem cel mult o muchie ı̂n E(w, A\{v}) şi am eliminat
|E(v, A\v)| ≥ 2. Deci numărul de muchii a scăzut sub m ı̂n subgraful indus de A0 ,
contradicţie.
Considerăm acum cazul m = 6. Dacă există ı̂n A cu |E(A)| = m un vârf de
grad cel puţin 2 procedăm la fel ca ı̂n cazul anterior. Altfel A va fi reuniunea a şase
muchii disjuncte, iar un v ∈ V \A va fi legat de cel mult un vârf w ∈ A. Luând la fel
A0 = (A\{w}) ∪ {v} obţinem un subgraf indus cu cinci muchii, din nou contradicţie cu
minimalitatea lui m şi deci concluzia.

14.4 Clasa a X-a


1. Vom demonstra mai ı̂ntâi o inegalitate ajutătoare şi anume:

a b c 1 1 1
+ + ≥ + + .
b c a ab bc ca
Aceasta din urmă este echivalentă cu

a2 c + b2 a + c2 b ≥ a + b + c,

care rezultă prin aplicarea inegalită ţii Cauchy-Schwartz astfel


 
1 1 1
+ + (a2 c + b2 a + c2 b) ≥ (a + b + c)2
c a a
227

şi folosind ipoteza problemei rezultă inegalitatea ajutătoare. Inegalitatea din


enunţ este echivalentă cu
 
2 1 1 1
(a + b + c) ≥ 3 + 2 + + .
ab bc ca

Folosindu-ne de inegalitatea ajutătoare vom ară ta că


 
1 1 1 1 1 1 a b c
(a + b + c) + + ≥3+ + + + + + .
a b c ab bc ca b c a

Această ultimă inegalitate se mai scrie

a b c b c a 1 1 1 a b c
+ + + + + +3≥3+ + + + + + .
b c a a b c ab bc ca b c a
ne rămâne să demonstrăm că
b c a 1 1 1
+ + ≥ + +
a b c ab bc ca
care este echivalentă cu

a2 b + b2 c + c2 a ≥ a + b + c.

La fel ca la inegalitatea ajutătoare, folosind inegalitatea Cauchy-Buniakovski-


Schwartz, avem
 
1 1 1
a2 b + b2 c + c2 a (a + b + c)2

+ +
b c a

de unde folosind ipoteza problemei rezultă imediat inegalitatea de mai sus.


Observaţie. La Barajul se selecţie al lotului Olimpic pentru juniori a fost dată
urmaătoarea problemă:
” Fie a, b, c trei numere reale strict pozitive astfel ı̂ncât

1 1 1
a+b+c≥ + + .
a b c
Să se arate că
3
a+b+c≥ .”
abc
Demonstraţia inegalităţii de mai sus este destul de simplă. Ipoteza mai poate fi
scrisă şi sub forma:
abc(a + b + c) ≥ ab + bc + ca.
228
p
Însă, folosind inegalitatea ab + bc + ca ≥ 3abc(a + b + c), avem
p
abc(a + b + c) ≥ 3abc(a + b + c) ⇔
abc(a + b + c) ≥ 3
adică problema enunţată mai sus. Folosind această problemă rezultă
2 2
(a + b + c) ≥
3 abc
şi astfel ne rămâne să arătăm că
3 2
a+b+c≥ + (a + b + c)
a+b+c 3
care estre echivalentă după calcule cu
 
1 1 1
a + b + c ≥ 3 care se deduce uşor din (a + b + c) + + ḑin ipoteză.
a b c
2. Problema este aproape identică cu problema 3, dată la primul baraj de selecţie al
lotului pentru OIM. Pentru detalii recomandăm RMC 2006, paginile 6-65.

3. Fie kl numărul de cifre de 0. Un număr de kn cifre dintre cele căutate poate


fi identificat, ı̂nlocuind fiecare secvenţă de k cifre de 0 cu s, cu o secvenţă de
k(n − l) de 1 şi l de s, având 1 pe prima poziţie. Notând x1 < x2 < . . . < xl
poziţiile pe care se află s ı̂n secvenţă, avem: x1 > 1. xi ∈ {1, 2, . . . k(n − l) + l} pt
orice i ∈ {1, . . . l}.
Numărul de numere cu kl cifre de 0 cu proprietăţile din ipoteză este egal cu
numărul de moduri de a alege numerele xi cu proprietaţile de mai sus, deci
Xn
l l
Ck(n−l)+l−1 . Deci numărul total de numere este: Ck(n−l)+l−1 .
l=1

4. Orice produs de a numere consecutive este divizibil cu a!. (22n + 2n+m + 22m )!
este un produs de 22n + 2n+m + 22m = 2n (2n + 2m−1 ) + 2m (2m + 2n−1 ) numere
consecutive, deci poate fi ı̂mpărţit ı̂n 2n +2m−1 secvenţe de 2n numere consecutive,
fiecare dintre ele divizibilă cu (2n )!, urmate de 2m + 2n−1 secvenţe de 2m numere
consecutive, fiecare divizibilă cu (2m )!:
(22n + 2n+m + 22m )! = 1 · 2 · . . . 2n · (2n + 1 · . . . (2 · 2n )) . . .
   

· (2n (2n +2m−1 −1)+1) . . . (2n (2n +2m−1 )) (22n +2m+n−1 +1) . . . (22n +2m+n−1 +2m ) . . .
  

· (22n + 2m+n−1 + 2m (2m + 2n−1 − 1) + 1) . . . (22n + 2m+n−1 + 2m (2m + 2n−1 )) .


 

Fiecare din primele 2n +2m−1 paranteze drepte este divizibilă cu (2n )! şi fiecare din
ultimele 2m +2n−1 paranteze drepte este divizibilă cu (2m )!, deci (22n +2n+m +22m )!
n m−1 m n−1
este divizibil cu (2n !)2 +2 · (2m !)2 +2 .
229

5. Problema de faţă nu este altceva decât o extindere a următoarei probleme date la


Barajul de Selecţie OIM 2002 şi care are următorul enunţ:
” Fie (an )n≥0 un şir definit astfel: a0 = a1 = 1 şi

an+1 = 14an − an−1 , ∀n ≥ 2.

Să se arate că numărul 2an − 1 este pătrat perfect. ”


În soluţionarea problemei de mai sus, vom mai defini ı̂ncă un şir şi anume: ((bn )n≥0
definit prin b0 = −1, b1 = 1 şi bn+1 = 4bn − bn−1 , ∀n ≥ 1. Vom arăta, prin inducţie,
că
2an − 1 = b2n .
Mai bine zis, vom mai arăta că

2bn bn−1 = an + an−1 − 4.

Evident că b20 = 2a0 − 1 = 1, b21 = 2a1 − 1 şi 2b0 b1 = −2 = a0 + a1 − 4. Acum


presupunem adevărate egalităţile de mai sus pentru n şi le demonstrăm pentru
n + 1. Avem
b2n+1 = (4bn −bn−1 )2 = 16(2an −1)+(2an−1 −1)−4(an +an−1 −4) = 2(14an −an−1 ) =
2an+1 − 1 şi

2bn+1 bn = 2(4bn − bn−1 )bn = 8b2n − 2bn bn−1 = 15an − an−1 − 4 = an+1 + an − 4.

Astfel inducţia este complet demonstrată.


Propunem cititorilor să adapteze problema mai sus rezolvată pentru problema
din lista scurtă. Pentru alte şiruri de acelaşi tip propunem consultarea RMC 2006,
problema 1 de la Barajul 2 de Selecţie Seniori , paginile 65-66.

6. Să considerăm funcţia f : R → (0, ∞), f (x) = log3 (2x +1) care este strict crescătoare
şi surjectivă, deci bijectivă, iar inversa ei este f −1 (x) = log2 (3x −1), f : (0, ∞) → R.
Astfel, ecuaţia se mai scrie

f (f (x)) = f −1 (x) ⇔

f (f (f (x))) = 1R .
Să presupunem prin absurd că există x0 ∈ R astfel ı̂ncât f (x0 ) 6= x0 . Fie x0 < f (x0 )
de unde f (x0 ) < f (f (x0 )) şi f (f (x0 )) < f (f (f (x0 ))) = x0 , ceea ce reprezintă o
contradicţie. Analog, dacă f (x0 ) > x0 . Prin urmare, f (x) = x şi ecuaţia devine
( 23 )x + ( 13 )x = 1 cu soluţia unică x = 1.

7.
230

8.

9. a) Luăm punctele de tangenţă al cercului C de rază r intersectate cu dreptele


A1 A2 , A2 A3 , A3 A4 respectiv A4 A1 . Intersecţia lor vor fi punctele B1 , B2 , B3 respec-
tiv B4 , iar cercurile C(A1 , A1 B1 ), C(A2 , A2 B2 ), C(A3 , A3 B3 ) respectiv C(A4 , A4 B4 )
satisfac concluzia problemei.
b) Folosind inegalitatea mediilor sau C.B.S avem
4
X 1 16
≥ 4
r
i=1 i
P
ri
i=1

de unde rezultă că


4
X
ri ≤ 4r.
i=1

Cum cercurile mai sus amintite sunt tangente două câte două, rezultă că
4
X
ri = p,
i=1

unde p este semiperimetrul


p patrulaterului. Cum patrulaterul este inscriptibil,
rezultă că S = (p − a)(p − b)(p − c)(p − d) şi avem

p ≤ 4r.

Vom demonstra că p ≥ 4r. Din inegalitatea mediilor, avem:


p
2p = (p − a) + (p − b) + (p − c) + (p − c) ≥ 4 4 (p − a)(p − b)(p − c)(p − d) ⇔

2p ≥ 4 S
de unde rezultă p ≥ 4r. Egalitatea are loc dacă şi numai dacă a = b = c = d, adică
dacă patrulaterul ABCD este pătrat.

10.

11. Vom algebriza un pic inegalitatea din enunţ. Notăm x = b2 +c2 −a2 , y = c2 +a2 −b2
şi z = a2 + b2 − c2 . Astfel vom obţine
xy + yz + zx 2 a2 b2 c2 (x + y)(y + z)(z + x) x+y+z xyz
S2 = ,R = = = − ,
16 16S 2 8(xy + yz + zx) 8 8(xy + yz + zx)
de unde vom obţine
1 xyz
R2 = − .
2 8(xy + yz + zx)
231

Astfel, inegalitatea care trebuie s-o demonstrăm devine


xy + yz + zx 3xyz
≥ ⇔
16 4(xy + yz + zx)
(xy + yz + zx)2 ≥ 3xyz(x + y + z)
care este echivalentă cu
x2 y 2 + y 2 z 2 + z 2 x2 ≥ xyz(x + y + z).
Dacă luăm m = xy, n = yz şi p = zx rezultă banala inegalitate m2 + n2 + p2 ≥
mn + np + pm.

12. Fie r1 , r2 respectiv r3 razele cercurilor ı̂nscrise ale triunghiurilor P BC, P CA re-
spectiv P AB. Avem
a b c ap1 bp2 cp3
+ + = + + ,
r1 r2 r3 S1 S2 S3
unde S1 , p1 reprezintă aria, respectiv semiperimetrul triunghiului P BC, S2 , p2
reprezintă aria, respectiv semiperimetrul triunghiului P CA şi S3 , p3 reprezintă
aria, respectiv semiperimetrul triunghiului P AB. Mai departe, avem
X a X a(P B + P C + a) X P B + P C + a
= = .
r
cyc 1 cyc
2S1 cyc
da

Mai departe, avem


X PB + PC X a
+ ,
cyc
da cyc
d a

unde da , db , dc sutn distanţele de la P la laturile BC, CA respectiv AB ale triunghi-


ului ABC.
√ Pe de altă parte, folosind inegalitaţile C.B.S. respectiv Mitrinovic,
2
p ≥ 3 3S rezultă
!
X a X X a √
2S = ada ≥ (a + b + c)2 ≥ 12S 3,
cyc
da cyc
da

de unde rezultă că


a b c √
+ + ≥ 6 3.
da db dc
Din inegalitatea mediilor, avem
sQ

r
X PB + PC 3 (P B + P C) 8P AP BP C 3
≥3 ≥33 ≥ 3 64 = 12.
cyc
da da db dc da db dc

Concluzia problemei decurge imediat prin adunarea celor două inegalităţi de mai
sus.
232

13. Din cunoscuta identitate

(a + b + c)3 − (a3 + b3 + c3 ) = 3(a + b)(b + c)(c + a),

obţinem că
(a + b + c)3 = 3 · 64 + 24
de unde, trecând la module, avem că |a + b + c| = 6. Pe de altă parte, trecând la
module ı̂na doua relaţie şi folosind-o pe cea de-a treia rezultă imediat că

|a + b| = |b + c| = |c + a| = 4.

Însă, din inegalitatea triunghiului aplicată pentru numerele a + b, b + c respectiv


c + a avem
|a + b| + |b + c| + |c + a| ≥ 2|a + b + c|,
cu egalitate dacă şi numai dacă a + b = k(b + c), b + c = k(c + a) c si c + a = k(a + b)
unde k ∈ R+ . Cum |a + b| = |b + c| = |c + a|, avem k = 1 şi deci, a = b = c, iar din
prima condiţie obţinem a3 = 8 care este o ecuaţie binomă uşor de rezolvat.

14. Ridicând la pătrat inegalitatea, trebuie să demonstrăm că

3R(ma + mb + mc )
p2 ≤ ⇔
2
(a + b + c)2
ma + mb + mc ≥ .
6R
Vom demonstra o inegalitate mai tare şi anume

a2 + b 2 + c 2
ma + mb + mc ≥ .
2R
Notăm cu A1 , B1 repectiv C1 picioarele medianelor duse din varfurile triunghiu-
lui ABC, iar cu A2 , B2 respectiv C2 punctele de intersecţie ale acestora cu cercul
circumscris. Evident că AA2 , BB2 , CC2 ≤ 2R Vom obţine că ma + A1 A2 ≤ 2R plus
analoagele. Acum, din puterea punctului A1 faţă de cerc avem

A1 A2 · AA1 = BA1 · A1 C,

de unde rezultă că


b2 + c 2
ma ≥ .
4R
Scriind şi analoagele şi adunându-le vom obţine concluzia.
Capitolul 15

Probleme avute ı̂n atenţia comisiei, 2007

15.1 Clasa a VII-a


1. Pentru orice N ∈ N∗ alegem n = 2N . Rezultă pentru a ∈ N că a + (a + 1) + ... +
.
(a + (2N − 1)) = 2N a + (2N −1)2N = 2N a + N (2N − 1) = N (2a + 2N − 1)..N . În
2
concluzie, pentru toate numerele naturale are loc proprietatea din enunţ.

2. Fie, prin absurd, 2007 = p2 + q 2 + r2 + s2 , p ≥ q ≥ r ≥ s prime. Evident printre


p, q, r, s sunt un număr impar de numere pare, deci ori s = 2, r > 2, ori q = r =
s = 2, p > 2, ultima situaţie fiind imposibilă. Deci s = 2 şi r ≥ 3. Dacă r > 3,
avem p, q, r = M3 ±1, deci p2 , q 2 , r2 = M3 +1 şi apoi 3|p2 +q 2 +r2 = 2003, fals. Deci
r = 3, iar p2 +q 2 = 1994. Cum 1994 = M3 +2, rezultă p, q = M3 ±1 şi apoi p, q ≥ 5.
Dacă q > 5, atunci p2 , q 2 = M5 + 1, M5 + 4 deci p2 + q 2 = M5 + 2, M5 + 8, M5 + 5,
adică p2 + q 2 = M5 , M5 + 2, M5 + 3. Dar p2 + q 2 = 1994 = M5 + 4, contradicţie.
Rezultă q = 5, iar apoi p2 = 1969, ceea ce reprezintă contradicţia dorită.

AB IB CB
3. a) Din Teorema Bisectoarei, AD
= ID
= CD
⇒ AB · CD = AD · BC.
AB AJ AB AD
b) Fie J intersecţia AC cu bisectoarea din B. Rezultă BC
= JC
, dar BC
= DC
, deci
AD AJ
DC
= JC , adică J aparţine şi bisectoarei din D.
c) Cum, de asemenea, d(I, AB) = d(I, AD) şi d(I, BC) = d(I, CD) rezultă I
centrul cercului ı̂nscris ı̂n ABCD, deci ABCD circumscriptibil ⇒ AB + CD =
BC + AD = BC + AB·CD BC
⇔ (BC − AB)(BC − CD) = 0. Dacă BC = CD,
[CI] fiind bisectoare ı̂n 4CDB isoscel ı̂n ∠C, CI ⊥ BD iar BI = ID, deci ı̂n

233
234

4ADB avem [AI] bisectoare şi mediană ⇒ AI ⊥ BD. Deci A, I, C coliniare


⇒ AC = CI ⊥ BD, ceea ce trebuia arătat. Dacă BC = AB, AB 2 = BC 2 ⇒
2 2
(AB 2 − AD2 )(AB 2 − BC 2 ) = 0 ⇔ AD2 + BC 2 = AB 2 + ADAB·BC
2 ⇔ AD2 + BC 2 =
AB 2 + CD2 , de unde concluzia.

4. a) Avem AB = AH = sinAE ∠C
= ABsincos
∠C
∠A
⇒ cos ∠A = sin ∠C. De aici cos ∠A > 0, iar
cos ∠A + cos ∠C = 1 ⇔ 60a b + 64a + 9b6 = 144a4 b2 , din Teorema Cosinusului.
2 2 2 4 6

Folosind b = 2c, obcţinem AC = 4OM ⇔ b = 4·R·cos ∠A ⇔ b = 2· sinb∠B ·cos ∠A ⇔


sin ∠B = 2 cos ∠A ⇔ b sinc ∠C = 2 cos ∠A ⇔ 2c sinc ∠C = 2 cos ∠A ⇔ sin ∠C = cos ∠A,
relaţie deja stabilită.
b) Avem AH = 2R cos ∠A, OM = R cos ∠A şi BM = a2 . Inegalitatea AH +
BM > OM ⇔ R cos ∠A + a2 > 0 este evidentă. Acum OM + BM > AH ⇔
a > 2R cos ∠A ⇔ a > sinc∠C · cos ∠A, iar cum cos ∠A = sin ∠C, trebuie arătat
a > c ⇔ 2a > b. Dacă, prin reducere la absurd, 2a ≤ b, rezultă b4 ≥ 16a4 , deci
144a4 b2 = 60a2 b4 + 64a6 + 9b2 · b4 ≥ 60a2 b4 + 64a6 + 9b2 · 16a4 = 60a2 b4 + 64a6 +
144a4 b2 ⇒ 0 ≥ 60a2 b4 + 64a6 , evident fals. Mai rămâne AH + OM > BM ⇔
6R cos ∠A > a ⇔ 6R sin ∠C > a ⇔ 3 · sinc∠C · sin ∠C > a ⇔ 3c > a ⇔ 3b > 2a. Din
nou, dacă prin absurd 3b ≤ 2a, obţinem 4a2 ≥ 9b2 , iar 144a4 b2 = 9b6 + 60a2 b4 +
16a4 · 4a2 ≥ 9b6 + 60a2 b4 + 16a4 · 9b2 = 9b6 + 60a2 b4 + 144a4 b2 ⇒ 0 ≥ 9b6 + 60a2 b4 ,
contradicţie.

5. Fie E, F punctele unde se ı̂ndoaie laturile [AD], respectiv [BC], iar α = m(∠
AEM ), x = AE. Presupunem fără pierdere de generalitate că AB = 1. Dacă
α ∈ {0◦ , 90◦ }, suprafaţa ı̂ndoită este jumătate din aria pătratului, deci concluzia
se verifică. În caz contrar, figura este nedegenerată şi fie M C ∩ BF = {N }. Avem
AE + EM = 1 ⇒ x + cosx α = 1 ⇒ x = coscosα+1 α
. Rezultă M E = cos 1α+1 , AM = x tg
α = cossinα+1
α
, apoi M B = 1−AM = cos cos α−sin α+1
α+1
, M N = cos α−sin α+1
cos2 α+cos α
, N C = 1−M C =
sin α−sin2 α
cos2 α+cos α
şi F C = cos α+1 . Avem [M N F E] > 4 ⇔ [M CF E] − [N CF ] > 14 ⇔ M C ·
1−sin α 1
M E+F C
> 14 + N C·CF ⇔ 2 + 2 sin2 α + 3 cos α > cos3 α + 4 sin α + 2 sin α cos α. Punem
2 2 √
cos α = x ∈ (0, 1), iar sin α = 1 − x2 . Inegalitatea se reduce la −x3 −2x2 +3x+4 >

2(x + 2) 1 − x2 ⇔ (−x3 − 2x2 + 3x + 4)2 > 2(x + 2)(1 − x2 ) ⇔ x(x5 + 4x4 + 2x3 −
4x2 +5x+8) > 0. Dar x > 0, deci trebuie arătat (x5 +4x4 +2x3 +5x)+(8−4x2 ) > 0.
Cum evident x5 + 4x4 + 2x3 + 5x > 0, iar x2 < 1 ⇒ −4x2 > −4 ⇒ 8 − 4x2 > 4 > 0,
concluzia urmează cu uşurinţă.

?
235

6. Notând n = 22007 + 1, cerinţa se reduce la a arăta că

1 1 1
S =1+ + + ... + ∈/ N.
2 3 n

Fie L = [1, 2, ..., n]. Evident 22007 ||L. Punând

n
X L
A= ,
k=1
k

avem S = A L
. Arătăm că A este impar. Atunci, cum 2|L, rezultă L - A şi deci
A
S= L ∈ / N. Observăm ı̂nsă că Lk este par pentru k ∈ 1, n − 2 ∪ {n} şi impar pentru
k = n − 1 = 22007 , iar concluzia rezultă cu uşurinţă.
Remarcă. Demonstraţia se poate extinde cu uşurinţă pentru a demonstra că ∀n ∈
N, n ≥ 2,

1 1 1
Sn = 1 + + + ... + ∈/ N.
2 3 n

7. Vom folosi următoarea


Lemă. Dacă x, y ∈ N, iar 7|x2 + y 2 , avem 7|x şi 7|y.
Demonstraţie. Un pătrat perfect este congruent doar cu 0, 1, 2 sau 4 modulo 7.
După cum se poate vedea din tabelul de mai jos, suma x2 + y 2 de două pătrate
perfecte este 0 modulo 7 doar când pătratele sunt 0 modulo 7, ceea ce ı̂nseamnă
7|x şi 7|y. 

+ x2 0 1 2 4
y2
0 0 1 2 4
1 1 2 3 5
2 2 3 4 6
4 4 5 6 1

Ecuaţia se scrie 74 ·13 = (2x)2 +(3y)2 . Din Lemă deducem 7|2x, 7|3y şi deci 7|x, 7|y,
iar x = 7x1 , y = 7y1 , x1 , y1 ∈ N. Astfel 72 ·13 = (2x1 )2 +(3y1 )2 . Aplicănd ı̂ncă o dată
Lema obţinem x1 = 7x2 , y1 = 7y2 , x2 , y2 ∈ N. Deci 4x22 + 9y22 = 13, cu unica soluţie
(x2 , y2 ) = (1, 1) şi astfel (x, y) = (49, 49) este unica pereche verificând enunţul.
236

8. Se arată că dacă a1 , a2 , a3 sunt primele trei zecimale (ı̂n ordine) ale lui A, atunci
  (p )
[1000 · A] [ (125(32n + 1))2 + 1252 · 128447]
0, a1 a2 a3 = = .
1000 1000

Dar n ≥ 250874 > 250873, 0155 ⇒ 1252 · 128447 < 250(32n + 1) + 1 ⇒ (125(32n +
1))2 ≤ (125(32n + 1))2 + 1252 · 128447 < (125(32n + 1) + 1)2 ⇒ 125(32n + 1) ≤
1
((125(32n + 1))2 + 1252 · 128447) 2 < 125(32n + 1) + 1 ⇒ [((125(32n + 1))2 + 1252 ·
1
128447) 2 ] = 125(32n + 1), deci
   
125(32n + 1) 1
0, a1 a2 a3 = = 4n + = 0, 125.
1000 8

În concluzie, primele trei zecimale sunt ı̂ntotdeauna 1, 2, respectiv 5.

9. Prin calcul, 9A2 = (2 · 102008 + 8)2 − 108. Se verifică (2 · 102008 + 7)2 < 9A2 <
(2 · 102008 + 8)2 . Însă 9A2 ∈ N, 9A2 6= k 2 , k ∈ N implică 3A ∈
/ Q, deci A ∈
/ Q, de
unde A ∈ / N, adică A nu este natural.

10. Fie AB = a, iar ∠A = 120◦ şi ∠B = ∠C = 30◦ . Din Teorema Sinusurilor, BC =


√ √
a 3. De asemenea, AD = AB · tan 30◦ = √a3 . Rezultă BC · AD = a 3 · √a3 = a2 =
AB 2 , ceea ce trebuia arătat.

11. a) Presupunând contrariul, din 3a2 + 223b2 = 223 · 9 · 10001 rezultă 223|3a2 , iar 223
fiind prim obţinem a = 223a1 , a1 ∈ N şi deci 3 · 223a21 + b2 = 32 · 10001. De aici 3|b,
b = 3b1 , b1 ∈ N şi 223a21 + 3b21 = 3 · 10001. Rezultă 3|223a21 , apoi a1 = 3a2 , a2 ∈ N iar
3 · 223a22 + b21 = 10001. Luând egalitatea modulo 3, avem b21 ≡ 10001 ≡ 2 (mod 3),
o contradicţie deoarece pătratele perfecte sunt 0 sau 1 (mod 3).
b) La fel ca la punctul precedent deducem 223 · 3 · a22 + b21 = k 2 , unde a = 223a1 ,
a1 = 3a2 , b = 3b1 , a1 , a2 , b1 ∈ N∗ . Dacă a2 6= 1, ∃p prim cu p|a2 ⇒ a2 = kp, k ∈ N∗ ⇒
a = 223 · 3 · p · k, deci 1, 3, 223, 3 · 223|a, iar cum a are exact 4 divizori obligatoriu
p ∈ {3, 223}, dar atunci fie 32 |a, fie 2232 |a iar a are şi ı̂n acest caz mai mult de 4
237

divizori, fals. Rezultă a2 = 1, şi deci 669 = (k − b1 )(k + b1 ), k + b1 > 0 şi apoi
k − b1 > 0, deci (k − b1 , k + b1 ) ∈ {(1, 669); (3, 223); (223, 3); (669, 1)}. Convin doar
(k − b1 , k + b1 ) ∈ {(1, 669); (3, 223)} ⇔ (k, b1 ) ∈ {(335, 334); (113, 110)}. Obţinem
mulţimea soluţiilor (a, b, k) ∈ S = {(669, 330, 113); (669, 1002, 335)}.

12. a) Avem ∠H 0 BC = ∠HBC = 90◦ − ∠C = ∠H 0 AC, deci H 0 ABC inscriptibil


⇒ H 0 ∈ ω.
b) O0 ∈ ω ⇔ ABO0 C inscriptibil ⇔ ∠A + ∠BO0 C = 180◦ ⇔ ∠A + ∠BOC =
180◦ ⇔ ∠A + 2 · ∠A = 180◦ ⇔ ∠A = 60◦ .
c) I 0 ∈ ω ⇔ ABI 0 C inscriptibil ⇔ ∠A + ∠BI 0 C = 180◦ ⇔ ∠A + ∠BIC = 180◦ ⇔
∠A + ∠A + ∠B+∠C2
= 180◦ ⇔ 90◦ + 3∠A2
= 180◦ ⇔ ∠A = 60◦ .
d) G0 ∈ ω ⇔ ABG0 C inscriptibil ⇔ ∠A+∠BGC = 180◦ ⇔ ∠A = 180◦ −∠BGC ⇔
2 2 +c2 2 2 2
cos ∠A = cos(180◦ − ∠BGC) ⇔ cos ∠A = − cos ∠BGC ⇔ −a +b bc
= 5a4m−bb ·m−cc ⇔
(2a2 + 2c2 − b2 )(2a2 + 2b2 − c2 )(−a2 + b2 + c2 )2 = b2 c2 (5a2 − b2 − c2 )2 ⇔ 2a8 −
b8 − c8 + 2b4 c4 − 3a6 b2 − 3a6 c2 − a4 b4 − a4 c4 + 3a2 b6 + 3a2 c6 − 10a4 b2 c2 + 5a2 b4 c2 +
5a2 b2 c4 = 0 ⇔ (2a2 − b2 − c2 )( sym a4 b2 + 6a2 b2 c2 −
P P 6
a ) = 0. Dar 4ABC
2 2 2 4 2 4 2 6
fiind ascuţitunghic, b + c > a ⇒ a b + a c > a , şi ı̂nsumând cu relaţiile
analoage rezulţă sym a4 b2 −
P 6
a > 0, deci sym a4 b2 + 6a2 b2 c2 −
P P P 6
a > 0.
2 2 2 4 2 2 2 2
P P 6 2 2 2
Astfel, (2a − b − c )( sym a b + 6a b c − a ) = 0 ⇔ 2a − b − c = 0 ⇔
(2a2 − b2 − c2 )(a2 + b2 + c2 ) = 0 ⇔ 2b2 c2 = a2 b2 + a2 c2 + 2a4 − b4 − c4 ⇔ 9b2 c2 =
(2b2 + 2a2 − c2 )(2c2 + 2a2 − b2 ) ⇔ 3bc = 4mb · mc , ceea ce trebuia arătat.

13. a) Presupunem x = M3 , y = M3 + 1, z = M3 + 2 şi verificăm prin calcul.


b) Dacă, prin absurd, 3|x, y, z, avem P ≡ 12 − 2 + 1 ≡ 0 (mod 3), fals.

14. a) Fie AD ∩ BC = {E}. Din Menelaus, PPM D


= M C
EC
· BE
BC
BC
= 2·EC · BE
BC
BE
= 2·EC . Dar
AB
DC = 2 , DC k AB implică [DC] linie mijlocie, deci BC = CE ⇒ BE = 2 · EC,
de unde PPM
D
BE
= 2·EC BE
= BE = 1, şi astfel DP = P M .
b) Fie DD1 k BC, D1 ∈ (AB) şi O intersecţia AC cu DD1 . Atunci AD1 =
DC, AD1 k DC de unde DO = OD1 . În paralelogramul D1 BCD, [OM ] este
linie mijlocie, deci OM k D1 B = AB. Rezultă că AC şi cele două paralele sunt
concurente ı̂n O.
238

15. Aplicăm Teorema lui Menelaus ı̂n 4CDN pentru transversala QP A. Rezultă
CQ CP
DQ
= k1 · N P
. Notând cu [XY Z] aria triunghiului determinat de punctele X, Y şi
CP [DCP ] [M CP ] [DCP ]+[M CP ] [DCM ]
Z, observăm că NP
=
[DN P ]
= [M NP ]
= [DN P ]+[M N P ]
= [DN M]
. Pentru S = [ABC],
[DCM ] [DCA] 1
avem ı̂n continuare [DN M ] = DN ·[DAM ] = S · (1−k )· AM ·[DAB] = (1−k1S)·k2 ·S = k2 (1−k
1
1)
,
DA 1 AB
CQ k1
şi astfel concluzia DQ = k2 (1−k1)
.

16. a) Considerăm Q, R pentru care ABCQ, ACBR paralelograme. Evident B, G,


N coliniare şi apoi Q ∈ GN . Să arătăm că Q ∈ P E. Din reciproca Teoremei lui
AE QB P G
Menelaus ı̂n triunghiul ABG pentru punctele E, P , Q avem BE · QG · P A = 2· 32 · 13 =
1 deci E, P , Q coliniare. Astfel {Q} = GN ∩ P E, iar analog {R} = GM ∩ P F , iar
concluzia este trivial verificată.
b) Cu observaţia precedentă că Q ∈ EP , Q ∈ GN mai rămâne de confirmat
Q ∈ DF , ceea ce conduce la faptul că dreptele respective sunt concurente ı̂n Q.
CD QB
Însă BD · QN · FF N
C
= 1 · 2 · 12 = 1, şi astfel D, F , Q coliniare, aplicând aceeaşi
reciprocă ı̂n 4CBN .

17. Relaţia lui Euler OI 2 = R(R − 2r) implică R = 2r. Dar R ≥ 2r, cu egalitate
⇔ a = b = c. Deci triunghiul este echilateral, şi toate unghiurile sale au măsura
60◦ .

?

18. a) Notăm b = CD, c = BD, a = b2 + c2 = BC, p = a+b+c 2
. Observăm că P
coincide cu punctul de tangenţă la latura [BC] al cercului ı̂nscris ı̂n 4CDB, deci
BP = p−b, CP = p−c. Astfel 2·BP ·P C = 2(p−b)(p−c) = 12 (a−(b−c))(a+(b−c)) =
1 2
2
(a − (b − c)2 ) = 12 (b2 + c2 − b2 − c2 + 2bc) = bc = BD · DC.
b) Fie M , N , Q proiecţiile A pe BC, CD, respectiv DB. Atunci din ipoteză AM =
AN = AQ, deci 4AM C ≡ 4AN C, 4AN D ≡ 4AQD, 4AQB ≡ 4AM B. Deci
M C = CN = x, N D = QD = y, BQ = BM = z, iar astfel x + y = b, y + z =
c, z + x = a, deci x = b−c+a 2
= p − c = CP şi astfel x = CM = CP . Deci
M = P , iar relaţia de demonstrat devine, folosind şi Teorema Înălţimii ı̂n 4ABC
dreptunghic ı̂n ∠A, AM2·BC · 2 · AP = bc2 · BC ⇔ 2 · AM · AP = bc ⇔ 2 · AM 2 = bc ⇔
2 · BM · M C = BD · DC ⇔ BD · DC = 2 · BP · P C, adică relaţia deja demonstrată
la punctul a).
239

19. Fie tetraedrul ABCD cu AB = a, AC = b, AD = c, BC = d, CD = e, DB = f .


Presupunem că cercurile ı̂nscrise ı̂n 4ABC, 4ACD, 4ADB, notate C1 , C2 respec-
tiv C3 sunt tangente. Dacă C1,2 sunt tangente la AC ı̂n M1,2 , avem M1 = M2 şi
deci a+b−d
2
= b+c−e2
, ceea ce implică a + e = c + d. Considerând C2,3 rezultă analog
b + f = a + e. Astfel, dacă cercul ı̂nscris ı̂n 4BCD este C4 , avem C1 , C4 tangente
⇔ a−b+d
2
= d+f2 −e ⇔ a + e = b + f , adevărat, şi analoagele.

20. Din Cauchy-Schwartz,

y2
 
2
 y 2 2
(x + 3y) = x + 9 · ≤ 10 x + 9 · = 10(x2 + y 2 ).
3 9
2 2
Din ipoteză, 10(x2 + y 2 ) ≤ 20(x + 3y). Cum x + 3y ≥ x +y 2
≥ 0, deducem
2 2 2
x + 3y = 0 < 20 sau x + 3y > 0. În acest caz, (x + 3y) ≤ 10(x + y ) ≤ 20(x + 3y) ⇒
x + 3y ≤ 20. Rezultă max E = 20, cu egalitate doar pentru x = 2 şi y = 6.

21. Fie yk = xk − 1 ≥ 0 (iar xk = yk + 1), pentru k ∈ 1,p n. Din inegalitatea lui Huygens
√ √
pentru două numere (şi ponderi egale) obţinem (yi + 1)(1 + yj ) ≥ yi + yj ,
cu i, j ∈ 1, n. Avem egalitate ⇔ yi yj = 1. Însumând inegalităţile rezultate de
√ √ p √
forma xi xj ≥ xi − 1 + xj − 1 pentru i < j, fiecare termen de forma xi − 1
apare de n − 1 ori, şi ı̂mpărţind prin n − 1 deducem
! n
1 X √ X √
xi xj ≥ xi − 1.
n − 1 1≤i<j≤n i=1

În concluzie, având egalitate trebuie ca yi yj = 1, oricare ar fi i, j ∈ 1, n, i 6= j.


Alegem i = 1 şi y2 = y3 = ... = yn = y11 , apoi y1 y3 = y2 y3 implică y1 = y2 , deci
yk = 1 şi astfel xk = 2, pentru orice k ∈ 1, n. Produsul căutat este egal cu 1.

22. a) Alegem

d−1
[

A = {kd|k ∈ N }, B = {kd + m|k ∈ N}.
m=1
240

Evident A, B nevide iar (A, B) este o partiţie a lui N∗ . Punem a = da1 , b = db1 , a1 ,
b1 ∈ N∗ . Rămâne de observat că x ∈ A ⇒ x = kd ⇒ x + a = kd + da1 = (k + a1 )d ∈
A, k ∈ N∗ , iar y ∈ B ⇒ y = kd + m ⇒ y + b = kd + m + db1 = (k + b1 )d + m ∈ B,
k ∈ N, m ∈ 1, d − 1.
b) Presupunem că există z ∈ N∗ cu z ∈ A, z + d ∈ B, celălalt caz tratându-se
analog. Deoarece (a, b) = d, există x, y ∈ Z pentru care ax + by = d. Evident x, y
nu pot fi simultan strict negative.
Dacă x ≥ 0, y ≤ 0 avem z + ax ∈ A, iar z + d + b(−y) ∈ B, x, −y ≥ 0. Dar
ax + by = d ⇒ z + ax = z + d + b(−y), deci z + ax ∈ A ∩ B, contradicţie.
În cazul x ≥ 0, y > 0, alegem k = ay + 1 şi deci ak > y, a, y, k ∈ N∗ . Avem
 

z + a(bk + x) ∈ A, z + d + b(ak − y) ∈ B, bk + x, ak − y ∈ N∗ . Din nou rezultă


z + a(bk + x) ∈ A ∩ B, fals.
În final, pentru x < 0, y ≥ 0 punem
nh x i hy i o
k = max − + 1, + 1 , a, b, −x, y ∈ N, a, b 6= 0,
b a

şi de aici bk > −x, ak > y, şi astfel bk + x > 0, ak − y > 0, ceea ce implică
z+a(bk+x) ∈ A, z+d+b(ak−y) ∈ B şi din nou contradicţie deoarece z+a(bk+x) ∈
A ∩ B.

23. Avem

X ab X a+b 3
LHS = ≤ = .
a+b 2(a + b) 2

Mai rămâne de arătat


 
3 3 1 1 1 1
≤ + + + ,
2 8 8 a b c

adică
X1 X  X 1 
≥9⇔ a ≥ 9,
a a

evident din Cauchy-Schwartz. Egalul se atinge pentru a = b = c = 13 .

?
241

24. Evident a2 + 1 ≥ 2a şi similarele pentru b, c. Rezultă, utilizând inegalitatea lui


Nesbitt:
X 2a X a 3
LHS ≥ =2 ≥ 2 · = 3.
b+c b+c 2

Avem egalitate dacă şi numai dacă a = b = c = 1.

25. Evident V D2 +V E 2 +V F 2 = 3V O2 +OD2 +OE 2 +OF 2 = 3V O2 +3R2 − 14 ·(M A2 +


M B 2 +M C 2 ), considerând D, E, F chiar proiecţiile V pe M A, M B, M C respectiv.
Rămâne de văzut că M A2 + M B 2 + M C 2 = 6R2 . Notând ∠ACM = α ∈ [0◦ , 60◦ ],
relaţia se reduce, utilizând Teorema Sinusurilor, la sin2 α+sin2 (60◦ −α)+sin2 (60◦ +
α) = 32 , care se poate demonstra uşor.

26. a) Implicaţia k = 1 ⇒ Q ∈ (M N P ) este evidentă, observând, de exemplu,


că M, N, P, Q ∈ (A1 B1 C1 D1 ), unde A1 mijlocul [AA0 ], ş.a.m.d. Reciproc, pre-
−−→ −−→
supunem M, N, P, Q coplanare, deci Q ∈ (M N P ) şi ∃λ1 , λ2 cu M Q = λ1 · M N +
−−→ −→ −−→ −−→
λ2 · N P . Notând ~ı = AB, ~ = AD, ~l = AA0 şi efectuând calculele, relaţia se re-
duce la ~ı(λ1 − λ2 ) + ~(kλ1 + λ2 ) + ~l(λ2 − 1)(k − 1) = 0. Cum ~ı, ~, ~l sunt liniar
independenţi peste R, rezultă λ1 = λ2 , λ1 (k + 1) = 0 şi cum k ≥ 0, avem k 6= −1
şi astfel λ1 = λ2 = 0, de unde k − 1 = 0, k = 1, ceea ce trebuia arătat.
b) Fie O = AB 0 ∩ A0 B. Avem ∠(C 0 M, √
(ADB 0 )) ≡ ∠M C 0 O. Să presupunem

k > 1 şi AB = 1. Rezultă M B = k+1 2 k−1
, M O = √2(k+1) , OC 0 = 26 , M C 0 =
q
3
2
+ M O2 . Aplicăm Teorema Cosinusului ı̂n 4M OC 0 pentru ∠C 0 . Egalând
√ √ √
cos ∠C 0 = cos 15◦ = 6+4 2 , deducem k = 3 + 1. Celălalt caz se tratează sim-
ilar.

15.2 Clasa a VIII-a


1. Pentru orice N ∈ N∗ alegem n = 2N . Rezultă pentru a ∈ N că a + (a + 1) + ... +
.
(a + (2N − 1)) = 2N a + (2N −1)2N = 2N a + N (2N − 1) = N (2a + 2N − 1)..N . În
2
concluzie, pentru toate numerele naturale are loc proprietatea din enunţ.

?
242

2. Fie, prin absurd, 2007 = p2 + q 2 + r2 + s2 , p ≥ q ≥ r ≥ s prime. Evident printre


p, q, r, s sunt un număr impar de numere pare, deci ori s = 2, r > 2, ori q = r =
s = 2, p > 2, ultima situaţie fiind imposibilă. Deci s = 2 şi r ≥ 3. Dacă r > 3,
avem p, q, r = M3 ±1, deci p2 , q 2 , r2 = M3 +1 şi apoi 3|p2 +q 2 +r2 = 2003, fals. Deci
r = 3, iar p2 +q 2 = 1994. Cum 1994 = M3 +2, rezultă p, q = M3 ±1 şi apoi p, q ≥ 5.
Dacă q > 5, atunci p2 , q 2 = M5 + 1, M5 + 4 deci p2 + q 2 = M5 + 2, M5 + 8, M5 + 5,
adică p2 + q 2 = M5 , M5 + 2, M5 + 3. Dar p2 + q 2 = 1994 = M5 + 4, contradicţie.
Rezultă q = 5, iar apoi p2 = 1969, ceea ce reprezintă contradicţia dorită.

AB IB CB
3. a) Din Teorema Bisectoarei, AD
= ID
= CD
⇒ AB · CD = AD · BC.
AB AJ AB AD
b) Fie J intersecţia AC cu bisectoarea din B. Rezultă BC
= JC
, dar BC
= DC
, deci
AD AJ
DC
= JC , adică J aparţine şi bisectoarei din D.
c) Cum, de asemenea, d(I, AB) = d(I, AD) şi d(I, BC) = d(I, CD) rezultă I
centrul cercului ı̂nscris ı̂n ABCD, deci ABCD circumscriptibil ⇒ AB + CD =
BC + AD = BC + AB·CD BC
⇔ (BC − AB)(BC − CD) = 0. Dacă BC = CD,
[CI] fiind bisectoare ı̂n 4CDB isoscel ı̂n ∠C, CI ⊥ BD iar BI = ID, deci ı̂n
4ADB avem [AI] bisectoare şi mediană ⇒ AI ⊥ BD. Deci A, I, C coliniare
⇒ AC = CI ⊥ BD, ceea ce trebuia arătat. Dacă BC = AB, AB 2 = BC 2 ⇒
2 2
(AB 2 − AD2 )(AB 2 − BC 2 ) = 0 ⇔ AD2 + BC 2 = AB 2 + ADAB·BC
2 ⇔ AD2 + BC 2 =
AB 2 + CD2 , de unde concluzia.

4. a) Avem AB = AH = sinAE ∠C
= ABsincos
∠C
∠A
⇒ cos ∠A = sin ∠C. De aici cos ∠A > 0, iar
cos ∠A + cos ∠C = 1 ⇔ 60a b + 64a + 9b6 = 144a4 b2 , din Teorema Cosinusului.
2 2 2 4 6

Folosind b = 2c, obcţinem AC = 4OM ⇔ b = 4·R·cos ∠A ⇔ b = 2· sinb∠B ·cos ∠A ⇔


sin ∠B = 2 cos ∠A ⇔ b sinc ∠C = 2 cos ∠A ⇔ 2c sinc ∠C = 2 cos ∠A ⇔ sin ∠C = cos ∠A,
relaţie deja stabilită.
b) Avem AH = 2R cos ∠A, OM = R cos ∠A şi BM = a2 . Inegalitatea AH +
BM > OM ⇔ R cos ∠A + a2 > 0 este evidentă. Acum OM + BM > AH ⇔
a > 2R cos ∠A ⇔ a > sinc∠C · cos ∠A, iar cum cos ∠A = sin ∠C, trebuie arătat
a > c ⇔ 2a > b. Dacă, prin reducere la absurd, 2a ≤ b, rezultă b4 ≥ 16a4 , deci
144a4 b2 = 60a2 b4 + 64a6 + 9b2 · b4 ≥ 60a2 b4 + 64a6 + 9b2 · 16a4 = 60a2 b4 + 64a6 +
144a4 b2 ⇒ 0 ≥ 60a2 b4 + 64a6 , evident fals. Mai rămâne AH + OM > BM ⇔
6R cos ∠A > a ⇔ 6R sin ∠C > a ⇔ 3 · sinc∠C · sin ∠C > a ⇔ 3c > a ⇔ 3b > 2a. Din
nou, dacă prin absurd 3b ≤ 2a, obţinem 4a2 ≥ 9b2 , iar 144a4 b2 = 9b6 + 60a2 b4 +
243

16a4 · 4a2 ≥ 9b6 + 60a2 b4 + 16a4 · 9b2 = 9b6 + 60a2 b4 + 144a4 b2 ⇒ 0 ≥ 9b6 + 60a2 b4 ,
contradicţie.

5. Fie E, F punctele unde se ı̂ndoaie laturile [AD], respectiv [BC], iar α = m(∠
AEM ), x = AE. Presupunem fără pierdere de generalitate că AB = 1. Dacă
α ∈ {0◦ , 90◦ }, suprafaţa ı̂ndoită este jumătate din aria pătratului, deci concluzia
se verifică. În caz contrar, figura este nedegenerată şi fie M C ∩ BF = {N }. Avem
AE + EM = 1 ⇒ x + cosx α = 1 ⇒ x = coscosα+1 α
. Rezultă M E = cos 1α+1 , AM = x tg
α = cossinα+1
α
, apoi M B = 1−AM = cos cos α−sin α+1
α+1
, M N = cos α−sin α+1
cos2 α+cos α
, N C = 1−M C =
2
sin α−sin α
cos2 α+cos α
şi F C = cos α+1 . Avem [M N F E] > 4 ⇔ [M CF E] − [N CF ] > 14 ⇔ M C ·
1−sin α 1
M E+F C
> 14 + N C·CF ⇔ 2 + 2 sin2 α + 3 cos α > cos3 α + 4 sin α + 2 sin α cos α. Punem
2 2 √
cos α = x ∈ (0, 1), iar sin α = 1 − x2 . Inegalitatea se reduce la −x3 −2x2 +3x+4 >

2(x + 2) 1 − x2 ⇔ (−x3 − 2x2 + 3x + 4)2 > 2(x + 2)(1 − x2 ) ⇔ x(x5 + 4x4 + 2x3 −
4x2 +5x+8) > 0. Dar x > 0, deci trebuie arătat (x5 +4x4 +2x3 +5x)+(8−4x2 ) > 0.
Cum evident x5 + 4x4 + 2x3 + 5x > 0, iar x2 < 1 ⇒ −4x2 > −4 ⇒ 8 − 4x2 > 4 > 0,
concluzia urmează cu uşurinţă.

6. Notând n = 22007 + 1, cerinţa se reduce la a arăta că

1 1 1
S =1+ + + ... + ∈/ N.
2 3 n

Fie L = [1, 2, ..., n]. Evident 22007 ||L. Punând


n
X L
A= ,
k=1
k

avem S = A L
. Arătăm că A este impar. Atunci, cum 2|L, rezultă L - A şi deci
A
S= L ∈ / N. Observăm ı̂nsă că Lk este par pentru k ∈ 1, n − 2 ∪ {n} şi impar pentru
k = n − 1 = 22007 , iar concluzia rezultă cu uşurinţă.
Remarcă. Demonstraţia se poate extinde cu uşurinţă pentru a demonstra că ∀n ∈
N, n ≥ 2,

1 1 1
Sn = 1 + + + ... + ∈/ N.
2 3 n

?
244

7. Vom folosi următoarea


Lemă. Dacă x, y ∈ N, iar 7|x2 + y 2 , avem 7|x şi 7|y.
Demonstraţie. Un pătrat perfect este congruent doar cu 0, 1, 2 sau 4 modulo 7.
După cum se poate vedea din tabelul de mai jos, suma x2 + y 2 de două pătrate
perfecte este 0 modulo 7 doar când pătratele sunt 0 modulo 7, ceea ce ı̂nseamnă
7|x şi 7|y. 

+ x2 0 1 2 4
y2
0 0 1 2 4
1 1 2 3 5
2 2 3 4 6
4 4 5 6 1

Ecuaţia se scrie 74 ·13 = (2x)2 +(3y)2 . Din Lemă deducem 7|2x, 7|3y şi deci 7|x, 7|y,
iar x = 7x1 , y = 7y1 , x1 , y1 ∈ N. Astfel 72 ·13 = (2x1 )2 +(3y1 )2 . Aplicănd ı̂ncă o dată
Lema obţinem x1 = 7x2 , y1 = 7y2 , x2 , y2 ∈ N. Deci 4x22 + 9y22 = 13, cu unica soluţie
(x2 , y2 ) = (1, 1) şi astfel (x, y) = (49, 49) este unica pereche verificând enunţul.

8. Se arată că dacă a1 , a2 , a3 sunt primele trei zecimale (ı̂n ordine) ale lui A, atunci

  (p )
[1000 · A] [ (125(32n + 1))2 + 1252 · 128447]
0, a1 a2 a3 = = .
1000 1000

Dar n ≥ 250874 > 250873, 0155 ⇒ 1252 · 128447 < 250(32n + 1) + 1 ⇒ (125(32n +
1))2 ≤ (125(32n + 1))2 + 1252 · 128447 < (125(32n + 1) + 1)2 ⇒ 125(32n + 1) ≤
1
((125(32n + 1))2 + 1252 · 128447) 2 < 125(32n + 1) + 1 ⇒ [((125(32n + 1))2 + 1252 ·
1
128447) 2 ] = 125(32n + 1), deci
   
125(32n + 1) 1
0, a1 a2 a3 = = 4n + = 0, 125.
1000 8

În concluzie, primele trei zecimale sunt ı̂ntotdeauna 1, 2, respectiv 5.

?
245

9. Prin calcul, 9A2 = (2 · 102008 + 8)2 − 108. Se verifică (2 · 102008 + 7)2 < 9A2 <
(2 · 102008 + 8)2 . Însă 9A2 ∈ N, 9A2 6= k 2 , k ∈ N implică 3A ∈
/ Q, deci A ∈
/ Q, de
unde A ∈ / N, adică A nu este natural.

10. Fie AB = a, iar ∠A = 120◦ şi ∠B = ∠C = 30◦ . Din Teorema Sinusurilor, BC =


√ √
a 3. De asemenea, AD = AB · tan 30◦ = √a3 . Rezultă BC · AD = a 3 · √a3 = a2 =
AB 2 , ceea ce trebuia arătat.

11. a) Presupunând contrariul, din 3a2 + 223b2 = 223 · 9 · 10001 rezultă 223|3a2 , iar 223
fiind prim obţinem a = 223a1 , a1 ∈ N şi deci 3 · 223a21 + b2 = 32 · 10001. De aici 3|b,
b = 3b1 , b1 ∈ N şi 223a21 + 3b21 = 3 · 10001. Rezultă 3|223a21 , apoi a1 = 3a2 , a2 ∈ N iar
3 · 223a22 + b21 = 10001. Luând egalitatea modulo 3, avem b21 ≡ 10001 ≡ 2 (mod 3),
o contradicţie deoarece pătratele perfecte sunt 0 sau 1 (mod 3).
b) La fel ca la punctul precedent deducem 223 · 3 · a22 + b21 = k 2 , unde a = 223a1 ,
a1 = 3a2 , b = 3b1 , a1 , a2 , b1 ∈ N∗ . Dacă a2 6= 1, ∃p prim cu p|a2 ⇒ a2 = kp, k ∈ N∗ ⇒
a = 223 · 3 · p · k, deci 1, 3, 223, 3 · 223|a, iar cum a are exact 4 divizori obligatoriu
p ∈ {3, 223}, dar atunci fie 32 |a, fie 2232 |a iar a are şi ı̂n acest caz mai mult de 4
divizori, fals. Rezultă a2 = 1, şi deci 669 = (k − b1 )(k + b1 ), k + b1 > 0 şi apoi
k − b1 > 0, deci (k − b1 , k + b1 ) ∈ {(1, 669); (3, 223); (223, 3); (669, 1)}. Convin doar
(k − b1 , k + b1 ) ∈ {(1, 669); (3, 223)} ⇔ (k, b1 ) ∈ {(335, 334); (113, 110)}. Obţinem
mulţimea soluţiilor (a, b, k) ∈ S = {(669, 330, 113); (669, 1002, 335)}.

12. a) Avem ∠H 0 BC = ∠HBC = 90◦ − ∠C = ∠H 0 AC, deci H 0 ABC inscriptibil


⇒ H 0 ∈ ω.
b) O0 ∈ ω ⇔ ABO0 C inscriptibil ⇔ ∠A + ∠BO0 C = 180◦ ⇔ ∠A + ∠BOC =
180◦ ⇔ ∠A + 2 · ∠A = 180◦ ⇔ ∠A = 60◦ .
c) I 0 ∈ ω ⇔ ABI 0 C inscriptibil ⇔ ∠A + ∠BI 0 C = 180◦ ⇔ ∠A + ∠BIC = 180◦ ⇔
∠A + ∠A + ∠B+∠C2
= 180◦ ⇔ 90◦ + 3∠A2
= 180◦ ⇔ ∠A = 60◦ .
d) G0 ∈ ω ⇔ ABG0 C inscriptibil ⇔ ∠A+∠BGC = 180◦ ⇔ ∠A = 180◦ −∠BGC ⇔
2 2 +c2 2 2 2
cos ∠A = cos(180◦ − ∠BGC) ⇔ cos ∠A = − cos ∠BGC ⇔ −a +b bc
= 5a4m−bb ·m−cc ⇔
(2a2 + 2c2 − b2 )(2a2 + 2b2 − c2 )(−a2 + b2 + c2 )2 = b2 c2 (5a2 − b2 − c2 )2 ⇔ 2a8 −
b8 − c8 + 2b4 c4 − 3a6 b2 − 3a6 c2 − a4 b4 − a4 c4 + 3a2 b6 + 3a2 c6 − 10a4 b2 c2 + 5a2 b4 c2 +
246

5a2 b2 c4 = 0 ⇔ (2a2 − b2 − c2 )( sym a4 b2 + 6a2 b2 c2 −


P P 6
a ) = 0. Dar 4ABC
2 2 2 4 2 4 2 6
fiind ascuţitunghic, b + c > a ⇒ a b + a c > a , şi ı̂nsumând cu relaţiile
analoage rezulţă sym a4 b2 −
P 6
a > 0, deci sym a4 b2 + 6a2 b2 c2 −
P P P 6
a > 0.
2 2 2 4 2 2 2 2
P P 6 2 2 2
Astfel, (2a − b − c )( sym a b + 6a b c − a ) = 0 ⇔ 2a − b − c = 0 ⇔
(2a2 − b2 − c2 )(a2 + b2 + c2 ) = 0 ⇔ 2b2 c2 = a2 b2 + a2 c2 + 2a4 − b4 − c4 ⇔ 9b2 c2 =
(2b2 + 2a2 − c2 )(2c2 + 2a2 − b2 ) ⇔ 3bc = 4mb · mc , ceea ce trebuia arătat.

13. a) Presupunem x = M3 , y = M3 + 1, z = M3 + 2 şi verificăm prin calcul.


b) Dacă, prin absurd, 3|x, y, z, avem P ≡ 12 − 2 + 1 ≡ 0 (mod 3), fals.

14. a) Fie AD ∩ BC = {E}. Din Menelaus, PPM D


= M C
EC
· BE
BC
BC
= 2·EC · BE
BC
BE
= 2·EC . Dar
AB
DC = 2 , DC k AB implică [DC] linie mijlocie, deci BC = CE ⇒ BE = 2 · EC,
de unde PPM
D
BE
= 2·EC BE
= BE = 1, şi astfel DP = P M .
b) Fie DD1 k BC, D1 ∈ (AB) şi O intersecţia AC cu DD1 . Atunci AD1 =
DC, AD1 k DC de unde DO = OD1 . În paralelogramul D1 BCD, [OM ] este
linie mijlocie, deci OM k D1 B = AB. Rezultă că AC şi cele două paralele sunt
concurente ı̂n O.

15. Aplicăm Teorema lui Menelaus ı̂n 4CDN pentru transversala QP A. Rezultă
CQ CP
DQ
= k1 · N P
. Notând cu [XY Z] aria triunghiului determinat de punctele X, Y şi
CP [DCP ] [M CP ] [DCP ]+[M CP ] [DCM ]
Z, observăm că NP
=
[DN P ]
= [M NP ]
= [DN P ]+[M N P ]
= [DN M]
. Pentru S = [ABC],
[DCM ] [DCA] 1
avem ı̂n continuare [DN M ] = DN ·[DAM ] = S · (1−k )· AM 1
= (1−k1S)·k2 ·S = k2 (1−k 1)
,
DA 1 AB ·[DAB]
CQ k1
şi astfel concluzia DQ = k2 (1−k1)
.

16. a) Considerăm Q, R pentru care ABCQ, ACBR paralelograme. Evident B, G,


N coliniare şi apoi Q ∈ GN . Să arătăm că Q ∈ P E. Din reciproca Teoremei lui
AE QB P G
Menelaus ı̂n triunghiul ABG pentru punctele E, P , Q avem BE · QG · P A = 2· 32 · 13 =
1 deci E, P , Q coliniare. Astfel {Q} = GN ∩ P E, iar analog {R} = GM ∩ P F , iar
concluzia este trivial verificată.
b) Cu observaţia precedentă că Q ∈ EP , Q ∈ GN mai rămâne de confirmat
Q ∈ DF , ceea ce conduce la faptul că dreptele respective sunt concurente ı̂n Q.
CD QB
Însă BD · QN · FF N
C
= 1 · 2 · 12 = 1, şi astfel D, F , Q coliniare, aplicând aceeaşi
reciprocă ı̂n 4CBN .
247

17. Relaţia lui Euler OI 2 = R(R − 2r) implică R = 2r. Dar R ≥ 2r, cu egalitate
⇔ a = b = c. Deci triunghiul este echilateral, şi toate unghiurile sale au măsura
60◦ .

?

18. a) Notăm b = CD, c = BD, a = b2 + c2 = BC, p = a+b+c 2
. Observăm că P
coincide cu punctul de tangenţă la latura [BC] al cercului ı̂nscris ı̂n 4CDB, deci
BP = p−b, CP = p−c. Astfel 2·BP ·P C = 2(p−b)(p−c) = 12 (a−(b−c))(a+(b−c)) =
1 2
2
(a − (b − c)2 ) = 12 (b2 + c2 − b2 − c2 + 2bc) = bc = BD · DC.
b) Fie M , N , Q proiecţiile A pe BC, CD, respectiv DB. Atunci din ipoteză AM =
AN = AQ, deci 4AM C ≡ 4AN C, 4AN D ≡ 4AQD, 4AQB ≡ 4AM B. Deci
M C = CN = x, N D = QD = y, BQ = BM = z, iar astfel x + y = b, y + z =
c, z + x = a, deci x = b−c+a 2
= p − c = CP şi astfel x = CM = CP . Deci
M = P , iar relaţia de demonstrat devine, folosind şi Teorema Înălţimii ı̂n 4ABC
dreptunghic ı̂n ∠A, AM2·BC · 2 · AP = bc2 · BC ⇔ 2 · AM · AP = bc ⇔ 2 · AM 2 = bc ⇔
2 · BM · M C = BD · DC ⇔ BD · DC = 2 · BP · P C, adică relaţia deja demonstrată
la punctul a).

19. Fie tetraedrul ABCD cu AB = a, AC = b, AD = c, BC = d, CD = e, DB = f .


Presupunem că cercurile ı̂nscrise ı̂n 4ABC, 4ACD, 4ADB, notate C1 , C2 respec-
tiv C3 sunt tangente. Dacă C1,2 sunt tangente la AC ı̂n M1,2 , avem M1 = M2 şi
deci a+b−d
2
= b+c−e2
, ceea ce implică a + e = c + d. Considerând C2,3 rezultă analog
b + f = a + e. Astfel, dacă cercul ı̂nscris ı̂n 4BCD este C4 , avem C1 , C4 tangente
⇔ a−b+d
2
= d+f2 −e ⇔ a + e = b + f , adevărat, şi analoagele.

20. Din Cauchy-Schwartz,

y2
 
2
 y 2 2
(x + 3y) = x + 9 · ≤ 10 x + 9 · = 10(x2 + y 2 ).
3 9
2 2
Din ipoteză, 10(x2 + y 2 ) ≤ 20(x + 3y). Cum x + 3y ≥ x +y 2
≥ 0, deducem
2 2 2
x + 3y = 0 < 20 sau x + 3y > 0. În acest caz, (x + 3y) ≤ 10(x + y ) ≤ 20(x + 3y) ⇒
x + 3y ≤ 20. Rezultă max E = 20, cu egalitate doar pentru x = 2 şi y = 6.
248

21. Fie yk = xk − 1 ≥ 0 (iar xk = yk + 1), pentru k ∈ 1,p n. Din inegalitatea lui Huygens
√ √
pentru două numere (şi ponderi egale) obţinem (yi + 1)(1 + yj ) ≥ yi + yj ,
cu i, j ∈ 1, n. Avem egalitate ⇔ yi yj = 1. Însumând inegalităţile rezultate de
√ √ p √
forma xi xj ≥ xi − 1 + xj − 1 pentru i < j, fiecare termen de forma xi − 1
apare de n − 1 ori, şi ı̂mpărţind prin n − 1 deducem
! n
1 X √ X √
xi xj ≥ xi − 1.
n−1 1≤i<j≤n i=1

În concluzie, având egalitate trebuie ca yi yj = 1, oricare ar fi i, j ∈ 1, n, i 6= j.


Alegem i = 1 şi y2 = y3 = ... = yn = y11 , apoi y1 y3 = y2 y3 implică y1 = y2 , deci
yk = 1 şi astfel xk = 2, pentru orice k ∈ 1, n. Produsul căutat este egal cu 1.

22. a) Alegem

d−1
[

A = {kd|k ∈ N }, B = {kd + m|k ∈ N}.
m=1

Evident A, B nevide iar (A, B) este o partiţie a lui N∗ . Punem a = da1 , b = db1 , a1 ,
b1 ∈ N∗ . Rămâne de observat că x ∈ A ⇒ x = kd ⇒ x + a = kd + da1 = (k + a1 )d ∈
A, k ∈ N∗ , iar y ∈ B ⇒ y = kd + m ⇒ y + b = kd + m + db1 = (k + b1 )d + m ∈ B,
k ∈ N, m ∈ 1, d − 1.
b) Presupunem că există z ∈ N∗ cu z ∈ A, z + d ∈ B, celălalt caz tratându-se
analog. Deoarece (a, b) = d, există x, y ∈ Z pentru care ax + by = d. Evident x, y
nu pot fi simultan strict negative.
Dacă x ≥ 0, y ≤ 0 avem z + ax ∈ A, iar z + d + b(−y) ∈ B, x, −y ≥ 0. Dar
ax + by = d ⇒ z + ax = z + d + b(−y), deci z + ax ∈ A ∩ B, contradicţie.
În cazul x ≥ 0, y > 0, alegem k = ay + 1 şi deci ak > y, a, y, k ∈ N∗ . Avem
 

z + a(bk + x) ∈ A, z + d + b(ak − y) ∈ B, bk + x, ak − y ∈ N∗ . Din nou rezultă


z + a(bk + x) ∈ A ∩ B, fals.
În final, pentru x < 0, y ≥ 0 punem
nh x i hy i o
k = max − + 1, + 1 , a, b, −x, y ∈ N, a, b 6= 0,
b a
249

şi de aici bk > −x, ak > y, şi astfel bk + x > 0, ak − y > 0, ceea ce implică
z+a(bk+x) ∈ A, z+d+b(ak−y) ∈ B şi din nou contradicţie deoarece z+a(bk+x) ∈
A ∩ B.

23. Avem

X ab X a+b 3
LHS = ≤ = .
a+b 2(a + b) 2

Mai rămâne de arătat


 
3 3 1 1 1 1
≤ + + + ,
2 8 8 a b c

adică
X1 X  X 1 
≥9⇔ a ≥ 9,
a a

evident din Cauchy-Schwartz. Egalul se atinge pentru a = b = c = 13 .

24. Evident a2 + 1 ≥ 2a şi similarele pentru b, c. Rezultă, utilizând inegalitatea lui


Nesbitt:
X 2a X a 3
LHS ≥ =2 ≥ 2 · = 3.
b+c b+c 2

Avem egalitate dacă şi numai dacă a = b = c = 1.

25. Evident V D2 +V E 2 +V F 2 = 3V O2 +OD2 +OE 2 +OF 2 = 3V O2 +3R2 − 14 ·(M A2 +


M B 2 +M C 2 ), considerând D, E, F chiar proiecţiile V pe M A, M B, M C respectiv.
Rămâne de văzut că M A2 + M B 2 + M C 2 = 6R2 . Notând ∠ACM = α ∈ [0◦ , 60◦ ],
relaţia se reduce, utilizând Teorema Sinusurilor, la sin2 α+sin2 (60◦ −α)+sin2 (60◦ +
α) = 32 , care se poate demonstra uşor.

?
250

26. a) Implicaţia k = 1 ⇒ Q ∈ (M N P ) este evidentă, observând, de exemplu,


că M, N, P, Q ∈ (A1 B1 C1 D1 ), unde A1 mijlocul [AA0 ], ş.a.m.d. Reciproc, pre-
−−→ −−→
supunem M, N, P, Q coplanare, deci Q ∈ (M N P ) şi ∃λ1 , λ2 cu M Q = λ1 · M N +
−−→ −→ −−→ −−→
λ2 · N P . Notând ~ı = AB, ~ = AD, ~l = AA0 şi efectuând calculele, relaţia se re-
duce la ~ı(λ1 − λ2 ) + ~(kλ1 + λ2 ) + ~l(λ2 − 1)(k − 1) = 0. Cum ~ı, ~, ~l sunt liniar
independenţi peste R, rezultă λ1 = λ2 , λ1 (k + 1) = 0 şi cum k ≥ 0, avem k 6= −1
şi astfel λ1 = λ2 = 0, de unde k − 1 = 0, k = 1, ceea ce trebuia arătat.
b) Fie O = AB 0 ∩ A0 B. Avem ∠(C 0 M, √
(ADB 0 )) ≡ ∠M C 0 O. Să presupunem

2
k > 1 şi AB = 1. Rezultă M B = k+1 , M O = √2(k+1) k−1
, OC 0 = 26 , M C 0 =
q
3
2
+ M O2 . Aplicăm Teorema Cosinusului ı̂n 4M OC 0 pentru ∠C 0 . Egalând
√ √ √
cos ∠C 0 = cos 15◦ = 6+4 2 , deducem k = 3 + 1. Celălalt caz se tratează sim-
ilar.

15.3 Clasa a IX-a

15.4 Clasa a X-a


1. Avem
h 2i h 2i h 2i h 2i
x n x2 n2
x n
+ nx = nx + nx + nx ≥ 2 + x
· nx ,
  
n
+ x
= n
+ x n
· x
≥2+ n

scris in forma echivalenta,


h 2 i  h 2 i 
x n
n
−1 x
− 1 ≤ −1,

h 2i h 2i h 2i
x n n
deci una dintre n
si x
este nula. Fie x
= 0. Atunci, rezulta ca x =
n2 + k, k ∈ N ∗ . Rescriind egalitatea initiala cu aceasta notatie, avem

h i h i h i
n4 +2n2 k+k2 n2 n2 +k n
n
+ n2 +k
= n
+ n2 +k
+ n3 + nk <=>
=0
h 2i
k
= n3 + nk + n + nk + n2n+k <=>
 
n3 + 2nk + n
h 2i 
n (k − 1) + kn = nk + n2n+k .


Dar
251
k n
 k
n
+ n2 +k
< n
+ 1,

k
si pentru ca n (k − 1) < n
+ 1, trebuie sa avem k = 1. Punand x = n2 + 1, ecuatia
se verifica:
1 1 n

n3 + 2n + n
= n3 + 2n + n
+ n2 +1
.
h 2i
x
Daca n
= 0, cum egalitatea este simetrica in x, n, vom ajunge la n = x2 + 1.

In concluzie, solutiile x ale ecuatiei sunt n2 + 1 si n − 1(cand n − 1 este patrat
perfect).

2. In primul rand, sa observam ca daca (x, y, z) este solutie, atunci si (−x, −y, −z)
este. Daca unul dintre x, y, z este 1, atunci toate sunt 1. Acum, daca unul dintre
x, y, z este negativ, atunci toate sunt. Asadar putem considera doar cazul x, y, z >
0, x, y, z diferite de 1. Fie x > 1(vom trata ulterior cazul x, y, z < 1). Atunci

x(x2 +3)
z= 3x2 +1
. (*)

Avem

(x − 1)3 > 0 <=> x3 − 3x2 + 3x − 1 > 0 <=> x (x2 + 3) > 3x2 + 1 <=> z > 1

x2 +3
si analog vom obtine si y > 1. De asemenea, din x > 1 rezulta 3x2 +1
< 1, deci
z < x si continuand analog se obtine contradictia z < x < y < z.
In cazul x, y, z < 1, similar se va obtine z > x > y > z.
Asadar (x, y, z) ∈ {(1, 1, 1) ; (−1, −1, −1)}.

3. Fie q un numar prim impar ≤ p−1 2


. Daca q nu ar divide p − 1 atunci 1 + k (p − 1)
ar parcurge toate resturile modq pentru k = 1, q(Deoarece daca ar exista k1 si k2
sa dea acelasi rest, atunci am avea ca q| (p − 1) (k1 − k2 ) Fals intrucat k1 − k2 < q).
Deci orice prim impar ≤ p−1 2
trebuie sa divida p−1(de fapt va divide p−1 2
deoarece
 p−1   p−1 
sunt impare). Pentru p ≥ 11, intre 4 si 2 4 exista un numar prim(Bertrand)
si in mod cert acesta nu va divide p−1 2
. Acum mai ramane sa analizam cazurile
p ∈ {3, 5, 7}
p = 3: avem de verificat doar daca 1 + 1(3 − 1) = 3 este prim.
p = 5: nu este solutie deoarece pentru k = 2 avem 1 + 2(5 − 1) = 9 neprim.
p = 7: avem de verificat ca 7, 13, 19 sunt prime.
Asadar numerele prime p cautate sunt 3 si 7.
252

4. Daca z1 , z2 , z3 sunt varfurile unui triunghi echilateral, cum z1 , z2 , z3 au acelasi


modul, centrul sau este in origine si din simetrie rezulta

|kz1 + z2 + z3 | = |z1 + kz2 + z3 | = |z1 + z2 + kz3 | ,

pentru orice k real. Acum, daca avem

|kz1 + z2 + z3 | = |z1 + kz2 + z3 | = |z1 + z2 + kz3 | <=>


|kz1 + z2 + z3 |2 = |z1 + kz2 + z3 |2 = |z1 + z2 + kz3 |2 <=>
(kz1 + z2 + z3 ) (kz1 + z2 + z3 ) = (z1 + kz2 + z3 ) (z1 + kz2 + z3 ) =
(z1 + z2 + kz3 ) (z1 + z2 + kz3 ) <=>
 
(S + (k − 1) z1 ) S + (k − 1) z1 = (S + (k − 1) z2 ) S + (k − 1) z2 =

(S + (k − 1) z3 ) S + (k − 1) z3 <=>
|S|2 + (k − 1) |z1 |2 + (k − 1) z1 S + z1 S = ... =


|S|2 + (k − 1) |z3 |2 + (k − 1) z3 S + z3 S <=>



  
(k − 1) z1 S + z1 S = (k − 1) z2 S + z2 S = (k − 1) z3 S + z3 S .

Cum k ∈ R\ {1} , putem elimina (k − 1) si deci avem

z1 S + z1 S = z2 S + z2 S = z3 S + z3 S <=>
z1 (z1 + z2 + z3 ) + z1 (z1 + z2 + z3 ) = ... = z3 (z1 + z2 + z3 ) + z3 (z1 + z2 + z3 ) <=>
2 |z1 |2 + z1 z2 + z1 z2 + z1 z3 + z1 z3 = ... = 2 |z3 |2 + z3 z2 + z3 z2 + z1 z3 + z1 z3 <=>
z1 z2 + z1 z2 + z1 z3 + z1 z3 = ... = z3 z2 + z3 z2 + z1 z3 + z1 z3 <=>
z3 z2 + z3 z2 = z1 z3 + z1 z3 = z1 z2 + z1 z2 <=>
|z2 |2 + |z3 |2 + z3 z2 + z3 z2 = ... = |z2 |2 + |z1 |2 + z1 z2 + z1 z2 <=>
(z2 + z3 ) (z2 + z3 ) = (z3 + z1 ) (z3 + z1 ) = (z2 + z1 ) (z2 + z1 ) <=>
|z2 + z3 |2 = |z3 + z1 |2 = |z1 + z2 |2 .

Dar acum, uitandu-ne in configuratia geometrica, am avea |z2 + z3 | = 2R cos A si


am ajuns la

cos2 A = cos2 B = cos2 C,

deci triunghiul ABC (de afixe z1 , z2 , z3 ) este echilateral.


253

5. Fie x = log3 2008


2007
2008
. Evident, x ∈ (0, 1). Alegem y = 1 − log3 2007 . Atunci, x, y > 0 si
x
log2 (x + y) = log2 1 = 0 ∈ M si deci 3 ∈ M . Acum mai ramane doar observatia
2008
ca 3log3 2007 = 2008
2007
si demonstratia se incheie.

6. Notand tan x = t, avem


1 1
2t + 2 t = t
− t.
1
Deriand f (a) = 2a + 2 a − a a1 + a = 0, obtinem ca f crescatoare, deci solutia este
unica. Se observa ca se afla in (−∞, −1), cu a → −1. 602 3
abc S
7. Inlocuim cu formulele R = şi r = si eliminam numitorii in inegalitatea de
4S p
demonstrat, care devine
3
abcp + S 2 + 9S 4 p4 ≥ 4S 2 p4 abcp + S 2 .


Inlocuind pe S din formula lui Heron, prelucram expresia abcp + S 2 care se trans-
formă succesiv in

p [abc + (p − a) (p − b) (p − c)] = p abc + p3 − p2 Σa + pΣab − abc




= p p3 − 2p3 + pΣab


= p2 Σab − p2


si astfel inegalitatea de demonstrat ia forma


3
p6 Σab − p2 + 9p6 (p − a)2 (p − b)2 (p − c)2 ≥ 4S 2 p6 Σab − p2 ,


care, dupa ce impartim prin p6 si folosim notaţiile p − a = x, p − b = y, p − c = z,


de unde p = x + y + z, a = y + z, b = z + x, c = x + y, Σab − p2 = xy + yz + zx,
devine

(xy + yz + zx)3 + 9x2 y 2 z 2 ≥ 4xyz (x + y + z) (xy + yz + zx) .

Această inegalitate o forma echivalenta Schur(pentru xy = A > 0, yz = B >


0, zx = C > 0):

(A + B + C)3 + 9ABC ≥ 4 (AB + BC + CA) (A + B + C) ,

ceea ce este usor de verificat deoarece

(A + B + C)3 = ΣA3 + 3 (A + B) (B + C) (C + A) ,
ΣAB (A + B) = (A + B) (B + C) (C + A) − 2ABC
= (AB + BC + CA) (A + B + C) − 3ABC.
254

8. Notam a = z −1. Avem |z| ≤ 1; |z 2 − 3z + 3| ≤ 1, deci |a + 1| ≤ 1; |a2 − a + 1| ≤ 1.


Acum, fie a = x + iy. Trebuie sa avem simultan

(x + 1)2 + y 2 ≤ 1 <=> x2 + 2x + y 2 ≤ 0;(1)


2
(x2 − y 2 − x + 1) + (2xy − y)2 ≤ 1 <=>
2
(x2 − y 2 − x) + 2x2 − 2y 2 − 2x + (2xy − y)2 ≤ 0 <=>
2
<=> (x2 − y 2 − x) + 2x2 − 2y 2 − 2x + 4x2 y 2 − 4xy 2 + y 2 ≤ 0 <=>
2
<=> (x2 − y 2 − x) + 2x2 − 2y 2 − 2x + 4x2 y 2 − 4xy 2 + y 2 ≤ 0.(2)

Adunand (1) si (2), avem

2
(x2 − y 2 − x) + 3x2 + 4x2 y 2 − 4xy 2 ≤ 0.(3)

Dar acum, din (1) trebuie ca x ≤ 0 si din (3) x ≥ 0, deci x = 0 si revenind la (1)
rezulta si y = 0. Asadar a = 0, deci z = 1.

9. Vom merge cu echivalenta: Sa presupunem ca exista θ astfel ca (X − z1 ) (X − z2 ) (X − z3 )−


θ sa nu aiba nici o radacina simpla. Atunci el trebuie sa aiba o radacina tripla, ad-
ica

(X − z1 ) (X − z2 ) (X − z3 ) − θ = (X − k)3 <=>
X 3 − ( zi ) X 2 + ( zi zj ) X − z1 z2 z3 − θ = X 3 − 3X 2 k + 3Xk 2 − k 3 .
P P

zi zj = 3k 2 ,
P P
Din aceasta egalitatea de polinoame trebuie sa avem zi = 3k si
deci

( zi )2 = (3k)2 = 3 (3k 2 ) = 3 ( zi zj ) <=>


P P P 2 P
zi = zi zj ,

deci z1 , z2 , z3 sunt varfurile unui triunghi echilateral. Reciproc, construim θ =


 P 3
3 zi
k = 3
.
Capitolul 16

Probleme avute ı̂n atenţia comisiei, 2008

16.1 Clasa a VII-a

16.2 Clasa a VIII-a

16.3 Clasa a IX-a


1. Ii dam lui x valoarea 0. Din aceasta obtinem ca:
f (f (y)) = y. Sa notam f (1) = a si sa inlocuim x = 1 si y = a.
Acum, revenind in relatia initiala, obtinem:
f (1 + f (f (1))) = f (1) + f (1) ⇔ f (2) = 2a.
Acum vom demonstra prin inductie ca f (n) = na. Verificarea a fost facuta mai
sus, acum sa presupunem adevarat f (n) = na si sa demonstram f (n + 1) = (n +
1)a. Inlocuim pe y cu f (n) si pe x cu 1. Obtinem ca: f (n +1) = f (n)+ a = (n +1)a.
Inlocuind in relatia initiala, obtinem:
x2 a + ya2 = x2 a + y ⇔ a2 = 1 ⇒ a = 1, deoarece a este numar natural, deci si
pozitiv.

2. a) Sa facem urmatoarea observatie: f (x) = x +1, ∀x ∈ [a, a+1) si f (x) = x − 1, x ∈


[b − 1, b). Acum, evident ne rezulta ca f (x) = x + 1, ∀x ∈ [a + 2, a + 3) si in mod
analog, deducem ca f (x) = x + 1, ∀x ∈ [a + 2k, a + 2k + 1), k ∈ N, a.i.a + 2k + 1 < b.
Deci, pentru ca functia sa fie injectiva trebuie ca [b−2, b−1)∩[a+2k−1, a+2k) = .
Sa presupunem ca a + 2k < b − 2, a + 2k + 2 > b − 2. daca translatam segmentul
[a + 2k − 1, a + 2k) cu un vector de lungime doi pe directia si in sensul axei reale,
obtinem segmentul [a + 2k + 1, a + 2k + 2), care se intersecteaza cu segmentul
[b − 2, b − 1). Aceasta inseamna ca intervalele [b − 2, b − 1) si [a + 2k + 1, a + 2k + 2)
nu sunt disjuncte, dar asta ar insemna ca functia nu este injectiva, fals. Daca

255
256

a + 2k − 1 > b − 1, aplicam din nou rationamentul de mai sus si obtinem ca functia


nu ar mai fi injectiva. Acum nu mai ramane decat cazul in care a + 2k = b − 2, de
unde obtinem ca b − a = 2k + 2.
b) Sa presupunem ca f nu ar fi injectiva. Inseamna ca ar exista doua numere
reale x1 , x2 ∈ [a, b), astfel incat f (x1 ) = f (x2 ). Insemna ca g(f (x1 )) = g(f (x2 )),
ceea ce este fals, pentru ca ar insemna ca g ◦ f nu ar fi injectiva. Acum, de la
punctul a cunoastem forma functiilor injective f cu proprietatea P. Inseamna ca
f (x) = x + 1, ∀x ∈ [a + 2k, a + 2k + 1), k ∈ N, a + 2k + 2 ≤ b si f (x) = x − 1, ∀x ∈
[a + 2k + 1, a + 2k + 2), k ∈ N, a + 2k + 2 ≤ b.
Atunci trebuie ca si g sa fie injectiva, deci f = g.

3. Folosim teorema sinusurilor dupa cum urmeaza: a = 2R sin A, b = 2R sin B, c =


2R sin C, R este raza cercului circumscris triunghiului ABC. Inlocuind obtinem:
tan A + tan B + tan C ≥ 2(sin A + sin B + sin C). Observatie:
sin x + sin y ≤ sin( x+y
2
). Aceasta relatie se demonstreaza imediat, folosind trans-
formari echivalente. Rezulta, conform inegalitatii Jensen, ca: 2(sin A + sin B +

sin C) ≤ 6 sin A+B+C
3
= 6 sin π
3
= 3 3. Acum, vom demonstra ca:
sin(x+y) sin( x+y )
tan x + tan y ≥ 2 tan( x+y
2
). Transformam pe tan x + tan y = cos x cos y
≥ 2 cos( x+y
2
)

2
cos2 ( x+y
2
) ≥ cos x cos y, deci 1+cos(x+y) ≥ 2 cos x cos y, deci rezulta sin(x−y) ≤ 1,
ceea ce este evident adevarat. Aplicand inegalitatea lui Jensen pentru functia tan-
genta avem ca:

tan A + tan B + tan C ≥ 3 tan( A+B+C ) = 3 tan( π3 ) = 3 3 ≥ 2( sin A) si problema
P
3
cyc
s-a incheiat.

4. a) Pentru k < n − 1, rezultatul este imediat deoarece 0 ≤ gk ≤ k ≤ n − 2, dar


functia este definita in n puncte, prin urmare, exista macar doua care sa fie luate
de doua ori.
Acum pentru k > 2n − n + 1. Sa presupunem ca exista astfel de functii. Exista 2n
functii in Fn . Din motive de simetrie, observam ca:
2n
fj (i) = 2n−1 , ∀i ∈ An , deci o constanta. Atunci daca luam restul de 2n − k
P
j=1
functii, observam, ca putem forma cu ele g2n −k (i) = 2n−1 −gk (i) injectiva, deoarece
gk e injectiva. Dar 2n − k < n − 1, dar din demonstratia de mai sus, rezulta ca nu
putem alege astfel de functii. Problema s-a incheiat.
b) Pentru a gasi functii care sa genereze functii gk injective trebuie sa ne uitam
in primul rand la felul in care gasim aceste functii pentru k = n − 1 si k = n.
Sa consideram pentru k = n − 1, urmatoarele valori ale functiilor: fi (1) = 0,
f1 (2) = 1, fi (2) = 0, ∀i > 1 si in mod analog fi (j) = 1, i < j, fi (j) = 0, i ≥ j.
Acum sa ne uitam la cazul in care k = n. Aici, configuratia este simpla daca luam
257

elementele unei matrici


1 2 3 4 ... n
1 1 1 1 1 1
2 0 1 1 1 1
3 0 0 1 1 1
4
.
.
.
n 0 0 0 0 1
Se observa in mod evident ca aceasta matrice convine cerintelor de injectivitate.
Acum pentru un numar k aflat in multimea {n − 1, n, ..., 2n − n + 1}, vom lua
un patrat de n × n, pe care il vom completa cu 1 si 0 ca mai sus in matricea
n × k ce simbolizeaza valorile functiilor fi in toate punctele lui An . Acum, restul
randurilor din matrice le completam doua cate doua astfel:
1 2 3 4 ... n
n+1 0 1 0 0 ... 0
n+2 1 0 1 1 ... 1
pentru a pastra suma egala fn+1 (i) + fn+2 (i) = fn+1 (j) + fn+2 (j) = 1, ∀i, j ∈ An
Alegerea acestor doua rtanduri a fost arbitrara. Putem alege oricate casute mar-
cate cu 1 si oricare ar fi pozitia lor in cadrul matricei. Evident, numarul acestor
randuri va fi 2 k−n
n
2
daca k-n ar fi par si 2 2
+ 1, atunci cand nu putem forma
perechi cu toate randurile adaugam randul
0, 0, 0..., 0

5.

6. Fie {H1 } = AD ∩ BC si in mod analog H2 , H3 . BH1 = cos B · c. Evident,


deoarece ABDC inscriptibil, avem ∠ADB = ∠C, ∠ADC = ∠B. Atunci, BD =
cos B·c
sin C
, CD = cos C·b
sin B
.
cos B cos Cbc sin A
σ[BCD] = sin B sin C
si in mod analog pentru celelalte triunghiuri exterioare.
Daca ariile sunt egale, atunci avem: cos B cos Cbc
sin B sin C
= cos A cos Cac
sin A sin C

cos B sin A = cos A sin B ⇔ cot B = cot A ⇔ A = B. In mod analog demonstram
ca B = C.
Daca perimetrele vor fi egale, atunci procedam in mod analog si initial avem:
b(1 + cos A
sin B
+ cos C
sin B
) = a(1 + cos C
sin A
+ cos B
sin A
), apoi dupa transformari trigonometrice
obtinem: √
sin A+B2
= 23 , deci C = 60◦ . Analog, obtinem ca triunghiul e echilateral.
258

7. tkP Q ⇔ tan α = tan ∠(t, M N ) = tan ∠(P Q, M N ) = tan β.


−r2
β = 2 · γ, γ = ∠(O1 O2 , M N ). tan γ = M Nr1+r 1 +r2
, unde r1 , O1 , r2 , O2 sunt razele
respectiv centrele cercurilor inscrise triunghiurilor BN C respectiv AM D.Deci
2 tan γ 2(r1 −r2 )(M N +r1 +r2 ) r1 −r2
tan β = 1−tan 2 γ = 4r r +M N 2 +2M N (r +r ) = tan α = M N . In cele din urma, aceasta
1 2 1 2
conduce la:

M N 2 (r1 − r2 ) = 4r1 r2 (r1 − r2 ). De aici rezulta ca M N = 2 r1 r2 sau r1 = r2 , de
unde rezulta ca trapezul e isoscel.

8. Sa observam ca relatia din enunt este echivalenta cu a spune ca:


CM
BC
= AN
AD
, acum deoarece M este mobil pe [BC], putem sa ne uitam mai intai la
cazul in care M = C. Rezulta ca N = A. De unde rezulta imediat ca punctul P1
se afla pe AC, astfel incat CP AC
1
= k. In mod analog, punand pe M = B, rezulta ca
N = D. Deci punctul P2 se va afla pe [BD] si BP BD
2
= k.
Acum vom demonstra ca locul geometric al punctelor P este segmentul [P1 P2 ].
Vom demonstra aceasta afirmatie vectorial. Sa scriem vectorul P~1 P , in functie de
λ, unde λ = BM CM
= DNAN
. P~1 P = (1 − k)P1~M + k P1~N = λ+11
(λ(P~1 C + k CP
~ 1 + k P~1 A) +
~ 1 + P~1 B + k P~1 D) = 1 (λ(P~1 C + k CA)
k BP ~ + k BD~ + P~1 B), dar stim ca P~1 C = ACk, ~
λ+1
de unde P~1 P = λ+1 1 ~ + P~1 B).
(k BD
In acelasi timp avem ca P1~P2 = P~1 B + k BD, ~ deci cei doi vectori sunt coliniari,
deci locul geometric este intocmai segmentul [P1 P2 ].

9.

10. Sa numaram mai intai cate functii f exista in total. Rezultatul este imediat, se pot
numara exact nn functii. Acum vom demonstra ca exista in total nn−1 functii care
sa respecte proprietatea din enunt. Sa consideram fixate primele n-1 valori ale
functieie f . Avem ca:
n−1
P
f (i) ≡ t (mod n), t ales arbitrar. Evident exista si este unic x ∈ An , a.i. x ≡
i=1
−t (mod n). Deci este de ajuns sa numaram in cate feluri putem sa dam valori
functiei numai pentru n − 1 puncte, deci nn−1 functii.

11. Vom demostra ca orice numar rational se poate scrie sub forma x. In rpimul rand
sa facem observatia ca x ∈ Q, deoarece toate numerele de forma ai ∈ N. Sa
alegem ai = i, ∀n > i ≥ 1. Dam factor comun pe a1n si aducand la acelasi numitor,
obtinem:
2
x = a1n a1n ( 1+1 +2!·2+3!·3+...+(n−1)!·(n−1)
(n−1)!
Se calculeaza simplu ca expresia de la numarator este egala cu n!. Deci, in final
ne va ramane:
n!
x = (n−1)! = ann . Stim ca orice numar rational q se poate scrie sub forma q = m
n
,
unde m, n ∈ N. Numarul natural an joaca rolul de numitor si poate fi ales arbitrar.
259

12. Sa facem urmatoarea ordonare a celor trei numere reale: a ≥ b ≥ c. Acum sa


observam ca:
1 1 1 1 1 1
c+b
≥ a+c ≥ a+b . Aplicam inegalitatea Chebyshev pentru c+b ≥ a+c ≥ a+b si
2 2 2
pentru a ≥ b ≥ c , obtinand astfel:
P a2
≥ 13 a2
P P 1 P 1 9
b+c b+c
= b+c
≥ 2(a+b+c) ,din inegalitatea Cauchy-Schwarz, dar
cyc cyc cyc
2
3 = a + b + c ≥ (a+b+c)
2 2 2
3
⇔ (a + b + c) ≤ 3, deci
9 3
2(a+b+c)
≥ 2 si problema s-a incheiat.

13. Vom rezolva aceasta problema vectorial, tinand cont ca produsul a doi vectori
perpendiculari este 0. Sa luam vectorul M~M 0 , unde M, M 0 reprezinta mijloacele
laturilor [AC 0 ] si [A0 C]. M~M 0 = AA~ 0 + C~0 C. In mod analog, considerand N, N 0
mijloacele laturilor [B 0 D] si [BD0 ], obtinem N~N 0 = B~0 B + DD~ 0 . Inmultind cele
doua relatii obtinute mai sus si egaland cu 0, obtinem:
AA0 · DD0 cos A + AA0 · B 0 B cos B + CC 0 · B 0 B cos C + CC 0 · DD0 cos D = 0, dar
CC 0 = AA0 si BB 0 = DD0 . Dam factor comun AA0 · BB 0 si obtinem:
4 cos B+D
2
sin C+D
2
sin A+D
2
= 0, de unde nu ne poate rezulta decat ca B + D = π,
deoarece sin x = 0 ⇒ x ∈ kπ, k ∈ Z, deci C + D sau A + D este 2π. FALS!
Patrulaterul este convex. Deci ABCD inscriptibil.

14.

15. Pentru a demonstra prima inegalitate:


P a
a2 +a+1
≥ 1/3, vom folosi urmatoarea observatie:
a
a2 +a+1
≥ a3 . Acest fapt se demonstreaza imediat folosind semnul functiei de
gradul doi rezultate in urma aducerii relatiei intr-un singur membru.
Cea de-a doua inegalitate se poate demonstra folosind concavitatea functiei f (x) =
x
x2 +x+1
pe intervalul [0, 1]. Sa demonstram aceasta concavitate, folosind semnul
celei de-a doua derivate a lui f.
2 2 2 +x+1)2 )0 (1−x2 )
f 00 (x) = −2x(x +x+1)(x−((x
2 +x+1)4 . Evident, cum (x2 + x + 1)2 are coefici-
enti pozitivi, atunci coeficientii derivatei vor fi si ei pozitivi, iar cum x ≥ 0, in-
seamna ca [(x2 + x + 1)2 ]0 ≥ 0, deci −[(x2 + x + 1)2 ]0 ≤ 0. (1 − x2 ) ≥ 0, deci
−[(x2 + x + 1)2 ]0 (1 − x2 ) − 2x(x2 + x + 1)2 ≤ 0
f 00 (x) ≤ 0, ∀x ∈ [0, 1], deci functia este concava pe acest interval. Acum, sa ne
uitam la:
f (a) + f (b) + f (c) ≤ 3f ( a+b+c
3
) = 9/13, problema s-a incheiat.

16.4 Clasa a X-a


260
Capitolul 17

Probleme avute ı̂n atenţia comisiei, 2009

17.1 Clasa a VII-a

17.2 Clasa a VIII-a

17.3 Clasa a IX-a


1. Se cunoaste faptul ca a − 1 < bac ≤ a şi b − 1 < bbc ≤ b. Prin urmare a + b <
bac + bbc + 2, ceea ce ne conduce la inegalitatea cunoscută ba + bc ≤ bac + bbc + 1.
Generalizăm această inegalitate prin inducţie astfel:
Fie numerele reale x1 , ..., xn , n ≥ 2. Atunci are loc

bx1 + x2 + ... + xn c ≤ bx1 c + bx2 c + ... + bxn c + n − 1.

Inegalitatea este demonstrată mai sus pentru n = 2. Presupunem acum că ine-
galitatea are loc pentru k ≥ 2 numere reale x1 , ..., xk . Deci

bx1 + x2 + ... + xk c ≤ bx1 c + bx2 c + ... + bxk c + k − 1.

Fie y ∈ R arbitrar. Atunci folosind cazul n = 2 avem

bx1 + x2 + ... + xk + yc ≤ bx1 + ... + xk c + byc + 1.

Folosim acum ipoteza de inducţie obţinem

bx1 + ... + xk c + byc + 1 ≤ bx1 c + bx2 c + ... + bxk c + k − 1 + byc + 1.

Combinând cele de mai sus obţinem

bx1 + ... + xk + yc ≤ bx1 c + bx2 c + ... + bxk c + byc + k,

261
262

pentru orice k+1 numere reale x1 , ...xk , y. Conform principiului inducţiei matem-
atice, inegalitatea are loc pentru orice n ≥ 2.
Acum, pentru a soluţiona problema este suficient să observăm că
a2 − a1 a3 − a2 an+1 − an an+1 − a1
+ + ... + = .
k k k k
Nu ne mai rămâne decât să aplicăm inegalitatea demonstrată mai sus pentru cele
n numere a2 −ak
, k , ..., an+1k−an .
1 a3 −a2

n √  n
X 2k − 1 X 2k − 1
2. Din inegalitatea Cauchy Buniakowski Schwartz avem ca 2
≤n .
k=1
(x + k ) k=1
(x + k 2 )2
n
2 2
X 2k − 1
Să observăm că x+k −(x+(k−1) ) = 2k−1. Aşadar din faptul că <
k=1
(x + k 2 )2
n
X 2k − 1
, combinat cu observaţia avem că ultima sumă o putem
k=1
(x + k )(x + (k − 1)2 )
2

n n n
X 2k − 1 X 1 X 1
telescopa ,adică explicit = − =
k=1
(x + k )(x + (k − 1) ) k=1 x + (k − 1) k=1 x + k 2
2 2 2

1 1 1
− 2
< .
x x+n x
3.

4. Calculând avem
an |1 − an |
|an+1 − 1| = |1 − an | 2
≤ .
1 + an 2
Din enunt avem inegalitatea evidenta |a1 − 1| < 1, care combinată cu inegalitatea
recursiva obţinută mai sus rezolvă complet problema.

5. a) Să observăm că au loc egalităţile


a2 a2 b a b2 b
= = , = .
a+1 ab + b b+1 b+1 a+1

Astfel inegalitatea de demonstrat este echivalentă cu


a b
+ ≥ 1 ⇔ a2 + a + b2 + b ≥ ab + a + b + 1 ⇔ a2 + b2 ≥ 1 + ab,
b+1 a+1
inegalitate care este adevărată pentru că ab = 1.
n
X a2i
b) Aplicăm Cauchy Buniakowski Schwartz pentru a obţine că ≥
i=1
s + 1 − ai
s2 s2
. Pentru a finaliza demonstraţia este suficient să demonstrăm că ns+n−s ≥
ns + n − s
263

1, ceea ce este echivalent cu s2 ≥ (n−1)s+n sau (s−n)(s+1) ≥ 0. Din inegalitatea


mediilor avem s ≥ n, şi demonstraţia se incheie.

17.4 Clasa a X-a


1. Transformăm relaţia din enunţ după cum urmează ţinând cont că ab = a + b şi
prin urmare (a − 1)(b − 1) = 1.

x[(b − 1)ax + (a − 1)bx − x − ab + 2] = (ax − 1)(bx − 1)


x[(b − 1)(ax − 1) + (a − 1)(bx − 1)] − x2 = (ax − 1)(bx − 1)
ax − 1 bx − 1 ax − 1 b x − 1
x +x − x2 =
 x a − 1   xb − 1  a−1 b−1
a −1 b −1
−x −x =0
a−1 b−1

Unul dintre factori este 0. Să presupunem că


ax − 1
− x = 0 ⇔ ax = (a − 1)x + 1.
a−1
Funcţia ax cu a > 1 este strict convexă. Graficul unei funcţii strict convexe este
intersectat de o dreaptă ı̂n cel mult două puncte (demonstraţia este imediată
folosind definiţia funcţiei strict convexe). În egalitatea noastră avem ı̂n mem-
brul drept ecuaţia unei drepte şi ı̂n membrul stâng o funcţie strict convexă. Din
cele amintite mai sus rezultă că ecuaţia are maxim două soluţii. Se verifică direct
că x = 0 şi x = 1 sunt soluţii. Anularea celui de-al doilea factor ne duce la aceeaşi
concluzie. Prin urmare ecuaţia considerată are soluţia S = {0, 1}.

2. a) Prelucrăm trigonometric relaţia următoare:

sin(x − y) + sin(y − z) + sin(z − x) =


x−z x + z − 2y x−z x−z
2 sin cos − 2 sin cos =
2 2 2 2
x−z x + z − 2y x−z
2 sin cos − cos =
2 2 2
 
x−z x−y z−y
2 sin (−2) sin sin =
2 2 2
x−y y−z x−z
4 sin sin sin
2 2 2
Revenind la problemă, folosim binecunoscuta relaţie pentru raza cercului circum-
abc
scris unui triunghi, R = , şi faptul că punctele A, B, C sunt pe cercul unitate,
4S
264

care are raza 1. Prin urmare, folosind şi teorema sinusurilor in triunghiul ABC
avem
x−y y−z x−z
16S 2 = a2 b2 c2 = 4 sin2 4 sin2 4 sin2 =
2 2 2
= 4[sin(x − y) + sin(y − z) + sin(z − x)]2 .
Simplificând cu 4 relaţia obţinem ceea ce trebuia demonstrat.
b) Dacă a + b + c = 0 există x, y, z ∈ (0, π) cu a = x − y, b = y − z, c = z − x, şi
considerăm triunghiul ABC ca la punctul a). Astfel | sin a + sin b + sin c| = √ 2S,
3 3
unde S este aria triunghiului ABC. Inegalitatea este echivalentă cu S ≤ .
4
Astfel trebuie sa demonstrăm că triunghiul de arie maximă ı̂nscris ı̂ntr-un cerc
de rază 1 este cel echilateral.
Pentru aceasta, considerăm un triungi ABC ı̂nscris ı̂n cerc. În primul rând, ob-
servăm că centrul cercului trebuie sa fie ı̂n interiorul triunghiului de arie maximă,
pentru că altfel putem găsi un alt triunghi cu baza şi ı̂nălţimea,şi prin urmare aria.
Acum, fie α, β, γ ∈ [0, π] unghiurile la centru determinate de cele două laturi.
Din formula ariei folosind două laturi şi sinusul unghiului cuprins ı̂ntre acestea,
putem deduce că aria triunghiului este 12 (sin α + sin β + sin γ). Deoarece funcţia
sin este concavă pe intervalul [0, π] avem inegalitatea

1 1 α+β+γ 3 3
(sin α + sin β + sin γ) ≤ 3 sin = ,
2 2 3 4
adică ceea ce trebuia demonstrat.

3. Să considerăm primele două ecuaţii ale sistemului. Atunci


(x + y + z)2 − x2 − y 2 − z 2 6
xy + yz + zx = = = 3 = x2 + y 2 + z 2 .
2 2
Este cunoscut faptul că dacă trei numere complexe verifică identitatea de mai sus,
atunci afixele lor determină un triunghi echilateral, posibil degenerat la un punct.
Astfel există a ∈ C şi r ∈ (0, ∞) cu proprietatea că x = a+r, y = a+εr, z = a+ε2 r,
2π 2π
unde ε = cos + i sin .
3 3
Din prima ecuaţie reiese că
3a + r(1 + ε + ε2 ) = 3 ⇔ 3a = 3 ⇔ a = 1.

Este evident că pentru orice r număr real, primele două ecuaţii sunt verificate.
Prin urmare, pentru n ∈ {1, 2} avem o infinitate de soluţii. Să presupunem acum
că n ≥ 3. Dezvoltând cu binomul lui Newton expresiile obţinem
   
n 3 n
r + ... + 3 n rb 3 c = 3.
n
3+3
3 b3c
265

Pentru n ≤ 5 ecuaţia de mai sus are soluţia unică r = 0. Dacă n ≥ 6 ecuaţia


mai are cel puţin o radacina complexă r0 . Atunci se poate verifica imediat că
x = 1 + r0 , y = 1 + r0 ε, z = 1 + r0 ε2 este soluţie a sistemului dat.
În concluzie, pentru ca sistemul să aibă soluţie unică este necesar şi suficient ca
3 ≤ n ≤ 5.
266
Capitolul 18

Probleme avute ı̂n atenţia comisiei, 2010

18.1 Clasa a VII-a

18.2 Clasa a VIII-a

18.3 Clasa a IX-a

1. Pentru a obţine restricţia asupra lui a, consider cazul când z = 0. Inegalitatea din
enunţ revine la (x+y)3 +axy(x+y) ≥ 0 sau echivalent (x+y)(x2 +(2+a)xy +y 2 ) ≥
0, de unde folosind x + y ≥ 0 avem x2 + (2 + a)xy + y 2 ≥ 0, ∀x, y ≥ 0. Putem sa
x
dezomogenizăm notând = t, iar inegalitatea devine t2 + (2 + a)t + 1 ≥ 0, ∀t ≥ 0.
y
Privind ca ecuaţie de gradul 2, avem ∆ = (2 + a)2 − 4 = (a2 + 4a). Dacă ∆ > 0
atunci ecuaţia are două radacini reale distincte, şi din ipoteze trebuie sa fie ambele
negative deci, daca le notăm −x1 , −x2 avem că x1 x2 = 1 si x1 + x2 = 2 + a. Avem
din inegalitatea mediilor x1 + x2 ≥ 2 de unde a ≥ 0. În celălalt caz, ∆ ≤ 0,
numărul a trebuie să se afle ı̂ntre rădăcini deci −4 ≤ a ≤ 0.
Aşadar a ≥ −4 de unde a = −4 + r cu r ≥ 0.
Rescriem acum inegalitatea x3 + y 3 + z 3 + r(x + y + z)(xy + yz + zx) + (b − 3)xyz ≥
(x + y + z)(xy + yz + zx), de unde mai departe x3 + y 3 + z 3 + 6xyz − (x + y +
z)(xy + yz + zx) + r[(x + y + z)(xy + yz + zx) − 9xyz] + (b + 9r − 9)xyz ≥ 0. Dacă
considerăm x = y = z obţinem (b + 9r − 9)x3 ≥ 0, de unde b + 9r − 9 ≥ 0, prin
urmare b = 9 − 9r + s, cu s ≥ 0.

267
268

2. Avem
BC CA AB a2 b2 c2
+ + = + + ≥
MD ME MF 2[BM C] 2[AM C] 2[AM B]
(a + b + c)2
≥ =
2[ABC]
a+b+c
= ,
r
unde am notat [XY Z] aria triunghiului XY Z şi am aplicat inegalitatea

x2 y 2 z 2 (x + y + z)2
+ + ≥ , x, y, z ∈ R, u, v, w > 0,
u v w u+v+w
care este o variantă a inegalităţii Cauchy Buniakowski Schwarz, cu egalitate dacă
şi numai dacă
x y z
= = .
u v w
Ipoteza ne asigură egalitatea ı̂n această inegalitate, de unde deducem că

a b c
= = ,
[BM C] [AM C] [AM B]

ceea ce este echivalent cu


1 1 1
= = ,
MD ME MF
adică M este centrul cercului ı̂nscris şi punctele D, E, F sunt punctele de tangenţă
ale acestui cerc cu laturile triunghiului. Astfel AD, BE, CF sunt concurente ı̂n
punctul lui Gergonne al triunghiului ABC. Pentru o demonstraţie imediată se
foloseşte teorema lui Ceva şi faptul că tangentele duse dintr-un punct la un cerc
dat au lungimi egale.

3. Deoarece Ia M este bisectoare ı̂n triunghiul BCIa avem, conform teoremei bisec-
toarei
BM Ia B BD sin C2
= = .
CM Ia C CD sin B2

Notăm α = ∠BAD, β = ∠CAD. Atunci, aplicând teorema sinusurilor ı̂n tri-


unghiurile ABD, ACD, AF P, AEP şi folosind că triunghiul AEF este isoscel obţinem:

FP sin α
=
EP sin β
sin α BM sin B
= .
sin β M C sin C
269

Deasemenea, tot din teorema sinusurilor aplicată ı̂n triunghiurile BF D, CED


obţinem
FD BD cos B2
= .
DE CD cos C2
Să combinăm acum rezultatele de mai sus:

FD BM sin B2 cos B2 BM sin B sin α FP


= C C
= = = .
DE CM sin 2 cos 2 CM sin C sin β EP
Conform reciprocei teoremei bisectoarei ı̂n triunghiul DEF , DP este bisectoarea
unghiului ∠F DE.

4. Se considera patru segmente cu originea comuna O, notate OA = x, OB = y, OC =


z, OD = t astfel ı̂ncât ∠AOB = ∠COD = 120◦ , ∠BOC = ∠AOD = 60◦ . Aplicând
teorema cosinusurilor ı̂n triunghiurile OAB, OBC, OCD, ODA inegalitatea de demon-
strat devine
AB · CD + AD · BC ≥ AC · BD,
care este inegalitatea lui Ptolemeu pentru un patrulater convex. Egalitatea are loc
daca şi numai dacă patrulaterul ABCD este inscriptibil, ceea ce se traduce prin
xz = yt.

5. După desfacerea
X X parantezelor
X şi folosirea faptului că abc = 1 inegalitatea se re-
3 3 3
scrie ca a + a b ≥2 a2 b. Din inegalitatea mediilor avem a3 +a3 b3 +1 ≥
cyc cyc cyc
X X X
2
3a b de unde prin sumare avem a3 + a3 b 3 + 3 ≥ 3 a2 b. Folosind din nou
cyc cyc cyc
X
2
medii avem a b ≥ 3. Cu aceasta problema se ı̂ncheie.
cyc

2bc A
6. Formulele pentru lungimea bisectoarelor au o scriere de forma ba = cos , şi
b+c 2
analoagele. Presupunem că triunghiul ABC nu este echilateral. Atunci va exista
un unghi, să zicem A cu măsura mai mare decât π/3 (cel mai mare unghi), şi va
exista un unghi, să zicem C cu măsura mai mică decât π/3(cel

mai mic unghi).

Atunci, folosind monotonia funcţiei cosinus, avem cos 2 < 2 şi cos C2 > 23 .
A 3

Atunci avem √ √ √
2bc 3 3 3
ba < ≤ max{b, c} ≤a ,
b+c 2 2 2
şi √ √ √
2ab 3 3 3
bc > ≥ min{a, b} ≥c ,
a+b 2 2 2
adică ceea ce trebuia demonstrat.
270

18.4 Clasa a X-a


1. Următoarea inegalitate ı̂n numere complexe este cunoscută:

|a + b + c|2 + |a|2 + |b|2 + |c|2 = |a + b|2 + |b + c|2 + |c + a|2 .

Facând substitutiile a = c, b = a − c, c = b − a avem

|b|2 + |c|2 + |a − c|2 + |a − b|2 = |a|2 + |b − c|2 + |b + c − a|2 ,

adică
2 + |a − c|2 + |a − b|2 ≥ 1 + |b − c|2 ,
ceea ce este echivalent cu ce dorim să demonstrăm.
ln a
2. Avem x = , şi cele similare. Astfel putem renota pentru a avea variabile
ln b + ln c
X 1
pozitive cu p, q, r respectiv − ln a, − ln b, − ln c. Inegalitatea revine la p q ≤
cyc +1+ +1
q+r p+r
X 1
1, ceea este echivalent cu ≤ p + q + r. D. Folosind acum ine-
1 1
cyc +
q+r p+r
1 x+y
galitatea intre media armonică şi aritmetică ≤ , demonstraţia este
1 1 4
+
x y
incheiată.
 
p−1
3. Principiul cutiei ne sugerează să alegem numărul căutat de forma N = ·
2
m+1. Împărţind mulţimea dată ı̂n submulţimi de forma {1, 2, ..., p}, {p+1, ..., 2p}, ..., {(m−
1)p + 1, ..., mp}, dacă alegem  N numere
 din mulţimea iniţială, conform principi-
p−1
ului cutiei, vor fi cel puţin + 1 elemente dintr-o submulţime de mai sus,
2
ceea ce ı̂nseamnă că vor exista două elemente care dau resturile k, respectiv p − k
la ı̂mpărţirea cu p. Aceste două elemente au suma divizibilă cu p.
 
p−1
Reciproc, dacă considerăm toate elementele care dau resturile 1, 2, ..., ,
2
acestea sunt ı̂n număr de N − 1 şi evident oricum am aduna două dintre ele, nu
putem obţine un număr divizibil cu p.

4.

S-ar putea să vă placă și